Vous êtes sur la page 1sur 340

Partnership Basic Considerations and Formation

CHAPTER 1
MULTIPLE CHOICE ANSWERS AND SOLUTIONS
1-1: a
1-2: b
1-2: c
1-3: a

1-4: b

1-5: d

Jose's capital should be credited for the market value of the computer contributed by
him.
(40,000 + 80,000) 2/3 = 180,000 x 1/3 = 60,000.

Cash
Land
Mortgage payable

P100,000
300,000
( 50,000)

Net assets (Julio, capital)

P350,000

Total Capital (P300,000/60%)


Perla's interest

P500,000
______40%

Perla's capital
Less:Non-cash asset contributed at market value
Land
P 70,000
Building
90,000
Mortgage Payable
( 40,000)

P200,000

Cash contribution

P 80,000

_120,000

- Zero, because under the bonus method, a transfer of capital is only required.

1-6: b

Reyes

Santos

Cash
Inventory
Building
Equipment
Mortgage payable

P200,000

150,000
________

P300,000
150,000
400,000
( 100,000)

Net asset (capital)

P350,000

P750,000

AA

BB

CC

P55,000
P55,000

1-7: c
Cash
Property at Market Value
Mortgage payable
Equipment at Market Value

P 50,000
_______

P 80,000
( 35,000)
_______

Capital

P 50,000

P 45,000

Chapter 1

1-8: a

1-9: c

PP

RR

SS

Cash
Computer at Market Value

P 50,000
__25,000

P 80,000
_______

P 25,000
__60,000

Capital

P 75,000

P 80,000

P 85,000

Maria

Nora

Cash
Merchandise inventory
Computer equipment
Liability
Furniture and Fixtures

P 30,000

200,000

P 90,000
160,000
( 60,000)
________

Total contribution

P230,000

P190,000

Total agreed capital (P230,000/40%)


Nora's interest

P575,000
______60%

Nora's agreed capital


Less: investment

P345,000
190,000

Cash to be invested

P155,000

1-10: d

Roy

Sam

Tim

Cash
Office Equipment
Note payable

P140,000

________

P220,000
_( 60,000)

______

Net asset invested

P140,000

P160,000

Agreed capitals, equally (P300,000/3) =

P100,000

1-11: a

Lara

Mitra

Cash
Computer equipment
Note payable

P130,000

________

P200,000
50,000
_( 10,000)

Net asset invested

P130,000

P240,000

Goodwill (P240,000 - P130,000) =

P110,000

1-12: a

Perez

Reyes

Cash
Office Equipment
Merchandise
Furniture
Notes payable

P 50,000
30,000

_______

P 70,000

110,000
100,000
( 50,000)

Net asset invested

P 80,000

P230,000

Partnership Basic Considerations and Formation

Bonus Method:
Total capital (net asset invested)

P310,000

Goodwill Method:
Net assets invested
Add: Goodwill (P230,000-P80,000)

P310,000
_150,000

Net capital
1-13: b

1-14: d

1-15: d

P460,000

Required capital of each partner (P300,000/2)


Contributed capital of Ruiz:
Total assets
P105,000
Less Liabilities
__15,000

P150,000

Cash to be contributed by Ruiz

P 60,000

Total assets:
Cash
Machinery
Building
Less: Liabilities (Mortgage payable)

P 70,000
75,000
_225,000

__90,000

P370,000
__90,000

Net assets (equal to Ferrer's capital account)


Divide by Ferrer's P & L share percentage

P280,000
____70%

Total partnership capital

P400,000

Required capital of Cruz (P400,000 X 30%)


Less Assets already contributed:
Cash
P 30,000
Machinery and equipment
25,000
Furniture and fixtures
__10,000

P120,000

Cash to be invested by Cruz

P 55,000

__65,000

Adjusted assets of C Borja


Cash
P 2,500
Accounts Receivable (P10,000-P500)
9,500
Merchandise inventory (P15,000-P3,000) 12,000
Fixtures
__20,000
Asset contributed by D. Arce:
Cash
P 20,000
Merchandise
__10,000

__30,000

Total assets of the partnership

P 74,000

P 44,000

1-16: a

Chapter 1

Cash to be invested by Mendez:


Adjusted capital of Lopez (2/3)
Unadjusted capital
Adjustments:
Prepaid expenses
Accrued expenses
Allowance for bad debts (5% X P100,000)

P158,400
17,500
( 5,000)
_( 5,000)

Adjusted capital

1-17: d

P165,900

Total partnership capital (P165,900/2/3)


Multiply by Mendez's interest

P248,850

Mendez's capital
Less Merchandise contributed

P 82,950
__50,000

Cash to be invested by Mendez

P 32,950

Total Capital:
Adjusted capital of Lopez
Contributed capital of Mendez

P165,900
__82,950

Total capital
Moran, capital (40%)
Cash
Furniture and Fixtures
Divide by Moran's P & L share percentage

P248,850
P 15,000
_100,000

Total partnership capital


Multiply by Nakar's P & L share percentage
Required capital of credit of Nakar:
Contributed capital of Nakar:
Merchandise inventory
Land
Building
Total assets
Less Liabilities
Required cash investment by Nakar
1-18: c

P115,000
______40%
P287,500
______60%
P172,500

P 45,000
15,000
__65,000
P125,000
__30,000

P 95,000
P 77,500

Garcia's adjusted capital (see schedule 1)


Divide by Garcia's P & L share percentage

P40,500
______40%

Total partnership capital


Flores' P & L share percentage

P101,250
______60%

Flores' capital credit


Flores' contributed capital (see schedule 2)

P 60,750
__43,500

Additional cash to be invested by Flores

P 17,250

Partnership Basic Considerations and Formation

Schedule 1:
Garcia, capital:
Unadjusted balance
Adjustments:
Accumulated depreciation
Allowance for doubtful account

P 49,500
( 4,500)
( 4,500)

Adjusted balance

P 40,500

Schedule 2:
Flores capital:
Unadjusted balance
Adjustments:
Accumulated depreciation
Allowance for doubtful accounts
1-19: d

P 57,000
( 1,500)
( 12,000)

Adjusted balance

P 43,500
Ortiz

Ponce

Total

( 60%)
( 40%)
P133,000
P108,000
P241,000

Unadjusted capital balances


Adjustments:
Allowance for bad debts
Inventories
Accrued expenses

( 2,700)
3,000
_( 2,400)

Adjusted capital balances

P130,900

( 1,800)
2,000
( 1,600)
P106,000

( 4,500)
5,000
( 4,000)
P237,500

Total capital before the formation of the new partnership (see above) P237,500
Divide by the total percentage share of Ortiz and Ponce (50% + 30%) ______80%

1-20: d

Total capital of the partnership before the admission of Roxas


Multiply by Roxas' interest

P296,875
______20%

Cash to be invested by Roxas

P 59,375

Merchandise to be invested by Gomez:


Total partnership capital (P180,000/60%)

P300,000

Gomez's capital (P300,000 X 40%)


Less Cash investment

P120,000
__30,000

Merchandise to be invested by Gomez

P 90,000

Cash to be invested by Jocson:


Adjusted capital of Jocson:
Total assets (at agreed valuations)
Less Accounts payable
Required capital of Jocson
Cash to be invested by Jocson

P180,000
__48,000

P132,000
_180,000
P 48,000

1-21: b

1-22: c

1-23: a

1-24: c

1-25: c

1-26: d

Chapter 1

Unadjusted Ell, capital (P75,000 P5,000)


Allowance for doubtful accounts
Accounts payable

P 70,000
( 1,000)
( 4,000)

Adjusted Ell, capital

P 65,000

Total partnership capital (P113,640/1/3)


Less David's capital

P340,920
_113,640

Cortez's capital after adjustments


Adjustments made:
Allowance for doubtful account (2% X P96,000)
Merchandise inventory
Prepaid expenses
Accrued expenses

P227,280

Cortez's capital before adjustments

P211,200

1,920
( 16,000)
( 5,200)
___3,200

Total assets at fair value


Liabilities
Capital balance of Flor

P4,625,000
(1,125,000)
P3,500,000

Total capital of the partnership (P3,500,000 70%)


Eden agreed profit & loss ratio
Eden agreed capital
Eden contributed capital at fair value
Allocated cash to be invested by Eden

P5,000,000
30%
1,500,000
812,000
P 688,000

__Rey
__Sam_ __Tim
__Total_
Contributed capital (assets-liabilities)P471,000 P291,000 P195,000 P957,000
Agreed capital (profit and loss ratio) 382,800 382,800 191,400 957,000
Capital transfer (Bonus)
P 88,200 P(91,800) P 3,600
Total agreed capital (P90,000 40%)
Contributed capital of Candy (P126,000+P36,000-P12,000)
Total agreed capital (P90,000 40%)
Candy, agreed capital interest
Agreed capital of Candy
Contributed capital of Candy
Withdrawal

P225,000
150,000
225,000
60%
135,000
150,000
P 15,000

Partnership Basic Considerations and Formation


1-27: a

1-28: a

Total agreed capital (210,000 70%)


Noras interest
Agreed capital of Nora
Cash invested
Cash to be invested by Nora

P300,000
30%
P 90,000
42,000
P 48,000

Contributed capital of May (P194,000 - P56,000)


Agreed capital of May (P300,000 x 70%)
Cash to be invested by May

P138,000
210,000
P 72,000

1-29: c
Contributed capital
Agreed capital
Capital invested

__Alex_
P100,000
92,000
P( 8,000)

_Carlos_
P84,000
92,000
P 8,000

__Total__
P184,000
184,000
-

Chapter 1

SOLUTIONS TO PROBLEMS
Problem 1 1
1.

a. Books of Pedro Castro will be retained by the partnership


To adjust the assets and liabilities of Pedro Castro.
1. Pedro Castro, Capital .............................................................
Merchandise Inventory......................................................

600

2. Pedro Castro, Capital .............................................................


Allowance for Bad Debts ..................................................

200

3. Accrued Interest Receivable ..................................................


Pedro Castro, Capital.........................................................

35

Computation:
P1,000 x 6% x 3/12 =
P2,000 x 6% x 2/12 =

600
200
35

P15
_20

Total ......................... ......P35


4. Pedro Castro, Capital .............................................................
Accrued Interest Payable...................................................
(P4,000 x 5% x 6/12 = P100)

100

5. Pedro Castro, Capital .............................................................


Accumulated Depreciation Furniture and Fixtures ........

800

6. Office Supplies ......................................................................


Pedro Castro, Capital.........................................................

400

100

800
400

To record the investment of Jose Bunag.


Cash.. ........................................................................................... 15,067.50
Jose Bunag, Capital ...............................................................
Computation:

Pedro Castro, Capital


(1)
P600 P31,400
(2)
200
35 (3)
(4)
100
400 (6)
(5) ___800
P1,700 P31,835
P30,135
Jose Bunag, Capital : 1/2 x P30,135 = P15,067.50

15,067.50

Partnership Basic Considerations and Formation

b.

A new set of books will be used


Books of Pedro Castro
To adjust the assets and liabilities.
See Requirement (a).
To close the books.
Notes Payable...............................................................................
Accounts Payable .........................................................................
Accrued Interest Payable..............................................................
Allowance for Bad Debts .............................................................
Accumulated Depreciation Furniture and Fixtures ...................
Pedro Castro, Capital ...................................................................
Cash .......................................................................................
Notes Receivable ...................................................................
Accounts Receivable .............................................................
Accrued Interest Receivable ..................................................
Merchandise Inventory ..........................................................
Office Supplies ......................................................................
Furniture and Fixtures............................................................

4,000
10,000
100
1,200
1,400
30,135

6,000
3,000
24,000
35
7,400
400
6,000

New Partnership Books


To record the investment of Pedro Castro.
Cash ...........................................................................................
Notes Receivable..........................................................................
Accounts Receivable ....................................................................
Accrued Interest Receivable.........................................................
Merchandise Inventory.................................................................
Office Supplies.............................................................................
Furniture and Fixtures ..................................................................
Notes Payable ........................................................................
Accounts Payable...................................................................
Accrued Interest Payable .......................................................
Allowance for Bad Debts.......................................................
Accumulated Depreciation Furniture and Fixtures.............
Pedro Castro, Capital .............................................................

6,000
3,000
24,000
35
7,400
400
6,000

4,000
10,000
100
1,200
1,400
30,135

To record the investment of Jose Bunag.


Cash.. ........................................................................................... 15,067.50
Jose Bunag, Capital ...............................................................

15,067.50

10

Chapter 1

2.

Castro and Bunag Partnership


Balance Sheet
October 1, 2008
Assets

Cash ..... ...... ... ...........................................................................................


Notes receivable ..........................................................................................
Accounts receivable .................................................................................... P 24,000
Less Allowance for bad debts...................................................................... ___1,200
Accrued interest receivable .........................................................................
Merchandise inventory ................................................................................
Office supplies ...........................................................................................
Furniture and fixtures ..................................................................................
6,000
Less Accumulated depreciation................................................................... ___1,400
Total Assets ........................................................................................

P21,067.50
3,000.00
22,800.00
35.00
7,400.00
400.00
__4,600.00
P59,302.50

Liabilities and Capital


Notes payable ...........................................................................................
Accounts payable ........................................................................................
Accrued interest payable .............................................................................
Pedro Castro, Capital...................................................................................
Jose Bunag, Capital .....................................................................................
Total Liabilities and Capital ...............................................................

Contributed Capitals:

P 4,000.00
10,000.00
100.00
30,135.00
_15,067.50
P59,302.50

Problem 1 2

Jose:

Capital before adjustment ...................................................... P 85,000


Notes Payable ........................................................................
62,000
Undervaluation of inventory..................................................
13,000
Underdepreciation.................................................................. ( 25,000)
Pedro: Cash .......................................................................................
Pablo: Cash .......................................................................................
11,000
Marketable securities ............................................................. _57,500
Total contributed capital..............................................................................
Agreed Capitals:
Bonus Method:
Jose (P231,500 x 50%)................................................................. P115,750
Pedro (P231,500 x 25%) ..............................................................
57,875
Pablo (P231,500 x 25%)............................................................... __57,875
Total . ........................................................................................... P231,500

P 135,000
28,000
___68,500
P 231,500

Partnership Basic Considerations and Formation

11

Goodwill Method. To have a goodwill, the only possible base is the capital of Pablo. The
computation is:
Contributed
Capital
Jose
Pedro
Pablo
Total

Agreed
Capital

P135,000
28,000
__68,500
P231,500

Goodwill

P137,000 (50%)
68,500 (25%)
__68,500 (25%)
274,000

2,000
40,500
_____
42,500

Total agreed capital (P68,500 25%) = 274,000


Jose, Pedro and Pablo Partnership
Balance Sheet
June 30, 2008
Assets:
Cash
Accounts receivable (net)
Marketable securities
Inventory
Equipment (net)
Goodwill
Total

Bonus Method

Goodwill Method

P 49,000
48,000
57,500
85,000
45,000
______
P284,500

P 49,000
48,000
57,500
85,000
45,000
__42,500
P327,000

P 53,000
115,750
57,875
__57,875
P284,500

P 53,000
137,000
68,500
__68,500
P327,000

Liabilities and Capital:


Accounts payable
Jose, capital (50%)
Pedro, capital (25%)
Pablo, capital (25%)
Total

Problem 1 3
1.

Books of Pepe Basco


To adjust the assets.
a. Pepe Basco, Capital......................................................................
Estimated Uncollectible Account ..........................................

3,200

b. Pepe Basco, Capital......................................................................


Accumulated Depreciation Furniture and Fixtures.............

500

3,200
500

12

Chapter 1

To close the books.


Estimated Uncollectible Account .......................................................
Accumulated Depreciation Furniture and Fixtures..........................
Accounts Payable................................................................................
Pepe Basco, Capital ............................................................................
Cash.. ...........................................................................................
Accounts Receivable ....................................................................
Merchandise Inventory.................................................................
Furniture and Fixtures ..................................................................
2.

4,800
1,500
3,600
31,500

400
16,000
20,000
5,000

Books of the Partnership


To record the investment of Pepe Basco.
Cash .... ... ...........................................................................................
Accounts Receivable ..........................................................................
Merchandise Inventory .......................................................................
Furniture and Fixtures.........................................................................
Estimated Uncollectible account..................................................
Accumulated Depreciation Furniture and Fixtures ...................
Accounts Payable .........................................................................
Pepe Basco, Capital......................................................................

400
16,000
20,000
5,000

4,800
1,500
3,600
31,500

To record the investment of Carlo Torre.


Cash .... ... ...........................................................................................
Carlo Torre, Capital .....................................................................
Computation:
Pepe Basco, capital (Base) ...........................................................
Divide by Pepe Basco's P & L ratio .............................................
Total agreed capital ......................................................................
Multiply by Carlo Torre's P & L ratio..........................................
Cash to be invested by Carlo Torre..............................................

47,250

47,250

P31,500
___40%
P78,750
___60%
P47,250

Problem 1 4
a.

Roces' books will be used by the partnership


1. Adjusting Entries

Books of Sales

(a) Sales, Capital .........................................................................


Accumulated Depreciation Fixtures...............................

3,200

(b) Goodwill ................................................................................


Sales, Capital.....................................................................

32,000

3,200
32,000

Partnership Basic Considerations and Formation

2.

13

Closing Entry
Allowance for Bad Debts .............................................................
Accumulated Depreciation Delivery Equipment ......................
Accumulated Depreciation Fixtures..........................................
Accounts Payable .........................................................................
Notes Payable...............................................................................
Accrued Taxes..............................................................................
Sales, Capital................................................................................
Cash .......................................................................................
Accounts Inventory................................................................
Merchandise Inventory ..........................................................
Prepaid Insurance...................................................................
Delivery Equipment...............................................................
Fixtures ..................................................................................
Goodwill ................................................................................

12,800
8,000
91,200
64,000
40,000
8,000
224,000

4,800
72,000
192,000
3,200
48,000
96,000
32,000

Books of Roces (Books of the Partnership)


1.

2.

Adjusting Entries
(a) Roces, Capital ..............................................................................
Allowance for Bad Debts.......................................................

1,600

(b) Accumulated Depreciation Fixtures..........................................


Roces, Capital........................................................................

16,000

(c) Merchandise Inventory.................................................................


Roces, Capital........................................................................

8,000

(d) Goodwill.......................................................................................
Roces, Capital........................................................................

40,000

1,600
16,000
8,000
40,000

To record the investment of Sales.


Cash .... ... ...........................................................................................
Accounts Receivable ..........................................................................
Merchandise Inventory .......................................................................
Prepaid Insurance................................................................................
Delivery Equipment............................................................................
Fixtures ... ...........................................................................................
Goodwill . ...........................................................................................
Allowance for Bad Debts .............................................................
Accumulated Depreciation Delivery Equipment ......................
Accumulated Depreciation Fixtures..........................................
Accounts Payable .........................................................................
Notes Payable...............................................................................
Accrued Taxes..............................................................................
Sales, Capital................................................................................

4,800
72,000
192,000
3,200
48,000
96,000
32,000

12,800
8,000
91,200
64,000
40,000
8,000
224,000

14

b.

Chapter 1

Sales' books will be used by the partnership


Books of Roces
1. Adjusting Entries
See Requirement (a).
2. Closing Entry
Allowance for Bad Debts .............................................................
Accumulated Depreciation Delivery Equipment ......................
Accumulated Depreciation Fixtures..........................................
Accounts Payable .........................................................................
Accrued Taxes..............................................................................
Roces, Capital ..............................................................................
Cash .......................................................................................
Accounts Receivable .............................................................
Merchandise Inventory ..........................................................
Prepaid Insurance...................................................................
Delivery Equipment...............................................................
Fixtures ..................................................................................
Goodwill ................................................................................

1,600
12,800
64,000
104,000
6,400
224,000

14,400
57,600
132,800
4,800
19,200
144,000
40,000

Books of Sales (Books of the Partnership)


1.

Adjusting Entries
See Requirement (a).

2.

To record the investment of Roces.


Cash .... ... ...........................................................................................
Accounts Receivable ..........................................................................
Merchandise Inventory .......................................................................
Prepaid Insurance................................................................................
Delivery Equipment............................................................................
Fixtures ... ...........................................................................................
Goodwill . ...........................................................................................
Allowance for Bad Debts .............................................................
Accumulated Depreciation Delivery Equipment ......................
Accumulated Depreciation Fixtures..........................................
Accounts Payable .........................................................................
Accrued Taxes..............................................................................
Roces, Capital ..............................................................................

14,400
57,600
132,800
4,800
19,200
144,000
40,000

1,600
12,800
64,000
104,000
6,400
224,000

Partnership Basic Considerations and Formation

c.

15

A new set of books will be opened by the partnership


Books of Roces
1. Adjusting Entries
See Requirement (a).
2. Closing Entry
See Requirement (b).
Books of Sales
1. Adjusting Entries
See Requirement (a).
2. Closing Entry
See Requirement (a).
New Partnership Books
To record the investment of Roces and Sales.
Cash .... ... ...........................................................................................
Accounts Receivable ..........................................................................
Merchandise Inventory .......................................................................
Prepaid Insurance................................................................................
Delivery Equipment (net) ...................................................................
Fixtures (net).......................................................................................
Goodwill ...........................................................................................
Allowance for Bad Debts .............................................................
Accounts Payable .........................................................................
Notes Payable...............................................................................
Accrued Taxes..............................................................................
Roces, Capital ..............................................................................
Sales, Capital................................................................................

19,200
129,600
324,800
8,000
46,400
84,800
72,000

14,400
168,000
40,000
14,000
224,000
224,000

16

Chapter 1

Problem 1 5

1.

To close Magno's books.


Allowance for Bad Debts....................................................................
Accounts Payable................................................................................
Notes Payable .....................................................................................
Accrued Interest Payable ....................................................................
R. Magno, Capital...............................................................................
Cash.. ...........................................................................................
Accounts Receivable ....................................................................
Merchandise Inventory.................................................................
Equipment ....................................................................................
Other Assets .................................................................................

2.

5,000
13,000
12,000
3,000
9,000

To adjust the books of Lagman.


Goodwill . ...........................................................................................
Allowance for Bad Debts .............................................................
J. Lagman, Capital........................................................................

3.

1,000
6,000
10,000
300
24,700

8,000

210
7,790

To record the investment of Magno.


Cash .... ... ...........................................................................................
Accounts Receivable ..........................................................................
Merchandise Inventory .......................................................................
Equipment...........................................................................................
Other Assets........................................................................................
Allowance for Bad Debts .............................................................
Accounts Payable .........................................................................
Notes Payable...............................................................................
Accrued Interest Payable..............................................................
R. Magno, Capital ........................................................................

5,000
13,000
12,000
3,000
9,000

1,000
6,000
10,000
300
24,700

To adjust the investments of the partners.


Cash .... ... ...........................................................................................
R. Magno, Capital ........................................................................
(P35,000 P24,700 = P10,300)

10,300

J. Lagman, Capital ..............................................................................


Cash.. ...........................................................................................
Accounts Payable to J. Lagman ...................................................
(P63,000 + P7,790 = P70,790 P35,000 = P35,790)

35,790

10,300

23,300
12,490

Partnership Basic Considerations and Formation

4.

17

Lagman and Magno


Balance Sheet
December 31, 2008
Assets
Cash .... ... ...........................................................................................
Accounts receivable............................................................................
Less Allowance for bad debts.............................................................
Merchandise inventory .......................................................................
Equipment...........................................................................................
Other assets.........................................................................................
Goodwill ...........................................................................................
Total Assets..................................................................................

P34,000
1,210

32,790
21,000
8,000
46,000
___8,000
P115,790

Liabilities and Capital


Accounts payable................................................................................
Notes payable......................................................................................
Accrued interest payable.....................................................................
Accounts payable to J. Lagman ..........................................................
J. Lagman, capital ...............................................................................
R. Magno, capital................................................................................
Total Liabilities and Capital.........................................................

P 18,000
15,000
300
12,490
35,000
__35,000
P115,790

Problem 1 6
1.

Books of Toledo
Toledo, Capital.............................................................................
Allowance for Bad Debts (15% x P32,000) ..........................

4,800

4,800

Books of Ureta
Ureta, Capital ...............................................................................
Allowance for Bad Debts (10% x P24,000) ..........................

2,400

Cash (90% x P12,000)..................................................................


Loss from Sale of Office Equipment............................................
Office Equipment...................................................................

10,800
1,200

Toledo, Capital (1/4 x P1,200) .....................................................


Ureta, Capital ...............................................................................
Loss from Sale of Office Equipment .....................................

300
900

2,400

12,000

1,200

18

2.

3.

Chapter 1

New Partnership Books


Cash.. ...........................................................................................
Accounts Receivable ....................................................................
Merchandise .................................................................................
Office Equipment .........................................................................
Allowance for Bad Debts.......................................................
Accounts Payable...................................................................
Notes Payable ........................................................................
Toledo, Capital ......................................................................
To record the investment of Toledo.

3,200
32,000
40,000
10,000

Cash.. ...........................................................................................
Accounts Receivable ....................................................................
Merchandise .................................................................................
Toledo, Capital.............................................................................
Allowable for Bad Debts .......................................................
Accounts Payable...................................................................
Ureta, Capital.........................................................................
To record the investment of Ureta.

22,800
24,000
36,000
300

Cash .... ... ...........................................................................................


Ureta, Capital ...............................................................................
To record Ureta's cash contribution.

3,400

Computation:
Toledo, capital (P68,400 P300).................................................
Divide by Toledo's profit share percentage..................................
Total agreed capital of the partnership .........................................
Multiply by Ureta's profit share percentage .................................
Agreed capital of Ureta ................................................................
Ureta, capital ................................................................................
Cash contribution of Ureta ...........................................................
or
Toledo, capital (P68,400 P300).................................................
Less Ureta, capital........................................................................
Cash contribution of Ureta ...........................................................

P 68,100
____50%
P136,200
____50%
P 68,100
__64,700
P 3,400
P 68,100
__64,700
P 3,400

4,800
10,000
2,000
68,400

2,400
16,000
64,700

3,400

Partnership Basic Considerations and Formation

4.

19

Toledo and Ureta Partnership


Balance Sheet
July 1, 2008
Assets
Cash .... ... ...........................................................................................
Accounts receivable............................................................................
Less Allowance for bad debts.............................................................
Merchandise........................................................................................
Office equipment ................................................................................
Total Assets..................................................................................

P56,000
__7,200

P 29,400
48,800
76,000
__10,000
P164,200

Liabilities and Capital


Accounts payable................................................................................
Notes payable......................................................................................
Toledo, capital ....................................................................................
Ureta, capital.......................................................................................
Total Liabilities and Capital.........................................................

P 26,000
2,000
68,100
__68,100
P164,200

20

Chapter 2

CHAPTER 2
MULTIPLE CHOICE ANSWERS AND SOLUTIONS

2-1: d

Jordan
P120,000
( 10,000)
P110,000

Annual salary
Balance, equally
Total
2-2: a

2-3: a

Bonus (.20 X P90,000)


Interest
JJ (.15 X P100,000)
KK (.15 X P200,000)
LL (.15 X P300,000)
Balance, equally
Total profit share

JJ
P18,000

KK

P15,000

P 30,000

( 6,000)
P27,000

( 6,000)
P 24,000

2-4: a

Allan

Interest
Allan - .10 X (P40,000 + 60,000 /2)
Michael - .10 X (P60,000 + 70,000/2)
Balance, equally
Total
2-5: a

Average Capital
Date
January 1
July 1
August 1

Capital
Balance
140,000
180,000
165,000

_14,000
P 19,000
Fred
P12,000
30,000
( 35,000)
P 7,000

Interest (.10 of average capital)


Salaries
Balance, equally
Total
2-6: b

P 5,000

Months
Unchanged
6
1
5
12

( 35,000)
( P29,000)

Peso
Months
P 840,000
180,000
__825,000
P1,845,000

Average capital - P1,845,000/12

P153,750

Interest

P 15,375

(P153,750 X 10%)

Greg
P 6,000

Pippen
Total
P80,000 P200,000
( 10,000) ( 20,000)
P 70,000 P180,000
LL

Total
P 18,000

)
)
P45,000)
90,000
( 6,000) ( 18,000)
P39,000 P 90,000

Michael

Total

)
P 6,500) P 11,500
_14,000 __28,000
P20,500 P 39,500
Henry
Total
P 4,000 P 22,000
20,000
50,000
( 35,000) (105,000)
(P11,000) (P 33,000)

Partnership Operations

2-7: c
Date
January 1
April 1
June 1
September 1

21

Capital
Balance
P16,000
17,600
19,200
15,200

Months
Unchanged
3
2
3
4
12

Average Capital(P201,600/12) =
2-8: a

P16,800

Net profit before bonus


Net profit after bonus (P24,000/120%)
Bonus to RJ
Balance (P24,000-P4,000)X3/5
Total profit share

2-9: a

LT
P3,200
15,000
(11,580)
P 6,620

P 24,000
__20,000
4,000
__12,000
P 16,000

AM
P 3,600
7,500
( 7,720)
P 3,380

Total
P 6,800
22,500
( 19,300)
P 10,000

Net income after salary, interest and bonus


Add back: Salary (P10,000 X 12)
P120,000
Interest (P250,000 X .05)
__12,500
Net income after bonus (80%)
Net income before bonus (P600,000/80%)
Paul's bonus

P467,500

Interest
Salaries
Balance, 3:2
Total
2-10: b

Peso
Months
P 48,000
35,200
57,600
__60,800
P201,600

2-11: b

_132,500
P600,000
_750,000
P150,000
CC

Salary
Balance
Additional profit to DD
Total

P14,000
( 1,500)
P12,500

Net income
Fees Earned
Expenses
Net Income

P90,000
_48,000
P42,000

DD
P 8,400
__2,100
P10,500

EE
Total
P 14,000 P 14,000
5,600
28,000
( 600) ______
P 19,000 P 42,000

22

Chapter 2

2-12: c
Interest
Annual Salary
Additional profit to give LL, P20,000
Additional profit to give MM, P14,000
Total
*(P9,500/50%) = P19,000

LL
P 2,000
8,500
9,500
_____
P20,000

MM
P 1,250

5,700
__7,050
P14,000

NN
P 750

3,800
_____
P 4,550

Total
P 4,000
8,500
19,000*
__7,050
P 38,550

RR

SS

TT

Total

P15,000

_47,500
P62,500

(P10,000)
_35,625
P25,625

)
)
_11,875
P11,875

P 5,000
__95,000
P100,000

BB

CC

Total

2-13: a
Excess (Deficiency)
RR (P80,000 - P95,000)
SS (P50,000 - P40,000)
Balance 4:3:1
Total
Net Income
2-14: b

2-15: a

2-16: c

(200,000 - 100,000) =

AA - 100,000 X 10%
150,000 X 20%
Remainder, 210,000
BB (60,000 X .05)
CC (60,000 X .05)
Balance, equally
Total

P100,000
AA
P 10,000
30,000

)
)

P 40,000

)
P 3,000
_68,000
P71,000

6,000
_204,000
P250,000

BJ

CJ

Total

P1,000

_6,800
P7,800

P4,000

12,000
__3,400
P19,400

P4,000
1,000
22,000
_17,000
P44,000

P 3,000
__68,000
P108,000

AJ
Bonus to CJ
Net profit before bonus
P44,000
Net profit after bonus (P44,000/110%)P40,000

Interest to BJ

Salaries
P 10,000
Balance, 4:4:2
__6,800
Total
P 16,800

_68,000
P71,000

Total profit share of Pedro


Less: Salary to Pedro
Interest
Share in the balance (40%)

P 50,000
__20,000

Net profit after salary and interest (130,000/40%)


Add: Total Salaries
Total Interest
Total Partnership Income

P150,000
__70,000

P200,000
__70,000
P130,000
P325,000
_220,000
P545,000

Partnership Operations

2-17: c

Net income before extraordinary gain and bonus (69,600-12,000)


Net income after bonus (57,600/120%)
Bonus to RR
Distribution of Net Income:

2-18: a

Bonus
Balance, equally
Net profit before extraordinary gain
Extraordinary gain
Total

P 24,000
P 24,000
__4,800
P 28,800

RR
P 9,600
24,000
P 33,600
__7,200
P 40,800

Interest
Annual Salary
Remainder 60:40
Total

Mel
P 20,000
36,000
__60,000
P116,000

Jay
P 12,000

_40,000
P 52,000

Interest on excess (Deficiency)


Remainder 5:3:2
Total

DV
P 15,000
( 36,875)
(P 21,875)

2-19: a

2-20: c

23

JJ

2-21: a
Salaries
Interest
Bonus (P360,000-P54,000)X.25
Remainder, 30:70
Total

Total
P 9,600
48,000
P 57,600
_12,000
P 69,600
Total
P 32,000
36,000
_100,000
P168,000

JE
FR
Total
P 3,750
(P 7,500) P 11,250
( 22,125) ( 14,750) ( 73,750)
(P 18,375) (P 22,250) (P 62,500)

Correction of 1998 profit:


Net income per books
Understatement of depreciation
Overstatement of inventory, December 31
Adjusted net income
Distribution of net income per book:
Equally
Distribution of adjusted net income
Equally
Required Decrease

P 57,600
_48,000
P 9,600

P 19,500
( 2,100)
( 11,400)
P 6,000
Pete

Rico

Total

P 9,750

P 9,750

P 19,500

( 3,000)
P 6,750

( 3,000) ( 6,000)
P 6,750 P 13,500

Tiger
P 64,000
24,000
76,500
__19,650
P184,150

Woods
P100,000
30,000

__45,850
P175,850

Total
P164,000
54,000
76,500
__65,500
P360,000

24

Chapter 2

2-22: a

Holly
P 20,000

32,000
30,000
__35,640
P117,640

Salaries
Commission
Interest
Bonus, schedule 1
Remainder, 60:40
Total

2-23: a

Field

P 25,000
33,600

_23,760
P 82,360

Schedule 1
Net income before salary, commission,
interest and bonus
Less: salaries
Net income before bonus
Net income after bonus (P180,000/120%)
Bonus

Total
P 20,000
25,000
65,600
30,000
__59,400
P200,000

P200,000
__20,000
P180,000
_150,000
P 30,000

Capital balance, beginning


Additional investment
Capital withdrawal
Capital balance before profit and loss distribution

Mike
P600,000
100,000
-200,000
P500,000

Tyson
Total
P400,000 P1,000,000
200,000
300,000
( 100,000) _-300,000
P500,000 P1,000,000

Net income:
Salary
Balance, 3:2
Total
Total
Drawings
Capital balance, end

P200,000
__60,000
P260,000
P760,000
( 200,000)
P560,000

P300,000 P 500,000
__40,000 __100,000
P340,000 P 600,000
P840,000 P1,600,000
( 300,000) ( 500,000)
P540,000 P1,100,000

Average Capital - King:


Date
January 1
April 1

Capital
Balance
P40,000
55,000

Average capital P615,000/12 = P51,250


Average Capital - Queen:
Date
January 1
April 1

Capital
Balance
P100,000
130,000

Average capital - P1,350,000 / 12 =P112,500

Months
Unchanged
3
9
12

Peso
Months
P120,000
_495,000
P615,000

Months
Unchanged
7
5
12

Peso
Months
P700,000
__650,000
P1,350,000

Partnership Operations

2-24: d

25

Distribution of Net Income - Schedule 1


Interest
Bonus, Schedule 2
Salaries
Residual, 50:50
Total

King
P 5,125
12,725
25,000
( 2,050)
P40,800

Queen
P11,250

30,000
_(2,050)
P39,200

Total
P16,375
12,725
55,000
_(4,100)
P80,000

Schedule 2
Net income before allocation
Less: Interest
Net income before bonus
Net income after bonus (P63,625/125%)
Bonus
Capital Balance December 31:
Capital balance, January 1
Additional investment
Capital balance before profit and
loss distribution
Net income (Schedule 2)
Drawings (P400 X 52)
Capital balance, December 31
2-25: d

P80,000
_16,375
P63,625
_50,900
P12,725
King
P40,000
_15,000

Queen
P100,000
__30,000

Total
P140,000
__45,000

P55,000
40,800
( 20,800)
P75,000

P130,000
39,000
( 20,800)
P148,400

P185,000
80,000
( 41,600)
P223,400

Total receipts (P1,500,000 + P1,625,000)


Expenses
Net income
Distribution to Partners
Red P1,500,000/P3,125,000 X P2,045,000 =
Blue P1,625,000/P3,125,000 X P2,045,000 =
Capital balance of Blue Dec. 31
Capital Balance, Jan. 1
Additional investment
Capital balance before profit and
loss distribution
Profit share
Drawings
Capital balance, Dec. 31

P3,125,000
( 1,080,000)
P2,045,000
P 981,600 (1)
_1,063,400
P2,045,000
P 374,000
___22,000
P 396,000
1,063,400
( 750,000)
P 709,400 (2)

26

Chapter 2

2-26: a
Ray
P150,000
_______

Sam
P180,000
__60,000

Total
P330,000
__60,000

150,000

240,000

390,000

15,000
51,000

20,000
34,000

35,000
85,000

66,000

54,000

120,000

Total
Salaries

216,000
_18,000

294,000
_24,000

510,000
_42,000

Total
Drawings

234,000
(18,000)

318,000
(24,000)

552,000
(42,000)

Capital balances, March 1


Additional investment, Nov. 1
Capital balances before salaries, profit and Drawings
Profit share:
Interest
Balance, 60:40
Total

Capital balances, Feb. 28

P216,000

P294,000

Capital balances, 1/1


Additional investment, 4/1
Capital withdrawals, 7/1

Susan
P150,000
8,000
_______

Tanny
P30,000

158,000

24,000

23,400
(1,725)

4,050
6,000
(1,725)

27,450
6,000
(3,450)

21,675

_8,325

30,000

179,675
(12,000)

32,325
(12,000)

212,000
(24,000)

2-27: a

Balances before profit distribution


Profit distribution:
Interest
Bonus (20% x P30,000)
Balance, equally
Total
Total
Drawings
Capital balances, 12/31

P167,675

P510,000

Total
P180,000
8,000
(6,000) _(6,000)

P20,325

182,000

P188,000

Partnership Operations
2-28: a
Capital balances, beg. 1st year
Loss distribution, 1st year:
Salaries
Interest
Balance, 5:3:2
Total
Total
Drawings
Capital balances, beg. 2nd year
Profit distribution, 2nd year:
Salaries
Interest
Balance, 5:3:2
Total
Total
Drawings
Capital balances, end of 2nd year
2-29: c
Capital balances, 1/1/06
Additional investment, 2006
Capital withdrawal, 2006
Capital balances
Profit distribution, 2006:
Interest
Salary
Balance, equally
Capital balances, 1/1/07
Additional investment, 2007
Capital withdrawal, 2002
Capital balances
Profit distribution, 2007:
Interest
Salary
Balance, equally
Capital balances, 1/1/08
Additional investment, 2008
Capital withdrawal, 2008
Capital balances
Profit distribution, 2008:
Interest
Salary
Balance, equally
Capital balances, 12/31/08 per books
Understatement of depreciation
Adjusted capital balances, 12/31/08

27
Sin
P110,000

Tan
P80,000

Uy
P110,000

Total
P300,000

20,000
11,000
(40,000)
( 9,000)
101,000
(10,000)
91,000

8,000
(16,000)
( 8,000)
72,000
(10,000)
62,000

10,000
11,000
(24,000)
( 3,000)
107,000
(10,000)
97,000

30,000
30,000
(80,000)
(20,000)
280,000
(30,000)
250,000

20,000
9,100
( 7,500)
21,600
112,600
_(10,000)
P102,600

6,200
( 4,500)
_1,700
63,700
(10,000)
P53,700

10,000
9,700
( 3,000)
16,700
113,700
_(10,000)
P103,700

30,000
25,000
(15,000)
40,000
290,000
_(30,000)
P260,000

Jay
P30,000

Kay
P30,000

_(5,000)
25,000

_(4,000)
26,000

Loi
P30,000
5,000
______
35,000

Total
P90,000
5,000
_(9,000)
86,000

3,000
7,000
_1,000
36,000
5,000
______
41,000

3,000

3,000

_1,000
30,000

_1,000
39,000

_(3,000)
27,000

_(8,000)
31,000

9,000
7,000
__3,000
105,000
5,000
(11,000)
99,000

3,600
7,000
_1,500
53,100

3,000

3,900

_1,500
31,500

______
53,100

_(4,000)
27,500

_1,500
36,400
6,000
_(2,000)
40,400

5,310
7,000
__3,300
P68,710
(2,000)
P66,710

3,150

3,640

__3,300
P33,950
(2,000)
P31,950

__3,300
P47,340
(2,000)
P45,340

10,500
7,000
__4,500
121,000
6,000
_(6,000)
121,000
12,100
7,000
___9,900
P150,000
(6,000)
P144,000

28

Chapter 2

2-30: a
Ken
Capital balances, 1/1/07
Additional investment, 2007
Capital withdrawal, 2007
Balances
Profit distribution, 2007 (Schedule 1)
Salary
Balance, beg. Capital ratio
Capital balances, 1/1/08
Capital withdrawal, 2008

P100,000

Len

P100,000

( 20,000)

_______

P300,000
40,000
( 20,000)

80,000

140,000

100,000

320,000

20,000

20,000

60,000
20,000

60,000
60,000

100,000
( 20,000)

160,000
( 40,000)

180,000
_______

440,000
( 60,000)

Balances
Profit distribution, 2008:
Salary
Balance, beg. capital ratio

80,000

120,000

180,000

380,000

__13,636

__21,818

60,000
__24,546

60,000
__60,000

Capital balances, 12/31/08

P 93,636

P141,818

P264,546

P500,000

Total profit for 2 years


Net profit per year (P240,000 / 2)

2-32: d

Total

P100,000
40,000
_______

Schedule 1 Computation of net profit:


Total capital, 2008 (P647,500 P147,500)
Total capital, 2007 (P300,000 + P40,000 P80,000)

2-31: d

Mon

_Nardo_
Capital balance, 1/1/08
P280,000
Additional investment
96,000
Withdrawals
Cap. bal. before P/L dist.
376,000
NP: Salary (16,500 x 12)
Interest on EC (15%)
42,000
Balance 25:30:45
( 19,875 )
Total
22,125
Capital balance 12/31/08 P398,125
Sam capital, beginning
Additional investment (Land)
Drawings
Capital balance before net profit (loss)
Capital balance, end
Profit share (40%)
Net profit (P50,000 40%)

P500,000
_260,000
P240,000

P120,000
__Orly
P300,000
60,000
( 90,000 )
270,000
198,000
45,000
( 23,850 )
219,150
P 489,150

__Pedro_
P170,000
( 72,000 )
98,000
25,500
( 35,775 )
( 10,275 )
P 87,72
P120,000
60,000
( 80,000 )
100,000
150,000
50,000
P125,000

_Total_
P750,000
156,000
(162,000)
744,000
198,000
112,500
(79,500 )
231,000
P975,000

Partnership Operations

2-33: a

__Joe__
Capital balance, 1/2/07
P 80,000
Net loss- 2007:
Annual salary
96,000
10% interest on beg. capital
8,000
Bal. beg. cap. ratio: 8:4
( 108,000)
Total
( 4,000)
Capital balance
76,000
Drawings
( 4,000)
Capital balance, 12/31/07
72,000
Net profit- 2008:
Annual salary
96,000
10% interest on BC
7,200
Bonus to JoeNPBB
P 22000
NPAB (22000/110%)20000 2,000
Balance equally
( 67,300)
Total
37,900
Total
109,900
Drawings
(
4,000)
Capital balance, 12/31/08

2-34: a

29

Decrease in capital
Drawings
Contribution
Profit share
Net income (45,000 30)

105,900

__Tom__
P 40,000

__Total__
P120,000

48,000
4,000
( 54,000)
( 2,000)
38,000
( 4,000)
34,000

144,000
12,000
( 162,000)
( 6,000)
114,000
( 8,000)
106,000

48,000
3,400

144,000
10,600

( 67,300)
( 15,900)
18,100
(
4,000)

2,000
( 134,600)
22,000
128,000
( 8,000)

14,100

120,000
P 60,000
( 130,000)
25,000
45,000
P150,000

30

Chapter 2

SOLUTIONS TO PROBLEMS
Problem 2 1

1.

Castro
Diaz

:
:

(P26,000/P42,500) x
(P16,500/P42,500) x

P23,800
P23,800

=
=

P14,560
__9,240
P23,800

2.

Castro
Diaz

:
:

(P31,250/P50,000) x
(P18,750/P50,000) x

P23,800
P23,800

=
=

P14,875
__8,925
P23,800

Computation of Average Capitals:


Castro:
Date
1/1 .....................................
4/10 ...................................
5/1 .....................................
8/1 .....................................

Capital
Balances
P26,000
29,000
36,000
32,000

Average capital = P375,000 12 months =


Diaz:

Date
1/1 .....................................
6/1 .....................................
9/1 .....................................

Capital
Balances
P16,500
21,500
19,500

Average capital = P225,000 12 months =


3.

Months
Unchanged
3
1
3
5
12

Peso
Months
P 78,000
29,000
108,000
_160,000
P375,000

P31,250
Months
Unchanged
5
3
4
12

Peso
Months
P 82,500
64,500
__78,000
P225,000

P18,750

Interest ........................................................
Salaries........................................................
Balance, equally..........................................
Total............................................................

Castro
P 7,500
36,000
( 24,100)
P19,400

Diaz
P4,500
24,000
(24,100)
P 4,400

Total
P12,000
60,000
( 48,200)
P23,800

Bonus (a) ....................................................


Interest (b)...................................................
Balance, 3:2 ................................................
Total............................................................

Castro
P 4,760
1,100
_10,764
P16,624

Diaz
P

_7,176
P7,176

Total
P 4,760
1,100
_17,940
P23,800

4.

Partnership Operations

31

Computations:
a. Net profit before bonus.................................................
Net profit after bonus (P23,800 125%).....................
Bonus............................................................................
b.

5.

Castro
Diaz

P23,800
_19,040
P 4,760

Average capital of Castro [(P26,000 + P32,000) 2] ...........................


Average of Diaz [(P16,500 + P18,500) 2]....... ..................................
Castro's excess..................................................... ..................................
Multiply by.......................................................... ..................................
Interest................................................................. ..................................

P29,000
_18,000
P11,000
___10%
P 1,100

:
:

P14,280
__9,520
P23,800

(P3,000/P5,000) x P23,800
(P2,000/P5,000) x P23,800

=
=

Problem 2 2
a.

Average Capital:
Robin:
Date
Jan. 1
Feb. 28
Apr. 30
Sept. 30

Balances
P135,000
95,000
175,000
195,000

Months
Unchanged
2
2
5
3
12

Peso
Months
P270,000
190,000
875,000
__585,000
P1,920,000

Months
Unchanged
3
3
2
2
2
12

Peso
Months
P420,000
600,000
300,000
440,000
__400,000
P2,160,000

Ave. Capital (P1,920,000 12) = P160,000


Hood:

Date

Balances

Jan. 1
Mar. 31
June 30
Aug. 31
Oct. 31

P140,000
200,000
150,000
220,000
200,000

Ave. Capital (P2,160,000 12) = P180,000


Profit Distribution:
Robin : P160,000 P340,000 x P510,000 =
Hood : P180,000 P340,000 x P510,000 =

P240,000
_270,000
P510,000

32

Chapter 2

b.

Robin
P 14,400
60,000
78,850
_119,775
P274,025

Hood
P 16,200
100,000

_119,775
P235,975

Total
P 30,600
160,000
79,850
_239,550
P510,000

Robin

Hood

Totals

P 6,000
249,000
255,000

P 12,000
498,000
510,000

Robin
Hood
Salaries.................................................................
P 80,000
P120,000
Bonus (see computations below) .........................
62,000
Balance, equally...................................................
_124,000
_124,000
Totals ...................................................................
P266,000
P244,000
Bonus Computations:
Net income before salaries and bonus......... ..................... .......................
Less Salaries................................................ ..................... .......................
Net income before bonus ............................ ..................... .......................
Net income after bonus (P310,000 125%)..................... .......................
Bonus .......................................................... ..................... .......................

Total
P200,000
62,000
_248,000
P510,000

Interest on ave. capital .........................................


Salaries.................................................................
Bonus (P510,000 30,600 160,000) x 25%)....
Balance, equally...................................................
Totals ...................................................................
c.

d.

Interest:
Robin (P195,000 P135,000) 10%.............
Hood (P200,000 P140,000) 10% .............
Balance, equally...................................................
Totals ...................................................................

P 6,000
249,000
255,000

P510,000
200,000
310,000
_248,000
P 62,000

Problem 2 3
a.

De Villa
P 30,000

De Vera

P 20,000
31,200
9,818
__44,182
P105,200

Salaries.................................................................
Commission (2% x P1,000,000)..........................
Interest of 8% on average capital.........................
32,800
Bonus (see computations below) .........................
9,818
Balance, equally...................................................
__44,182
Total.....................................................................
P116,800
Bonus Computations:
Income before salary, commissions, interest & bonus...... .......................
Salary and commission (P30,000 + P20,000) ................... .......................
Interest......................................................... ..................... .......................
Income before bonus................................... ..................... .......................
Income after bonus (P108,000 110%) ..... ..................... .......................
Bonus .......................................................... ..................... .......................
b.

Income Summary.................................................
De Villa, capital ..........................................
De Vera, capital...........................................

P 222,000

Total
P 30,000
20,000
64,000
19,636
__88,364
P222,000
P222,000
( 50,000)
( 64,000)
108,000
_98,182
P 9,818
116,800
105,200

Partnership Operations

33

Problem 2 4
a.
Salaries................................................
Bonus (see computation below)..........
Interest (see computation below)........
Balance, 3:3:4 .....................................
Total....................................................

East
P15,000
3,760
2,800
__3,180
P24,740

North
P20,000

West
P18,000

4,000
__3,180
P27,180

4,800
__4,240
P27,040

Bonus computations:
Net income before bonus ........... .................... ..................... .....................
Net income after bonus (P78,960 105%)..... ..................... .....................
Bonus ......................................... .................... ..................... .....................
Interest computations:
East (10% x P28,000)................. .................... ..................... .....................
North (10% x P40,000) .............. .................... ..................... .....................
West (10% x P48,000) ............... .................... ..................... .....................
Total ........................................... .................... ..................... .....................
b.
Interest (see computations below) ......
Salaries................................................
Bonus (see computations below) ........
Balance, equally..................................
Total....................................................
Interest computations:
Average capitals:
East:
Date
1/1
5/1
9/1

Balances
P30,000
36,000
28,000

East
P 3,133
24,000
( 6,056)
P 21,077

North
P 3,633
21,000
4,280
( 6,055)
P 22,858

West
P 5,200
25,000
( 6,055)
P 24,145

Months
Unchanged
4
4
4
12

Average capital (P376,000 12) ..........................................


North:

Date
1/1
3/1
7/1
9/1

Balances
P40,000
31,000
36,000
40,000

Months
Unchanged
2
4
2
4
12

Average capital (P436,000 12)...........................................

Total
P53,000
3,760
11,600
_10,600
P78,960
P78,960
_75,200
P 3,760
P 2,800
4,000
__4,800
P11,600
Total
P11,966
70,000
4,280
( 18,166)
P 68,080

Pesos
Months
P120,000
144,000
_112,000
P376,000
P 31,333
Pesos
Months
P80,000
124,000
72,000
_160,000
P436,000
P 36,333

34

Chapter 2

West:

Date
1/1
4/1
6/1
8/1

Months
Unchanged
3
2
2
5
12

Balances
P50,000
57,000
60,000
48,000

Pesos
Months
P150,000
114,000
120,000
_240,000
P624,000

Ave. capital (P624,000 12)....................................

P 52,000

Interest Computations:
East (10% x P31,333)............ ...............................................
North (10% x P36,333) ......... ...............................................
West (10% x P52,000)........... ...............................................
Total ... .................................. ...............................................

P 3,133
3,633
__5,200
P 11,966

Bonus Computations:
Net income ............................ ...............................................
Less Salary ............................ ...............................................
Net income before bonus....... ...............................................
Net income after bonus (P47,080 110%) ...........................
Bonus to North ...................... ...............................................
* To Total
c.

East

West
P 8,990

5,000
__8,237.50
P22,227.50

Total
P 8,990
39,000
12,000
_32,950
P92,940

Bonus Computations:
Net income before salaries & bonus ............... ..................... .....................
Less Salaries (P21,000 + P18,000) ................. ..................... .....................
Net income before bonus ........... .................... ..................... .....................
Net income after bonus (P53,940 120%)..... ..................... .....................
Bonus to West ............................ .................... ..................... .....................

P92,940
_39,000
P53,940
_44,950
P 8,990

Bonus (see comp. below)....................


Salaries ...........................................
Interest on beginning capital...............
Remainder, 8:7:5.................................
Total........ ...........................................

North

P 68,000
_21,000
47,080
_42,800
P 4,280

P21,000 P 18,000
3,000
4,000
_13,180 _11,532.50
P37,180 P33,532.50

Problem 2 5
a.

Schedule of Income Distribution:


Salaries.... ...........................................
Interest (see computation on p. 30).....
Balance, equally..................................
Total........ ...........................................

Maria
P12,000
7,200
__3,133
P22,333

Clara
P10,000
9,600
__3,133
P22,733

Rita
P 8,000
13,800
__3,134
P24,934

Total
P30,000
30,600
__9,410
P70,000

Partnership Operations

35

Interest on Average Capital:


Maria:
P80,000 x 8% x 6 months.. ....................
P100,000 x 5% x 6 months ....................
Clara:
P120,000 x 8% .................. ....................
Rita:
P180,000 x 8% x 9 Mos. ... ....................
P150,000 x 8% x 3 Mos. ... ....................
Total ........................................... ....................
b.

Statement of Partners Capital:


Balances, Jan. 1...................................
Additional Investment ........................
Capital Withdrawal.............................
Net Income..........................................
Drawings ...........................................
Balance, Dec. 31 .................................

Maria
P 80,000
20,000

22,333
( 10,000)
P112,333

P 3,200
__4,000

P 7,200
9,600

P10,800
__3,000

_13,800
P30,600

Clara
P120,000

22,733
( 10,000)
P132,733

Rita
P180,000

( 30,000)
24,934
( 10,000)
P164,934

Total
P380,000
20,000
( 30,000)
70,000
( 30,000)
P410,000

Benny

Celia

Total
P20,000

Problem 2 6
1.

Allocation of net loss for 2008:


Salary to Alvin....................................
Interests on average capital:
Alvin (P120,000 x 10%) ............
Benny (P200,000 x 10%) ...........
Celia (P220,000 x 10%) .............
Balance, 30:30:40 ...............................
Total........ ...........................................

2.

Statement of Partnership Capital


Year Ended December 31, 2008
Capitals, January 1, 2008....................
Additional investments .......................
Capital withdrawals ............................
Balances.. ...........................................
Net loss (see above) ............................
Balances.. ...........................................
Drawings. ...........................................
Capitals, December 31, 2008..............

Alvin
P 20,000
12,000
(29,400)
P 2,600

Alvin
P120,000
_______
120,000
__2,600
122,600
_(16,000)
P106,600

20,000
_(29,400)
P( 9,400)

22,000
_(39,200)
P(17,200)

54,000
_(98,000)
P(24,000)

Benny
P180,000
60,000
________
240,000
__(9,400)
230,600
_______
P230,600

Celia
P220,000
40,000
_(20,000)
240,000
_(17,200)
222,800
_______
P222,800

Total
P520,000
100,000
_(20,000)
600,000
_(24,000)
576,000
_(16,000)
P560,000

36

3.

Chapter 2

Correcting entry:
Celia capital ........................................
2,400
Alvin capital...............................
2,200
Benny capital .............................
200
To correct capital accounts for error in loss allocation computed as follows:
Alvin
Benny
Celia
Correct loss allocation ........................
P2,600
P(9,400) P(17,200)
Actual loss allocation..........................
__(400)
__9,600
__14,800
Adjustment..........................................
P2,200
P 200
P ( 2,400)
Problem 2 7
Dino
P45,000
_15,000
60,000
(1,800)
(17,000)
41,200
_____
41,200
10,800
(17,000)
35,000
______
35,000
56,365
(19,000)
P72,365

Nelson
P45,000
_15,000
60,000
( 1,800)
( 7,000)
51,200
_____
51,200
8,100
( 7,000)
52,300
______
52,300
42,272
( 9,000)
P86,572

Oscar
P45,000
__6,000
51,000
( 1,800)
( 3,200)
46,000
__6,000
52,000
8,100
( 3,200)
56,900
___6,000
62,900
20,363
( 3,200)
P80,063

Total
P135,000
__36,000
171,000
( 5,400)
( 27,200)
138,400
___6,000
144,400
27,000
( 27,200)
144,200
___6,000
150,200
120,000
( 31,200)
P239,000

Dino
P48,000

3,600
_* 4,765
P56,365

Nelson
P24,000
10,909
3,600
__4,763
P43,272

Oscar
P12,000

3,600
__4,763
P20,363

Total
P84,000
10,909
10,800
__14,291
P120,000

Bonus computations:
Net income before bonus ........... ................ ..................... .....................
Net income after bonus (P120,000 110%)..................... .....................
Bonus to Nelson......................... ................ ..................... .....................

P120,000
_109,091
P 10,909

Capital balances, 1/2/06...............................


Additional investment, 2006 .......................
Balances.......................................................
Net income (Loss) - 2006, equally ..............
Withdrawals, 2006.......................................
Capital balances, 12/31/06...........................
Additional investment, 2007 .......................
Balances.......................................................
Net income - 2007, 40: 30: 30 .....................
Withdrawals, 2007.......................................
Capital Balances, 12/31/07..........................
Additional investment, 2008 .......................
Balances.......................................................
Net income, 2008 (schedule 1)....................
Withdrawals, 2008.......................................
Capital balances, 12/31/08...........................
Schedule 1:
Annual salaries....................................
Bonus (see computations below) ........
Interest ................................................
Balance, equally..................................
Totals ..................................................

* To Total

Partnership Operations

37

Problem 2 8
Red, White & Blue Partnership
Statement of Partners' Capital
For Year Ended December 31, 2008

Balances, beginning of year


Add: 20% of fees billed to personal clients
Green's share of fees (Exhibit A)
Remaining net income (Exhibit A)
Subtotals
Less: Withdrawals
Uncollectible accounts identified
with clients of each partner
Excess rent charged to Blue
Total deductions
Balances, end of year

Red
40,200
8,800

White
20,200
4,800

Blue
40,600
4,400

_22,800
_71,800
10,400

_22,800
_47,800
8,800

_11,400
_56,400
11,600

2,400

900

P12,800
P59,000

P 9,700
P38,100

1,800
P13,400
P43,000

Green
3,200
______
__3,200
5,000

Total
P101,000
18,000
3,200
_57,000
179,200
35,800

P 5,000
P (1,800)

3,300
1,800
P 40,900
P138,300

Red, White & Blue Partnership


Exhibit A Computation and Division of Net income
For Year Ended December 31, 2008
Total revenue from fees
Expenses, excluding depreciation and doubtful
accounts expense
Less: Excess rent charged to N ($300 x 6)
Subtotal
Add: Depreciation, computed as follows:
$26,000 x 0.10
$10,000 x 0.10 x 1/2
Total expenses, excluding doubtful accounts expense
Add: Doubtful accounts expense ($3,000 x 0.60)
Total expenses
Net income for year ended Dec. 31, Year 1
Division of net income:
Fees billed to personal clients:
Red P44,000 x 20%
White P24,000 x 2%
Blue, P22,000 x 20%
Green's share of fees:
Gross fees from new clients after April 1, Year 1
Less: Allocated expenses ($40,000 x $24,000/
$120,000)
Net income from new clients
Green's share (P16,000 x 20%)
Total divided pursuant to special agreement
Balance, divided in income-sharing ratio as follows:
To Red, 40%
To White, 40%
To Blue, 20%
Total

P120,000
P38,700
__1,800
36,900
2,600
____500
P40,000
__1,800
41,800

P 8,800
48,000
4,400

________
P 78,200

P18,000

24,000
__8,000
P16,000

P 3,200
P22,800
22,800
_11,400
P57,000

__21,200
P 57,000

38

Chapter 2

Problem 2 9
Allan, Eman and Gino Partnership
Statement of Profit Distribution
Year Ended December 31, 2008
Allan

Eman

Gino

Total

Interest
Commission (P16,120 P5,000) x 10%
Balance, equally

P 4,000

__5,926

P 750
1,112
_5,925

P 250
1,112
_5,925

P 5,000
2,224
_17,776

Total
Adjustments (50% of P25,000 to Allan)

P 9,926
__2,574

P7,787
(1,287)

P7,287
(1,287)

P25,000
_____

Total

P12,500

P6,500

P6,000

P25,000

Sonny

Letty

Total

Problem 2 10
Gary, Sonny, and Letty Partnership
Statement of Partners' Capital Accounts
Year Ended December 31, 2008
Gary
Capital balances, 1/1/08
Additional investments

P210,000
___9,100

P180,000
_______

P 90,000
_______

P480,000
__9,100

Total
Profit distribution:
Salaries
Interest
Bonus to Gary and Sonny (Schedule 1)
Balance, equally

_219,100

_180,000

_90,000

489,100

10,640
10,800

13,680
25,920

__(9,720)

11,520
21,600

_(9,720)

_(9,720)

35,840
58,320

(29,160)

__29,880

_23,400

_11,720

_65,000

Total
Drawings

248,980
_(21,000)

203,400
101,720
554,100
(18,000) __(9,000) _(48,000)

Capital balances, 12/31/08

P227,980

Total

P185,400

Schedule 1: Computation of the bonus.


Net profit before interest, salaries and bonus
Less:Salaries
Interest
Net profit (loss) before bonus
Therefore no bonus is to be given to Gary and Sonny.

P35,840
_58,320

P 65,000
__94,160
P(29,160)

P 92,720

P506,100

Partnership Operations

39

Problem 2 11

a. Entries to record the formation of the partnership and the events that occurred during 2008:
Cash
Inventory
Land
Equipment
Mortgage payable
Installment note payable
Kobe, capital (P600,000 + P800,000
+ P1,000,000 P200,000)
Lebron, capital (P500,000 + P1,300,000
- P500,000)

1,100,000
800,000
1,300,000
1,000,000

2,200,000
1,300,000

(1)

Inventory
Cash
Accounts payable

(2)

Mortgage payable
Interest expense
Cash

50,000
20,000

Installment note payable


Interest expense
Cash

35,000
20,000

(3)

(4)

Accounts receivable
Cash
Sales

300,000

210,000
1,340,000

(5)

Selling and general expenses


Cash
Accrued expenses payable

340,000

(6)

Depreciation expense
Accumulated depreciation

60,000

(7)

Kobe, drawing
Lebron, drawing
Cash

(8)

Sales

(9)

Cost of goods sold


Inventory
P900,000 = P800,000 beginning inventory
+ 300,000 purchases
- 200,000 ending inventory

Income summary

500,000
200,000

104,000
104,000
1,550,000
900,000

240,000
60,000

70,000

55,000

1,550,000
278,000
62,000
60,000

208,000
1,550,000
900,000

40

Chapter 2

Income summary
Cost of good sold
Selling and general expenses
Depreciation expense
Interest expense

1,340,000

Income summary
Kobe, capital
Lebron, capital

210,000

Kobe, capital
Lebron, capital
Kobe, drawing
Lebron, drawing

104,000
104,000

900,000
340,000
60,000
40,000
105,000
105,000

104,000
104,000

Schedule to allocate partnership net income for 2008:


Kobe
Profit percentage
60%
Beginning capital balance
P2,200,000
Net income (P1,550,000 revenue
- P 1,340,000 expenses)
Interest on beginning capital
balances (3%)
66,000
Salaries
Residual deficit
Total
b.

Lebron
40%
P1,300,000

Total
100%
P3,500,000
210,000

39,000

120,000

120,000

(81,000)
P105,000

(54,000)
P105,000

(105,000)
P105,000
(240,000)
P(135,000)
(135,000)
-0-

Kobe-Lebron Partnership
Income Statement
For the Year Ended December 31, 2008
Sales
Less: Cost of goods sold:
Inventory, January 1
Purchases
Goods available for sale
Less: Inventory, December 31
Gross profit
Less: Selling and general expenses
Depreciation expenses
Operating income
Nonoperating expense- interest
Net income

P1,550,000
P800,000
300,000
P1,100,000
(200,000)
340,000
60,000

(900,000)
P650,000
400,000
P250,000
(40,000)
P210,000

Partnership Operations

41

Kobe-Lebron Partnership
Balance Sheet
At December 31, 2008

c.

Cash
Accounts receivable
Inventory
Land
Equipment (net)
Total assets
Liabilities:
Accounts payable
Accrued expenses payable
Installment note payable
Mortgage payable
Total liabilities
Capital:
Kobe, capital
Lebron, capital
Total capital
Total liabilities and capital

Assets

P1,589,000
210,000
200,000
1,300,000
940,000
P4,239,000

Liabilities and Capital


P60,000
62,000
165,000
450,000
P737,000
P2,201,000
1,301,000

3,502,000
P4239,000

42

Chapter 3

CHAPTER 3
MULTIPLE CHOICE ANSWERS AND SOLUTIONS
3-1: c

Implied capital of the partnership (P90,000/20%)


Actual value of the partnership
Goodwill

P450,000
( 420,000)
P 30,000

Capital balances before Goodwill


Goodwill to old partners
Total
Purchase by Hizon (20%)
Capital balances after admission

AQUINO
P252,000
__18,000
P270,000
( 54,000)
P216,000

LOCSIN
P126,000
___9,000
P135,000
( 27,000)
P108,000

DAVID
P42,000
__3,000
P45,000
( 9,000)
P36,000

Capital balances before admission


Purchase by Hizon (20%)
Capital balances after admission

AQUINO
P252,000
( 50,400)
P201,600

LOCSIN
P126,000
( 25,200)
P100,800

DAVID
HIZON
P42,000

( 8,400) _84,000
P33,600 P 84,000

Capital transferred
Excess divided using profit and loss ratio
Cash distribution

AQUINO
P 50,400
__3,600
P 54,000

LOCSIN
P 25,200
__1,800
P 27,000

DAVID
P 8,400
___600
P 9,000

3-2: b

3-3: d

HIZON

_____

_90,000
P 90,000

TOTAL
P 84,000
__6,000
P 90,000

3-4: b
Selling price
Interest sold (444,000X1/5)
Combine gain

P132,000
( 88,800)
P 43,200

3-5: b
Implied value of the partnership (P40,000/1/4)
Actual value
Goodwill
Cash balances
Goodwill, Profit and Loss ratio
Total
Capital Transfer (1/4)
Capital balances after admission

P160,000
( 140,000)
P 20,000
BERNAL
P 80,000
__12,000
P 92,000
( 23,000)
P 69,000

CUEVAS
DIAZ
P40,000 P 20,000
__6,000
__2,000
P46,000 P 22,000
( 11,500) ( 5,500)
P34,500 P 16,500

Partnership Dissolution Changes in Ownership

43

3-6: b
Capital Transfer (20%)
Excess, Profit and Loss ratio
Cash distribution

BANZON
P 16,000
__6,000
P 22,000

CORTEZ
P 4,000
__4,000
P 8,000

Capital balances beginning


Net profit, 1:2
Drawings
Capital balances before admission
Capital transfer (squeeze)
Capital balances after admission 1:2

PEREZ
P 24,000
5,430
( 5,050)
P 24,380
( 5,570)
P 18,810

CADIZ
TOTAL
P 48,000 P 72,000
10,860
16,290
( 8,000) ( 13,050)
P 50,860 P 75,240
( 13,240) (18,810) (1/4)
P 37,620 P 56,430

Capital transfer
Excess, 1:2
Cash

P 5,570
__3,730
P 9,300

P 13,240
__7,460
P 20,700

3-7: d

3-8: a
Total agreed capital (P150,000/5/6)
Diana's Interest
Cash distribution

P180,000
1/6
P 30,000

Total agreed capital (P36,000/1/5)


Total contributed capital (80,000+40,000+36,000)
Unrecognized Goodwill

P180,000
( 156,000)
P 24,000

3-9: a

3-10: b
Old partners
New partner
Total

Contributed
Agreed
Capital
Capital
P110,000
P100,000
__40,000
__50,000
P150,000
P150,000

Increase
(Dec.)
(P 10,000)
_10,000
P

Ben, capital balance before admission


Bonus share to new partner (10,000X60%)
Ben, capital after admission

P 60,000
( 6,000)
P 54,000

Total agreed capital (P40,000+20,000+17,000)


Pete's interest
Pete's agreed capital balance

P 77,000
1/5
P 15,400

3-11: c

TOTAL
P20,000
_10,000
P30,000

P18,810
_11,190
P30,000

44

Chapter 3

3-12: b
Old partner
New partner
Total

Contributed
Capital
P 65,000
25,000 (1/3)
P 90,000

Agreed
Capital
P60,000
30,000
P90,000

Increase
(Dec.)
(P 5,000)
_5,000
P

Capital balances before admission


Investment by Lory
Bonus to Lory
Capital balances after admission

FRED
P 35,000

(
3,500)
P 31,500

RAUL
P30,000

( 1,500)
P28,500

LORY

25,000
__5,000
P 30,000

3-13: c
Total agreed capital (90,000+60,000+70,000)
Augusts' interest
Agreed capital
Contributed capital
Bonus to June & July

P220,000
_____1/4
P 55,000
__70,000
P 15,000
JUNE
P90,000
__7,500
P97,500

Capital balances before admission


Bonus from August, equally
Capital balances after admission

JULY
P 60,000
__7,500
P 67,500

3-14: a
Total agreed capital (52,000 + 88,000)/80%)
Total capital of Mira & Nina after admission
Cash paid by Elma

P175,000
( 140,000)
P 35,000

Total agreed capital (P41,600/2/3)


Total contributed capital (P23,000+18,600+16,000)
Goodwill to new partner, Ang

P 62,400
( 57,600)
P 4,800

3-15: a

Capital balances before admission


Investment by Ang
Goodwill to August
Capital balances after admission

LIM
P23,000

_____
P23,000

ONG
P 18,600

______
P 18,600

ANG

16,000
__4,800
P20,800

Partnership Dissolution Changes in Ownership

45

3-16: a
Capital balances before
admission
Admission by Dong:
By Purchase (1/2)
By Investment
Capital balances before
Goodwill and Bonus
Goodwill to Old Partners (sch. 1)
Bonus to Old Partners (sch. 1)
Capital balances after
admission
Schedule 1:
Old Partners
New Partner
Total
3-17: b
Capital balances before
admission of Alma
Admission of Alma:
Investment
Goodwill to old partner,
70:30 (sch. 1)
Capital balances before
admission of Lorna
Admission of Lorna:
Goodwill Written off, 5:3:2
Investment
Goodwill to old partners,
5:3:2 (sch. 2)
Capital balances after
admission

ANG

BENG

CHING

DONG

TOTAL

P600,000

P 400,000

P 300,000

P1,300,000

( 300,000)
_______

_______

_______

300,000
_300,000

___300,000

P300,000
150,000
__37,500

P 400,000
150,000
__37,500

P 300,000
100,000
__25,000

P600,000

( 100,000)

P1,600,000
400,000
________

P487,500

P 587,500

P 425,000

P500,000

P2,000,000

CC
AC
P 1,000,000 P1,500,000
600,000 (25%) __500,000
P 1,600,000 P2,000,000

Inc. (Dec.)
P500,000
( 100,000) Bonus
P400,000 GW

MONA

LIZA

ALMA

LORNA

TOTAL

P150,000

P 50,000

P 200,000

80,000

80,000

__28,000

___12,000

_______

______

___40,000

P178,000

P 62,000

P 80,000

P 320,000

(P 20,000)

(P 12,000) (

P8,000)

75,000

( P40,000)
75,000

__10,000

____6,000

____4,000

______

___20,000

P168,000

P 56,000

P 76,000

P 75,000

P 375,000

Schedule 1:
Total agreed capital (80,000/25%)
Total capital contributed (200,000+80,000)
Goodwill to old partners, 70:30

P 320,000
( 280,000)
P 40,000

Schedule 2:
Total agreed capital (75,000/20%)
Total contributed capital (280,000+75,000)
Goodwill to old partners, 5:3:2

P 375,000
( 355,000)
P 20,000

46

Chapter 3

3-18: c
Unadjusted capital balances
Overvaluation of Marketable Securities
Allowance for Bad Debts
Adjusted capital balances before admission

RED
P175,000
( 12,500)
( 12,500)
P150,000

Total agreed capital (270,000/2/3)


Green's interest
Investment

P405,000
1/3
P135,000

3-19: b
Capital balances before
admission
Capital transfer
to WW (1/6)
Balances
Equalization of capital
Balances
Net profit, equally
Drawings (2 months)
Capital balances before
WWs Investment

3-20: a

WHITE
P100,000
( 7,500)
( 7,500)
P 85,000

BLUE
P 45,000
( 5,000)
( 5,000)
P 35,000

TOTAL
P320,000
( 25,000)
( 25,000)
P270,000

XX

YY

ZZ

WW

TOTAL

P360,000

P225,000

P135,000

P720,000

( 60,000)
P300,000
( 100,000)
P200,000
3,150
_( 1,500)

( 37,500)
P187,500
__12,500
P200,000
3,150
_( 2,000)

( 22,500)
P112,500
__87,500
P200,000
3,150
_( 1,500)

_120,000 ______
P120,000 P720,000
______ ______
P120,000 P720,000
3,150
12,600
_( 2,000) _( 7,000)

P201,650

P201,150

P201,650

P121,150

Total agreed capital (201,650+201,150+201,650)/2/3


WW's interest
Agreed capital of WW
Contributed capital (see above)
Cash to be invested

P725,600

P906,675
1/3
P302,225
_121,150
P181,075

Capital balances
Understatement of assets, P12,000
Balances before settlement to A

A
P 20,750
__3,000
P 23,750

Settlement to A
A's interest (23,750+5,000)
Partial Goodwill to A

P 30,250
_28,750
P 1,500

B
P 19,250
__3,000
P 22,250

Therefore:
1. Under partial Goodwill method the capital balances of B is P 22,250
2. Under Bonus method the capital balances of B would be:
B, capital balances before settlement to A
P 22,250
Bonus to A (1,500X25/75)
_( 500)
B, capital after retirement of A
P 21,750

C
P 45,000
__6,000
P 51,000

Partnership Dissolution Changes in Ownership

47

3-21: a
Capital balances
Net income, P140,000
Undervaluation of inventory, P20,000
Capital balances before settlement to Perez
Settlement to Perez
Bonus to Perez
Capital balances after retirement

Perez
P 100,000
70,000
___10,000
P 180,000
( 195,000)
___15,000
P

Capital balances
Settlement to Ely
Total Goodwill (P40,000/50%)P80,000
Capital balances after retirement of Ely

ELY
FLOR
GLOR
P 320,000 P 192,000 P 128,000
( 360,000)

__40,000 ___24,000 ___16,000


P
P 216,000 P 144,000

3-22: c

3-23: c
Capital balance 3/1/07
Net loss-2007:
Salary (10 months)
Interest (10 months)
Bal. beg. cap. ratio: 48:24
Total
Capital balance
Drawings
Capital balance, 12/31/07
Net profit- 2008:
Salary
Interest
Balance, equally
Total
Capital balance
Drawings
Capital balance 12/31/08

Reyes
Suarez
P 150,000 P 200,000
42,000
28,000
____6,000 ____4,000
P 198,000 P 232,000

_( 9,000) _( 6,000)
P 189,000 P 226,000

_Alma_
480,000

_Betty_
240,000

480,000
40,000
( 544,000)
( 24,000)
456,000
( 24,000)
432,000

240,000
20,000
( 272,000)
( 12,000)
228,000
( 24,000)
204,000

720,000
60,000
( 816,000)
( 36,000)
684,000
( 48,000)
636,000

576,000
43,200
( 397,800)
221,400
653,400
( 24,000)
629,400

288,000
20,400
( 397,800)
( 89,400)
114,600
( 24,000)
90,600

864,000
63,600
( 795,600)
132,000
768,000
( 48,000)
720,000

Total contributed capital (720,000 + 400,000)


Coras interest
Coras agreed capital
Coras contributed capital
Bonus to Cora, from Alma and Betty 4:2
Therefore entry (c) is correct.

_Total_
720,000

1,120,000
40%
448,000
400,000
48,000

48

Chapter 3

3-24: a
Capital balance, beg. 2007
2007 net profit (90,000 59,000):
Interest
Compensation
Balance, 4:6
Total
Balance
Withdrawal
Repairs (charge to Pete)
Capital balance, 12/31/07

_Pete_
P80,000
8,000
5,000
( 2,000)
11,000
91,000
( 8,000)
( 5,000)
78,000

1/1/08: Admission of Sammy


Total agreed capital (P117,000 +43,000)
Sammys interest
Sammys agreed capital
Sammys contributed capital
Bonus to Pete & Carlos, 4:6
Therefore entry (a) is correct.

_Carlos_
P30,000
3,000
20,000
( 3,000)
20,000
50,000
( 11,000)
39,000

_Total_
P110,000
11,000
25,000
( 5,000)
31,000
141,000
(19,000)
( 5,000)
117,000
P160,000
20%
32,000
43,000
11,000

Partnership Dissolution Changes in Ownership

49

SOLUTIONS TO PROBLEMS
(a)

Problem 3 1
1. Goodwill Method:
Total agreed capital (P75,000 25%) .....................................P300,000
Total contributed capital .......................................................... _275,000
Goodwill to old partners, P/L ratio ..........................................P 25,000
Entry
Goodwill............................................................................
Cash...................................................................................
Red, capital ...................................................................
White, capital ................................................................
Blue, capital ..................................................................
Green, capital ................................................................

25,000
75,000

5,000
10,000
10,000
75,000

2. Bonus Method:
Contributed capital of Green ....................................................P 75,000
Agreed capital of Green (P275,000 x 25%)............................... _68,750
Bonus to old partners, P/L ratio................................................P 6,250
Entry:
Cash...................................................................................
Green, capital ................................................................
Red, capital ...................................................................
White, capital ................................................................
Blue, capital ..................................................................

75,000

68,750
1,250
2,500
2,500

(b) 1. Implicit Goodwill Method:


Total Implied Capital (P75,000 25)......................................P300,000
Total existing capital................................................................ _200,000
Implied Goodwill to old partners.............................................P100,000
Entries:
Goodwill............................................................................
Red, capital ...................................................................
White, capital ................................................................
Blue, capital ..................................................................

100,000

Red, capital (25% x P80,000)............................................


White, capital (25% x p120,000).......................................
Blue, capital (25% x P100,000).........................................
Green, capital ................................................................

20,000
30,000
25,000

2. Red, capital (25% x P10,000).......................................................


White, capital (25% x P80,000) ...................................................
Blue, capital (25% x P60,000) .....................................................
Green, capital.........................................................................

15,000
20,000
15,000

20,000
40,000
40,000

75,000

50,000

50
a.

Chapter 3

Problem 3 2

(1) Bonus Method:


Contributed capital of Tomas ......................................................... ..................
Agreed capital of Tomas (P640,000 x 20%)................................... ..................

P140,000
_128,000

Bonus to old partners, P/L ratio ...................................................... ..................

P 12,000

BRUNO

MARIO

TOMAS

TOTAL

Balances before admission ....................


Admission of Tomas..............................

P200,000
___9,000

P300,000
___3,000

_128,000

P500,000
_140,000

Balances after admission .......................

P209,000

P303,000

P128,000

P640,000

(2) Goodwill Method:


Total agreed capital (P140,000 20%) . .................. .....................
Total contributed capital ........................ .................. .....................

P700,000
_640,000

Goodwill to old partners, P/L ratio ........ .................. .....................

P 60,000

BRUNO

MARIO

TOMAS

TOTAL

Balances before admission ....................


Admission of Tomas..............................

P200,000
__45,000

P300,000
__15,000

_140,000

P500,000
_200,000

Balances after admission .......................

P245,000

P315,000

P140,000

P700,000

(3) Goodwill with subsequent write-off.

BRUNO

MARIO

TOMAS

TOTAL

Balances from A-2.................................


Goodwill written off, 6:2:2 ....................

P245,000
( 36,000)

P315,000
( 12,000)

P140,000
( 12,000)

P700,000
( 60,000)

Balances.................................................

P209,000

P303,000

P128,000

P640,000

BRUNO

MARIO

TOMAS

TOTAL

Balances from A-2.................................


Goodwill written off, 4:4:2 ....................

P245,000
( 24,000)

P315,000
( 24,000)

P140,000
( 12,000)

P700,000
( 60,000)

Balances.................................................

P221,000

P291,000

P128,000

P640,000

b.

Problem 3 3
a.

b.

Total capital after admission (P76,000 + P104,000) ..................................... ..................


Total capital before admission (P60,000 + P80,000) .................................... ..................
Goodwill recorded ........................................................................................ ..................

P180,000
_140,000
P 40,000

Total capital of the partnership (P180,000 75%) ....................................... ..................


Less: Total capital of old partners plus Goodwill (P140,000 + 40,000) ....... ..................
Cash payment by Barry................................................................................. ..................

P240,000
_180,000
P 60,000

Total capital after admission (P52,000 + P68,000) ....................................... ..................


Total capital before admission ...................................................................... ..................
Bonus to Barry .............................................................................................. ..................

P120,000
_140,000
P 20,000

Agreed capital of Barry (P120,000 75%) x 25% ....................................... ..................


Less: Bonus .............................................................................................. ..................
Cash payment by Barry................................................................................. ..................

P 40,000
__20,000
P 20,000

Partnership Dissolution Changes in Ownership

51

Problem 3 4
a.

Total agreed capital (P60,000 20%) ..................................................P300,000


Total contributed capital (P100,000 + P40,000 + P60,000) ................. _200,000
Goodwill to old partners, P/L ratio .......................................................P100,000
Entry:
Cash.. .... ......................................................................................
Goodwill ......................................................................................
Gene, capital ..........................................................................
Nancy, capital ........................................................................
Ellen, capital ..........................................................................

b.

Cash ..... .. .... ......................................................................................


Ellen, capital.................................................................................

60,000
100,000

60,000

80,000
20,000
60,000
60,000

No Goodwill, no bonus because the total agreed capital is equal to the total contributed
capital.
c.

Gene, capital ......................................................................................


Nancy, capital .....................................................................................
Ellen, capital.................................................................................

20,000

d.

Cash .... ... .... ......................................................................................


Ellen, capital.................................................................................

32,000

8,000
28,000
32,000

Since the total agreed capital (P172,000) is equal to the total contributed capital (P172,000),
then no Goodwill or bonus is to be recorded.
e.

Total agreed capital (P140,000 80%) ................................................P175,000


Total contributed capital (P140,000 + P32,000)................................... _172,000
Goodwill to new partner .......................................................................P 3,000
Entry:
Cash.. .... ......................................................................................
Goodwill ......................................................................................
Ellen, capital ..........................................................................

32,000
3,000

35,000

Problem 3 5
a.

Cash ..... .. .... ......................................................................................


Cherry capital ...............................................................................

b.

Total agreed capital (P120,000 + P50,000) ..........................................P170,000


Cherry's interest ....................................................................................____25%
Cherry's agreed capital.............................................................................. 42,500
Contributed capital................................................................................ __50,000
Bonus to old partners, 70:30.................................................................P 7,500

40,000

40,000

52

Chapter 3

Entry:
Cash.. .... ......................................................................................
Cherry, capital .......................................................................
Helen, capital .........................................................................
Cathy, capital .........................................................................
c.

25,000
7,875
3,375

36,250

Total agreed capital (P50,000 25%) ..................................................P200,000


Total contributed capital (P120,000 + 50,000) ....................................... 170,000
Goodwill to old partners, 70:30 ............................................................P 30,000
Entry:
Cash
......................................................................................
Goodwill ......................................................................................
Cherry, capital .......................................................................
Helen, capital .........................................................................
Cathy, capital .........................................................................

e.

42,500
5,250
2,250

Total agreed capital (P120,000 + P25,000) ..........................................P145,000


Cherry's interest ....................................................................................____25%
Agreed capital of Cherry .......................................................................... 36,250
Contributed capital................................................................................ __25,000
Bonus to new partner ............................................................................P 11,250
Entry:
Cash.. .... ......................................................................................
Helen, capital................................................................................
Cathy, capital................................................................................
Cherry, capital .......................................................................

d.

50,000

50,000
30,000

50,000
21,000
9,000

Total agreed capital (P120,000 75%) ................................................P160,000


Total contributed capital (P120,000 + P25,000)................................... _145,000
Goodwill to new partner .......................................................................P 15,000
Entry:
Cash
......................................................................................
Goodwill ......................................................................................
Cherry, capital .......................................................................

25,000
15,000

40,000

Problem 3 6
a.

Total agreed capital (P600,000 3/4) .................................................................


Santos interest......................................................................................................
Contribution of Santos.........................................................................................

P800,000
_____1/4
P200,000

b.

Total agreed capital (P630,000 3/4) .................................................................


Santos' interest .....................................................................................................
Contribution of Santos.........................................................................................

P840,000
_____1/4
P210,000

Partnership Dissolution Changes in Ownership


c.

d.

e.

a.

b.

c.

53

Total agreed capital (P624,000 3/4) ....................................................................... ....................


Less: Contributed capital of old partners ................................................................... ....................

P832,000
_600,000

Contributed capital of Santos .................................................................................... ....................

P232,000

Total agreed capital (P600,000 3/4) ....................................................................... ....................


Less: Goodwill ........................................................................................................ ....................

P800,000
__10,000

Contributed capital .................................................................................................... ....................


Contributed capital of old partners ............................................................................ ....................

790,000
_600,000

Contributed capital of Santos .................................................................................... ....................

P190,000

Total agreed capital (Contributed)............................................................................. ....................


Less: Contributed capital of old partners ................................................................... ....................

P820,000
_600,000

Contributed capital of Santos .................................................................................... ....................

P220,000

Problem 3 7

Tony, capital
........................................................................................................
Noel, capital ......................................................................................................

40,000

Cash

........................................................................................................
Noel, capital ......................................................................................................
(P180,000 2/3) x 1/3 = P90,000.

90,000

Cash....... ..... .... ........................................................................................................


Goodwill ..... .... ........................................................................................................
Noel, capital ......................................................................................................

56,000
4,000

40,000

90,000

60,000

Total agreed capital (P180,000 3/4) ....................................................................... ..... P240,000


Total contributed capital (P180,000 + P56,000)........................................................ ..... _236,000
Goodwill to new partner............................................................................................ ..... P
d.

Subas, capital
.....
Tony, capital
..
Inventory.............

4,000
14,400
9,600

Cash....... ..... .... ........................................................................................................


52,000
Noel, capital ......................................................................................................
Total agreed capital (P52,000 1/4) ......................................................................... ..... P208,000
Total capital before inventory write-down (180,000 + 52,000)................................. .....(232,000)
e.

Write-down to old partners capital ............................................................................ .....( 24,000)


Land.. 92,000
Subas, capital
Tony, capital.
Subas, capital (P155,200 x 1/4) .................................................................................
38,800
Tony, capital (P116,800 x 1/4) ..................................................................................
29,200
Noel, capital ......................................................................................................
Total resulting capital (P68,000 1/4) ...................................................................... ..... P272,000
Total capital of old partner (net assets)...................................................................... ..... _180,000
Increase in value of land............................................................................................ ..... P 92,000
Capital of old partner after revaluation of land:
Subas (P100,000 + P55,200) ............................................................................. ..... P155,200
Tony (P80,000 + P36,800) ................................................................................ ....... 116,800

24,000

52,000

55,200
36,800

68,000

54

f.

Chapter 3

Cash .... ... .... ......................................................................................


Subas, capital ......................................................................................
Tony, capital ......................................................................................
Noel, capital .................................................................................

40,000
2,400
1,600

44,000

Agreed capital of Noel (P220,000 x 1/5)...............................................P 44,000


Contributed capital of Noel .................................................................... _40,000
Bonus to Noel ........................................................................................P 4,000
g.

Cash .... ... .... ......................................................................................


Goodwill . .... ......................................................................................
Noel, capital .................................................................................
Subas, capital (P60,000 x 3/5)......................................................
Tony, capital (P60,000 x 2/5).......................................................

P60,000
60,000

P 60,000
36,000
24,000

Total agreed capital (P60,000 1/5) ....................................................P300,000


Total contributed capital (P180,000 + P60,000)................................... _240,000
Goodwill to old partner, 3:2..................................................................P 60,000
Problem 3 8
a.

b.

c.

Conny, capital .....................................................................................


Andy, capital (P8,000 x 3/4)...............................................................
Benny, capital (P8,000 x 1/4) .............................................................
Cash.. .... ......................................................................................

40,000
6,000
2,000

Goodwill . .... ......................................................................................


Conny, capital .....................................................................................
Cash.. .... ......................................................................................

10,000
40,000

Goodwill (P5,000 1/5) .....................................................................


Conny, capital .....................................................................................
Andy, capital (P25,000 x 3/5) ......................................................
Benny, capital (P25,000 x 1/5).....................................................
Cash
......................................................................................

25,000
40,000

48,000

50,000

15,000
5,000
45,000

Problem 3 9
a.

Spade, capital......................................................................................
Jack, capital ..................................................................................

120,000

b.

Goodwill (P30,000 50%).................................................................


Ace, capital...................................................................................
Jack, capital ..................................................................................
Spade, capital ...............................................................................

60,000

Spade, capital (P120,000 + P30,000)..................................................


Jack, capital ..................................................................................

150,000

120,000
12,000
18,000
30,000
150,000

Partnership Dissolution Changes in Ownership

55

Problem 3-9 (Continued)

c.

d.

e.

f.

g.

Spade, capital......................................................................................
Cash.. .... ......................................................................................

180,000

Ace, capital (P60,000 x 2/5) ...............................................................


Jack, capital (P60,000 x 3/5)...............................................................
Spade, capital ...............................................................................

24,000
36,000

Land .... ... .... ......................................................................................


Ace, capital (20%)........................................................................
Jack, capital (30%) .......................................................................
Spade, capital (50%) ....................................................................

20,000

Spade, capital......................................................................................
Ace, capital (P50,000 x .40) ...............................................................
Jack, capital (P50,000 x .60)...............................................................
Cash.. .... ......................................................................................
Land.. .... ......................................................................................

130,000
20,000
30,000

Goodwill . .... ......................................................................................


Spade, capital......................................................................................
Cash.. .... ......................................................................................

30,000
120,000

Goodwill (P30,000 50%).................................................................


Spade, capital......................................................................................
Ace, capital (P60,000 x 20%).......................................................
Jack, capital (P60,000 x 30%)......................................................
Cash.. .... ......................................................................................

60,000
120,000

Land .... ... .... ......................................................................................


Ace, capital (20%)........................................................................
Jack, capital (30%) .......................................................................
Spade, capital (50%) ....................................................................

P40,000

Spade, capital (P120,000 x P20,000)..................................................


Ace, capital (P10,000 x 40%) .............................................................
Jack, capital (P10,000 x 60%) ............................................................
Land.. .... ......................................................................................
Note payable.................................................................................

140,000
4,000
6,000

180,000

60,000
4,000
6,000
10,000

60,000
120,000

150,000

12,000
18,000
150,000
8,000
12,000
20,000

100,000
50,000

56

Chapter 3

Problem 3 10
Case 1: Bonus of P10,000 to Eddy:
Eddy, capital.................................................................................
Charly, capital (P10,000 x 3/5) ....................................................
Danny, capital (P10,000 x 2/5).....................................................
Cash ......................................................................................

70,000
6,000
4,000

Case 2: Partial Goodwill to Eddy:


Goodwill ......................................................................................
Eddy, capital.................................................................................
Cash ......................................................................................

4,000
70,000

Case 3: Bonus of P5,000 to remaining partner:


Eddy, capital.................................................................................
Charly, capital (P5,000 x 3/5)................................................
Danny, capital (P5,000 x 2/5) ................................................
Cash ......................................................................................
Case 4: Total Implied Goodwill of P24,000:
Goodwill ......................................................................................
Eddy, capital.................................................................................
Charly, capital (P24,000 x 3/6)..............................................
Danny, capital (P24,000 x 2/6) ..............................................
Cash ......................................................................................
Case 5: Other assets disbursed:
Eddy, capital.................................................................................
Other assets ..................................................................................
Charly, capital (P60,000 x 3/6)..............................................
Danny, capital (P60,000 x 2/6) ..............................................
Cash ......................................................................................
Case 6: Danny purchases Eddy's capital interest:
Eddy, capital.................................................................................
Danny, capital........................................................................

70,000

24,000
70,000

70,000
20,000

70,000

80,000

74,000

3,000
2,000
65,000

12,000
8,000
74,000

30,000
20,000
40,000

70,000

Partnership Dissolution Changes in Ownership

57

Problem 3 11
a. 1/1/06

Building ...............................................................
Equipment............................................................
Cash ....................................................................
Santos capital ..............................................
To record initial investment.

52,000
16,000
12,000

12/31/06 Reyes capital........................................................


Santos capital ..............................................
Income summary.........................................
To record distribution of loss as follows:

22,000

1/1/07

40,000

12,000
10,000

Interest .................................................................
Additional profit ..................................................
Balance to Reyes..................................................

Santos
P 8,000
4,000
______

Reyes
P

(22,000)

Total
P 8,000
4,000
(22,000)

Total ....................................................................

P12,000

P(22,000)

(P10,000)

Cash ....................................................................
Santos capital (15%) ............................................
Reyes capital (85%).............................................
Cruz capital .................................................

15,000
300
1,700

17,000

(new investment by Cruz brings total capital to P85,000 after 2006 loss [80,000
10,000 + 15,000]. Cruz's 20% interest is P17,000 [85,000 x 20%] with the extra
P2,000 coming from the two original partners [allocated between them according
to their profit and loss ratio].)
12/31/07 Santos capital.......................................................
Reyes capital........................................................
Cruz capital..........................................................
Santos drawings ..........................................
Reyes drawings ...........................................
Cruz drawings .............................................

10,340
5,000
5,000

10,340
5,000
5,000

To close drawings accounts for the year based on distributing 20%. Of each
partner's beginning capital balances [after adjustment for Cruz's investment] or
P5,000 whichever is greater. Santos's capital Is P51,700 [40,000 + 12,000 300].)
12/31/07 Income summary .................................................
Santos capital ..............................................
Reyes capital ...............................................
Cruz capital .................................................
To allocate P44,000 income figure as computed below:
Santos
Interest (20% of P51,700).................................... P10,340
15% of P44,000 income.......................................
6,600
Balance, 60:40 .....................................................
______
Total ....................................................................

P16,940

44,000

16,940
16,236
10,824

Reyes

Cruz

P16,236

P10,824

P16,236

P10,824

58

Chapter 3

Capital balances as of December 31, 2008

1/1/08

Initial investment, 2007 .......................................


2007 profit ...........................................................
Cruz investment ...................................................
2007 drawings......................................................
2007 profit ...........................................................

Santos
P40,000
12,000
(300)
(10,340)
_16,940

Reyes
P40,000
(22,000)
(1,700)
(5,000)
_16,236

P17,000
(5,000)
_10,824

Capital, 12/31/07 .................................................

P58,300

P27,536

P22,824

Cruz capital..........................................................
Diaz capital .................................................
To transfer capital purchase from Cruz to Diaz

22,824

Cruz

22,824

12/31/08 Santos capital.......................................................


11,660
Reyes capital........................................................
5,507
Diaz capital ..........................................................
5,000
Santos drawings ..........................................
11,660
Reyes drawings ...........................................
5,507
Diaz drawings .............................................
5,000
To close drawings accounts based on 20% of beginning capital Balances (above) or
P5,0000 (whichever is greater).
12/31/08 Income summary .................................................
Santos capital ..............................................
Reyes capital ...............................................
Diaz capital .................................................
To distribute profit for 2008 computed as follows:

1/1/09

b. 1/1/06

61,000

20,810
24,114
16,076

Interest (20% of P58,300)....................................


15% of P61,000 profit..........................................
Balance, P40,190, 60:40 ......................................

Santos
P11,660
9,150
______

Reyes

Diaz

P24,114

P16,076

Total ....................................................................

P20,810

P24,114

P16,076

Diaz capital ..........................................................


33,900
Santos capital (15%) ............................................
509
Reyes capital (85%).............................................
2,881
Cash.............................................................
37,290
Diaz capital is [33,900 (P22,824 P5,000 + P16,076)]. Extra 10% is deducted
from the two remaining partners' capital accounts.
Building ...............................................................
Equipment............................................................
Cash ....................................................................
Goodwill ..............................................................
Santos capital ..............................................
Reyes capital ...............................................
To record initial investments. Reyes is credited with goodwill of
Santos investment.

52,000
16,000
12,000
80,000

80,000
80,000
P80,000 to match

Partnership Dissolution Changes in Ownership

59

12/31/06 Reyes capital ..............................................................


30,000
Santos capital ..............................................
20,000
Income summary.........................................
10,000
Interest of P16,000 is credited to Santos (P80,000 x 20%) along with a base of
P4,000. The remaining profit is now a P30,000 loss which is attributed entirely to
Reyes.
1/1/07

Cash ....................................................................
15,000
Goodwill ..............................................................
22,500
Cruz capital .................................................
37,500
Cash and goodwill contributed by Cruz are recorded. Goodwill is Computed
algebraically as follows:
P15,000 + goodwill =
P15,000 + goodwill =
P15,000 + goodwill =
.80 goodwill
=
goodwill
=

20% (current capital + P15,000 + goodwill)


20% (P150,000 + P15,000 + goodwill)
P33,000 + .20 goodwill
P18,000
P22,500

12/31/07 Santos capital.......................................................


20,000
Reyes capital........................................................
10,000
Cruz capital..........................................................
7,500
Santos drawings ..........................................
Reyes drawings ...........................................
Cruz drawings .............................................
To close drawings accounts based on 20% of beginning capital
Balances: Santos, p100,000; Reyes, P50,000; and Cruz, P37,500.
12/31/07 Income summary .................................................
Santos capital ..............................................
Reyes capital ...............................................
Cruz capital .................................................
To allocate P44,000 profit as follows:

44,000

20,000
10,000
7,500

26,600
10,400
6,960

Interest (20% of P100,000)..................................


15% of P44,000 profit..........................................
Balance of P17,400, 60:40...................................

Santos
P20,000
6,600
______

Reyes

Cruz

P10,440

P 6,960

Total ....................................................................

P26,600

P10,440

P 6,960

Capital balances as of December 31, 2004:

Santos
Initial investment, 2006 ....................................... P80,000
2006 profit allocation...........................................
20,000
Additional investment..........................................
2007 drawings...................................................... (20,000)
2007profit allocation............................................ __26,600

Reyes
P80,000
(30,000)

Cruz

(10,000)
_10,440

P37,500
(7,500)
__6,960

Capitals, 12/31/07................................................ P106,600

P50,440

P36,960

60

Chapter 3
1/1/08

Goodwill ......................................................................
26,588
Santos capital .....................................................
3,988
Reyes capital ......................................................
13,560
Cruz capital ........................................................
9,040
To record goodwill implied of Cruz's interest. In effect, the profit Sharing ratio is 15% to
Santos, 51% to Reyes (60% of 85% remaining after Santos's income), and 34% to Cruz
(40% of the 85% remaining after Santos' income). Diaz is paying P46,000, P9,040 in excess
of Cruz's capital (P36,960). The additional payment for this 34% income Interest indicates
total goodwill of P26,588 (P9,040/34%).

1/1/08

Cruz capital..................................................................
Diaz capital ........................................................
To transfer of capital purchase.

46,000

12/31/08 Santos capital ...............................................................


Reyes capital................................................................
Diaz capital ..................................................................
Santos drawings .................................................
Reyes drawings ..................................................
Diaz drawings ....................................................
To close drawings accounts based on 20% of beginning capitals.

22,118
12,800
9,200

12/31/08 Income summary .........................................................


Santos capital .....................................................
Reyes capital ......................................................
Diaz capital ........................................................
To allocate profit for 2008 as follows:

61,000

31,268
12,800
9,200

Santos
P22,118
9,150
______

Reyes

Diaz

P17,839

P11,893

Totals ...........................................................................

P31,268

P17,839

P11,893

Santos
P106,600
3,988

Reyes
P50,440
13,560

Diaz

(22,118)
__31,268

(12,800)
_17,839

P46,000
(9,200)
_11,893

P119,738

P69,039

P48,693

12/31/07 balances ........................................................


Goodwill ......................................................................
Capital purchased ........................................................
Drawings......................................................................
Profit allocation ...........................................................
12/31/08 balances ........................................................

1/1/09

22,118
12,800
9,200

Interest (20% of P110,588)..........................................


15% of P61,000 ...........................................................
Balance of P29,732, 60:40...........................................

Capital balances as of December 31, 2008:

1/1/09

46,000

Goodwill ......................................................................
14,321
Santos capital .....................................................
2,148
Reyes capital ......................................................
7,304
Diaz capital ........................................................
4,869
To record implied goodwill. Diaz will be paid P53,562 (110% of the capital balance for his
interest. This amount is P4,869 in excess of the capital account. Since Diaz is only entitled
to a 34% share of profits and losses, the additional P4,869 must indicate that the partnership
as a whole is undervalued by P14,321 (P4,869/34%) which is treated as goodwill.
Diaz capital ..................................................................
53,562
Cash....................................................................
53,562
To record settlement to Diaz.

Partnership Dissolution Changes in Ownership

61

Problem 3 12
Partnership Books Continued as Books of Corporation
Entries in the Books of the Corporation
(1) Inventories ..... .................................................................... .................
Land ...... ........ .................................................................... .................
Building. ........ .................................................................... .................
Accumulated depreciation bldg. ...................................... .................
Accumulated depreciation equipment.............................. .................
Equipment .................................................................. .................
Jack capital ................................................................ .................
Jill capital................................................................... .................
Jun capital.................................................................. .................
To adjust assets and liabilities of the partnership
to their current fair values.

26,000
40,000
20,000
20,000
30,000

(2) Cash ...... ........ .................................................................... .................


Jack capital .... .................................................................... .................
Jill capital................................................................... .................
Jun capital.................................................................. .................
To adjust capital accounts of the partners to 4:3:3 ratio.

4,000
18,000

(3) Jack capital .... .................................................................... .................


Jill capital ...... .................................................................... .................
Jun capital...... .................................................................... .................
Capital stock............................................................... .................
To record issuance of stock to the partners.

100,000
75,000
75,000

20,000
58,000
34,800
23,200

20,200
1,800

250,000

New Books Opened for the New Corporation


Entries in the Books of the Partnership
(1) Inventories ..... .................................................................... .................
Land ...... ........ .................................................................... .................
Building. ........ .................................................................... .................
Accumulated depreciation bldg. ...................................... .................
Accumulated depreciation equipment.............................. .................
Equipment .................................................................. .................
Jack capital ................................................................ .................
Jill capital................................................................... .................
Jun capital.................................................................. .................
To adjust assets and liabilities of the partnership.

26,000
40,000
20,000
20,000
30,000

(2) Cash ...... ........ .................................................................... .................


Jack capital .... .................................................................... .................
Jill capital................................................................... .................
Jun capital.................................................................. .................
To adjust capital accounts of the partners.

4,000
18,000

20,000
58,000
34,800
23,200

20,200
1,800

62

Chapter 3

(3) Stock of JJJ Corporation .................................................... .................


Accounts payable ................................................................ .................
Loans payable Jill ............................................................ .................
Cash in bank............................................................... .................
Accounts payable ....................................................... .................
Inventories.................................................................. .................
Land....... .................................................................... .................
Building. .................................................................... .................
Equipment .................................................................. .................
To record transfer of assets and liabilities to
The corporation and the receipt of capital stock

250,000
30,000
40,000

(4) Jack capital .... .................................................................... .................


Jill capital ...... .................................................................... .................
Jun capital...... .................................................................... .................
Stock of JJJ Corporation............................................ .................
To record issuance of stock to the partners.

100,000
75,000
75,000

44,000
26,000
60,000
60,000
70,000
60,000

250,000

Entries in the Books of the Corporation


(1) To record the acquisition of assets and liabilities from the partnership:
Cash in bank .. .................................................................... .................
Accounts receivable ............................................................ .................
Inventories ..... .................................................................... .................
Land ...... ........ .................................................................... .................
Building (net) . .................................................................... .................
Equipment (net)................................................................... .................
Accounts payable ....................................................... .................
Loans payable ............................................................ .................
Capital stock............................................................... .................

a. 1/1/06

12/31/06

44,000
26,000
60,000
60,000
70,000
60,000

30,000
40,000
250,000

Problem 3 13
Building
1,040,000
Equipment
320,000
Cash
240,000
Lim, capital
800,000
Sy, capital
800,000
(To record initial investment. Assets recorded at market value with two equal
capital balances.
Sy, capital
440,000
Lim, capital
240,000
Income summary
200,000
(The allocation plan specifies that Lim will receive 20% in interest [or 160,000
based on P800,000 capital balance] plus P80,000 more [since that amount is

Partnership Dissolution Changes in Ownership

63

greater than 15% of the profits from the period]. The remaining P440,000 loss is
assigned to Sy.)
1/1/07

Cash
300,000
Lim, capital (15%)
6,000
Sy, capital (85%)
34,000
Tan, capital
340,000
(New investment by Tan brings total capital to P1,700,000 after 2006 loss
[P1,600,000 P200,000 + P300,000]. Tans 20% interest is P340,000
[P1,700,000 x 20%] with the extra P40,000 coming from the two original
partners [allocated between them according to their profit and loss ratio].)

12/31/07

Lim, capital
206,800
Sy, capital
100,000
Tan, capital
100,000
Lim, drawings
206,800
Sy, drawings
100,000
Tan, drawings
100,000
(To close out drawings accounts for the year based on distributing 20% of each
partners beginning capital balances [after adjustment for Tans investment] or
P100,000 whichever is greater. Lims capital is P1,034,000 [P800,000 +
P240,000 P6,000])

12/31/07

Income summary
880,000
Lim, capital
Sy, capital
Tan, capital
(To allocate P880,000 income figure for 2007 as determined below.)

Interest (20% of P1,034,000


beginning capital balance)
15% of P880,000 income
60:40 split of remaining P541,200 income
Total
Capital balances as of December 31, 2007:
Initial 2006 investment
2006 profit allocation
Tans investment
2007 drawings
2007 profit allocation
12/31/07 balances
1/1/08

338,800
324,720
216,480

Lim

Sy

Tan

P206,800
132,000
P338,800

324,720
P524,720

216,480
P216,480

Lim
P800,000
240,000
(6,000)
(206,800)
338,800
P1,166,000

Sy
P800,000
440,000
(34,000)
(100,000)
324,720
P550,720

Tan, capital
456,480
Ang, capital
(To reclassify balance to reflect acquisition of Tans interest.)

Tan
P340,000
(100,000)
216,480
P456,480
456,480

64

Chapter 3

12/31/08

Lim, capital
233,200
Sy, capital
110,140
Ang, capital
100,000
Lim, drawings
233,200
Sy, drawings
110,140
Ang, drawings
100,000
(To close out drawings accounts for the year based on 20% of beginning capital
balances [above] or P100,000 [whichever is greater].)

12/31/08

Income summary
1,220,000
Lim, capital
416,200
Sy, capital
482,280
Ang, capital
321,520
(To allocate profit for 2008 determined as follows)
Lim
Sy
Ang
Interest (20% of P1,166,000 beg. capital)
P233,200
15% of P1,220,000 income
183,000
60:40 split of remaining P803,800
482,280
321,520
Totals
P416,200
P482,280
P321,520

b.

1/1/09

Ang, capital
678,000
Lim, capital (15%)
10,180
Sy, capital 85%)
57,620
Cash
745,800
(Angs capital is P678,000 [P456,480 P100,000 + P321,520]. Extra 10%
payment is deducted from the two remaining partners capital accounts.)

1/1/06

Building
1,040,000
Equipment
320,000
Cash
240,000
Goodwill
1,600,000
Lim, capital
1,600,000
Sy, capital
1,600,000
(To record initial capital investments. Sy is credited with goodwill of P1,600,000
to match Lims investment.)

12/31/06

Sy, capital
600,000
Lim, capital
400,000
Income summary
200,000
(Interest of P320,000 is credited to Lim [P1,600,000 x 20%] along with a base of
P80,000. The remaining amount is now a P600,000 loss that is attributed entirely
to Sy.)

1/1/07

Cash
300,000
Goodwill
450,000
Tan, capital
750,000
(Cash and goodwill being contributed by Tan are recorded. Goodwill must be
calculated algebraically.)

Partnership Dissolution Changes in Ownership

65

P300,000 + Goodwill = 20% (Current capital + P300,000 + Goodwill)


P300,000 + Goodwill = 20% (P3,000,000 + P300,000 + Goodwill)
P300,000 + Goodwill = P660,000 + .2 Goodwill
.8 Goodwill = P360,000
Goodwill = P450,000
12/31/07

Lim, capital
400,000
Sy, capital
200,000
Tan, capital
150,000
Lim, drawings
400,000
Sy, drawings
200,000
Tan, drawings
150,000
(To close out drawings accounts for the year based on 20% of beginning capital
balances: Lim- P2,000,000, Sy- P100,000, and Tan- P750,000.)

12/31/07

Income summary
Lim, capital
Sy, capital
Tan, capital
(To allocate P880,000 income figure as follows)

Interest (20% of P2,000,000)


beginning capital balance)
15% of P880,000 income
60:40 split of remaining P348,000
Totals
Capital balances as of December 31, 2007:
Initial 2006 investment
2006 profit allocation
Additional investment
2007 drawings
2007 profit allocation
12/31/07 balances
1/1/08

Lim
P400,000
132,000
P532,000

880,000

Sy

532,000
208,800
139,200
Tan

P208,800
P208,800

P139,200
P139,200

Lim
P1,600,000
400,000

Sy
P1,600,000
(600,000)

Tan

(400,000)
532,000
P2,132,000

(200,000)
208,800
P1,008,800

Goodwill
531,760
Lim, capital (15%)
Sy, capital (51%)
Tan, capital (34%)
(To record goodwill indicated by purchase of Tans interest.)

P750,000
(150,000)
139,200
P739,200
79,760
271,200
180,800

In effect, profits are shared 15% to Lim, 51% to Sy (60% of the 85% remaining after Lims
income), and 34% to Tan (50% of the 85% remaining after Lims income). Ang is paying
P920,000, an amount P180,800 in excess of Tans capital (P739,200). The additional payment for
this 34% income interest indicates total goodwill of P531,760 (P180,800/34%). Since Tan is
entitled to 34% of the profits but only holds 19% of the total capital, an implied value for the

66

Chapter 3

company as a whole cannot be determined directly from the payment of P920,000. Thus,
goodwill can only be computed based on the excess payment.
1/1/08

Tan, capital
Ang, capital
(To reclassify capital balance to new partner.)

12/31/08

Lim, capital
442,360
Sy, capital
256,000
Ang, capital
184,000
Lim, drawings
442,360
Sy, drawings
256,000
Ang, drawings
184,000
(To close out drawings accounts for the year based on 20% of beginning capital
balances [after adjustment for goodwill].)

12/31/08

Income summary
Lim, capital
Sy, capital
Ang, capital

12/31/07 balances
Adjustment for goodwill
Drawings
Profit allocation
12/31/08 balances

920,000

1,220,000

To allocate profit for 2008 as follows:


Lim
Interest (20% of P2,211,760
beginning capital balance)
P442,360
15% of P1,220,000 income
183,000
60:40 split of remaining P594,640
Totals
P625,360
Capital balances as of December 31, 2008:

920,000

Lim
P2,132,000
79,760
(442,360)
625,360
P2,394,760

Sy

625,360
356,780
237,860
Ang

356,780
P356,780

237,860
P237,860

Sy
P1,008,00
271,200
( 256,000)
356,780
P1,380,780

Ang
P739,200
180,800
(184,000)
237,860
P973,860

Ang will be paid P1,071,240 (110% of the capital balance) for her interest. This amount is
P97,380 in excess of the capital account. Since Ang is only entitled to a 34% share of profits and
losses, the additional P97,380 must indicate that the partnership as a whole is undervalued by
P286,420 (P97,380/34%). Only in that circumstance would the extra payment to Ang be justified:
1/1/09

Goodwill
Lim, capital (15%)
Sy, capital (51%)
Ang, capital (34%)
(To recognize implied goodwill.)

286,420

42,960
146,080
97,380

Partnership Dissolution Changes in Ownership

1/1/09

Ang, capital
Cash
(To record final distribution to Ang.

67

1,071,240

1,071,240

68

Chapter 4

CHAPTER 4
MULTIPLE CHOICE ANSWERS AND SOLUTIONS
4-1: a
Capital balances before realization
Loss on liquidation, P40,000
Cash distribution

PAR
P 20,000
( 20,000)
P

BOOGIE
P 16,000
( 12,000)
P 4,000

BIRDIE
P 10,000
( 8,000)
P 2,000

Capital balances before liquidation


Gain of P10,000 (150,000-140,000)
Cash distribution

PING
P 50,000
__6,000
P 56,000

PANG
P 50,000
__2,000
P 52,000

PONG
P 10,000
__2,000
P 12,000

Capital balances before liquidation


Loss of P40,000 (P140,000-P100,000)
Cash distribution

PING
P 50,000
( 24,000)
P 26,000

PANG
P 50,000
( 8,000)
P 42,000

PONG
P 10,000
( 8,000)
P 2,000

Capital balances before liquidation


Loss of P70,000 (P140,000-P70,000)
Balances
Absorption of Pong's deficiency, 6:2
Cash distribution

PING
P 50,000
( 42,000)
P8,000
( 3,000)
P 5,000

PANG
P 50,000
( 14,000)
P 36,000
( 1,000)
P 35,000

PONG
P 10,000
( 14,000)
( 4,000)
__4,000

COLT
MARK
Capital balances before liquidation (net of loans)P290,000 P200,000
Loss of P130,000, 4:3:3
( 52,000)
( 39,000)
Cash distribution
P238,000
P161,000

CLOCK
P220,000
( 39,000)
P181,000

4-2: c

4-3: b

4-4: a

4-5: b

4-6: c
Capital balances before liquidation
Loss of P60,000, 40:50:10
Cash distribution

JONAS
P160,000
( 24,000)
P136,000

CARLOS
P 45,000
( 20,000)
P 25,000

TOMAS
P 55,000
( 6,000)
P 49,000

Partnership Liquidation

4-7: a
Capital balances before liquidation
Loss of P100,000, 4:3:3
Cash distribution
4-8: b
Capital balances before realization
Additional investment by Nory for
the unpaid liabilities (33,000-18,000)
Loss on realization (schedule 1)
Payment by Oscar to Nory
Schedule 1
Total capital before liquidation
Unpaid liabilities
Total loss on realization
4-9: d
Capital balances before liquidation (net)
Loss on realization (schedule 1) P27,500
Balances, cash distribution

69

ARIEL
P40,000
( 40,000)
P

BERT
P180,000
( 30,000)
P150,000

NORY
P23,000

OSCAR
P 13,500

15,000
( 30,900)
P 7,100

( 20,600)
( P7,100)
P 36,500
15,000
P 51,500

BLACK
P99,000
( 13,750)
P85,250

Schedule 1:
Capital balances of white (net)
Cash received by White
White's share of total loss (30%)

WHITE
P 91,500
( 27,500)
P 64,000

GREEN
P138,000
_( 5,500)
P132,500

P 91,500
_83,250
P 8,250

Total loss on realization (P8,250/39%)


4-10: c

CESAR
P 30,000
( 30,000)
P

P 27,500

Capital balances before liquidation (net)


Loss on realization, P63,600
Balances
Unrecorded liabilities, P500
Balances
Elimination of Nora's deficiency
Payment to partners

ANA
P27,000
( 25,320)
P 1,680
( 200)
P 1,480
( 1,380)
P 100

EVA
P 43,000
( 25,320)
P 17,680
( 200)
P 17,480
( 1,380)
P 16,100

NORA
P 10,000
( 12,660)
( 2,660)
( 100)
( 2,760)
__2,760
P

Capital balances before liquidation (net)


Loss on realization (schedule 1) P45,000
Payment to partners

ARIES
P33,500
( 22,500)
P11,000

LEO
P 49,000
( 13,500)
P 35,500

TAURUS
P 36,500
( 9,000)
P 27,500

4-11: d

70

Chapter 4

Schedule 1:

Taurus capital (net)


Payment to Taurus
Share of total loss (20%)

P36,500
( 27,500)
P 9,000

Total loss on realization (9,000/20%)

P45,000

4-12: c
Capital balances, June 11
Net loss from operation (squeeze)
Capital balances, August 30 before
liquidation (48,500-25,600)
Loss on realization (47,500-30,000)
Balances
Additional investment by Olga
Balances
Elimination of Olga's deficiency
Payment to partners

TOTAL
P32,700
( 9,800)

MONA
P15,000
( 4,200)

NORA
P13,500
( 2,800)

OLGA
P 4,200
( 2,800)

P22,900
( 17,500)
P 5,400
_1,500
P 6,900
______
P 6,900

P10,800
( 7,500)
P 3,300
_____
P 3,300
( 1,260)
P 2,040

P10,700
( 5,000)
P 5,700
_____
P 5,700
( 840)
P 4,860

P 1,400
( 5,000)
( 3,600)
_1,500
( 2,100)
_2,100
P

Capital balances before liquidation


Operating loss, P21,000
Drawings
Loans
Loss on realization, P12,000
Balances
Absorption of Tita's deficiency
Payment to Nora

RITA
P49,000
( 3,500)
( 10,000)

( 2,000)
P33,500
__1,500
P32,000

SARA
P18,000
( 7,000)
( 15,000)
8,000
( 4,000)
P

_____
P

TITA
P10,000
( 10,500)
( 20,000)
25,000
( 6,000)
( 1,500)
_1,500
P

CLARO
P45,000

PEDRO
P27,000

ANDRO
P50,000

( 24,000)
P21,000

( 24,000)
P 3,000

( 12,000)
P38,000

TOTAL
P47,500
( 38,500)
P 9,000

MONA
P28,500
( 23,100)
P 5,400

LISA
P19,000
( 15,400)
P 3,600

4-13: b

4-14: a
Capital balances before liquidation
Loss on realization
Accounts Receivable (P50,000 X 40%)
Investment (P30,000 - P20,000)
Equipment (P60,000-P30,000)
Total
Payment to partners

P20,000
10,000
_30,000
P60,000

4-15: c
Capital balances before liquidation (inclusive loans)
Loss on realization, (squeeze)
Capital balances - cash distribution

Partnership Liquidation

Cash after realization


Less Liabilities (P36,000-P7,500)
Total capital after realization

71

P 37,500
( 28,500)
P 9,000

4-16: a
FF capital before distribution of net loss
Add: share of net loss (P10,000 X 40%)
FF capital before liquidation
Cash settlement to FF
FF share of total loss on realization (40%)

P100,000
_( 4,000)
96,000
( 80,000)
P 16,000

Total loss on realization (P16,000/40%)

P 40,000

Total capital before liquidation (P260,000-P10,000)


Add: Liabilities
Total assets
Cash before liquidation
Non-cash assets
Loss on realization
Cash to be realized

P250,000
_100,000
P350,000
( 50,000)
P300,000
( 40,000)
P260,000

4-17: d
Capital balances before realization (net)
Loss on realization (squeeze)
Capital balances after realization
(liabilities-unpaid)
Elimination of CC's deficiency
Balances
Investment by DD
Payment to EE

TOTAL
P100,000
( 125,000)

CC
P 15,000
( 62,500)

DD
P22,500
( 37,500)

EE
P62,500
( 25,000)

(P 25,000)
_______
(P 25,000)
__43,500
P 18,500

( 47,500)
__47,500

______
P

( 15,000)
( 28,500)
(P43,500)
_43,500
P

P37,500
( 19,000)
P18,500
_____
P18,500

4-18: d
Total capital before liquidation
Liabilities
Total assets
Less: Cash balance before realization
Cash after payment of liabilities
payment of liabilities
Cash realized
Non-cash asset
Less: cash realized
Loss on realization

P 30,000
__1,500
P 31,500
P 11,100
1,500
( 11,600)

__1,000
P 30,500
_11,600
P 18,900

72

Chapter 4

4-19: d
Capital balances
Salary of LL (P600 X 8 months)
Capital balances before liquidation
Loss on realization
Balances
Additional investment by NN
Payment to partners

LL
P 50,000
__4,800
P 54,800
( 44,880)
P 9,920
______
P 9,920

MM
P 20,000
_______
P 20,000
( 14,960)
P 5,040
_____
P 5,040

NN
TOTAL
P 10,000 P 80,000
_______ ___4,800
P 10,000 P 84,800
( 14,960)
(P 4,960)
__4,960
P

4-20: b
KK's total interest (P60,000-P10,000)
Less: Cash to be paid to KK
Share of total loss (1/3)

P 50,000
__10,000
P 40,000

Total loss on realization (P40,000/1/3)

P120,000

Total assets:
Total interest of the partners before liquidation:
JJ (P70,000+P30,000+P10,000)
KK (P60,000-P10,000)
LL (P30,000+P10,000)
Divide by
Total
Loss on realization
Cash to be realized
4-21: a
Capital balances, July 1
Advances to NN, August 1
OO Loan, September 1
Interest, December 31 (6%)
NN (5 mos.)
OO (4 mos.)
Compensation to PP
Capital balances before liquidation
Loss on realization (squeeze)
Cash distribution

TOTAL
P 75,000
( 10,000)
20,000
(

250)
400
__2,500
P 87,650
_56,250
P 35,000

P110,000
50,000
__40,000

NN
P 25,000
( 10,000)

250)

_______
P 14,750
( 17,550)
( 2,800)

P200,000
______50%
P400,000
_120,000
P280,000
OO
P 25,000

20,000

PP
P 25,000

400
_______ ___2,500
P 45,400 P 27,500
( 17,550) ( 17,550)
P 27,850 P 9,950

NN should pay P2,800 and this is to be divided to OO & PP equally or P1,400 each.

Partnership Liquidation

73

4-22: a
Capital balances before realization
Loss on realization (squeeze)
Capital balances after realization
(unpaid liabilities)
Elimination of AS's deficiency
Cash to be absorbed

TOTAL
P 950,000
( 1,000,000)

PG
P350,000
__20,000

JR
AS
P250,000 P350,000
( 200,000) _500,000

(P 50,000)
_______
P

P 50,000
( 90,000)
(P 40,000)

P 50,000 ( 150,000)
( 60,000) P150,000
(P 10,000) P

RM
P500,000
( 490,000)
P 10,000

ST
P825,000
( 735,000)
P 90,000

TOTAL
P 27,500
__37,500
P 65,000

LT
P 20,000
_18,750
P 38,750

AM
P 5,000
__-9,375
P 14,375

AG
P 420,000
( 300,000)
P 120,000

BM
P375,000
( 300,000)
P 75,000

4-23: a
Capital balances before realization (net)
Loss on realization, P1,225,000
Payment to Partners
4-24: a
Capital balances before realization (net)
Gain on realization (squeeze)
Capital balances after realization
4-25: c
Capital balances before realization (net)
Loss on realization, P1,000,000
Balances
Additional investment by DJ
4-26: a

Settlement to Uy
Uy capital before liquidation (net):
Uy capital
Receivable from Uy
Loss of Uy (50%)
Total loss on realization (P70,000 50%)
CB before liquidation
Receivable from Uy
Loan to Wi
Salary payable to Vi
Interest before realization
Loss on realization
Settlement to partners

__Uy__
553,500
(132,000)
421,500
( 70,000)
351,500

CP
DJ
P205,000 P150,000
(200,000) (200,000)
P 5,000 P(50,000)
50,000
P351,500

P553,500
( 132,000)

421,500
P 70,000
P140,000

__Vi__
452,500
135,000
587,500
( 42,000)
545,500

__Wi__
486,000
( 40,500)
445,500
( 28,000)
417,500

ZP
P 2,500
__9,375
P 11,875

__Total__
1,492,000
(132,000)
(40,500)
135,000
1,454,500
( 140,000)
1,314,500

74

Chapter 4

SOLUTIONS TO PROBLEMS
Problem 4 1
Case 1
Rivas and Briones
Statement of Liquidation
December 31, 2008
Assets
Rivas,
Cash
Others Liabilities
Loan
Balances before liquidation ... P 20,000 P200,000 P132,000 P 18,000
Realization of assets and
distribution of loss .......... _134,000 ( 200,000) _______ _______
Balances................................. 154,000
132,000
18,000
Payment of liabilities............. ( 132,000) ______ ( 132,000) ______
Balances.................................
22,000

18,000
Offset Rivas' loan against his
capital deficiency ............ _______ _______ _______ ( 18,000)
Balances.................................
22,000

Additional loss to Briones ..... _______ _______ _______ _______


Balances.................................
22,000

Payment to partner................. P(22,000)

Partners' Capitals
Briones, Rivas Briones
Loan
(90%)
(10%)
P 20,000 P40,000 P10,000
_______ ( 59,400) ( 6,600)
20,000 ( 19,400) 3,400
_______ _______ ______
20,000 ( 19,400) 3,400
_______ _18,000 ______
20,000 ( 1,400) 3,400
_______ __1,400 ( 1,400)
20,000

2,000
P(20,000)
P(2,000)

Case 2
Rivas and Briones
Statement of Liquidation
December 31, 2008
Partners' Capitals
Assets
Rivas, Briones, Rivas Briones
Cash
Others Liabilities
Loan
Loan
(70%)
(30%)
P20,000 P200,000 P132,000 P 18,000 P 20,000 P40,000 P10,000

Balances before liquidation ...


Realization of assets and
distribution of loss .......... 134,000 ( 200,000) _______ ______ _______
Balances................................. 154,000
132,000
18,000
20,000
Payment of liabilities............. ( 132,000) _______ ( 132,000) ______ _______
Balances.................................
22,000

18,000
20,000
Offset loan against capital
deficiency........................ ________ _______ _______ ( 6,200) ( 9,800)
Balances.................................
22,000

11,800
10,200
Payment to partner................. P(22,000)

P(11,800) P(10,200)

( 46,200) ( 19,800)
( 6,200) 9,800
_______ ______
( 6,200) 9,800
__6,200 __9,800

Partnership Liquidation

75

Case 3
Rivas and Briones
Statement of Liquidation
December 31, 2008
Rivas,
Loan
P 18,000

Briones,
Loan
P20,000

Partners' Capitals
Rivas
Briones
(50%)
(50%)
P40,000 P10,000

( 200,000) _______ _______

132,000
18,000
_______ ( 132,000)

18,000

______
20,000
__
20,000

( 33,000) ( 33,000)
( 7,000) ( 23,000)
_
_______
( 7,000) ( 23,000)

Assets
Cash
Others Liabilities
P 20,000 P200,000 P132,000

Balances before liquidation ........


Realization of assets and
distribution of loss ............... _134,000
Balances .....................................
154,000
Payment of liabilities .................. ( 132,000)
Balances .....................................
22,000
Offset Briones'' loan against
his capital deficiency ........... _______
Balances .....................................
22,000
Additional loss to Rivas.............. _______
Balances .....................................
22,000
Payment to partner...................... P(22,000)

_______

_______

_______

_______

_______ ( 20,000)
18,000

_______ _______
18,000

P(18,000)

______ _20,000
7,000 ( 3,000)
( 3,000) __3,000
4,000

P( 4,000)

Journal Entries
Case 1:
Cash ..... .... ...................................................................................................
Rivas, Capital................................................................................................
Briones, Capital ............................................................................................
Other Assets...........................................................................................
Liabilities .. ...................................................................................................
Cash ... ...................................................................................................
Rivas, Loan ...................................................................................................
Rivas, Capital.........................................................................................
Briones, Capital ............................................................................................
Rivas, Capital.........................................................................................
Briones, Loan................................................................................................
Briones, Capital ............................................................................................
Cash ...................................................................................................
Case 2:
Cash ..... .... ...................................................................................................
Rivas, Capital................................................................................................
Briones, Capital ............................................................................................
Other Assets...........................................................................................
Liabilities .. ...................................................................................................
Cash ... ...................................................................................................
Rivas, Loan ...................................................................................................
Briones, Loan................................................................................................
Rivas, Capital.........................................................................................
Briones, Capital .....................................................................................
Rivas, Loan ...................................................................................................
Briones, Loan................................................................................................
Cash ... ...................................................................................................

134,000
59,400
6,600
132,000
18,000
1,400
20,000
2,000
134,000
46,200
19,800
132,000
6,200
9,800
11,800
10,200

200,000
132,000
18,000
1,400
22,000

200,000
132,000
6,200
9,800
22,000

76

Chapter 4

Case 3:
Cash .... ... ...........................................................................................
Rivas, Capital......................................................................................
Briones, Capital ..................................................................................
Other Assets .................................................................................
Liabilities ...........................................................................................
Cash.. ...........................................................................................
Briones, Loan......................................................................................
Briones, Capital............................................................................
Rivas, Capital......................................................................................
Briones, Capital............................................................................
Rivas, Loan.........................................................................................
Rivas, Capital......................................................................................
Cash.. ...........................................................................................

134,000
33,000
33,000
132,000
20,000
3,000
18,000
4,000

200,000
132,000
20,000
3,000
22,000

Problem 4 2
Blando and Castro
Statement of Liquidation
April 30, 2008
A s s e t s
Cash Receivables Inventory
Balances before
liquidation....................
Collection of
receivables and
distribution of loss .......
Balances............................
Realization of
inventory and
distribution of
loss...............................
Balances............................
Realization of other
assets and distribution
of loss ..........................

Others

Accounts
Payable

Blando,
Loan

Partners'
Blando
(60%)

Capitals
Castro
(40%)

P 18,000

P75,000

P90,000

P84,000

P42,000

P 24,000

P102,000

P99,000

_37,500

( 75,000)

_______ _______

_______

_______

( 22,500)

( 15,000)

84,000

42,000

24,000

79,500

84,000

( 90,000) _______

_______

_______

( 36,000)

( 24,000)

42,000

24,000

43,500

60,000

_______

_______

( 26,400)

( 17,600)

42,000

24,000

17,100

42,400

_______

_______

_______

55,500

_30,000

_______

85,500

_40,000

_______

Balances............................ 125,500
Payment of accounts
payable......................... ( 42,000)
Balances............................
83,500
Payments to partners.. P(83,500)

_______

90,000

84,000

_______ ( 84,000)

_______ _______

( 42,000)

24,000
17,100
P(24,000) P( 17,100)

42,400
P(42,400)

Partnership Liquidation
a.

77

Problem 4 3

Electric Company
Statement of Partnership Realization and Liquidation
June 30, 2008

Balances
Sale of
assets at a loss
Payment to
creditors

Capital Balances
Volt
Watt
30%
20%

Cash

Amp.
Loan

Noncash
Assets

Liabilities

Volt,
Loan

Amp
50%

20,000

15,000

135,000

30,000

10,000

80,000

36,000

14,000

_95,000
115,000

______
15,000

(135,000)
-0-

______
30,000

______
10,000

(20,000)
60,000

(12,000)
24,000

( 8,000)
6,000

_(30,000)
85,000

______
15,000

_______
-0-

(30,000)
-0-

______
10,000

_______
60,000

______
24,000

______
6,000

(24,000)
-0-

( 6,000)
-0-

Offset Amp,
receivable
(15,000)
Payments to partners:
Loan
(10,000)
Capitals
_(75,000) ______
_______
Balances
-0-0-0b. (1) Cash
Amp, Capital
Volt, Capital
Watt, Capital
Noncash Assets
Sell noncash assets at a loss of P40,000.

(15,000)
_______
-0-

(10,000)
______
-0-

(45,000)
-095,000
20,000
12,000
8,000

(2) Liabilities
Cash
Pay creditors.

30,000

(3) Amp, Capital


Amp, Loan
Offset receivable from Amp against his capital credit.

15,000

(4) Volt, Loan


Amp, Capital
Volt, Capital
Watt, Capital
Cash
Final lump-sum distribution to partners.

10,000
45,000
24,000
6,000

135,000

30,000

15,000

85,000

Note: All partners permitted Amp to offset his receivable against his capital credit. Alternatively, Amp
could be required to pay the partnership the P15,000 receivable; the partnership would then pay him an
additional P15,000 for his capital credit. In this case, an offset of the receivable against the capital credit is
reasonable, provided the receivable is not interest-bearing, Amp has a sufficient capital credit, Amp is
personally solvent, and the note is not secured against specific assts of Amp. The offset is not automatic,
but must be determined by the terms of the initial note, and by the partners.

78

Chapter 4

Problem 4 4
a.

b.

Bina, capital before liquidation ..................................................................... ..................


Payment to Bina............................................................................................ ..................

P320,000
_128,000

Loss absorbed by Bina (40%) ....................................................................... ..................

P192,000

Loss on realization (P192,000 40%).......................................................... ..................

P480,000

AIDA, BINA & CELIA


Statement of Partnership Liquidation
January 1, 2008

P720,000
( 720,000)

Aida
(5)
P320,000
( 240,000)

Capital
Bina
(4)
P320,000
( 192,000)

Celia
(1)
P160,000
( 48,000)

_______

80,000
( 80,000)

128,000
( 128,000)

112,000
( 112,000)

Cash

Other Assets

Balances before liquidation.


Realization & dist. of loss ...

P80,000
240,000

Balances .... .... ....................


Settlement to partners .........

320,000
(320,000)

Problem 4 5
a.

b.

LL, capital before liquidation........................................................................ ..................


Settlement to LL ........................................................................................... ..................

P 70,000
__98,000

Gain realized by LL (20%) ........................................................................... ..................

P 28,000

Total gain on realization (P28,000 20%) ................................................... ..................


Other assets sold ........................................................................................... ..................

P140,000
_500,000

Selling price

P640,000

.............................................................................................. ..................

JJ, KK & LL
Statement of Liquidation
Cash
Balances before liquidation...
Realization & Dist. of gain ...

P50,000
640,000

Balances .... .... ...................... 690,000


Payment of liabilities ............ ( 60,000)
Payment to Partners .............. (630,000)

Other
Assets

Liabilities

P500,000 P60,000
( 520,000) _______

_______

JJ (4)
P180,000
__56,000

60,000
236,000
( 60,000)
_______ ( 236,000)

Capital
KK(4)
(LL(2)
P240,000
__56,000

P70,000
_28,000

296,000

98,000

( 296,000)

( 98,000)

Partnership Liquidation

79

Problem 4 6
a.

BB ................................................... P160,000
CC ................................................... P20,000
DD................................................... P60,000
EE ...................................................
P 0

b.

BB, CC, DD, & EE


Statement of Liquidation
Cash
Balances before liquidation...
P
0
Advances by BB to pay liabilities
Deposit by DD ......................
60,000
Balances .... .... ......................
60,000
Elimination of EE's deficiency
Elimination of DD's deficiency
Payment to partners...............

Liabilities

C a p i t
CC (10%)DD (20%)

BB (30%)

P60,000 P160,000
( 60,000)
60,000
______ _______

P80,000

220,000
( 90,000)
______ __( 90,000)

60,000

l
EE (40%)

(P120,000) P(180,000)

_______

__60,000

80,000
( 30,000)
( 30,000)

( 60,000) ( 180,000)
( 60,000)
180,000
120,000

40,000

20,000

________

Problem 4 7
Sayson and Company
Statement of Liquidation
Date
Assets
Cash Noncash

Liabilities
Accounts
Notes
Payable Payable

Pea
Loan

P a r t n e r s' C a p i t a l s
Sayson
Zobel
Ayala
(45%)
(30%)
(15%)

Balances before liquidation...


Realization of assets and
distribution of gain ..........

P 15,000

P155,250

P11,250

P9,000

P 1,500

P 75,345

185,000

( 155,250)

_______

______

______

17,850

11,900

Balances................................
Payment of liabilities ............

200,000
( 20,250) ________

11,250
( 11,250)

9,000
( 9,000)

1,500
______

93,195
______

98,398 ( 14,993)
______ _______

1,650
______

1,500

93,195

98,398 ( 14,993)

1,650

Balances................................
Additional loss to Sayson,
Zobel and Pea;
45:30:10 ..........................

179,750

_______ ________ ________

Balances................................
Offset Pea's loan against
his capital deficiency.......

179,750

_______ ________ ________

Balances................................

179,750

Payments to partners.............

P(179,750)

______

______

P 86,498 P(14,993)

Pea
(10%)

______

( 7,937)

( 5,292)

14,993

1,500

85,258

93,106

______

( 114)

______

______

1,386

85,258

93,106

P(1,386) P(85,258) P(93,106)

_______
-

P1,650
______

( 1,764)
(114)
114

80

Chapter 4

Problem 4 8
a.

Art, Bea and Cid Partnership


Statement of Liquidation
June 4, 2008

Balances before liquidation


(including Bea loan, P4,000)......
Realization of assets
at a loss of P63,300 ..................
Unrecorded accounts payable .........
Payment to creditors .......................
Balances .... .... ................................
Eliminate Cid's deficit.....................
Balances .... .... ................................
Payment to Partners ........................
b.
2008
July 5

c.

Cash

Assets
Other

Liabilities

Partners' Capital
Art (40%) Bea (40%) Cid (20%)

P 6,000

P94,000

P20,000

P27,000

P43,000

30,000

( 94,000)

(20,500)
16,200
______
16,200
(16,200)

______
______
-

(25,320)
500
(20,500)
______
-

(25,320)
(200)
______
1,480
(1,380)
100
_( 100)

(12,660)
(200)
______
17,480
(1,380)
16,100
( 16,100)

Cash .... .... ................................ ............. .................. ..................


Art capital (P63,300 x 40%)....... ............. .................. ..................
Bea capital (P63,300 x 40%)...... ............. .................. ..................
Cid capital (P63,300 x 20%) ...... ............. .................. ..................
Other assets....................... ............. .................. ..................
To record realization of other assets at a loss of P63,300.

30,700
25,320
25,320
12,660

Art capital (P500 x 40%)............ ............. .................. ..................


Bea capital (P500 x 40%)........... ............. .................. ..................
Cid capital (P500 x 20%) ........... ............. .................. ..................
Liabilities .......................... ............. .................. ..................
To record trade accounts payable.

200
200
100

Liabilities .. ................................ ............. .................. ..................


Cash . ................................ ............. .................. ..................
To record payment of liabilities.

20,500

Art capital.. ................................ ............. .................. ..................


Bea capital. ................................ ............. .................. ..................
Cid capital......................... ............. .................. ..................
To eliminate Cid's capital deficit.

1,380
1,380

Art capital.. ................................ ............. .................. ..................


Bea capital. ................................ ............. .................. ..................
Cid capital . ................................ ............. .................. ..................
Cash . ................................ ............. .................. ..................
To record payments to partners to complete liquidation.

100
4,000
12,100

P10,000
(100)
______
(2,760)
_2,760

94,000

500

20,500

2,760

16,200

Cid's loss must be limited to P5,000, or P25,000 for the partnership (P5,000 / 20% = P25,000).
Because the liquidation of liabilities results in a loss of P500, only P24,500 may be lost on the
realization of other assets. This requires that other assets realize P69,500 (P94,000 24,500) to
enable Cid to receive P5,000 from the partnership to pay personal creditors in full.

Problem 4 9
KGB Partnership
Statement of Realization and Liquidation
Lump-sum Liquidation on June 30, 2008

Preliquidation balances
Sale of assets
and distribution
of 430,000 loss
Cash contributed
by B
Distribution of deficit
of insolvent partner:
20/60 (P2,000)
40/60 (P2,000)
Offset deficit with loan
Contribution by G
Payment of creditors
Distribution to K
Postliquidation
balances

Capital Balances
K
G
20%
40%
(240,000) (100,000)

B
40% (120,000)

Cash
50,000

Noncash
Assets
950,000

Liabilities
(480,000)

G
Loan
(60,000)

520,000
570,000

950,000
-0-

(480,000)

(60,000)

86,000
(154,000)

172,000
72,000

172,000
52,000

(480,000)

(60,000)

(154,000)

72,000

50,000
2,000

50,000
620,000

620,000
620,000
13,334
633,334
(480,000)
153,334
(153,334)
-0-

-0-

-0-0-0-0-

-0-

666

(480,000)
(480,000)

(60,000)
60,000
-0-

(153,334)
(153,334)

(480,000)
480,000
-0-

-0-

(153,334)
(153,334)
153,334

-0-

-0-

-0-

-0-

(2,000)
1,334
73,334
(60,000)
13,334
(13,334)
-0-0-

-0-

-0-0-

-0-0-

-0-

82

Chapter 4
KGB Partnership
Schedule of Distribution of Personal Assets
June 30, 2008
Personal assets, excluding partnership
capital and loan interests
Personal liabilities
Personal net worth, excluding
partnership capital and loan
interests
Contribution to partnership
Distribution from partnership
Personal capacity

500,000
(460,000)

600,000
(480,000)

700,000
(650,000)

40,000

120,000
(13,334)
-0- 106,666

50,000

153,334
193,334

-0- -0- -

Partnership Liquidation by Installment

83

CHAPTER 5
MULTIPLE CHOICE ANSWERS AND SOLUTIONS
5-1: b
Capital balances before liquidation
Loan balances
Total interest
Possible loss (40,000+10,000)
Balances
Additional loss to RJ & SJ, 5:3
Cash distribution

RJ
P22,000
_10,000
32,000
( 25,000)
7,000
( 1,250)
P 5,750

SJ
P30,000
______
30,000
( 15,000)
15,000
( 750)
P14,250

TJ
P 8,000
______
8,000
( 10,000)
( 2,000)
__2,000
P

Capital balances
Loan balances
Total interest
Possible loss (23,000-6,000)
Balances
Additional loss to BR, CR, DR, 3:2:1
Balances
Additional loss to CR & DR, 2:1
Payment to partners

AR
P 5,500
_1,000
6,500
( 6,800)
( 300)
___300

_____
P

BR
P 5,150
_____
5,150
( 5,100)
50
( 150)
( 100)
___100
P

CR
P 6,850
_____
6,850
( 3,400)
3,450
( 100)
3,350
_( 67)
P 3,283

Total liabilities
Total Capital
Total Assets

P 1,000
_22,000
P23,000

5-2: a

5-3: c
Capital balances
Loan balances
Advances
Total interest
Divided by P/L Ratio
Loss Absorption balances
PI - TO GG
Balances
PII - TO EE & GG, 30:10
Balances
PIII - TO EE, FF, GG, 3:1:1
Balances
PIV - P/L Ratio

DD
P40,000
5,000
_____
45,000
____50%
90,000
_____
90,000
_____
90,000
_____
P90,000

BALANCES
EE
FF
P30,000
P15,000
10,000

_____
( 4,500)
40,000
10,500
____30%
____10%
133,333
105,000
_____
( 91,667)
133,333
105,000
( 28,333)
_____
105,000
105,000
(15,000)
( 15,000)
P90,000
P90,000

DR
P 4,500
_____
4,500
( 1,700)
2,800
( 50)
2,750
_( 33)
P 2,717

GG
P25,000

( 2,500)
22,500
____10%
225,000
__ __
133,333
( 28,333)
10,500
( 15,000)
P90,000

Chapter 5

84
DD
PI - To GG
PII - To EE (28,833 X 30%)
GG (28,833 X 10%)
PIII To EE (15,000 X 30%)
FF (15,000 X 10%)
GG (15,000 X 10%)

_____

Total
PIV - P/L Ratio
Distribution of P18,000
PI - TO GG
PII - TO EE & GG, 3:1, P8,833
Cash distribution

5-4: a

DD

GG
P 9,167

2,833

__1,500

P12,933

P 1,500

P13,500

EE

FF

GG

_____

_6,625

_____

P 9,167
__2,208

P 6,625

P11,375

Capital balances before liquidation


Loss on realization, P40,000
Capital balances before cash distribution
Possible loss, P90,000
Balances
Additional loss to Lim & Wan, 4:2
Cash distribution

TAN
P40,000
( 16,000)
24,000
( 36,000)
( 12,000)
_12,000
P

LIM
P65,000
( 16,000)
49,000
( 36,000)
13,000
( 8,000)
P 5,000

WAN
P48,000
( 8,000)
40,000
( 18,000)
22,000
( 4,000)
P18,000

Capital balances before cash distribution


Possible loss (90,000+3,000)
Balances
Additional loss to Lim & Wan, 4:2
Cash distribution

TAN
P24,000
( 37,200)
( 13,200)
_13,200
P

LIM
P49,000
( 18,600)
30,400
( 8,800)
P21,600

WAN
P40,000
( 18,600)
21,400
_( 4,400)
P17,000

CARPIO
P72,000
( 5,000)
67,000
( 55,000)
12,000
( 6,000)
P 6,000

LOBO
P54,000
( 5,000)
49,000
( 55,000)
( 6,000)
__6,000
P

5-5: b

5-6: d

CASH PAYMENT
EE
FF

P 8,433

4,500

1,500
_____
_____

Tan (14,000 X 40%)


Lim (14,000 X 40%)
Wan (14,000 X 20%)

5-7: a
Capital balances before liquidation
Goodwill written-off
Cash balance
Possible loss (100,000+10,000), 110,000
Capital balances before liquidation
Additional loss to Carpio
Cash distribution

P5,600
P5,600
P2,800

Partnership Liquidation by Installment

85

5-8: d

JACOB
P40,000
( 15,000)
( 1,000)
24,000
__8,000
32,000
( 45,000)
( 13,000)
_13,000
P

SANTOS
P72,000
( 9,000)
( 600)
62,400
_____
62,400
27,000
35,400
( 7,800)
P27,600

HERVAS
P 7,000
( 6,000)
( 400)
63,600
_____
63,600
( 18,000)
45,600
( 5,200)
P40,400

A
P16,200
_____
16,200
( 600)
15,600
( 150)
15,450
12,000
27,450
( 27,000)
450
( 780)
( 330)
___330
P

B
P12,000
___160
12,000
( 600)
11,400
( 150)
11,250
14,400
25,650
( 27,000)
( 1,350)
__1,350

_____
P

C
P37,700
___240
37,860
( 600)
37,260
( 150)
37,110
_____
37,110
( 27,000)
10,110
( 780)
9,330
( 330)
P 9,000

D
P17,700
_______
( 17,940)
( 600)
17,340
( 150)
17,190
__9,600
26,790
( 27,000)
( 210)
____210

_____
P

DY
P22,000
2/4
44,000
_____
44,000
_____
P44,000

BALANCES
SY
P15,500
1/4
62,000
( 6,000)
56,000
( 12,000)
P44,000

LEE
P14,000
1/4
56,000
_____
56,000
( 12,000)
P44,000

Capital balances before liquidation


Loss on realization (120,000-90,000)
Liquidation expenses, P2,000
Capital balances before cash distribution
Loan balances
Total interest
Possible Loss (210,000-120,000)
Balances
Additional loss to Santos & Hervas
Cash distribution

5-9: d
Capital balances before liquidation
Salary payable
Balances
Loss on realization (P2,400)
Balances
Liquidation expenses (P600)
Balances
Loan balances
Total interest
Possible Loss (126,000-18,000)
Balances
Additional loss to A & C
Balances
Additional loss to C
Cash distribution

5-10: a
Total interest
Profit and Loss ratio
Loan absorption balances
Priority I - to Sy
Balances
Priority II - to Sy & Less
Total

DY
Priority I - to Sy (6,000 X 1/4)
Priority II - to Sy (12,000 X 1/4)
to Lee (12,000 X 1/4)
Total

_____
P

CASH PAYMENTS
SY
LEE
1,500

3,000

_____
_3,000
P 4,500
P 3,000

Chapter 5

86
Further cash distribution, profit and loss ratio
Cash distribution to Dy
Divided by Dy's Profit and Loss ratio
Amount in excess of P7,560
Total payment under priority I & II
Total cash distribution to partner

P 6,250
2/4
12,500
__7,500
P20,000

5-11: d
Cash before liquidation
Cash realized
Total
Less:
Payment of liquidation expense
Payment of liability
Payment to partners (Q 5-10)
Cash withheld

P 1,000
5,400
20,000

P12,000
_32,000
44,000
_26,400
P17,600

5-12: c
Loss absorption balances:
Cena (18,000/50%)
Batista (27,000/30%)
Excess of Batista
Multiply by Batista's Profit & Loss ratio
Priority I to Batista

P36,000
90,000
54,000
____30%
P16,200

5-13: c
Capital balances
Loan balances
Total interest
Divided by Profit and Loss Ratio
Loss Absorption balances
Priority I to CC
Balances
Priority II to BB & CC, 2:1
Total interest

AA
P15,000
10,000
25,000
2/5
62,500
_____
62,500
_____
P62,500
AA

Priority I to CC (12,500 X 1/5)


Priority II to BB (25,000 X 2/5)
to CC (25,000 X 1/5)
Total
Priority III P/L Ratio
Cash distribution to CC:
Priority I
Priority II (12,000-2,500) X 1/3
Total cash paid to CC

____
P

BALANCES
BB
P30,000
_5,000
35,000
2/5
87,520
_____
87,520
( 25,000)
P62,500

CC
P10,000
10,000
20,000
1/5
100,000
( 12,500)
100,000
( 25,000)
P62,500

CASH PAYMENTS
BB
CC

2,500
10,000

_____
_5,000
P10,000
P 7,500
P2,500
3,167
P5,667

Partnership Liquidation by Installment

87

5-14: c
JJ
P 60,000
_18,000
_78,000
____40%
195,000
______
195,000
______
195,000
______
P195,000

Capital balances
Loan balances
Total interest
Divided by Profit and Loss Ratio
Loss Absorption balances
Priority I to LL
Balances
Priority II to LL, MM, 15:10
Balances
Priority II to KK, LL, MM, 35:15:10
Total

JJ
Priority I to LL (30,000 X 15%)
Priority II to LL (30,000 X 15%)
to MM (30,000 X 10%)
Priority II to KK (75,000 X 35%)
to LL (75,000 X 15%)
to MM (75,000 X 10%)
Total

______
P

BALANCES
KK
LL
MM
P 64,500
P 54,000
P 30,000
_30,000
______
______
_94,500
_54,000
_30,000
_____35%
_____15% _____10%
270,000
360,000
300,000
______
( 60,000)
______
270,000
300,000
300,000
______
( 30,000) ( 30,000)
270,000
270,000
270,000
( 75,000)
( 75,000) ( 75,000)
P195,000
P195,000
P195,000
CASH PAYMENT
KK
LL

9,000

4,500

1,750

11,250
______
______
P 1,750
P 24,750

MM

3,000

___7,500
P 10,500

Further cash distribution, Profit and Loss ratio


Cash distribution to Partners (P38,100-9,000), P29,100
Priority I to LL
Priority II to LL, MM, 15:10
Priority II to KK, LL, MM, 35:15:10
(29,100-16,500), 12,600
Cash distribution

JJ

_____
P

KK

MM

LL
P 9,000
4,500

3,000

TOTAL
P 9,000
7,500

__7,350
P 7,350

___3,150
P 16,650

__2,100
P 5,100

__12,600
P 29,100

ARCE
P 20,000
_10,000
_32,000
_____50%
64,000
______
64,000
______
P 64,000

BALANCES
BELLO
P 24,900
______
_24,900
_____30%
83,000
( 8,000)
75,000
( 11,000)
P 64,000

CRUZ
P 15,000
______
_15,000
_____20%
75,000
______
75,000
( 11,000)
P 64,000

5-15: a
Capital balances
Loan balances
Total interest
Divided by Profit and Loss Ratio
Loss Absorption balances
Priority I to Bello
Balances
Priority II to Bello & cruz, 3:2
Total

Chapter 5

88
CASH PAYMENTS
ARCE

BELLO

CRUZ

P - I to Bello (8,000 X 30%)


P - II to Bello (11,000 X 30%)
to Cruz (11,000 X 20%)

_____

2,400
3,300
_____

_2,200

Total

P 5,700

P2,200

Further Cash distribution, Profit and Loss ratio


Based on the above cash priority program, the P2,000 is only a partial payment to Bello who
is entitled to a maximum of P2,400 under Priority I. Only after satisfying Priority I, Cruz will
receive payment and only after P7,900 has been distributed to Bello and Cruz will Arce receive
payment. Therefore no payments are made to Arce and Cruz.

5-16: a
Cash paid to Arce
Divide by Profit & Loss ratio

P2,000
_____5%

Amount in excess of P7,900


Add: cash paid under PI and PII

40,000
_7,900

Total cash distribution to partners


Cash paid to Creditor (30,000-10,000)

47,900
20,000

Total
Less cash before realization

67,900
_6,000

Cash realized from sale of asset

P61,900

5-17: b
Cash distribution to Cruz
Divide by profit and loss ratio

P 6,200
2/5

Cash distribution under Priority II


Multiply by Bello's Profit and Loss ratio

15,500
3/5

Cash distribution to Bello under Priority II


Cash distribution to Bello under Priority I

9,300
__2,400

Total cash distribution to Bello

P11,700

5-18: b

BALANCES
MONZON

NIEVA

Total Interest

P22,500

P17,500

Profit and Loss ratio

_____60%

_____40%

Loss absorption balances


Priority I - to Nieka

37,500
______

Total

P37,500

CASH PAYMENT
MONZON

NIEVA

43,750
( 6,250)

_____

_2,500

P37,500

P2,500

Further cash distribution - Profit and Loss ratio


All the P2,000 should be paid Nieva, since she is entitled to P2,500 under Priority I

Partnership Liquidation by Installment

5-19: b
Cash distribution
PI to Nieva (2,500-2,000)
Balances, 6:40
Cash distribution

89

CASH MONZON
P12,500

( 500)

_12,000
__7,200
P

P 7,200

5-20: a
Cash before liquidation
June: Cash realized
Payment to creditor
Payment to Partners
Cash balances, June 30
July: Cash realized
Payment of liquidation expense
Payment to Partners
Cash balances, July 31
Aug: Cash realized
Cash distribution for August,
Profit and Loss ratio
Distribution to Partners - August
Monzon (22,500 X 60%)
Nieva (22,500 x 40%)

P 5,000
18,000
( 20,000)
__2,000
1,000
12,000
( 500)
( 12,500)

_22,500
P22,500
P13,500
P 9,000

NIEVA

500
_4,800
P5,300

Chapter 5

90

SOLUTIONS TO PROBLEMS
Problem 5 1
Suarez, Tulio and Umali
Statement of Liquidation
January 1 to april 31, 2008
Assets
Cash
Others Liabilities
Balances before liquidation.P 2,000.00 P46,000.00 P6,000.00
January Installment:
Realization of assets and
distribution of loss .... 10,500.00 ( 12,000.00) _______
Balances......................... 12,500.00 34,000.00 6,000.00
Payment of expenses of
realization and distribution
to partners ...................... ( 500.00) _______ _______
Balances......................... 12,000.00 34,000.00 6,000.00
Payment of liabilities ..... ( 6,000.00) _______ ( 6,000.00)
Balances......................... 6,000.00 34,000.00

Payments to partners
(Schedule 1) ............. ( 4,000.00) _______ _______
Balances......................... 2,000.00 34,000.00

February Installment:
Realization of assets and
distribution of loss .... 6,000.00 ( 7,000.00) _______
Balances......................... 8,000.00 27,000.00

Payment of expenses of
realization and distribution
to partners ...................... ( 750.00) _______ ______
Balances......................... 7,250.00 27,000.00

Payments to partners
(Schedule 2) ............. ( 6,000.00) _______ ______
Balances......................... 1,250.00 27,000.00

March Installment:
Realization of assets and
distribution of loss .... 10,000.00 ( 15,000.00)______
Balances......................... 11,250.00 12,000.00

Payment of expenses of
realization and distribution
to partners ...................... ( 600.00) _______ ______
Balances......................... 10,650.00 12,000.00

Payments to partners,
P & L ratio................( 10,150.00) ______
______
Balances.........................
500.00 12,000.00

April Installment:
Realization of assets and
distribution of loss .... 4,000.00 ( 12,000.00)______
Balances......................... 4,500.00

Payment of expenses of
realization and distribution
to partners ...................... _(400.00) ______
______
Balances......................... 4,100.00

Final Payments to partnersP(41,100.00)_____


_____

Tulio,
Umali,
Partners' Capitals
Loan
Loan Suarez (40%) tulio (35%)Umali (25%)
P5,000.00 P2,500.00 P14,450.00 P12,550.00 P7,500.00
_______
______
5,000.00 2,500.00

( 600.00) ( 525.00)
13,850.00 12,025.00

( 375.00)
7,125.00

_______ _______
5,000.00 2,500.00
_______ _______
5,000.00 2,500.00

( 200.00)
13,650.00
_______
13,650.00

( 175.00)
11,850.00
________
11,850.00

( 125.00)
7,000.00
_______
7,000.00

( 3,812.50) ( 187.50)
1,187.50 2,312.50

_______
13,650.00

_______
11,850.00

_______
7,000.00

_______ _______
1,187.50 2,312.50
_______
1,187.50

_______
2,312.50

__(400.00) ( 350.00)
13,250.00 11,500.00

( 250.00)
6,750.00

( 300.00) ( 262.50)
12,950.00 11,237.50

( 187.50)
6,562.50

( 1,187.50) ( 1,812.50) ( 1,650.00) ( 1,350.00)

500.00
11,300.00
9,887.50

_______
6,562.50

______

______
500.00

( 2,000.00) ( 1,750.00) ( 1,250.00)


9,300.00
8,137.50
5,312.50

______

_______
500.00

______

( 500.00) ( 4,060.00) ( 3,552.50) ( 2,037.50)

5,000.00
4,375.00
3,125.00

( 240.00) ( 210.00)
9,060.00
7,927.50

( 150.00)
5,162.50

______

______

( 3,200.00) ( 2,800.00) ( 2,000.00)


1,800.00
1,575.00
1,125.00

______

_____

______

_____

___(160.00) ( 140.00) ( 100.00)


1,640.00
1,435.00
1,025.00
P( 1,640.00) P( 1,435.00) P(1,025.00)

Partnership Liquidation by Installment

91

Schedule 1
Suarez (40%)
Capital balances ......................................
P13,650.00
Loan balances..........................................
_____ _
Total interests..........................................
13,650.00
Possible loss (P2,000 + P34,000)...........
( 14,400.00)
Balances ..................................................
( 750.00)
Additional loss to Tulio and Umali 35:25
___750.00
Payments to partners ...............................

Apply to loan...........................................
__ __

Schedule 2
Capital balances ......................................
Loan balances..........................................
Total ........................................................
Possible loss (P1,250 + P27,000)...........
Payments to partners ...............................
Apply to loan...........................................
Apply to capital.......................................

Suarez (40%)
P12,950.00

12,950.00
( 11,300.00)
P 1,650.00

P 1,650.00

Tulio (35%) Umali (25%)


P11,850.00
P7,000.00
__5,000.00
_2,500.00
16,850.00
9,500.00
( 12,600.00)
( 9,000.00)
4,250.00
500.00
( 437.50)
( 312.50)
P 3,812.50
P 187.50
P 3,812.50
P 187.50
Tulio (35%) Umali (25%)
P11,237.50
P6,562.50
__1,187.50
_2,312.50
12,425.00
8,875.00
( 9,887.50)
( 7,062.50)
P 2,537.50
P1,812.50
_1,187.50
_1,812.50
P 1,350.00
P

Chapter 5

92

Problem 5 2
Miller and Bell Partnership
Statement of Partnership Realization and Liquidation

Cash
25,000
40,000

Balances
Sale of inventory
Payment to
creditors
(10,000)
55,000
Payments to
partners
(Schedule 1)
(50,000)
5,000
Sale of inventory 30,000
Payment to
creditors
( 5,000)
30,000
Offset deficit
with loan
______
30,000
Payments to
partners:
Loan
( 6,000)
Capitals
(24,000)
Balances
0

Inventory
120,000
( 60,000)

Accounts
Payable
15,000

Bell
Loan
60,000

Capital
Miller
Bell
80%
20%
65,000
5,000
(16,000)
(4,000)

______
60,000

(10,000)
5,000

______
60,000

______
49,000

______
1,000

______
60,000
( 60,000)

______
5,000

(49,000)
11,000

_(1,000)
48,000
(24,000)

______
1,000
6,000)

______
0

( 5,000)
0

______
11,000

______
24,000

______
(5,000)

______
0

______
0

( 5,000)
6,000

______
24,000

(5,000)
0

______
0

______
0

( 6,000)
______
0

(24,000)
0

______
0

Schedule 1:
Miller and Bell Partnership
Schedule of Safe Payments to Partners

Capital and loan balances


Possible loss of 60,000 on remaining inventory
Safe payment

Miller
80%
49,000
(48,000)
1,000

Bell
20%
61,000
(12,000)
49,000

Partnership Liquidation by Installment

HORIZON PARTNERSHIP
Statement of realization and Liquidation
May July, 2008

93

Problem 5 3

Partners Capital
TT
(1/3)

Other

Liabilities

SS
(1/3)

20,000
75,000

280,000
(105,000)

80,000
______

60,000
(10,000)

70,000
(10,000)

90,000
(10,000)

Balances
Payment to creditors

95,000
(80,000)

175,000
______

80,000
(80,000)

50,000
______

60,000
______

80,000
______

Balances
Payments to PP (Exhibit A)

15,000
(15,000)

175,000
______

______

50,000
______

60,000
______

80,000
(15,000)

0
25,000

175,000
(61,000)

______

50,000
(12,000)

60,000
(12,000)

65,000
(12,000)

25,000
(25,000)

114,000
______

______

38,000
______

48,000
(10,000)

53,000
(15,000)

114,000

38,000

38,000

38,000

(114,000)

(11,000)

(11,000)

(11,000)

27,000
(27,000)

27,000
(27,000)

27,000
(27,000)

SS

TT

PP

60,000
1
60,000

70,000
1
70,000

90,000
1
90,000

______

______

(20,000)

Cash
Balances before liquidation
May sale of assets at a loss of P30,000

Balances
June sale of assets at a loss of P36,000
Balances
Payment to partners (Exhibit A)
Balances
July sale of remaining assets at a loss of
P33,000
Balances
Payment to partners

Assets

81,000
81,000
(81,000)

Exhibit A Cash distributions to partners during liquidation:


Capital account balances before liquidation
Income sharing ratio
Loss absorption balances
Required reduction to bring
capital account balance for PP
to equal the next highest balance for TT PI.

PP
(1/3)

Balances
Required reduction to bring the balances for
TT and PP to equal the balance for SS PII.

60,000

70,000

70,000

______

(10,000)

(10,000)

Balances

60,000

60,000

60,000

10,000

20,000
10,000

1/3

1/3

Summary of cash distribution program:


To creditors before partners receive anything
To partners:
(1) First distribution to PP
(2) Second distribution to TT and PP equally
(3) Any amount in excess of $120,000
to the three partners in incomesharing ratio

80,000
20,000
20,000
1/3

b. After the cash distribution in June, the partners capital accounts had balances corresponding to the income-sharing
ratio (38,000 each). From this point on any cash payments to partners may be made in the income-sharing ratio or
equally in this problem. In other words, after the creditors are paid and TT and PP receive 10,000 and 30,000,
respective, any additional cash that becomes available may be paid to the three partners equally.

Chapter 5

94

Problem 5 4
1. X, Y and Z
Cash Priority Program
January 1, 2008
X

Balances
Y

Capital balances ..................................


Loan balances .....................................

P60,000
22,5000

P45,000
15,000

P20,000
6,500

Total interests......................................

P82,500

P60,000

P26,500

P200,000
(35,000)

P132,500

Loss absorption balances .................... P165,000


Priority I to Y...................................
Balances..............................................
Priority II to X and Y .......................

165,000
(32,500)

Total.................................................... P132,500

165,000
132,500
(32,500) ________
P132,500

P132,500

Any amount in excess of P36,500.......

X (50%)

Cash Payments
Y (30%) Z (20%)

Total

P10,500

P10,500

P16,250

9,750

26,000

P16,250

P20,250

P36,500

50%

30%

20%

100%

2. January
Cash
Available for distribution ..............................
Priority I to Y .............................................
Payment to partner.........................................
February.......................................................
Available for distribution ..............................
Priority I to Y (P10,500 P7,500) .............
Priority II to X and Y; 5:3 ..........................

Cash
P20,000
( 3,000)
( 17,000)

Payments to partners......................................
March ...........................................................
Available for distribution ..............................
Priority II to X and Y; 5:3
(P26,000 P17,000).................................
Excess; 5:3:2..................................................

Cash
P45,000
( 9,000)
( 36,000)

Payments to partners......................................
April..............................................................
Available for distribution ..............................
Excess; 5:3:2..................................................
Payments to partners......................................

P 7,500
( 7,500)

Cash
P15,000
( 15,000)

P 7,500

P 7,500

P10,625

P 3,000
6,375

_____

P10,625

P 9,375

P 5,625
18,000

P 3,375
10,800

P7,200

P23,625

P14,175

P7,200

P 7,500

P 4,500

P3,000

P 7,500

P 4,500

P3,000

Partnership Liquidation by Installment

95

Problem 5 5

AB, CD & EF Partnership


Statement of Partnership Realization and Liquidation
Capital
Able
Other Accounts CD
AB
CD
Cash
Loan
Assets Payable Loan
50%
30%
18,000 30,000 307,000 53,000 20,000 118,000 90,000

Balances before liquidation


January transactions:
1. Collection of accounts
receivable at loss
of 15,000
51,000
( 66,000)
2. Sale of inventory at
loss of 14,000
38,000
( 52,000)
3. Liquidation expenses paid ( 2,000)
4. Share of credit memorandum
5. Payments to creditors
( 50,000) _____ ______
55,000 30,000 189,000
Sale payments to partners
(Schedule 1
( 45,000) ______ _____
10,000 30,000 189,000
February transactions:
6. Liquidation expenses paid ( 4,000) ______ ______
6,000 30,000 189,000
Safe payments to partners
(Schedule 2)
-0- _____ ______
6,000 30,000 189,000
March transactions:
8. Sale of mac. & equip. at a
loss of 43,000
146,000
(189,000)
9. Liquidation expenses paid ( 5,000) ______ _______
147,000 30,000
-010. Offset AB's loan
receivable against capital
(30,000)
Payments to partners
(147,000) ______ _______
Balances at end of liquidation
0
0
0

EF
20%
74,000

( 7,500) ( 4,500) ( 3,000)


( 7,000) ( 4,200)
( 1,000) ( 600)
( 3,000)
1,500
900
(50,000) _____ ______ _____
-0- 20,000 104,000 81,600

( 2,800)
( 400)
600
______
68,400

______ (20,000) ______ ( 6,600) (18,400)


-0-0- 104,000 75,000 50,000
______ ______ ( 2,000) ( 1,200) ( 800)
-0-0- 102,000 73,800 49,200
______ ___
0
-0-0- 102,000

0
73,800

0
49,200

( 21,500) (12,900) ( 8,600)


______ ______ ( 2,500) ( 1,500) ( 1,000)
-0-0- 78,000 59,400 39,600
( 30,000)
______ ______ ( 48,000) (59,400) (39,600)
0
0
0
0
0

Chapter 5

96

Partnership
Schedules of Safe Payments to Partners
Schedule 1: January
Capital and loan balancesa
Possible loss:
Other assets (189,000) and possible liquidation
costs (10,000)
Balances
Absorption of AB's potential deficit balance
CD : (25,500 x 3/5 = 15,300)
EF : (25,500 x 2/5 = 10,200)
Safe payment
a = (104,000) capital less 30,000 loan receivable
= (81,600) capital plus 20,000 loan payable
= (68,400) capital
Schedule 2: February
Capital and loan balancesb
Possible loss:
Other assets (189,000) and possible liquidation
costs (6,000)
Absorption of AB's potential deficit balance
CD : (25,500 x 3/5 = 15,300)
EF : (25,500 x 2/5 = 10,200)
Safe payment
b = (102,000) capital less 30,000 loan receivable
= (73,800) capital
= (49,200) capital

AB
50%

CD
30%

EF
20%

P74,000

P101,600

P68,400

( 99,500)
( 25,500)
25,500

( 59,700)
41,900

( 39,800)
28,600

( 15,300)
_______
P 26,600

( 10,200)
P 18,400

______
P
-0-

72,000
( 97,500)
( 25,500)
25,500
_______
0

73,800

49,200

( 58,500)
15,300

( 39,000)
10,200

( 15,300)
________
0

( 10,200)
0

Partnership Liquidation by Installment

97

Problem 5 6
1.

M, N, O and P
Cash Priority Program
January 1, 2008

M
Capital balances.. P 70,000
Loan balances ..... 20,000
Total interests ..... P 90,000
Loss absorption
balances .........P240,000
Priority I to O .. _______
Balances ............. 240,000
Priority II to O
and P .............. _______
Balances ............. 240,000
Priority III to
M, O and P.....( 40,000)
Total ...................P200,000

Balances
N
O
P 70,000 P 30,000
5,000
25,000
P 75,000 P 55,000

P
P 20,000
15,000
P 35,000

Cash Payments
M (3/8) N (3/8) O (1/8)
P (1/8)

P200,000 P440,000 P280,000


_______ ( 160,000) ________
200,000 280,000
280,000
_______ ( 40,000) ( 40,000)
200,000 240,000
240,000

P20,000

5,000

_______ ( 40,000) ( 40,000) P15,000


P200,000 P200,000 P200,000 P15,000

Any amount in excess of P55,000

3/8

5,000
P30,000
3/8

Total

P20,000
P5,000

10,000

5,000 25,000
P10,000 P55,000

1/8

1/8

8/8

2.
Schedule 1

Available for distribution ....................


Priority I to O ...................................
Priority II to O and P; 1:1 .................

Cash

P25,000
( 20,000)
( 5,000)

________

_______

P20,000
2,500

P2,500

P22,500
( 22,500)

2,500
( 2,500)

P15,000
3,750

P3,750

P 2,500
5,000
1,250

P2,500
5,000
1,250

18,750
( 18,750)

P3,750
( 3,750)

8,750
( 2,500)
P 6,250

8,750
( 8,750)

Payments to partners............................
Apply to loan .......................................
Apply to capital ...................................
Schedule 2
Cash
Available for distribution ....................
Priority II to O and P; 1:1 .................
Priority III to M, O and P; 3:1:1 .......
Excess, 3:3:1:1.....................................
Payments to partners............................
Apply to loan .......................................
Apply to capital

P40,000
( 5,000
( 25,000)
( 10,000)

Chapter 5

98

Problem 5 7

Bronze, Gold & Silver


Cash Distribution Plan
June 30, 2008
Loss Absorption Balances
Bronze
Gold
Silver

Profit and loss ratio


Pre-liquidation capital and
loan balances
Loss absorption balances
(Capital and loan
balances/P& L ratio)
P110,000
Decrease highest LAB
to next highest:
Gold: (30,000 x .30) _______
110,000
Decrease LAB's
to next highest:
Gold: (10,000 x .30)
Silver: (10,000 x .20) _______
P110,000

Capital and Loan Accounts


Bronze
Gold
Silver
50%
30%
20%
P55,000

P45,000

P24,000

( 9,000)
36,000

______
24,000

P150,000

P120,000

( 30,000)
120,000

_______
120,000

______
55,000

( 10,000)
________
P110,000

( 10,000)
P110,000

_______
P 55,000

Accounts
Payable

Bronze
50%

Gold
30%

P37,500
P37,500

P 9,000
3,000
22,500
P34,500

( 3,000)
_______ _( 2,000)
P 33,000 P 22,000

Summary of Cash Distribution


(If Offer of P100,000 is Accepted)

Cash available
First
Next
Next
Additional paid in P&L ratio

P106,000
( 17,000)
( 9,000)
( 5,000)
( 75,000)
P
-0-

P 17,000
_______
P 17,000

Silver
20%

P 2,000
15,000
P17,000

Partnership Liquidation by Installment

99

Problem 5 8
Part A
North

Balances
South
East

Total Interest (capital and loan


balances
P120,000
P 88,000
Divided by P/L ratio
30%
10%
Loss absorption potential
P400,000 P880,000
Priority II To South
(335,000)
Balances
400,000
545,000
Priority II To South and East, 10:20
(145,000)
Balances
400,000
400,000
Priority III To North, South, and
east 30:10:20
(250,000) (250,000)
Total
150,000
150,000

West

North

P109,000 P 60,000
20%
40%
P545,000 P150,000
________
545,000
150,000
(145,000)
400,000
150,000
(250,000)
150,000

______
150,000

Cash Payments
South
East

33,500
14,500 29,000
75,000 25,000 50,000 _____
75,000 73,000 79,000

Further cash distribution P/L ratio


Part B
(1) Cash
65,600
North capital (30% of P16,400 loss)
4,920
South capital (10%)
1,640
East capital (20%)
3,280
West capital (40%)
6,560
Accounts receivable
To records collection of receivables with losses allocated to partners.
(2)

(3)

Cash
North capital (30% x P103,000)
South capital (10%)
East capital (20%)
West capital (40%)
Property and equipment
To record sale of property and equipment.

150,000
30,900
10,300
20,600
41,200

82,000

253,000

North capital
31,800
South capital
58,600
East capital
35,000
West capital
15,200
Cash
140,600
To record cash installment to partners of P230,600 based on the cash distribution plan in Part A.
First P90,000 is held to pay liabilities (P74,000) and estimated liquidation expenses of P16,000.
Next P33,500 goes entirely to South.
Next P43,500 is split between to South (P14,500) and East (P29,000).
Remaining P63,600 is allocated to North (P31,800), South (P10,600) and East (P21,200)

(4)

Liabilities
Cash
To record payment of liabilities.

74,000

West

74,000

Chapter 5

100

(5)

Cash
North capital (30% of P30,000 loss)
South capital (10%)
East capital (20%)
West capital (40%)
Inventory
To record inventory sold.

71,000
9,000
3,000
6,000
12,000

101,000

(6)

North capital
35,500
South capital
11,833
East capital
23,667
Cash
71,000
To record distribution of cash according to cash distribution plan. Although P87,000 cash
is being held, P16,000 must be retained to pay liquidation expenses. The Remaining
P71,000 is divided among North, South, and East on a 30:20 basis.

(7)

North capital (30% of expenses)


South capital (10%)
East capital (20%)
West capital (40%)
Cash
To record liquidation expenses paid.

(8)

North capital (30/60 of deficit)


2,080
South capital (10/60)
693
East capital (10/60)
1,387
West capital
To eliminate capital deficiency of West as computed below:

Capital balances, beginning


Loss on accounts receivable
Loss on property and equipment
Cash distribution
Liquidation expenses
Subtotal
Elimination of West deficiency
Capital balances
(9)

3,300
1,100
2,200
4,400

North capital
South capital
East capital
Cash
To record final cash distribution.

11,000

4,160

North
P120,000
(4,920)
(30,900)
(31,800)
( 3,300)

South
P88,000
( 1,640)
(10,300)
(58,600)
( 1,100)

East
P109,000
( 3,280)
(20,600)
(50,200)
( 2,200)

West
P60,000
( 6,560)
(41,200)
0
( 4,400)

4,580
( 2,090)

1,527
( 693)

3,053
( 1,666)

( 4,160)
4,160

P 2,500

P 834
2,500
834
1,666

P 1,666

5,000

P 0

Partnership Liquidation by Installment

101

Problem 5 9
DR Company
Schedule of Safe Payments to Partners

Capital and loan balances, August 1, 2008


Write-off of P24,000 in goodwill
Write-off of P12,000 of receivables
Gain of P6,000 on sale of P32,000 of
inventory (one-half of P64,000 book
value)
Capital and loan balances, August 31, 2008
Possible loss of P16,000 for remaining
receivables and P32,000 for
remaining inventory
Possible liquidation costs of P4,000
Balances (* = deficit)
Distribute Bens potential deficit
To Dan: P7,600 x 40/70
To Red: P7,600 x 30/70
Safe payments to partners

Dan
(40%)

Red
(30%)

Ben
(30%)

(42,000)
9,600
4,800

(45,000)
7,200
3,600

(17,000)
7,200
3,600

(2,400)
(30,000)

(1,800)
(36,000)

(1,800)
(8,000)

19,200
1,600
(9,200)

14,400
1,200
(20,400)

14,400
1,200
7,600*
(7,600)

4,343
(4,857)

3,257
(17,143)

-0-

Of the P84,000 in cash at the end of August, P58,000 will be required to liquidate the debts to
outside creditors, and P4,000 must be held in reserve to pay possible liquidation costs. Thus, a
total of P22,000 in cash can be safely distributed to partners as of August 31, 2008.

Problem 5 10
(1)

Journal entry to record Jennys contribution:


Cash
Equipment
Jenny, capital

40,000
60,000

100,000

Journal entry to record Kennys contribution:


Cash
Inventory
Equipment
Notes payable
Kenny, capital

60,000
10,000
180,000

50,000
200,000

Chapter 5

102

(2)

Capital balances of Jenny and Kenny before admission of Lenny:


Beginning capital balance
Interest on beginning capital balance
Annual salary
Remainder
Ending capital balance

Jenny
P100,000
10,000
15,000
48,000
P173,000

Kenny
P200,000
20,000
20,000
72,000
P312,000

Explanation:
Each partner receives 10% on beginning capital balance. Each partner receives
her respective income (P15,000 to Jenny and P20,000 to Kenny). The amount distributed
thus far is P65,000. The remainder to be distributed is P120,000 (P185,000 30,000
35,000). Two-fifths of this remainder of P129,000 (48,000) is allocated to Jenny; 3/5 x
P120,000 (72,000) is allocated to Kenny. The total income allocated to Jenny and Kenny
is P73,000 and P112,000 respectively.
The admission of Lenny can now be recorded by the following entry:
Cash

175,000

Lenny, capital
Jenny, capital
Kenny, capital

110,000
26,000
39,000

Explanation:
The book value of the partnership after the income distribution in 2006 was
P485,000 (P173,000 + P312,000). After Lennys contribution, the value of the
partnership is P485,000 + P175,000 = P660,000. A one-sixth interest in the partnership is
P660,000 x 1/6 = P110,000. Using the bonus method, we compute a bonus of P175,000
P110,000 = P65,000. Using the 2:3 profit sharing ratio, the amount allocated to Jenny is
P26,000 (2/5 x P65,000) and the amount allocated to Kenny is P39,000 (3/5 x P65,000).
(3)

Schedule of Safe Payments


Capital balances
Partners loan
Gain on realization
Possible loss
Safe payments to partners

Jenny
P200,000
9,000
(156,000)
P 53,000

Kenny
P400,000
(50,000)
15,000
(260,000)
P105,000

Lenny
P200,000
6,000
(104,000)
P102,000

Explanation:
The sale of assets realized a gain of P30,000 (P210,000 P180,000) which is
distributed to the partners on the new profit sharing ratio: 30% to Jenny, 50% to Kenny,
and 20% to Lenny. Liabilities are paid. A possible loss on the unsold assets (P520,000) is
distributed to partners in their profit and loss ratio of 30:50:20 to Jenny, Kenny and
Lenny respectively.

Joint Venture

103

CHAPTER 6
SOLUTIONS TO MULTIPLE CHOICES

6-1: a

6-2: a

Assets per Jessica Company- balance sheet


Jessicas proportionate interest in assets of JV (50%)
Total assets of Jessica

P3,550,000
1,000,000
P4550,000

Total liabilities only of Jenny Co.

6-3: b
6-4: b

6-5: a

6-6: a

Investment of Heart
Profit share:
Sales
Cost of sales (150,800 125%)
Gross profit
Expenses
Net Profit
Profit/loss ratio
Balance of investment in JV

P80,000
150,800
120,640
30,160
10,000
20,160
x 40%

8,064
P88,064

Cash
Merchandise inventory
Accounts receivable
Total assets
Sweet Cos, proportionate interest
Sweet Companys share in total asset
Sales
Cost of sales
Purchases
Merchandise inventory, end (50% of P10,000)

P190,000
29,360
150,800
370,160
x 60%
P222,096
7,200
P10,000
__5,000

_5,000

Gross profit
Expenses

2,200
___500

Net profit

P 1,700

104

6-7: b

6-8: a

6-9: b

Chapter 6

Original investment (cash)


Profit share (P1,700 / 2)

P10,000
___850

Balance of Investment account

P10,850

Joint venture account before profit distribution (credit balance)


Unsold merchandise

P 9,000
__2,500

Joint venture profit before fee to Salas

P11,500

Joint venture profit after fee to Salas (P11,500 / 115%)

P10,000

Fee of Salas (P10,000 x 15%)


Profit share of Salas (P10,000 x 25%)

P 1,500
_2,500

Total

P 4,000

6-10: b

6-11: d

6-12: b

Salas

Salve

Balance before profit distribution


Profit share:Sabas (P10,000 x 40%)
Salve (P10,000 x 35%)

P 500 (dr) P 2,000 (cr)


4,000
______
_3,500

Balance

P 3,500 (cr) P 5,500 (cr)

Joint venture account balance before profit distribution (debit)


Joint venture profit (P4,500 x 3)

P 6,000
_13,500

Cost of unsold merchandise (inventory) taken by Dante

P19,500

Edwin Capital:
Debits: Balance before profit distribution
Credits: Profit share

P14,000
__4,500

Due from Edwin (debit balance)

P 9,500

Joint Venture

105

Settlement to Ferdie (Balance of capital account)


Debits:
Credits: Balance before profit distribution
Profit share

P16,000
__4,500

Due to Ferdie (credit balance)


P30,000
__9,500

Balance

P19,000
P 4,600
__2,000

Joint venture profit before fee to Jerry


Joint venture profit after fee (P6,600 / 110%)
Fee to Jerry

P 6,600
__6,000
P 600

Harry Capital
Balances before profit distribution
Profit distribution:
Harry P6,000 x 50%)
Isaac (P6,000 x 20%)

(P 200)

Cash settlements

P 2,800

Isaac Capital
P 1,800

3,000

1,200
P 3,000

Sales
Cost of sales:
Merchandise inventory, beg (contributions)
Freight
Purchases

P14,000
300
__4,000

Goods available for sale


Merchandise inventory, end (P8,300/2)

P18,300
__4,150

Gross profit (loss)


Expenses (P400 + P200)
6-16: c

P39,500
_20,500

JV account balance before profit distribution (cr)


Unsold merchandise (required dr balance after profit distribution)

6-14: d

6-15: b

_20,500
P20,500

Settlement to Dante (balance of JV Cash account)


Debits: Balance before cash settlement
Due from Edwin
Credits: Due to Ferdie

6-13: a

P 0

Net profit (loss)

P14,000

14,150
(150)
__600
P( 750)

Contributions to the Joint Venture (P5,000 + P8,000)


Loss share (P750 x 50%)
Unsold merchandise taken (withdrawal)

P13,000
( 375)
( 4,150)

Final settlement to jack

P 8,475

106

Chapter 6

SOLUTIONS TO PROBLEMS
Problem 6 1
Books of Blanco (Manager)
JV Cash
Joint Venture
Cash
Ablan Capital

100,000
90,000

Joint Venture
JV cash

60,000

Joint Venture
JV cash

20,000

JV cash
Joint Venture

200,000

Books of Ablan

100,000
90,000

Investment in JV
Merchandise inventory

90,000

Investment in JV
Profit from JV

15,000

90,000

60,000
20,000
200,000

Computation of JV Profit
Total debit to JV
Total credit to JV

P170,000
P200,000

Credit balance (Profit)

P 30,000

Distribution
Joint Venture
Profit from JV
Ablan capital
Ablan capital
JV cash
Cash
JV cash

30,000

105,000
155,000

15,000
15,000
105,000
155,000

Cash
Investment in JV

105,000

15,000

105,000

Joint Venture

107

Problem 6 2
Books of the Joint Venture
1.

Computer equipment
Ella capital
Fabia capital

2.

Purchases
Supplies
Diaz capital

80,000
2,000

3.

Expenses
Diaz capital

9,000

4.

Cash

5.

Expenses
Cash

30,000

6.

Merchandise inventory
Ella capital

20,000

7.

Fabia capital
Cash

10,000

8.

Adjusting and closing entries:

105,000

150,000

Sales

(a)

Expenses
Supplies

(b)

Sales

Income summary

500
150,000

Income summary
Merchandise inventory
Purchases

77,500
2,500

Income summary
Expenses

39,500

Distribution of profit:
Income summary
Diaz capital
Ella capital
Fabia capital

33,000

60,000
45,000

82,000
9,000
150,000
30,000
20,000
10,000

500
150,000

80,000
39,500

11,000
11,000
11,000

108

Chapter 6

Books of Diaz
(1)

Investment in Joint Venture


Cash

82,000

(2)

Investment in Joint Venture


Cash

9,000

(3)

To record profit share:


Investment in Joint Venture
Profit from Joint Venture

11,000

82,000
9,000

11,000

Books of Ella:
(1)

Investment in Joint Venture


Computer equipment

60,000

(2)

Investment in Joint Venture


Merchandise inventory

20,000

(3)

To record profit share:


Investment in Joint Venture
Profit from Joint Venture

11,000

60,000
20,000

11,000

Books of Fabia:
(1)

Investment in Joint Venture


Computer equipment

45,000

(2)

Cash

10,000

(3)

To record profit share:

Investment in Joint Venture

Investment in Joint Venture


Profit from Joint Venture

11,000

45,000
10,000

11,000

Joint Venture

109

Problem 6 3
(1)

No Separate Set of Joint Venture Books is Used

Books of Duran (Manager)


May 1:

Joint Venture
Castro capital
Cash

12,500

7:

JV cash
Bueno capital

10,000

26: Joint Venture


JV cash

9,500

30: JV accounts receivable


Joint Venture

16,000

June 30: JV cash


JV accounts receivable

15,000

27: JV cash
Joint Venture

9,000

12,000
500
10,000
9,500
16,000
15,000
9,000

30: To record unsold merchandise taken by Duran:


Merchandise inventory
Joint Venture

3,000

3,000

To record profit distribution:


Joint Venture
Profit from JV
Bueno capital
Castro capital

6,000

2,000
2,000
2,000

To record settlements:
Bueno capital
Castro capital
JV cash
Cash
Accounts receivable
JV accounts receivable

12,000
14,000

1,000

24,500
1,500
1,000

110

Chapter 6

Books of Bueno
May 7:

Investment in Joint Venture


Cash

June 30: Investment in Joint Venture


Profit from Joint Venture
Cash

Investment in Joint Venture

10,000
2,000
12,000

10,000
2,000
12,000

Books of Castro
May 1:

Investment in Joint Venture


Merchandise inventory

June 30: Investment in Joint Venture


Profit from Joint Venture
Cash
(2)

Investment in Joint Venture

12,000
2,000
14,000

12,000
2,000
14,000

A Separate Set of Books is used:

Books of the Joint Venture


May 1:

7:

Merchandise inventory
Castro capital
Duran capital

12,500

Cash

10,000

Bueno capital

26: Purchases
Cash
30: Accounts receivable
Sales
June 20: Cash
27: Cash

Accounts receivable
Sales

9,500
16,000
15,000
9,000

12,000
500
10,000
9,500
16,000
15,000
9,000

Joint Venture

111

June 30: Closing entries:


Sales

Income summary

Income summary
Merchandise inventory, end
Merchandise inventory
Purchases

25,000
19,000
3,000

25,000

12,500
9,500

Distribution of profit:
Income summary
Bueno capital
Castro capital
Duran capital

6,000

2,000
2,000
2,000

Settlements to Venturers:
Bueno capital
Castro capital
Duran capital
Merchandise inventory
Accounts receivable
Cash

12,000
14,000
2,500

3,000
1,000
24,500

Books of Duran (Manager/Operator)


May 1:

Investment in Joint Venture


Cash

June 30: Investment in Joint Venture


Profit from Joint Venture
Cash

Investment in Joint Venture

Books of Bueno and Castro (Same as in No. 1 requirement)

500
2,000
2,500

500
2,000
2,500

112

Chapter 6

Problem 6 4
(1)

Books of Seiko (Manager/Operator)

April1:

JV Cash
Notes payable PNB
Roles capital
Timex capital

May:

Joint venture
Cash
Rolex capital

64,100

June:

Rolex capital
JV cash

30,000

Joint venture
Cash
Rolex capital
Timex capital
July:

August:

102,000

111,400

Cash
Rolex capital
Timex capital
JV cash

40,000
15,000
10,000

Joint venture
Cash
Rolex capital
Timex capital

55,770

Cash
Rolex capital
Timex capital
JV cash

45,000
67,000
13,500

Joint venture
Cash
Rolex capital
Timex capital

30,600

34,000
34,000
34,000
16,300
7,800
30,000
37,400
64,700
9,300

65,000
13,970
31,240
10,560

125,500
9,730
16,560
4,310

To record sales:
JV cash (P421,000 x 96%)
Joint venture

404,160

404,160

Joint Venture

113

To record payment of loan to PNB:


Notes payable PNB
Rolex capital
Timex capital
Joint venture (Interest expense)
JV cash

34,000
34,000
34,000
8,000

110,000

To record distribution of profit:


Joint venture
Gain from JV (30%)
Rolex capital (60%)
Timex capital (10%)

134,290

40,287
80,574
13,429

Computed as follows:
Total debits tot he JV account
Total credits to the JV account

P269,870
_404,160

Gain (credit balance)

P134,290

To record settlement:
Cash
Rolex capital
Times capital
JV cash

32,687
128,874
14,099

175,660

Computations:
Settlement to Rolex - Balance of capital account:
Debits: June
July
August
Payment of note payable

P30,000
15,000
67,000
_34,000

P146,000

Credits: April 1
May
June
July
August
Profit share

P34,000
47,800
64,700
31,240
16,560
_80,574

__274,874

Credit balance

P 128,874

114

Chapter 6

Settlement to timex Balance of capital account


Debits: July
August
Payment of loan

P 10,000
13,500
__34,000

P 57,500

Credits: April 1
June
July
August
Profit share

P 34,000
9,300
10,560
4,310
__13,429

_71,599

Credit balance

P 14,099

Settlement to Seiko Balance of JV cash account


Debits: April 1
Loan proceeds

P102,000
_404,160

P506,160

Credits: June
July
August
Payment of loan

P 30,000
65,000
125,500
_110,000

_330,500

Balance of JV cash
Less:Settlement to Rolex
Settlement to Timex

P128,874
__14,099

Settlement to Seiko
(2)

175,660
_142,973
P 32,687

Partial Balance Sheet


June 30, 2008
Books of Seiko (Manager/operator)
Current assets:
Investment in joint Venture:
Joint Venture assets:
Cash
Joint Venture
Less:Equity of other venturers
(P116,500 + P43,300)
Current liabilities:
Notes payable PNB

P 72,000
_175,500

P247,500
_159,800

87,700
34,000

Joint Venture

115

Computation of balances as of June 30, 2008:


JV Cash
April 1 P102,000
Balance P 72,000

P30,000

Joint Venture
June

May
June

P 64,100
_111,400

Balance P175,500
Notes Payable
P34,000

Rolex capital
April

June

P 30,000
_______

P 34,000
47,800
__64,700

P 30,000

P146,500

April 1
May
June

P116,500
Timex capital
P34,000
__9,000

April
June

P43,300
Problem 6 5
Consolidated Balance Sheet
Cash
Receivables
Inventory
Other assets

P 61,000
122,000
102,500
__40,500

Total assets

P326,000

Accounts payable
Other liabilities
Capital stock
Retained earnings

P 61,000
96,500
50,000
_118,500

Total liabilities and stockholders' equity

P326,000

Consolidated Income Statement


Sales
Cost of sales

P246,750
_124,750

Gross profit
Operating expenses

122,000
__58,250

Consolidated net income

P 63,750

116

Chapter 6

Problem 6 6
(a)

Journal entries on venture books


June 15:

Cash

1,000,000

July 1:

Land

2,400,000

Aug 1:

Cash

1,100,000

Land

950,000

Mortgage payable
Interest expense- Mortgage
Cash
Reduced mortgage and paid interest.

250,000
3,750

Mortgage payable
Interest expense- Mortgage
Cash
Reduced mortgage and paid interest.

400,000
8,000

Mortgage payable
Interest expense- Mortgage
Cash
Reduced mortgage and paid interest.

300,000
7,500

MacDo
Initial contribution at 6%

Mortgage payable
Cash
Purchased land for cash and 6% mortgage.
MacDo
Additional contribution at 6%.
Cash
Paid for improvements.

Sept 30:

Oct 31:

Nov 30:

Dec 31:

Mortgage payable
200,000
Interest expense- Mortgage
21,000
Cash
Reduced mortgage and make semi-annual
interest payment.

1,000,000

1,650,000
750,000

1,100,000

950,000

253,750

408,000

307,500

221,000

Joint Venture

(b)

117

31:

Cash

31:

Commissions
Cash
P2,600,000 x 5%

130,000

31:

Expenses
Cash
Paid expenses

628,100

31:

Interest expense- Venturer


MacDo
6% on P1,000,000 from June 15 to
December 31, and on P1,100,000
from August 1 to December 31.

31:

Sales

31:

Income summary
MacDo
MacEn
To divide gain, 60:40.

596,650

31:

MacDo

801,650

Sales
Sales to date.

Land (cost of land sold)


Expenses
Commissions
Interest expense- mortgage
Interest- venturer
Income summary
To close income and expense accounts.

Cash
Payment on account.

2,600,000

60,000

2,600,000

2,600,000

130,000

628,100

60,000

1,145,000
628,100
130,000
40,250
60,000
596,650

596,650
238,660

801,650

Journal entries on MacDos books:


June 15:

Investment in Joint Venture


Cash
Initial contribution.

1,000,000

Aug 1:

Investment in Joint Venture


Cash
Additional contribution.

1,100,000

1,000,000

1,100,000

118

Chapter 6

Dec 31:

(c)

Investment in Joint Venture


Interest income
Interest earned on cash advanced.

60,000

31:

Investment in Joint Venture


Gain on Joint Venture
60% of gain on venture.

357,990

31:

Cash

801,650

Investment in Joint Venture


Repayment in part of advances.

60,000

357,990

801,650

MacDo and MacEn Joint Venture


Income Statement
For the period from June 15 to December 31, 2008
Sales
Cost of land sold:
Land
Improvements
Total
Unsold land
Gross profit
Expenses:
Advertising and office expenses
Interest on mortgage
Interest on advances
Commissions
Net gain
Distributions:
MacDo (P596,650 x 60%)
MacEn (P596,650 x 40%)

P2,600,000
P2,400,000
950,000
P3,350,000
2,205,000
P 628,100
40,250
60,000
130,000

1,145,000
1,455,000

858,350
P 596,650
P 357,990
238,660

Mac Do and MacEn Joint Venture


Balance Sheet
December 31, 2008
Assets
Cash
Land
Total Assets

P 250,000
2,205,000
P2,455,000

Liabilities and equity:


Mortgage payable
MacDo
MacEn
Total liabilities and equity

P 500,000
1,716,340
238,660
P2,455,000

Joint Venture

Venturers equity (interest)


Invested
Shares:
Gain
Interest on advances
Commissions
Total
Balances
Withdrawn
Equity (interests)

119

MacDo
P2,100,000

MacEn

Total
P2,100,000

P 357,990
60,000

P238,660

P 596,650
60,000
130,000
786,650
2,886,650
(931,650)
P1,955,000

417,990
2,517,990
(801,650)
P1,716,340

130,000
368,660
368,660
(130,000)
P238,660

120

Chapter 7

CHAPTER 7
MULTIPLE CHOICE ANSWERS AND SOLUTIONS
7-1: c

7-2: d

7-3: d
7-4: a

7-5: c

7-6: a

Amount realized secured by inventory


Unsecured claim (P10,000 x 25%)

P 30,000
__2,500

Total amount received

P 32,500

Amount realized secured by inventory


Unsecured claim (P88,000 x 75%)

P120,000
__66,000

Total amount received

P186,000

(P15,000,000 + P200,000)
Realizable value:
Current assets
Land and building
Less mortgage payable

P240,000
_200,000

P 50,000
__40,000

Total
Less accounts payable

90,000
_160,000

Estimated deficiency to unsecured creditors

P 70,000

Total realizable value to unsecured creditors (P90,000)/total unsecured


Claims (P160,000) = 56.25%
Free assets:
Current assets
Buildings and equipment
Total
Liabilities with priority:
Administrative expenses
Salary payable
Income taxes
Total

P 33,000
_110,000
P143,000
P 20,000
6,000
__8,000
P 34,000

Corporation in Financial Difficulty Liquidation

Free assets after payment of liabilities with priority:


(P143,000 P34,000)
Unsecured liabilities
Notes payable
Accounts payable
Bonds payable
Total

121

P109,000
P 30,000
83,000
__70,000
P183,000

Percentage of Unsecured liabilities to be paid: P109,000 / P183,000 = 60%


Payment of notes payable:
Value of security (land)
60% of remaining P30,000
Total collected
7-7: c

Free assets:
Other assets
Excess from assets pledged with secured
Creditors (P116,000 P70,000)
Total
Liabilities with priority
Free assets after payment of liabilities with priority
(P126,000 P42,000)
Unsecured liabilities:
Excess of partially secured liabilities over pledge
Assets (P130,000 P50,000)
Unsecured creditors
Total

P 90,000
__18,000
P108,000

P 80,000
__46,000
P126,000
P 42,000
P 84,000
P 80,000
_200,000
P280,000

Recovery percentage: P84,000 / P280,000 = 30%


Payment of partially secured debt:
Value of pledged assets
30% of remaining P80,000
Total collected

P 50,000
__24,000
P 74,000

122

7-8: a

Chapter 7

The holder of Debt Two will receive P100,000 from the sale of the pledged
asset. Since the holder wants to receive P142,000 out of the total debt of
P170,000, the company must be able to generate enough cash to pay off
60% of the unsecured liabilities (P42,000/P70,000) after paying 100% of
the liabilities with priority (P110,000).
Unsecured liabilities:
Unsecured creditors
Excess liability of Debt One in excess of pledged
Asset (P210,000 P180,000)
Excess liability of Debt Two in excess of pledged
Asset (P170,000 P100,000)

P230,000
30,000
__70,000

Total unsecured liabilities


Necessary percentage

P330,000
____60%

Cash needed for these liabilities

P198,000

In order for the holder of Debt Two to received exactly P142,000, the other free assets
must be sold for P308,000. With that much money, the liabilities with priority
(P110,000) can be paid with the remaining P198,000 going to the unsecured debts of
P330,000. This 60% figure would insure that the holder of Debt Two would get
P100,000 from the pledged asset and P42,000 (P70,000 x 60%) from the free assets.
7-9: c

Estate equity, beg. (P100,000 P85,000)


Loss on realization (P100,000 P75,000)
Unrecorded liabilities:
Interest expense
Administrative expense
Estate deficit

7-10: c

P 15,000
( 25,000)
P

250
4,000

4,250)

P( 14,250)

Total assets at net realizable value


Fully secured liabilities
Estimated administrative expense

P 75,000
(40,000)
_( 4,000)

Estimated amount available


Unsecured claims (P45,000 + P250)

P 31,000
(45,250)

Estimated deficiency to unsecured creditors

P 14,250

Corporation in Financial Difficulty Liquidation

7-11: b

7-12: b

7-13: b

7-14: a
7-15: a
7-16: b
7-17: d
7-18: d

123

Assets pledged with fully secured creditors


Fully secured creditors
Free assets
Total free assets
Less: Liabilities with priority
Available to unsecured non-priority claims

P185,000
_130,000

Machinery
Recoveries of unsecured claims (50,000 - 10,000) X .50
Amount to be realized
Notes Payable
Less: Inventories
Unsecured Liabilities
% of recovery
Recovery
Add: Inventories
Amount to be received by Wood

55,000
_160,000
215,000
__35,000
P180,000
P 10,000
__20,000
P 30,000

P 23,940
19,200
4,740
____78%
3,697
_19,200
P 22,897

- P7,000
- P30,000
- P57,200 [52,000 + (8,000 X .65)]
- P72,800 (112,000 X .65)
Estimated loss:
Account Receivable
Inventories (28,000 - 18,500)
Building (59,000 - 22,000)
Equipment (5,600 - 2,000)
Goodwill
Prepaid expenses
Less: Stockholder's equity
Common stock
Deficit
Estimated deficiency

P 8,160
9,500
7,000
3,600
5,650
___430
P 72,000
( 16,660)

P 64,340
_55,340
P 9,000

124

7-19: d

Chapter 7

Accounts Receivable (39,350 - 16, 110)


Notes Receivable (18,500 - 12,500)
Inventories (87,850 - 45,100)
Prepaid expenses
Equipment (48,800 - 9,000)
Total estimated loss

P 23,240
600
42,750
950
__39,800
P112,740

7-20: b P33,750 (95,000 - 61,250) on Land and Building


7-21: d

7-22: d

Total Free Assets:


Balance of Assets Pledged to
Fully Secured Creditor (95,000 - 90,000)
Free Assets:
Cash
Accounts Receivable
Inventories
Equipment
Total
Less: Unsecured liabilities with priority (1,850 + 4,650)
Net Free Assets
Divide by Unsecured creditors:
Balance of Partially Secured Creditor
Notes Payable - PNB
P 15,000
Notes Receivable
__12,500
Accounts Payable
52,500
Notes Payable
__51,250
Estimated recovery %
Fully secured (Notes Payable)
Partially secured:
Notes Payable - PNB
Add (2,500 X 67%)
Unsecured Creditor with Priority
Unsecured Creditor without Priority (103,750 X 67%)
Total

P 5,000
P 2,700
16,110
45,100
__9,000

__72,910
77,910
___6,500
P 71,410

2,500
103,750 P106,250
67%
P 90,000
P12,500
__1,675

14,175
6,500
__69,513
P180,188

Corporation in Financial Difficulty Liquidation

7-23: a

7-24: a

7-25: b

7-26: a

7-27: a

7-28: a

Unsecured creditors without priority


Estimated deficiency to unsecured creditors:
Loss on realization
Estimated liquidation expenses
Total
Stockholders equity
Net free assets
Liabilities with priority
Free assets
Estimated net gain (loss) on realization:
Gain on realization
Loss on realization
Estimated claims
Total
Stockholders equity
Estimated deficiency

125

P1,102,500
551,250
55,125
606,375
441,000

78,750
(336,700)

Notes payable (175,000 140,000)


Unsecured liabilities (420,000 52,500)
Total
Free assets (157,500 + 210,000)
Estimated deficiency

165,375
937,125
122,500
P 1,059,625

(257,950)
( 43,750)
(301,700)
295,750
P( 5,950)
P 35,000
367,500
402,500
367,500
35,000

Old receivable (net)


Marketable securities
Old inventory
Depreciable assets- net
Total assets to be realized

P 38,000
12,000
60,000
96,000
P206,000

Old receivable
New receivable
Marketable securities
Sales of inventory
Total asset realized

P 21,000
47,000
10,500
75,000
P153,500

Gain on sale of inventory (P75,000 60,000)


Loss on realization:
Marketable securities (12,000 10,500)
Trustees expenses
Depreciation
Net loss

15,000
1,500
4,300
16,000

(21,800)
P( 6,800)

126

(A)

SOLUTIONS TO PROBLEMS
Laguna Company
Statement of Affairs
October 31, 2008

Problem 7 1

Book
Value

Estimated
Assets
Realizable Value
Assets pledge for fully secured creditors:
P107,000 ... Plant assets .................................................. P67,400
Less; Fully secured liabilities...................... _ 50,400
Assets pledged for partially secured creditors:
39,000 . ... Inventories................................................... P18,000
4,000 .. ...
46,000 .. ...
2,000 .. ...

P198,000
Book
Value

Chapter 7

Free Assets:
Cash............................................................. P 4,000
Accounts, receivable ...................................
46,000
Supplies....................................................... __1,500
Total free assets...............................................
Less: Unsecured liabilities with priority..........
Net Free Assets................................................
Estimated deficiency to unsecured creditors (to balance)

Creditors'
Liabilities & Stockholders' Equity
Claim
Fully secured liabilities:
P50,400... ... Mortgage payable (including interest, P400) P50,400
Partially secured liabilities:
21,000... ... Notes payable..............................................
P21,000
Less: Inventory............................................
_18,000
Unsecured creditors with priority:
5,800... ... Wages payable
P 5,800
1,200... ... Property taxes payable ................................
_1,200
Total ............................................................
P 7,000
Unsecured creditors without priority:
60,000... ... Accounts payable ........................................
19,000... ... Notes payable..............................................
Stockholders' Equity........................................
P198,000
(B)
Creditor Group
Amount of
Claim
Unsecured liabilities with priority ....................................
P7,000
Fully secured creditors......................................................
50,400
Partially secured creditors.................................................
21,000
Unsecured creditors without priority ................................
79,000
* P18,000 + (P3,000 X 0.75) = P20,250
(C) See statement of affairs in requirement (A)

Free Assets
P17,000

_51,500
P68,500
__7,000
P61,500
_20,500
P82,000
Unsecured
Liabilities

P 3,000

60,000
19,000
_____
P82,000
Amount to
be Paid
P7,000
50,400
20,250 *
59,250

Percentage
to be paid
100.0%
100.0%
96.4%
75.0%

Corporation in Financial Difficulty Liquidation

127

Problem 7 2

VC Corporation
Statement of Realization and Liquidation
Month Ended January 31, 2008
Assets to be realized:
Land ....................... P10,000
Building ................. 43,000
Equipment.............. 28,000
Patents.................... __4,400
Assets Acquired..............

P85,400
0

Liabilities Liquidated:
Account payable .... P14,000
Loans payable ........ __7,000

21,000

Liabilities not Liquidated:


Accounts payable... 66,000
Loans payable ........ 33,000

99,000

Gain on realization ......... ............... ___7,600


Total ............................... ............... P213,000

Assets realized:
land.............................. P
0
Building ......................
0
Equipment ...................
8,800
Patents......................... _12,000
Assets not realized:
Land ............................ P10,000
Building ...................... 43,000
Equipment ................... _13,000

P20,800

66,000

Liabilities to be Liquidated:
Accounts payable ........ P80,000
Loans payable ............. _40,000

120,000

Loss on realization ...... ..............


Total ............................ ..............

___6,200
P213,000

Accounts payable .........................


Loans payable ..............................
Estate deficit.................................

P 66,000
33,000
( 26,300)

VC Corporation
Balance Sheet
January 31, 2008
Cash ............................................... P 6,700
Land ...............................................
10,000
Building..........................................
43,000
Equipment ...................................... _13,000
Total ............................................... P 72,700

P 72,700

VC Corporation
Estate Deficit
January 31, 2008
Gain on realization ....................................................................
Loss in realization ....................................................................
Trustee's expenses ....................................................................
Net gain on realization...............................................................
Estate deficit, January 1, 2008...................................................
Estate deficit, January 31, 2008.................................................

P 7,600
( 6,200)
( 1,300)
P 100
( 26,400)
P(26,300)

128

Chapter 7

Problem 7 3

Rizal Corporation
Statement of Affairs
Book
Values
Assets
Assets pledged to fully secured creditors:
P 80,000 ...... .... Land and building ..............................................
Less: Mortgage payable .....................................
50,000 ...... .... Finished Goods ..................................................
Less: Loan payable.............................................
32,000 ...... ....
12,000 ...... ....

Assets pledged to partially secured creditors:


Accounts receivable (80% x 30,000) .................
Trucks ................................................................
Totals..................................................................

Free Assets:
Cash....................................................................
AR (20% x 30,000) ............................................
Inventory Materials .........................................
Prepaid expense..................................................
Trucks ................................................................
Equipment ..........................................................
Intangible ...........................................................
Total Free Assets ....................................................
Less: Unsecured liability with priority (12,000 + 8,000)
Net free assets .........................................................
________
Estimated deficiency to unsecured creditors (to Balance)
P 292,000 ...... ....Total unsecured liabilities .......................................
4,000 ...... ....
8,000 ...... ....
36,000 ...... ....
1,000 ...... ....
8,000 ...... ....
45,000 ...... ....
16,000 ...... ....

Book
Values

Liabilities and Equity


Fully secured creditors:
P 43,000 ...... .... Mortgage payable...............................................
50,000 ...... .... Loans payable ....................................................
Total ...................................................................

Estimated
Realizable Value
P102,000
43,000
P 55,000
50,000

4,000
6,000
27,000
0
2,500
25,000
_______

Creditors'
Claim

Unsecured creditors with Priority:


Wages payable ...................................................
Taxes payable.....................................................
Totals..................................................................

12,000
8,000
20,000

Unsecured creditors:
77,000 ...... .... Accounts payable ...............................................
110,000 ...... .... Stockholder Loan ...............................................
( 38,000) ...... ....Stockholder Equity .................................................
P 292,000
Total........................................................................

5,000

64,500
P128,500
20,000
108,500
81,000
P189,500
Unsecured
Liabilities

94,000
50,000
144,000
25,000
24,000
5,000
3,500

12,000 ...... ....


8,000 ...... ....

P 59,000

24,000
3,500
27,500

Partially secured creditors':


Bank Loan ..........................................................
Less: Receivable (80% x 30,000) .......................
5,000 ...... .... Truck Loan.........................................................
Less: trucks ........................................................

25,000 ...... ....

Free
Assets

77,000
110,000

P 1,000
1,500

187,000

P189,500

Corporation in Financial Difficulty Liquidation

129

Problem 7 4
Mapayapa Corporation
Statement of Affairs
November 1
Book
Value

Assets
Assets pledged to fully secured creditors:
P60,000.... ... Investments .................................................
180,000.... ... Accounts receivable ....................................
Total ............................................................
Less: Note payable......................................
66,000.... ...
248,000.... ...
291,000.... ...
870,000.... ...
114,000.... ...
.... ...

_________
P1,839,000

Free
Assets

P 69,000
171,000
240,000
210,000

P 30,000

Free assets:
Cash............................................................. P 66,000
Accounts receivable .................................... 193,500
Merchandise inventory................................ 180,000
Plant & equipment ...................................... 330,000
Notes receivable.......................................... 108,300
Patent........................................................... __12,000
Total free assets...........................................
Less: Unsecured liabilities with priority..........
Net free asset...............................................
Estimated deficiency (to balance) ...................
Total ................................................................

_889,800
919,800
__13,800
906,000
60,300
P966,300

Creditor's
Claim

Unsecured
Liabilities

Book
Value

Liabilities & Equity


Fully secured creditors:
P 210,000.... ... Notes payable..............................................
Unsecured creditor with priority:
Accrued wages ............................................
Accrued property tax...................................
Total ............................................................
Unsecured creditor:
Account payable..........................................
Accrued expenses........................................
300,000.... ... Capital stock
__369,000.... ... Retained earnings ............................................
P1,839,000
Total ................................................................
960,000.... ...

Estimated
Realizable Value

P210,000
P 7,200
___6,600
P 13,800
P960,000
6,300
_______
P966,300

130

Chapter 7

Problem 7 5
a.

b.

Total fair value of assets (estimated proceeds) ..........................


Less:Fully and partially secured creditors claim:
Notes payable, interest (secured by receivable and
inventory) ................................................................... 125,000
Bonds payable (secured by land & building).................... 231,000
Available to unsecured creditors................................................
Less:Unsecured creditors with priority:
Wages payable ..................................................................P 9,500
Taxes payable....................................................................__14,000
Amount available to unsecured creditors...................................

P471,000

__23,500
P 91,500

Unsecured portion of notes payable and interests (P195-P125)


Accounts payable.......................................................................
Total claims of unsecured creditors ...........................................

P 70,000
__95,000
P165,000

356,000
115,000

P91,500
= 55.45%
P165,000
c.

Distribution of P471,000:
Creditors
Accounts payable
Wages payable
Taxes payable
Notes payable & interests

Amount
P 95,000....
9,500 ....
14,000.....
125,000 ....
70,000
Bonds payable & interests
231,000 ....
Total estimated payment........................................

Percent
Realized
55.45%
100%
100%
100%
55.45%
100%

Total
Payment
P 52,678
9,500
14,000
125,000
38,815
_231,000
P470,993

Corporation in Financial Difficulty Liquidation

1.

Evergreen Company
Statement of Affairs
June 30, 2008

Book
Values
P460,000
80,000
140,000
100,000
120,000
100,000

131

Problem 7 6

Estimated
Realizable
Values

ASSETS
Pledged with fully secured creditors:
Land and building.....................................
P340,000
Less: Mortgage payable (including accrued interest)
(330,000)
Free Assets:
Cash .........................................................
P 80,000
Accounts receivable net.........................
126,000
Inventories ................................................
84,000
Machinery net ........................................
40,000
Goodwill ...................................................
_ _____0_

Available for
Unsecured
Creditors
P 10,000

330,000

Total free assets ........................................ ...................


Less: liabilities with priority..................... ...................

340,000
_140,000

Net free assets .......................................... ...................


Estimated deficiency (Squeeze figure) ..... ...................

200,000
_130,000

P1,000,000

P330,000
LIABILITIES AND STOCKHOLDERS' EQUITY

Secured &
Priority
Claims

P120,000
20,000

Liabilities with priority


Wages payable ..........................................
Property taxes payable..............................

300,000
30,000

Total .........................................................
Fully secured creditors
Mortgage payable .....................................
Interest on mortgage payable....................

220,000
100,000
10,000

Total .........................................................
P330,000
Unsecured creditors
Accounts payable...................................... ...................
Note payable-unsecured............................ ...................
Interest payable-unsecured ....................... ...................

Stockholders' Equity
400,000 Capital stock .............................................
(200,000) Retained earnings (deficit)........................ ...................

Unsecured
Non-priority
Liabilities

P120,000
__20,000
P140,000
300,000
__30,000

P220,000
100,000
10,000
___
P330,000

P1,000,000
2.

Settlement per peso of unsecured creditors is P.6250 (P200,000/P320,000). No payment is


made for the P10,000 unsecured interest claim.

Chapter 7

132 ____
Problem 7 7
1.

Entries on trustee's books.


2008
March 1:
Cash ............................................... ........P8,000
Accounts receivable net .............. ........16,000
Inventories ..................................... ........72,000
Land ............................................... ........40,000
Buildings net ............................... ......200,000
Intangible assets............................. ........52,000
Accounts payable .................... ...................
Note payable............................ ...................
Deferred revenue ..................... ...................
Wages payable......................... ...................
Mortgage payable.................... ...................
Estate equity ............................ ...................
To record custody of Kimerald Corporation.
March 1 to 31:Cash ............................................... ........15,200
Estate equity................................... .............800
Accounts receivable-net .......... ...................
To record collection of receivables and recognize loss.
Cash ............................................... ........38,800
Estate equity................................... ........33,200
Inventories............................... ...................
To record sale of inventories at a loss.
Cash ............................................... ......180,000
Estate equity................................... ........60,000
Land......................................... ...................
Buildings-net ........................... ...................
To record sale of land and buildings at a loss.
Estate equity................................... ........52,000
Intangible assets ...................... ...................
To write off intangible assets.
Estate equity ........................................ ......... 16,400
Administrative expenses payable . ....................

To accrue trustee expenses.

P100,000
80,000
2,000
6,000
160,000
40,000

16,000

72,000

40,000
200,000

52,000

16,400

Corporation in Financial Difficulty Liquidation

2.

133

Financial Statements
Kimerald Corporation in Trusteeship
Balance Sheet
March 31, 2008
Assets
Cash ..................... ................................................. ...................

P242,000

Liabilities and Deficit


Accounts payable. ................................................. ...................
Note payable-unsecured......................................... ...................
Revenue received in advance................................. ...................
Wages payable ..... ................................................. ...................
Mortgage payable ................................................. ...................
Administrative expense payable-new .................... ...................

P100,000
80,000
2,000
6,000
160,000
__16,400

Total liabilities ..... ................................................. ...................


Less: Estate deficit................................................. ...................

P364,400
_122,400

Total liabilities net of deficit.................................. ...................

P242,000

Kimerald Corporation in Trusteeship


Statement of Cash Receipts and Disbursements
March 1 to 31, 2008
Cash balance, March 1, 2008................................. ...................
Add: Cash receipts
Collections of receivables ............................. .....P 15,200
Sale of inventories......................................... ........38,800
Sale of land and buildings............................. ......180,000

P 8,000

Total..................... ................................................. ...................


Less: Cash disbursements ...................................... ...................

242,000
____0

Cash balance, March 31, 2008............................... ...................

P242,000

_234,000

Kimerald Corporation in Trusteeship


Statement of Changes in Estate Equity
March 1 to 31, 2008
Estate equity, March 1 ........................................... ...................
Less:Loss on uncollectible receivables.................. .......P 800
Loss on sale of inventories............................ ........33,200
Loss on sale of land and buildings ................ ........60,000
Loss on write off of intangibles .................... ........52,000
Administrative expenses ............................... ......_16,400

P 40,000

Estate deficit, March 31......................................... ...................

P122,400

_162,400

134

3.

Chapter 7

Entries on trustee's books:


2008
April: Mortgage payable ..................................... ......160,000
Cash.................................................. ...................
To record payment of secured creditors from
proceeds from sale of Land and buildings.
Administrative expenses payable-new...... ........16,400
Deferred revenue ...................................... ..........2,000
Wages payable .......................................... ..........6,000
Cash.................................................. ...................
To record payment of priority liabilities.
Accounts payable...................................... ........32,000
Note payable-unsecured............................ ........25,600
Cash.................................................. ...................
To record payment of P.32 per peso to unsecured
creditors (available Cash of P57,600 divided by
unsecured claims of P180,000).
Accounts payable...................................... ........68,000
Note payable-unsecured............................ ........54,400
Estate equity..................................... ...................
To write-off remaining liabilities and
close trustee's records.

160,000

24,400

57,600

122,400

CHAPTER 8
Reorganization and Troubled Debt Restructuring
MULTIPLE CHOICE ANSWERS AND SOLUTIONS
8-1: a

Trade accounts payable (P52,000 + P62,700)


12% preferred stock (5,000 x P1)
Paid in capital in excess of par (5,000 x P9)
Cash (P62,700 x P0.80)
Gain from discharge of indebtedness

P 5,000
45,000
_50,160

P114,700
_100,160
P 14,540

8-2: c
8-3: c
8-4: b

8-5: d

8-6: a

8-7: a

8-8: a

Carrying value of the note payable:


Principal
Interest
Restructured value:
Principal
Interest
Gain on debt restructuring

P600,000
__60,000
P400,000
_110,000
P150,000

Other income:
Fair value of land
Books value of land
Other income

P450,000
_360,000

Extraordinary gain:
Book value of note payable
Principal
Interest
Fair value of land
Extraordinary gain

P500,000
__60,000

Book value of bonds payable


Par value of preferred stock (5,000 shares x P100)
No gain no loss
Book value of notes payable:
Principal
Interest
Par value of common stock issued (200 shares x P5)
Additional paid in capital
Add gain on payment of accounts payable:
Book value
Payment
Total gain on debt discharge
Carrying value of debt:
Note payable
Interest payable
Fair value machinery
Balance of debt
Restructured debt:
Note payable

P660,000
_510,000

P 90,000

P560,000
_450,000
P110,000
P500,000
_500,000
P
0

P 2,500
___500

P 10,000
__8,000

P100,000
__12,000

P 50,000

P 3,000
__1,000
P 2,000
__2,000
P 4,000

P112,000
_(36,000)
P 76,000

Interest (P50,000 x .08 x 2)


Restructuring difference (gain)
8-9: d

8-10: c

___8,000

Principal
Interest payable (300,000 x 10%)
Carrying value

__58,000
P 18,000
P300,000
__30,000
P330,000

Should be P310,600
Restructured principal of note payable
Interest payable:
On book value (P300,000 x 10% 30%)
On restructured (P260,000 x 8% x 2)
Future cash flows to liquidate the debt

P260,000
P 9,000
_41,600

__50,600
P310,600

8-11: d
8-12: d

Loss on transfer of land:


Original cost
Market value
Gain on restructuring of debt:
Carrying value of debt
Market value of land

8-13: a

8-14: d

8-15: d

8-16: c

8-17: a

8-18: a

P290,000
_270,000
P300,000
_270,000

Transfer gain (loss):


Carrying amount of equipment
Fair value of equipment
Transfer loss

P80,000
75,000
P(5,000)

Restructuring gain:
Carrying amount of the debt
Fair value of equipment transferred
Restructuring gain

P100,000
75,000
P 25,000

Carrying amount of real estate transferred


Fair value of real estate
Loss on restructuring of payables

P100,000
90, 000
P(10,000)

Carrying amount of liability


Fair value of real estate transferred
Restructuring gain

150,000
90,000
P 60,000

Gain on revaluation of land (120,000 85,000)


Gain on the extinguishment of debt (185,000 120,000)
Total gain

P 35,000
65,000
P100,000

Carrying value of debt (P800,000 + 80,000)


Total future payments (P700,000 + 80,000)
Restructuring gain

P880,000
780,000
P100,000

First determine the expected future cash flows as follows:


70,000 x .79719 =
P55,803
5,600 x 1.69005 =
9,464

P 30,000

P 20,000

Present value of future cash flow

P65,267

The interest revenue can be computed using the effective interest method
as follows:
Present value at 12/31/06
P65,267
Interest income at 12/31/07 (65,267 x 12%)
7,832
Interest receivable at 12/31/07 (70,000 x 8%)
5,600
2,232
Present value at 12/31/07
P67,499
Interest income at 12/31/08 (67,499 x 12%)

P 8,100

SOLUTIONS TO PROBLEMS
Problem 8 1
Journal entries for com pany em erging from bankruptcy using fresh start accounting:

Receivables . ....... ...... .............................................................................................. 10,000


Inventory ..... ....... ...... .............................................................................................. 10,000
Building ..... ....... ...... . ... ........................................................................ 100,000
Reorganization value in excess of amount
Allocable to tangible assets ............................................................................... 60,000
Additional paid in capital ..........................................................................
180,000
To adjust accounts to market value as part of fresh start accounting. Since the company has a reorganization value of
P760,000 but the assets have a market value of only P700,000 (P90,000 + P210,000 + P400,000), and account entitled
Reorganization Value in Excess of Amount Allocable to Tangible Assets must be recorded for P60,000.
Liabilities .... ........ .....
300,000
Common stock (P330,000 x 80%).....................................................................
Gain on debt discharge ......................................................................................
To record settlement of liabilities.

2008
July 14:

264,000
36,000

Problem 8 2
Costs of reorganization...................................................................................... 50,000
Cash with escrow agent .............................................................................

50,000

Common stock .............................................................................................. 580,000


Common stock (60,000 x P1) ....................................................................
Additional paid in capital ..........................................................................

60,000
520,000

Note payable 10% ........................................................................................ 120,000


Interest payable (P120,000 x 10% x 3/12)......................................................... 3,000
Note payable 12%...................................................................................

123,000

Trade accounts payable .................................................................................... 100,000


Cash P100,000 x 0.80)...............................................................................
Gain on debt discharge ..............................................................................

80,000
20,000

Additional paid in capital ................................................................................. 290,000


Gain on debt discharge ...................................................................................... 20,000
Retained earnings ......................................................................................
260,000
Costs of reorganization..............................................................................
50,000

Jade Corporation
Balance Sheet
December 31, 2008

Problem 8 3

ASSETS
Current assets:
Cash .............. ..... ....... ...... ....................................................................P 23,000
Inventory.......... ..... ....... ...... ....................................................................__45,000 P 68,000
Property and equipment:
Land .............. ..... ....... ...... ....................................................................140,000
Buildings.......... ..... ....... ...... ....................................................................220,000
Equipment........ ..... ....... ...... ...................................................................._154,000_514,000
Total assets ...... ..... ....... ...... ....................................................................
P582,000
LIABILITIES AND STOCKHOLDERS' EQUITY
Liabilities not subject to compromise
Current liabilities:
Accounts payable... ....... ...... ....................................................................P 60,000
Long-term liabilities:
Note payable (2006)....... ...... P100,000
Note payable (2003)....... ...... _100,000..................................................... 200,000
Liabilities subject of compromise
Accounts payable... ....... ...... ....................................................................123,000
Accrued expenses .. ....... ...... .................................................................... 30,000
Income taxes payable..... ...... .................................................................... 22,000
Note payable (due 2008) ...... _170,000..................................................... 345,000
Total liabilities . ..... ....... ...... ....................................................................
Stockholders' Equity
Common stock . ..... ....... ...... ....................................................................200,000
Retained earnings (deficit).... .................................................................... (223,000)
Total liabilities and stockholders' equity (deficit) ......................................
Problem 8 4
Preliminary computations:
Book values prior to reorganization:
Total assets (P100,000 + P112,000 + P420,000 + P78,000)......................................
Total liabilities (P80,000 + p35,000 + P100,000 + P200,000 +
P185,000 + P200,000).......................................................................................
Common stock (given) ..............................................................................................
Deficit (given)
..............................................................................................
Book values after reorganization:
Total assets (reorganization value) ............................................................................
Total liabilities (P5,000 + P4,000 + P100,000 + P50,000 +
P71,000 + P110,000).........................................................................................
Common stock (returned shares are reissued) ...........................................................
Deficit (eliminated) ..............................................................................................
Additional paid in capital (squeeze) ..........................................................................

P260,000

605,000

(23,000)
P582,000

P710,000
P800,000
P240,000
P330,000
P780,000
P340,000
P240,000
0
P200,000

Since the company will have 30,000 shares outstanding after the reorganization, the additional paid in capital equals P6.66 per
share.
Because the company has a reorganization value of P780,000 but the assets have a market value of only P735,000, an account
entitled Reorganization Value in Excess of Amount allocable to Tangible Assets must be recognized for P45,000.
JOURNAL ENTRIES:
1.
Land and buildings
.............................................................................................. 80,000
Reorganization Value in excess of amount
allocable to tangible assets ................................................................................ 45,000
Accounts receivable...................................................................................
Inventory ..............................................................................................
Equipment ..............................................................................................
Additional paid in capital ..........................................................................
To adjust accounts to market value as part of fresh start accounting.

20,000
22,000
13,000
70,000

2.

Common stock .... ...... .............................................................................................. 144,000


Additional paid in capital ..................................................................................
144,000
To record shares turned in to the company by the owners as part of the reorganization plan. 18,000 shares at P8 par
value.

3.

Accounts payable ...... ............................................................................................ 80,000


Note payable ...... ..............................................................................................
Common stock, P8 par value.............................................................................
Additional paid in capital (P6.66 per share) ......................................................
Gain on debt discharge ......................................................................................
To record settlement of accounts payable.

5,000
8,000
6,666
60,334

4.

Accrued expenses ...... .............................................................................................. 35,000


Note payable ...... ..............................................................................................
Gain on debt discharge ......................................................................................
To record settlement of accrued expenses.

4,000
31,000

5.

Note payable ....... ...... .............................................................................................. 200,000


Note payable ...... ..............................................................................................
Common stock, P8 par value.............................................................................
Additional paid in capital (P6.66 per share) ......................................................
Gain on debt discharge ......................................................................................
To record settlement of note payable due in 2007

50,000
80,000
66,667
3,333

6.

Note payable ....... ...... .............................................................................................. 185,000


Note payable ...... ..............................................................................................
Common stock, P8 par value.............................................................................
Additional paid in capital, P6.66 per share........................................................
Gain on debt discharge ......................................................................................
To record settlement of note payable due in 2008

71,000
56,000
46,667
11,333

Problem 8 5
7.

Note payable ....... ...... .............................................................................................. 200,000


Note payable ...... ..............................................................................................
Gain on debt discharge ......................................................................................
To record settlement of note payable due in 2009

8.

Additional paid in capital (P334,000 P200,000)..................................................... 134,000


Gain on debt discharge .............................................................................................. 196,000
Retained earnings (deficit) ................................................................................
330,000
To adjust additional paid in capital to appropriate balance, close out gain, and eliminate deficit balance as part of fresh
start accounting.

110,000
90,000

Since the Company has a reorganization value of P800,000 but only P653,000 can be assigned to specific assets based on market
value, the remaining P147,000 is reported as a Reorganization Value in Excess of Amount Allocable to Identifiable Assets.
Sun Corporation
Balance Sheet Fresh Start Accounting
December 31, 2008
ASSETS
Current assets
Accounts receivable....... ...... .............................................................................................. P 18,000
Inventory ....... ..... ....... ...... .............................................................................................. _111,000
Property and equipment
Land and buildings ....... ...... .............................................................................................. 278,000
Machinery ....... ..... ....... ...... .............................................................................................. 121,000
Intangible assets
Patents
....... ..... ....... ...... .............................................................................................. 125,000
Reorganization value in excess of amount allocable To identifiable assets ......................... _147,000

P129,000
399,000
_272,000

Total assets ...... ..... ....... ...... ..............................................................................................

P800,000

LIABILITIES AND STOCKHOLDERS' EQUITY


Current liabilities
Accounts payable... ....... ...... .............................................................................................. P 97,000
Long-term liabilities
Note payable (due in 2 years) .............................................................................................. P 35,000
Note payable (due in 5 years) .............................................................................................. 50,000
Note payable (due in 8 years) .............................................................................................. 100,000
Total liabilities . ..... ....... ...... ..............................................................................................

_185,000
P282,000

Stockholders' Equity:
Common stock . ..... ....... ...... .............................................................................................. P500,000
Additional paid in capital (squeeze)..................................................................................... __18,000
Total liabilities and stockholders' equity..............................................................................

_518,000
P800,000

CHAPTER 9
Installment Sales
9-1: d

MULTIPLE CHOICE ANSWERS AND SOLUTIONS


Deferred gross profit, Dec. 31 (before adjustment)
Less:
Deferred gross profit, Dec. 31 (after adjustment)
Installment accounts receivable, Dec. 31
Gross profit rate
Realized gross profit, 2008
OR
Installment Sales (P1,050,000
Less: Installment account receivable, Dec. 31
Collection
Gross profit rate
Realized gross profit, 2008

9-2: a

9-3: c

9-4: b

9-5: a

9-6: a

Deferred gross profit, before adjustment


Deferred gross profit, end
2006 (6,000 X 35%)
2007 (61,500 X 33%)
2008 (195,000 X 30%)
Realized gross profit, December 31, 2008
(Total P107,235)

P1,050,000
P1,500,000
____

__375,000
P 675,000
P4,200,000
__1,500,00
P2,700,000
___X 25%
P 675,000

2006
P7,230
2,100
P5,130

2007
P 60,750
20,295
P 40,455

2008
P 120,150

___58,500
P 61,650

Deferred gross profit balance, end


Divide by Gross profit rate based on sales (25%
Installment Accounts Receivable, end
Collection
Installment Sales

P 202,000
____
P1,010,000
___440,000
P1,450,000

Sales
Cost of installment sales
Deferred gross profit
Less:
Deferred gross profit, end
Installment accounts receivables, 12/31
(1,000,000-400,000)
Gross profit rate (300,000
Realized gross profit
Operating expenses
Operating income
Interest and financing charges
Net income

P1,000,000
__700,000
P 300,000

Market value of repossessed merchandise


(before reconditioning cost)
Less: unrecovered cost
Unpaid balance (80,000-30,000)
Less: Deferred gross profit (50,000X20%)
Loss on repossession
Installment sales
Less: collection on installment sales
Installment account receivables, 12/31/08
Gross profit rate (500,000
Deferred gross profit, 12/31/08

P 600,000
___X 30%

__180,000
P 120,000
___80,000
40,000
__100,000
P 140,000
P 30,000

P 50,000
___10,000

__40,000
(P 10,000)
P1,000,000
__200,000
800,000
___X 50%
P 400,000

OR

9-7: d

9-8: b

Deferred gross profit (1,000,000-500,000)


Less: Realized Gross Profit (200,000 X 50%)
Deferred gross profit, 12/31/08

P500,000
_100,000
P400,000

Fair value of repossessed merchandise


Less:
unrecovered cost
Unpaid balance
Less: Deferred gross profit (200,000 X 32.5%)
Loss on repossession

P120,000

Realized gross profit:


Collections:
Downpayment
Installment received (205,000-200,000)
Total
Gross Profit Rate (150,000
Realized gross profit
Gain (loss) on repossession:
Appraised value of repossessed merchandise
Less: unrecovered cost
unpaid balance
less: deferred gross profit (200,000 X 62.5%)
Gain on repossession

9-9: b

P 200,000
___65,000

Sch.1
Date
Apr-1
Apr-1
May-1
Jun-1
Jul-1
Aug-1

Collection

P 35,000
___5,000
40,000
_X 62.5%
P 25,000
P165,000
P 200,000
__125,000

Applying
to
Interest

Applying
to
principal

125.00
115.00
104.80
__94.40
P439.20

750.00
500.00
510.00
520.20
___530.60
P2,810.80

750
625
625
625
625

Gain (loss) on repossession:


Market value of repossessed merchandise
Less: unrecovered cost
unpaid balance of principal (sch. 1)
less: deferred gross profit (4,189 X 35%)
Loss on repossession (rounded)

Deferred gross profit (Sales X Gross Profit Rate)


2007
(P300,000 X 30%)
2008
(P450,000 X 40%)
2007:
Accounts written-off (P25,000 X 30%)
Realized gross profit (P100,000 X 30%)

__75,000
P 90,000

Balance
of
principal
P7,000.00
6,250.00
5,750.00
5,240.00
4,719.80
4,189.00

P 1,875
P 4,189
__1,466

Realized gross profit:


Collection applying to principal (sch. 1)
Gross profit rate
Realized gross profit
9-10: c

_135,000
(P 15,000)

___2,723
(P 848)
P2,810.80
__X 35%
P 983.78

Year of Sales
2007
P 90,000
( 7,500)
( 30,000)

2008

P 180,000

2008:

Accounts written-off, 2007 (P75,000 X 30%)


Accounts written-off, 2008 (P50,000 X 40%)
Realized gross profit, 2007 (P50,000 X 30%)
Realized gross profit, 2008 (P150,000 X 40%)
Deferred gross profit, 12/31/08 (P75,000)
9-11: a

( 22,500)
( 15,000)
________
P 15,000

Deferred gross profit, 2007 (P1,050,000 - 735,000)


Realized gross profit, 2007 (P150,000 X 30%)
Deferred gross profit, 12/31/07
Realized gross profit, 2008 (P390,000-90,000) X 30%
Deferred gross profit, 12/31/08

2007
P 480,000
( 252,000)
( 180,000)
_______
P 48,000

Deferred gross profit (Sales - Cost of Installment Sales)


Realized gross profit, 2007 (P630,000 X 40%)
Realized gross profit, 2007 (P450,000 X 40%)
Realized gross profit, 2008 (P900,000 X 30%)
Deferred gross profit, 12/31/08 (P228,000)
Trade-in value
Less:
Actual value
Estimated selling price
Less: reconditioning cost
normal gross profit (25,000 X 15%)
Overallowance
Realized gross profit:
Collection:
Downpayment
Actual value of merchandise-Trade In
Installment collected (5,000 X 3)

9-15: a

P 1,250
__3,750

9-16: a

( 270,000)
P180,000

P 25,000
___5,000

P 5,000
20,000
_15,000

__20,000
P 10,000

P 40,000
P 85,000
( 10,000)
P 75,000
_60,000
P 15,000
___X 20%
P 8,000

Collection excluding interest (P900,000-P300,000)


Gross profit rate (P1,200,000
00,000)
Realized Gross Profit, December 31, 2008
Add Interests
Total Revenue
Wholesale value of repossessed merchandise
Less:
unrecovered cost
Unpaid balance:
Sales, 10/1/07
Collection, 2007 (6,000
Collection, 2008 (1,000 X 7)
Deferred gross profit (9,000 X 25%)
Loss on repossession

2008
P450,000

P 30,000

Gross Profit Rate:


Sales
Overallowance
Net Sales
Cost of Installment Sales
Gross Profit
Gross Profit Rate (15,000
Realized Gross Profit
9-14: c

( 60,000)
P 60,000
P 315,000
( 45,000)
270,000
( 90,000)
P 180,000

9-12: a

9-13: c

( 60,000)

P 600,000
X 33 1/3%
200,000
__300,000
P 500,000
P
P 24,000
( 8,000)
( 7,000)

P 9,000
__2,250

4,000

___6,750
(P 2,750)

Trade-in Value (P300 X 6)


Less:
Actual value
Estimated selling price (P315 X 6)
Less: Reconditioning cost (P25 X 6)
Gross Profit (P1,890 X 10%)
Over-allowance
9-17: a

9-18: d

9-19: b

Deferred gross profit, before adjustment


Deferred gross profit, end
2007: P32,500 X (30%
2008: P180,000 X (33 1/3%
Realized gross profit on installment sales

P
P150
_189

1,800

P 1,890
___339

___1,551
P
249
P 76,000

P 7,500
_45,000

__52,500
P 23,500

Unpaid balance (P27,000 - P16,000)


Multiply by gross profit rate (P734,400
Deferred gross profit to be cancelled on repossession

P 11,000
___X 34%
P 3,740

Collection:
2007
Downpayment
2008
Installment collection
Interest
Total

P 600,000
600,000
__540,000
P1,740,000

Cost to be recovered

P4,000,000

Since cost is not yet fully recovered, then no gross profit is to be recognized in 2008.
9-20: d

9-21: a

9-22: a

Regular Sales
Cost of regular sales
Gross profit on regular sales
Add: Realized gross profit on installment sales
2007 (25,000 X 50%)
2008 (62,500 X 55%)
Total realized gross profit
Operating expenses
Net income, 12/31/08
Installment sales 2007
Collections:
Down payment (20% x 785,000)
Installment (40% x 628,000)
Installment accounts receivable 2007, 12/31/07
Gross profit rate on sales
Deferred gross profit- 2007, 12/31/07

Regular sales
Cost of regular sales
Gross profit on regular sales
Realized gross profit on installment sales:
Installment sales (1,093,750 x 240%)
Installment accounts receivable-12/31/08
Collections
Gross profit on rate on sales

P 187,500
__112,500
P 75,000
P12,500
_34,375

__46,875
121,875
___31,250
P 90,625

P785,000
P157,000
251,200 408,200
376,800
35/135
P 97,689

P1,575,000
1,050,000
525,000
2,625,000
1,575,000
1,050,000
140/240

612,500

9-23: a

Total realized gross profit


Operating expenses (1,137,500 x 70%)
Net income

1,137,500
796,250
P 341,250

Regular sales
Cost of regular sales
Gross profit on regular sales
Realized gross profit on installment sales:
Collections excluding Interest (312,000 24,000)
Gross profit rate (270,000/900,000)
Total realized gross profit
Loss on repossession
Fair value of repossessed merchandise
Less: Unrecovered cost (100,000 x 70%)
Total realized GP after loss on repossession
Less: Operating expenses
Installment accounts written-off (44,000 x .70)
Net operating income
Interest income
Net income

P375,000
215,000
160,000
288,000
30%

86,400
246,400

54,000
70,000

( 16,000)
230,400

72,000
30,800

102,800
127,600
24,000
P151,600

SOLUTIONS TO PROBLEMS
Problem 9 1

Journal Entries:
Installment A/R2006...........................
Installment A/R2007
Installment A/R2008...........................
Installment Sales ...........................

2006
104,000

104,000

2007

116,000

116,000

2008

121,000
121,000

Cost of Installment Sales ......................


Inventory.......................................

64,480
64,480

68,440
68,440

73,810
73,810

Cash ......................................................
Installment A/R2006
Installment A/R2007...................
Installment A/R2008...................
Interest Revenue ...........................

66,980
57,200

9,780

125,520
29,120
71,920
_
24,480

145,460
15,000
26,680
76,230
27,550

Installment Sales ...................................


Cost of Installment Sales ..............
Deferred Gross Profit2006 ..........
Deferred Gross Profit2007 ..........
Deferred Gross Profit2008 ..........

104,000
64,480
39,520

116,000
68,440

47,560

121,000
73,810

47,190

Deferred Gross Profit2006..................


Deferred Gross Profit2007..................
Deferred Gross Profit2008..................
Realized Gross Profit ....................
Computations:
2006:
P57,200 X .38

21,736

21,736

P21,736

2007:

P29,120 X .38
P71,920 X .41
Total RGP

=
=

P11,066
29,987
P40,553

2008:

P15,000 X .38
P26,680 X .41

=
=

P 5,700
10,939

11,066
29,487

40,553

5,700
10,939
29,730
46,369

P76,230 X .39
Total RGP

2007:

2008:

29,730
P46,369

Problem 9 2
Inventory ........................................................................................................... 45,200
Cash...........................................................................................................
Notes Receivable 2007 (P32,000 + P62,000 + 3,600) ....................................... 97,600
Unearned Interest Revenue (P7,167 + P3,600)..........................................
Installment Sales........................................................................................
Cost of Installment Sales (P45,200 P2,000 inventory increase) ..................... 43,200
Inventory ...................................................................................................
Cash..... ............................................................................................................. 35,600
Notes Receivable 2007 ..............................................................................
Unearned Interest Revenue 2007....................................................................... 3,600
Interest Revenue ........................................................................................
Installment Sales ............................................................................................... 86,833
Cost of Installment Sales ...........................................................................
Deferred Gross Profit on Installment Sales2007......................................
Deferred Gross Profit on Installment Sales2007 ............................................. 16,080*
Realized Gross Profit on Installment Sales................................................
*Gross profit percentage: 50.25% (P43,633
3)
.5025 x 32,000 = P16,080
Inventory ........................................................................................................... 52,020
Cash...........................................................................................................
Notes Receivable2008 ..................................................................................... 89,5001
Unearned Interest Revenue........................................................................
Installment Sales........................................................................................
160,000 + (P50,000 + P5,500) P26,000* = 89,500
*2007 Notes receivable collected in 2008
2Interest revenue from 2007 notes: P7,167 P5,579 = P1,588
Interest revenue from 2008 notes: P5,500 P1,588 = P3,912
Discount on notes receivable at end of 2008 ..................................................... P 8,043
Interest revenue from 2008 notes (see above) ................................................... 3,912
Total discount at time of sale............................................................................. P11,955
Cost of Installment Sales (P52,020 P8,000) ................................................... 44,020
Inventory ...................................................................................................
Cash..... ............................................................................................................. 55,500
Notes Receivable2007 (P62,000 P36,000) ...........................................
Notes Receivable2008 .............................................................................
* P89,500 P60,000 = P29,500
Discount on Notes Receivable2007................................................................. 1,588
Discount on Notes Receivable2008................................................................. 3,912
Interest Revenue ........................................................................................
Installment Sales ............................................................................................... 77,545
Cost of Installment Sales ...........................................................................
Deferred Gross Profit on Installment Sales2008......................................
Deferred Gross Profit on Installment Sales2007 (P26,000
P1,538 = P24,412; P24,412 x .5025) .............................................................. 12,267
Deferred Gross Profit on Installment Sales2008 ............................................. 11,062*
Realized Gross Profit on Installment Sales................................................
profit percentage: 43.23% (P33,525 P77,545)
.4323 x (P29,500 P3,912) = P11,062

1.

2006:

Gross profit rate

2007:

Gross profit rate

Deferred gross profit, 1/1

Install. contracts rec'l, 1/1


Deferred gross profit, 1/1

P24,000

P60,000
P24,000

40%

42%

45,200
10,767
86,833
43,200
35,600
3,600
43,200
43,633
16,080

52,020
11,9552
77,545

44,020
26,000
29,500*

5,500
44,020
33,525

23,329

Install. contracts rec'l, 1/1


2008:
2.

Gross profit rate

Gross profit
=
=
Installment sales

P140,000
P86,000
=
P200,000

Journal Entries:
Accounts Receivable .................................................................................................
Sales .... .............................................................................................................
Installment Contracts Receivable 2008 ..................................................................
Installment Sales ...............................................................................................
Cost of Installment Sales...........................................................................................
Shipments on Installment Sales.........................................................................
Purchases..... .............................................................................................................
Cash..... .............................................................................................................
Selling Expenses .......................................................................................................
Cash..... .............................................................................................................
Cash....... ..... .............................................................................................................
Accounts Receivable .........................................................................................
Installment Contracts Receivable 2006 ..........................................................
Installment Contracts Receivable 2007 ..........................................................
Installment Contracts Receivable 2008 ..........................................................
Adjusting Entries:
Installment Sales .......................................................................................................
Cost of Installment Sales ...................................................................................
Deferred Gross Profit on Installment sales 2008 ............................................
Deferred Gross Profit 2006 (P40,000 x 40%).........................................................
Deferred Gross Profit 2007 (P80,000 x 42%).........................................................
Deferred Gross Profit 2008 (P110,000 x 43%).......................................................
Realized Gross Profit.........................................................................................
Doubtful Accounts Expense (1/4 x 1% x P600,000) .................................................
Allowance for Doubtful Accounts.....................................................................
Closing Entries:
Sales ...... ..... .............................................................................................................
Merchandise Inventory, December 31.......................................................................
Shipments on Installment Sales.................................................................................
Merchandise Inventory, January 1.....................................................................
Purchases...........................................................................................................
Selling Expenses ...............................................................................................
Doubtful Accounts Expense ..............................................................................
Income Summary ..............................................................................................
Realized Gross profit.................................................................................................
Income Summary ..............................................................................................
Income Summary ......................................................................................................
Retained Earnings .............................................................................................

3.

43%
600,000
200,000
114,000
476,000
210,000
790,000

200,000
16,000
33,600
47,300
1,500

600,000
260,000
114,000

96,900
143,400

600,000
200,000
114,000
476,000
210,000
560,000
40,000
80,000
110,000

114,000
86,000

96,900
1,500

240,000
476,000
210,000
1,500
46,500
96,900
143,400

Good Buy Mart


Income Statement
Year Ended December 31, 2008
Sales ...... ..... .............................................................................................................
Cost of sales:
Merchandise inventory, January 1.....................................................................
Purchases...........................................................................................................
Cost of goods available for sale.........................................................................
Less Shipments on installment sales..................................................................
Cost of goods available for regular sales ...........................................................
Less Merchandise inventory, December 31.......................................................
Gross profit on regular sales......................................................................................
Add Realized gross profit on installment sales (Schedule 1).....................................
Total realized gross profit..........................................................................................

P600,000
P240,000
476,000
716,000
114,000
602,000
260,000

342,000
258,000
96,900
354,900

Operating expenses:
Selling expenses ................................................................................................
Doubtful accounts expense................................................................................
Net income .............................................................................................................
Schedule 1
Collections ..............................................
Multiply by Gross profit rate......................
Realized gross profit...................................
4.

2006
P40,000
40%
P16,000

210,000
1,500

Years of Installment Sales


2007
2008
P80,000
P110,000
42%
43%
P33,600
P 47,300

Good Buy Mart


Balance Sheet
December 31, 2008
A s s e t s
Cash....... ..... .............................................................................................................
Merchandise inventory..............................................................................................
Accounts receivable ..................................................................................................
Allowance for doubtful accounts...............................................................................
Installment contracts receivable 2006 ....................................................................
Installment contracts receivable 2007 ....................................................................
Installment contracts receivable 2008 ....................................................................
Other assets .............................................................................................................
Total Assets .......................................................................................................
Liabilities and Equity
Liabilities:
Accounts payable ..............................................................................................
Deferred gross profit on installment sales 2006 .............................................
Deferred gross profit on installment sales 2007 .............................................
Deferred gross profit on installment sales 2008 .............................................
Total Liabilities .................................................................................................
Equity:
Capital stock......................................................................................................
Retained earnings ..............................................................................................
Total Liabilities and Equity ...............................................................................

P 62,000
3,500

200,000

211,500
P143,400

Total
P 96,900

P144,000
260,000
58,500
20,000
60,000
90,000
P832,500
P 60,000
8,000
25,200
38,700
131,900

P406,000
294,600

700,600
P832,500

Problem 9 4
1.

2.

2007: GP rate

Deferred gross profit, 1/1

Install. contracts rec'l, 1/1

2008: GP rate

Gross profit

Installment sales

=
=

P21,600 + P1,200

P24,000 + P52,000
P150,000 P97,500

P150,000

Installment Sales .......................................................................................................


Cost of Installment Sales ...................................................................................
Deferred Gross Profit, 2008 ..............................................................................
Deferred Gross profit, 2007.......................................................................................
Deferred Gross Profit, 2008 ......................................................................................
Realized Gross Profit.........................................................................................
Computation:
Installment contracts receivable, 1/1 ............................
Less Installment contracts receivable, 12/31 ................
Total credit for the period.............................................
Less Credit representing repossession ..........................
Credit representing collections .....................................

2007
Sales
P76,000
24,000
52,000
4,000
P48,000

=
=

P22,800

P76,000

30%

P52,500

P150,000

35%

150,000
14,400
25,900

2008
Sales
P150,000
76,000
74,000

P 74,000

97,500
52,500
40,300

Total

Multiply by Gross profit rate ........................................


Realized gross profit.....................................................

30%
P14,400

35%
P 25,900

Sales ...... ..... .............................................................................................................


Realized Gross Profit ................................................................................................
Loss on Repossession........................................................................................
Cost of Sales......................................................................................................
Selling and Administrative Expenses ................................................................
Income Summary ..............................................................................................
Income Summary ......................................................................................................
Retained Earnings .............................................................................................
3.

212,000
40,300

20,900

P 40,300

400
165,000
66,000
20,900
20,900

Apple Company
Income Statement
Year Ended December 31, 2008
Sales ...... ..... ............................................................................................................. ....................
Cost of sales ............................................................................................................. ....................
Gross profit on regular sales...................................................................................... ....................
Add Realized gross profit on installment sales (Schedule 1)..................................... ....................
Total realized gross profit.......................................................................................... ....................
Less Loss on repossession......................................................................................... ....................
Total realized gross profit after adjustment for loss on repossession ........................ ....................
Selling and administrative expenses.......................................................................... ....................
Net income .. ............................................................................................................. ....................

P212,000
165,000
47,000
40,300
87,300
400
86,900
66,000
P 20,900

Problem 9 4
Schedule 1
2007
Sales
P76 000
24,000
52,000
4,000
P48,000
30%
P14,400

Installment contracts receivable, 1/1 ..........................................


Less Installment contracts receivable, 12/31 ..............................
Total credit for the period...........................................................
Less Credit representing repossession ........................................
Credit representing collections ...................................................
Multiply by Gross profit rate......................................................
Realized gross profit...................................................................

2008
Sales
P150,000
76,000
74,000

P 74,000
35%
P 25,900

Total

P40,300

Problem 9 5
1.

Cost of Installment Sales...........................................................................................


Shipments on Installment Sales.........................................................................

54,400

Installment Sales .......................................................................................................


Cost of Installment Sales ...................................................................................
Deferred Gross Profit, 2008 ..............................................................................
Gross profit = P25,600
32%
Deferred Gross Profit, 2007 ......................................................................................
Deferred Gross Profit, 2008 ......................................................................................
Realized Gross Profit.........................................................................................

80,000

Computation:
Installment contracts receivable, 1/1 ............................
Less Installment contracts receivable, 12/31 ................
Total credit for the period.............................................
Less Credit representing repossession ..........................
Credit representing collections .....................................

2007
Sales
P82,000
_ 36,000
46,000
__6,000
P40,000

14,000
8,000

2008
Sales
P 80,000
_55,000
25,000
___

P 25,000

54,400
54,400
25,600

22,000

Total

Multiply by Gross profit rate ........................................


Realized gross profit.....................................................
*2007 Gross profit rate=

2.

DGP, 1/1

ICR, 1/1

__35%*
P14,000

___32%
P 8,000

P 22,000

P28,700 (26,600 + 2,100)

=
35%
P82,000 (36,000 + 40,000 + 6,000)

Sales ...... ..... .............................................................................................................


Merchandise Inventory, December 31.......................................................................
Shipments on Installment Sales.................................................................................
Merchandise Inventory, January 1.....................................................................
Purchases...........................................................................................................
Repossessed Merchandise .................................................................................
Loss on Repossession........................................................................................
Operating Expenses...........................................................................................
Income Summary ..............................................................................................

200,000
52,000
54,400

Realized Gross Profit ................................................................................................


Income Summary ..............................................................................................

22,000

Income Summary ......................................................................................................


Retained Earnings .............................................................................................

31,500

60,000
180,000
3,000
900
53,000
9,500
22,000
31,500

PPG Discount Center, Inc.


Income Statement
Year Ended December 31, 2008
Sales ...... ..... ..............................................................
Cost of sales:
Inventory, January 1 ...........................................
Purchases............................................................
Repossessed merchandise...................................
Cost of goods available for sale..........................
Less Shipments on installment sales...................
Cost of goods available for regular sales ............
Less Inventory, December 31.............................
Gross profit ..............................................................
Less Deferred gross profit on installment
sales, 2008 ..........................................................
Realized gross profit, 2008.........................................
Add Realized gross profit on 2007
installment sales .................................................
Total realized gross profit...........................................
Less Loss on repossession..........................................
Total realized gross profit after adjustment
for loss on repossession......................................
Operating expenses ....................................................
Net income ..............................................................

P 60,000
180,000
__3,000
243,000
_54,400
188,600
_52,000

Regular
P200,000

Installment
P80,000

Total
P280,000

_136,600
P 63,400

54,400
25,600

191,000
89,000

17,600
8,000

17,600
71,400

14,000
22,000
___900

14,000
85,400
__900

P21,100

84,500
_53,000
P31,500

Problem 9 6
1.

London Products
Schedule of Cost of Goods Sold
Year Ended December 31, 2008
Merchandise inventory, January 1............................................................................. ....................
Purchases
............................................................................................................. ....................
Freight-in
............................................................................................................. ....................
Repossessed merchandise.......................................................................................... ....................
Cost of goods available for sale................................................................................. ....................

P 48,000
238,000
12,000
14,000
312,000

Less Merchandise inventory, December 31............................................................... ....................


Cost of goods sold..................................................................................................... ....................
2.

London Products
Schedule of Allocation of Cost of Goods Sold
Year Ended December 31, 2008
Amount
Cash sales ....................
P60,000
Charge sales ....................120,000
Installment sales ...............
300,000

3.

52,000
P260,000

On Cash
Price Basis
P 60,000
100,000
240,000
P 400,000

120%
125%

Ratio to
Total
60/400
100/400
240/400

Allocated
Cost
P 39,000
65,000
156,000
P260,000

London Products
Income Statement
Year Ended December 31, 2008

Sales ...... ..... ...........................................


Cost of goods sold...................................
Gross profit ...........................................
Less Unrealized gross profit:
On installment contracts
receivable,12/31 (192,000 x 144/300)
Realized gross profit................................
Add Realized gross profit on
prior years' sales (Schedule 1):
2006.........................................
19,200
2007.........................................
14,700
Total realized gross profit........................
Less Loss on repossession
(Schedule 2) ....................................
Total realized gross profit after
adjustment for loss on
repossession.....................................
Less Operating expenses .........................
Net income ...........................................

Installment
Total
Sales
P480,000
P 300,000
260,000
156,000
P 220,000 P 144,000
92,160
127,840

92,160
51,840

33,900
161,740

33,900
85,740

10,200

10,200

151,540
93,000
P 58,540

P 75,540

Schedule 1
Installment contracts receivable, January 1:
2006 P32,000
.....................................................................................
2007 P56,000
.....................................................................................
Less Installment contracts receivable, December 31 .................................................
Total credits .............................................................................................................
Less Credit representing repossession .......................................................................
Total collections ........................................................................................................
Multiply by Gross profit rate.....................................................................................
Realized gross profit..................................................................................................
Schedule 2
Fair market value of repossessed merchandise.....................
Less Unrecovered cost:
Unpaid balance.............................................................
Less Unrealized profit
2006 P10,000 x
40% ....................................
2007 P28,000 x
35% ....................................
Balances
........................................................................
Gain (loss) on repossession ..................................................

Charge
Sales
P120,000
65,000
P 55,000

2006
P80,000

Cash
Sales
P 60,000
39,000
P 21,000

2007

_22,000
58,000
_10,000
P48,000
___40%
P19,200

P160,000
__90,000
70,000
28,000
P 42,000
___35%
P 14,700

2006
P 2,000

2007
P12,000

Total
P 14,000

10,000

28,000

38,000

4,000
__6,000
P(4,000)

9,800
18,200
P( 6,200)

13,800
__24,200
P( 10,200)

1.

Problem 9 7
2007
2007 installment sales (P400,000 x 42%*)........................................................
2008:
2007 installment sales (P173,000 x 42%)..........................................................
2008 installment sales (P560,000 x 38.5%*).....................................................
Deferred gross profit .................................................................................................

2007
P 168,000
________
P 168,000

*Computation of Gross profit percentages (see next page)

2007
Installment sales ........................................................................................................ P2,210,000
Less Trade-in allowances (P226,000 P158,000) ....................................................
_______
Adjusted installment sales .........................................................................................
2,210,000
Cost of sales:
Inventories, January 1 (new) .............................................................................

Purchases (new).................................................................................................
1,701,800
Repossessed merchandise..................................................................................

Cost of goods available for sale.........................................................................


1,701,800
Less: Inventories, December 31
New merchandise ......................................................................................
Repossessed merchandise..........................................................................
Total ..........................................................................................................
Cost of sales ......................................................................................................
Gross profit .............................................................................................................
Gross profit percentages............................................................................................
*2007 : P195,000 x 20%
=P39,000
2008 : P110,000 x 40%
=_44,000
P83,000
Uncollectible installment contracts expense, per books ...................
Correct Uncollectible installment contracts expense:
Fair market value of repossessed merchandise
2007 sales (P195,000 x 20%) ...........................................
2008 sales (P110,000 x 40%) ...........................................
Unrecovered cost
2007 sales [P105,000 x (100% 42%)] ...........................
2008 sales [P82,000 x (100% 38.5%)] ..........................
Adjustment to Uncollectible installment contracts expense .............

2008

P 72,660
__215,600
P 288,260
2008
P3,100,000
____68,000
_3,032,000
420,000
1,767,000
_83,000*
2,270,000

420,000
_______
420,000
1,281,800
P 928,200

358,820
____46,500
405,320
_1,864,680
P1,167,320

42%

38.5%

P 99,000
P 39,000
__44,000

P 83,000

60,900
__50,430

__111,330

__28,330
P 70,670

Installment
Sales
P3,032,000
1,864,680
1,167,320

Total
Sales
P3,237,000
_2,022,680
1,214,320

__247,170
920,150

__247,170
967,150

___51,240
971,390
___28,330
P 943,060

___51,240
1,018,390
___28,330
990,060
__592,960
P 397,100

Fortune Sales Corporation


Income Statement
Year Ended December 31, 2008
Cash
Sales
Sales
.................................................................................... P205,000
Cost of sales .................................................................................... _158,000_
Gross profit .................................................................................... P 47,000
Less Unrealized gross profit on 2005 installment
sales (Schedule 1) .....................................................................
Realized gross profit on 2008 sales ..................................................
Add Realized gross profit on 2007 installment
sales (Schedule 2) .....................................................................
Total realized gross profit.................................................................
Less Uncollectible installment contracts expense ............................
Total realized gross profit after adjustment ......................................
Operating expenses ..........................................................................
Net income ....................................................................................

Schedule 1
Installment contracts receivable 2008, December 31 ........................... ...............
Installment contracts receivable 2008 defaulted................................... ...............
Total ...... ..... ........................................................................................ ...............
Multiply by 2008 gross profit percentage............................................. ...............
Unrealized gross profit on 2008 installment sales ................................ ...............

P 560,000
___82,000
P 642,000
___38.5%
P 247,170

Schedule 2
Installment contracts receivable 2007, January 1 ......................................................
Less Installment contracts receivable 2007, December 31 ........................................
Total credits for the period ........................................................................................
Less Installment contracts receivable 2007 defaulted ...............................................
Total collections ........................................................................................................
Multiply by 2007 gross profit percentage..................................................................
Realized gross profit on 2007 installment sales.........................................................
1.

P 400,000
__173,000
227,000
__105,000
P 122,000
_____42%
P 51,240

Apportionment of cost (P600,000) to Lots 1, 2 and 3:


Lot 1 : 2/3 x P360,000 ................................................
Lot 2 : 2/3 x P240,000 ................................................
Lot 3 : 1/3 ...................................................................
1/3 x P240,000......................................................
Total cost......................................................................

P120,000
__80,000

Journal Entries for 2007


March 31
Cash....... ..... ........................................................................................................
Notes Receivable (Lot 2)......................................................................................
Lot 2 ........................................................................................................
Deferred gain on Sale of Land......................................................................
June 30
Cash....... ..... ........................................................................................................
Notes Receivable (Lot 3)......................................................................................
Lot 3 .... ........................................................................................................
Deferred Gain on Sale of Land.....................................................................
Cash....... ..... ........................................................................................................
Interest Income (P364,000 x 12% x 3/12) ....................................................
Notes Receivable (Lot 2)..............................................................................
September 30
Cash....... ..... ........................................................................................................
Interest Income (P358,920 x 12% x 3/12) ....................................................
Notes Receivable (Lot 2)..............................................................................
October 31
Cash....... ..... ........................................................................................................
Notes Receivable (Lot 1)......................................................................................
Lot 1 .... ........................................................................................................
Deferred Gain on Sale of Land.....................................................................
December 31
Cash....... ..... ........................................................................................................
Notes Receivable (Lot 1)..............................................................................
Notes Receivable (Lot 2)..............................................................................
Notes Receivable (Lot 3)..............................................................................
Interest Income.............................................................................................
Computation:
Collections ...................................................

Total
P78,000.00

Lot 1
P12,000.00

P 240,000
160,000
__200,000
P 600,000

36,000.00
364,000.00

120,000.00
720,000.00
16,000.00

16,000.00

72,000.00
288,000.00

78,000.00

Lot 2
P16,000.00

160,000.00
240,000.00

200,000.00
640,000.00
10,920.00
5,080.00
10,767.60
5,232.40

240,000.00
120,000.00
6,240.00
5,389.37
6,800.00
59,570.63
Lot 3
P50,000.00

Apply to interest:
Lot 1 P288,000.00 x 12% x 2/12..........
5,760.00
Lot 2 P353,687.60 x 12% x 3/12..........
59,570.63
10,610.63
Lot 3 P720,000.00 x 12% x 6/12..........______________________________________________ _43,200.00
Apply to principal ........................................
P18,429.37
P 6,240.00
P 5,389.37
P 6,800.00
2.

Deferred Gain on Sale of Land (Lot 1)......................................................................


Deferred Gain on Sale of Land (Lot 2)......................................................................
Deferred Gain on Sale of Land (Lot 3)......................................................................
Realized Gain on Sale of Land ..........................................................................
Computation:

Lot 1
P78,240.00

Collections applied to principal ...................


Multiply by Gross profit rates:
Lot 1 P120,000
................
Lot 2 P240,000
................
Lot 3 P640,000
................
Realized gain................................................
3.

33.33%
_________
P26,080.00

Lot 3 (80% x P200,000) ............................................................................................


Deferred Gain on Sale of Land (Lot 3) (P640,000 P96,368)..................................
Loss on Repossession................................................................................................
Notes Receivable (Lot 3) (P720,000 P6,800).................................................

Galaxy Investment Company


Income Statement
Year Ended December 31, 2008

26,080.00
31,021.06
96,368.00

153,469.06

Lot 2
P51,701.77

Lot 3
P126,800.00

60%
_________
P31,021.06

_____76%
P96,368.00

160,000.00
543,632.00
9,568.00

713,200.00

Problem 9 9

Sales Schedule 1) .............................................................................................................


Cost of sales (Schedule 2)....................................................................................................
Gross profit ...... ..... .............................................................................................................
Less Sales commissions.......................................................................................................
Gross profit ...... ..... .............................................................................................................
Less Deferred gross profit
Installment Notes Balance
P5,370,000

=
=67% x P6,227,000
Installment Sales
P8,060,000
Realized gross profit ............................................................................................................
Expenses:
Advertising and promotion........................................................................................
Sales manager's salary...............................................................................................
General office expenses (1/4 x P236,000).................................................................
Net profit ....... ..... .............................................................................................................
Schedule 1
A lots : 26 @ P150,000.........................................................
B lots : 32 @ P100,000 .........................................................
C lots : 12 @ P80,000 ...........................................................
..............................................................
Schedule 2
Class
A............ ..... ..............................................................
B............ ..... ..............................................................
C............ ..... ..............................................................
Total .... ..............................................................

Total
Sales Price
P3,900,000
3,200,000
960,000
P8,060,000
Number of
Lots
80
100
120
300

P 8,060,000
1,612,000
6,448,000
221,000
6,227,000
4,172,090
2,054,910
P 730,000
120,000
59,000

909,000
P 1,145,910

Cash
Received
P1,650,000
800,000
240,000
P2,690,000

Installment
Notes Balance
P 2,250,000
2,400,000
720,000
P 5,370,000

Unit
Price
P150,000
100,000
80,000

Total
Sales Value
P12,000,000
10,000,000
9,600,000
P31,600,000

Cost of tract:
Cost of land .............................................................................................................
Legal fees, etc............................................................................................................
Grading contract........................................................................................................
Water and sewerage system contract.........................................................................
Paving contract..........................................................................................................
General office expenses (3/4 x P236,000).................................................................
Total ...... ..... .............................................................................................................

P 4,800,000
600,000
225,000
184,900
266,300
177,000
P 6,253,200

P6,253,200
Cost rate : = 20% (rounded off)
P31,600,000
Cost of sales (P8,060,000 x 20%) ...................................................................................... P 1,612,000
Problem 9 10
Rizal Company
Income Statement
Year Ended December 31, 2008
Installment sales [(P14,300 x 7) + (P725 x 4)] ....................................................................
Cost of goods sold on installment (schedule 1)....................................................................
Gross profit ...... ..... .............................................................................................................
Less Deferred gross profit on 19x8 sales
(P103,000 P21,000 = P82,000 x 23%*) ................................................................
Realized gross profit on 2008 sales......................................................................................
Add Realized gross profit on prior years' sales
2006 : P60,000 x 33-1/3* ..........................................................................................
2007 : P115,000 x 35%* ...........................................................................................
Total realized gross profit ....................................................................................................
Less Loss on repossession (Schedule 4)...............................................................................
Total realized gross profit after adjustment..........................................................................
General and administrative expenses ...................................................................................
Net income (loss) .............................................................................................................

P103,000
__79,310
23,690
__18,860
4,830
P20,000
_40,250

__60,250
65,080
__33,100
31,980
__50,000
P(18,020)

*See Schedule 3
Schedule 1
Purchases (P10,500 x 8).......................................................................................................
Repossessed merchandise ....................................................................................................
Cost of goods available for sale ...........................................................................................
Less Inventory, December 31
Number of units on hand ...........................................................................................
Multiply by average unit cost (Schedule 2) ...............................................................
Cost of goods sold on installment ........................................................................................

P 84,000
___2,520
86,520
1
P 7,210

___7,210
P 79,310

Schedule 2
Purchases during 2008 (P10,500 x 8) ..................................................................................
Add Repossessed merchandise ............................................................................................
Total .............. ..... .............................................................................................................
divide by Number of units (8 + 4) .......................................................................................
Average unit cost .............................................................................................................
Schedule 3
Sales
2006 :
2007 :
2008 :

...................................................................
P15,000 x 10....................................................
P14,000 x 20....................................................
P14,300 x 7......................................................

2006
P150,000

P 84,000
___2,520
P 86,520
_____12
P 7,210
2007
P280,000

2008

100,100

P725 x 4...........................................................
...................................................................

_______
150,000

_______
280,000

__2,900
103,000

Cost of goods sold:


Inventory, January 1 ........................................................
Purchases
...................................................................
Repossessed merchandise................................................
Cost of goods available for sale.......................................
Less Inventory, December 31..........................................
Cost of goods sold...........................................................
Gross profit ...... ..... ...................................................................
Gross profit rates.... ...................................................................

120,000
_____
120,000
_20,000
100,000
P 50,000
33-1/3%

20,000
162,000
_____
182,000
_____
182,000
P 98,000
35%

84,000
_2,520
86,520
_7,210
79,310
P23,690
23%

Sales

Schedule 4
Fair market value of repossessed merchandise ....................................................................
Less Unrecovered cost
Unpaid balance:
Original sales amount (P14,000 x 4) .................................................................
Collections prior to repossession .......................................................................
Total .... .............................................................................................................
Less Unrealized profit (P54,800 x 35%) ...................................................................
Loss on repossession............................................................................................................

P 2,520
P 56,000
__1,200
54,800
_19,180

_35,620
P33,100

Long-Term Construction Contracts

163

CHAPTER 10
MULTIPLE CHOICE ANSWERS AND SOLUTIONS
10-1: a

Percentage of Completion Method:


Contract Price
Less:Total estimated cost
Cost incurred
Estimated remaining cost
Gross profit estimated
% of completion (200,000/600,000)
Gross profit to be recognized
Zero Profit Method:

10-2: a P100,000
Contract Price
Less: Total estimated cost
Estimated gross profit
% of completion:
2007 (3,900,000/7,800,000)
2008(6,300,000/8,100,000)
Gross profit earned to date
Less: Gross profit earned in prior year
Gross profit earned each year
10-3: a

10-4: b

P1,000,000
P 200,000
_400,000

__600,000
400,000
__33 1/3%
P 133,333
0

2007
2008
P9,000,000 P9,000,000
_7,800,000 _8,100,000
1,200,000
900,000
50%
_________ ______78%
600,000
700,000
________ ___600,000
P 600,000 P 100,000

Contract Price
Less: Total estimated cost (3,600,000 + 1,200,000)
Estimated gross profit
% of completion (3,600,000/4,800,000)
Gross profit earned to date
Less: Gross profit earned in 2007
Gross profit earned in 2008

P6,000,000
_4,800,000
1,200,000
_____75%
900,000
__600,000
P 300,000

Contract Price
Less: Total estimated cost (930,000 + 2,170,000)
Loss

P3,000,000
_3,100,000
(P 100,000)

164

10-5: b

10-6: a

Chapter 10

Total cost to date, 2008 (4,800,000 X 60%)


Less: Cost incurred in 2007 (4,500,000 X 20%)
Cost incurred in 2008

P2,880,000
__900,000
P1,980,000

Percentage of Completion Method:


Contract Price
Less: Total estimated cost (900,000/1,800,000)
Estimated gross profit
% of completion (900,000/2,700,000)
Gross profit recognized, 2007
Add: Cost Incurred
Construction in Progress - 2007

P3,000,000
_2,700,000
300,000
___33.33%
100,000
___900,000
P 1,000,000

Zero Profit Method:


Cost incurred to Construction in Progress - 2007

P 900,000

10-7: a
Contract Price
Less: Total estimated cost
Estimated gross profit
% of completion
Gross Profit earned to date
Gross Profit earned in prior year
Gross Profit earned this year
10-8: b

Collections:
Contract Billings
Less: Accounts receivable
Collections
Initial Gross Profit:
Contract Price
Gross Profit rate:
Income recognized
Divide by Construction in Progress
Initial Gross Profit

2007
2008
P4,200,000 P4,200,000
_3,000,000 _3,750,000
1,200,000
450,000
_____20% ____100%
240,000
450,000
_______ __240,000
P 240,000 P 210,000

P 47,000
___15,000
P 32,000
P 800,000
10,000
50,000 =_____20%
P 160,000

Long-Term Construction Contracts

165

10-9: a
Gross profit (loss) earned in 2008
Gross profit earned in prior years
Gross profit earned to date - 2008
Divide by percentage of completion - 2008
Estimated gross profit - 2008
Less: Contract price
Total estimated cost
Less: Cost incurred - 2008
Cost incurred to date - 2007
Less: Cost incurred - 2006
Cost incurred in 2007

(P 20,000)
_180,000
160,000
___100%
160,000
2,000,000
1,840,000
_820,000
1,020,000
__360,000
P 660,000

Gross profit earned to date - 2007 (P40,000 + P140,000)


Divide by estimated gross profit - 2007:
Contract price
P2,000,000
Gross profit rate [180,000/(1,020,000 + 180,000)]
___X 15%
Percentage of completion - 2007

P 180,000

10-10: b

__300,000
60%

10-11: a, Refer to Q 10-10 solutions.


10-12: d
Contract price
Estimated gross profit - 2007 (Refer to Q 10-10)
Total estimated cost
Less: Cost incurred to date - 2007 (refer to Q 10-9)
Estimated cost to complete - 2007

P2,000,000
__300,000
1,700,000
1,020,000
P 680,000

2007: Construction in progress


Less: Construction costs
Gross profit recognized - 2007

P 244,000
__210,000
P 34,000

2008: Construction in progress (P728,000-P244,000)


Less: Construction costs
Gross profit recognized - 2008

P 484,000
__384,000
P 100,000

10-13: d

166

Chapter 10

10-14: d

Project 1

Percentage of Completion Method:


Contract price
Less: Total estimated cost
Cost incurred to date - 2008
Estimated cost to complete
Total
Estimated gross profit (Loss)
Percentage of completion
Profit (loss) to be recognized
Total is (P10,000)

Project 2

P 420,000 P 300,000
P 240,000
__120,000
__360,000
60,000
__66.67%
P 40,000

P 280,000
___70,000
__350,000
(50,000)
_______
(P 50,000)

Zero Profit Method - The loss (P50,000) for project 2 only.


10-15: a
Contract price (cost X 120%)
Less: Total estimated costs
(1) Cost incurred to date
Estimated cost to complete
(2) Total
Estimated gross profit
Percentage of completion (1 2)
Gross profit earned to date
Gross profit earned in prior years
Gross profit earned this year
10-16: d
Contract price
Less: Total estimated cost
Cost incurred to date
Estimated cost to complete
Total
Estimated gross profit
Percentage of completion:
2007 (1,425,000 - 50,000) 5,500,000
2008 (3,040,000 - 50,000) 5,000,000
Profit earned to date
Less: Gross profit earned in prior year
Gross profit earned this year

2006
2007
2008
P3,744,000 P3,744,000 P3,744,000
546,000 1,544,400 3,120,000
_2,054,000 _1,315,000 ________
_2,600,000 _2,860,000 _3,120,000
1,144,000
884,000
624,000
_____20% _____54% ____100%
240,240
477,360
624,000
_______ __240,240 __477,360
P 240,240 P 237,120 P 146,640
2007
2008
P6,300,000 P6,300,000
1,425,000
_4,075,000
P5,500,000
800,000

3,040,000
_1,960,000
P5,000,000
1,300,000

25%
________ __59.80%
200,000
777,400
________ __200,000
P 200,000 P 577,400

Long-Term Construction Contracts

167

10-17: a
Cash collections:
Progress billings
Less: Accounts receivable, end
Collection

P1,500,000
__500,000
P1,000,000

Cost incurred to date:


Construction in Progress
Less: Gross profit earned
Cost incurred to date

P1,600,000
__200,000
P1,400,000

10-18: d
Percentage of Completion Method:

Apartment A
2007
2008
1,620,000 1,620,000

Contract price
Less: Total Estimated Costs
(1)
Cost incurred to date
Estimated cost to complete
(2)
Total estimated cost
Estimated Gross Profit
Percentage of completion (1 2)
Gross profit earned to date
Less: Gross profit earned in Prior years
Gross Profit earned this year
Total Gross Profit 20 (P75,000 + P22,800)

Apartment B
2007
2008
2,520,000 2,520,000

P 600,000 P1,200,000 P1,560,000 P2,310,000


840,000
240,000
690,000

1,440,000 1,440,000 2,250,000 2,310,000


180,000
180,000
270,000
210,000
_41.67%
_83.33%
_69.33% _100.00%
75,000
150,000
187,200
210,000
_______ ___75,000 _______ __187,200
P 75,000 P 75,000 P 187,000 P 22,800
P97,800

Zero Profit Method - P210,000 gross profit earned in 2008 for Apartment B.
10-19: d
Contract price:
2007
2008 (P6,000,000-P50,000)
Less: Total estimated costs
(1) Cost incurred to date
Estimated cost to complete
(2) Total estimated cost
Estimated Gross Profit
Percentage of completion (1 2)
Gross profit earned to date
Less: Gross profit earned in Prior year
Gross Profit earned this year

2007

2008

P6,000,000
_________ P5,950,000
2,340,000
260,000
2,600,000
3,400,000
____90%
3,060,000
_______
P3,060,000

2,650,000

2,650,000
3,300,000
___100%
3,300,000
3,060,000
P 240,000

168

Chapter 10

10-20: a
(1)Cost incurred to date
(2)Estimated cost to complete
(3)Total Estimated Costs

10-21: c

2006
P3,400,000
1,600,000
5,000,000

Percentage of completion (1 3)

68%

Contract price
Less: Total estimated cost
Estimated Gross Profit
Percentage of completion
Gross profit earned (loss) to date
Add: Cost incurred to date
Construction in Progress
Less: Contract billings
Balance

P6,000,000
5,000,000
1,000,000
68%
680,000
3,400,000
4,080,000
3,200,000
P 880,000

Construction in Progress:
Cost incurred to date, 2007
Gross profit earned, 2007 (Schedule 1)
Less: Contract billings, 2006 (P3,250,000 x 75%)

2007
2008
P5,950,000 P6,150,000
150,000

6,100,000 6,150,000
98%

100%

P6,000,000 P6,000,000
6,100,000 6,150,000
(100,000) (150,000)
100%
100%
(100,000) (150,000)
5,950,000 6,150,000
5,850,000 6,000,000
5,200,000 6,000,000
P 650,000

P2,625,000
100,000 P2,725,000
2,437,500

Excess of Construction in Progress over Contract Billings (CA)

P 287,500

Schedule 1 Computation of gross profit earned, 2006

2006
2007
P3,250,000 P3,250,000

Contract price
Total estimated cost:
Cost to date
Estimated cost to complete

1,075,000
1,612,500

2,625,000
750,000

2,687,500

3,375,000

Estimated gross profit (loss)


% of completion

562,500
40%

(125,000)

Gross profit (loss) to date


Gross profit earned in prior years

225,000

(125,000)
225,000

P 225,000

P 100,000

Total

Gross profit earned this year

10-22: a
Contract price
Estimated cost:
Cost to date
Estimated costs to complete

2005
P2,800,000

2006
2007
P2,800,000 P2,800,000

1,300,000
1,360,000

1,960,000
780,000

2,440,000
380,000

2,660,000

2,740,000

2,820,000

Estimated gross profit

140,000

60,000

(20,000)

% of completion

48.87%

71.53%

Total

Long-Term Construction Contracts

10-23: b

169

2007
Contract price
Estimated costs:
Cost to date
Estimated cost to complete

Project A
P2,900,000

Project B
P3,400,000

Project C
P 1,700,000

1,680,000
1,120,000

1,440,000
1,760,000

320,000
960,000

2,800,000

3,200,000

1,280,000

100,000
60%

200,000
45%

420,000
25%

P 60,000

P 90,000

P 105,000

Project B
P3,400,000

Project C
P1,700,000

Project D
P 2,000,000

2,120,000
0

1,183,000
1,360,000

560,000
117,000

1,040,000

2,640,000

3,480,000

1,300,000

1,600,000

Total
Estimated gross profit
% of completion
Gross profit earned this year (P255,000)
2008
Contract price
Estimated costs
Cost to date2,640,000
Estimated costs to complete
Total

Project A
P2,900,000

Estimated gross profit (loss)


% of completion

260,000
100%

(80,000)

400,000
91%

400,000
35%

Gross profit (loss) to date


Gross profit earned in prior year

260,000
60,000

(80,000)
90,000

364,000
105,000

140,000
0

P 200,000

P 10,000

P 259,000

P 140,000

2007
P 255,000
120,000

2008
P 609,000
120,000

Gross profit earned this year(P609,000)


Gross profit earned
General and administrative expenses
Net income

10-24: c

P 135,000

Contract price
Gross profit earned to date, 2008 (P900,000 P100,000)

P 489,000
P10,000,000
800,000

Total cost to date, 2008


Less: cost incurred in 2008

10-25: d

9,200,000
4,100,000

Cost to date, 2007

P 5,100,000

Gross profit earned to date


Divided by % of completion:
(P5,100,000 + P900,000) / P10,000,000

Estimated gross profit, 2007

P 1,500,000

Construction in progress:
Cost incurred to date
Gross profit earned to date (P2,500,000 P2,000,000)
Total
Less: Contract billings (P2,500,000 x 30%)
Excess of contract billings over construction in progress (CL)

900,000
60%

P 440,000
110,000
550,000
750,000
P( 200,000)

170

10-26: a

10-27: a

Chapter 10

Contract price
Total estimated cost:
Cost incurred to date:
Site labor cost
Cost of construction materials
Depreciation of special plant & equip
Total
Estimated cost to complete
Estimated gross profit
Percentage of completion (45/100)
Gross profit to be recognized
Cost incurred to date- 2007
Total estimated cost (8,000,000 / 40%)
Estimated cost to complete
Cost incurred in 2007
Cost incurred in 2006
Estimated cost at completion- 2006
Total estimated cost- 2006

P120,000,000
10,000,000
30,000,000
5,000,000
45,000,000
55,000,000

20,000,000
8,000,000

100,000,000
20,000,000
45%
P 9,000,000

P12,000,000
3,700,000
8,300,000
12,450,000
P20,750,000

Percentage of completion- 2006 (8,300,000/ 20,750,000) = 40%


10-28: a
Contract price
Total estimated cost:
Cost incurred to date
Estimated cost to complete
Total estimated cost
Estimated gross profit
Percentage of completion
Gross profit recognized

Contract price
Total estimated cost
Estimated gross profit
Percentage of completion
Gross profit earned to date
Gross profit earned in 2007
Gross profit earned this year

2007
Contract 1
Contract 2
P600,000
P450,000
150,000
150,000
300,000
300,000
50%
P150,000
Contract 1
600,000
350,000
250,000
80%
200,000
150,000
50,000

87,500
162,500
250,000
200,000
35%
P70,000
2008
Contract 2
450,000
300,000
150,000
60%
90,000
70,000
20,000

CIP-2007
P237,500

P220,000
Contract 3
900,000
500,000
400,000
36%
144,000
144,000

Long-Term Construction Contracts

10-29: a

Bicol
Contract price
P875,000
Total estimated cost
Cost incurred
656,250
Est. cost to complete
Total estimated cost
656,250
Estimated gross profit
218,750
Percentage of completion
100%
Gross profit earned
P218,750
Total cost incurred
Total gross profit earned
Construction in progress
Less: Billings
Due from (to)

10-30: a

171

Davao
Aklan
P1,225,000 P437,500
175,000
700,000
875,000
350,000
20%
P 70,000

Percentage of completion
1,006,250
332,500
1,338,750
1,312,500
26,250

Contract price
Total estimated cost:
Cost incurred
Estimated cost to complete
Estimated gross profit
Percentage of completion
Gross profit recognized

175,000
175,000
350,000
87,500
50%
P43,750

Total
1,006,250

332,500

Zero Profit
1,006,250
218,750
1,225,000
1,312,500
(87,500)
P40,825,000

8,475,000
28,400,000

36,875,000
3,950,000
22.983%
P 907,830

172

Chapter 10

SOLUTIONS TO PROBLEMS
Problem 10 1

(a)

Contract Price
Less: Total estimated cost
(1) Cost incurred to date
Estimated costs to complete
(2) Total
Estimated gross profit
Percentage of completion (1 2)
Estimated gross profit to date
Less: Gross profit earned in prior year
Gross profit earned this year
(b)

Contract Price
Less: Total cost incurred
Gross profit

(c)

2007: Construction in Progress


Cost of construction
Construction Revenue
2008: Construction in Progress
Cost of Construction
Construction Revenue

(a)

(b)

2007
P 450,000

2008
P 450,000

200,000
__100,000
__300,000
150,000
______2/3
100,000
_______
P 100,000

320,000
_______
_320,000
130,000
___100%
130,000
__100,000
P 30,000
P 450,000
__320,000
P 130,000

100,000
200,000
30,000
320,000
Problem 10 2

300,000
350,000

Construction Revenue
Less: Cost incurred
Gross profit 2008

P1,250,000
_1,250,000
P
0

Construction in Progress (cost incurred)


Less: Contract billings (P5,800,000 x 30%)
Billings in excess of related costs

P1,250,000
_1,740,000
P(490,000)

Contract price
Less: Total estimated costs
Cost incurred to date
Estimated costs to complete
Estimated gross profit
Percentage of Completion (P1,250,000 500,000)
Gross profit

P5,800,000

Construction on Progress (P1,250,000 + P200,000)


Less: Contract billings
Billings in excess of related costs

P1,250,000
3,740,000

5,000,000
800,000
_____25%
P 200,000
P1,450,000
_1,740,000
P(290,000)

Long-Term Construction Contracts

173

Problem 10 3
2005
2006
P55,000,000 P55,000,000

(a)

Contract Price
Less: Total estimated costs
(1) Cost incurred to date
Estimated costs to complete
(2) Total
Estimated gross profit
Percentage of completion (1 2)
Gross profit earned to date
Gross profit earned in prior yr(s)
Gross profit earned the year

15,000,000
_35,000,000
_50,000,000
5,000,000
______30%
1,500,000
________
P 1,500,000

25,000,000
25,000,000
50,000,000
5,000,000
_____50%
2,500,000
_1,500,000
P 1,000,000
2007

(b)
(1) Construction in Progress
Cash or Payable

15,000,000

(2) Accounts Receivable


Contract Billings

15,000,000

(3) Cash
Accounts Receivable

12,000,000

(4) Construction in Progress


Cost of Construction
Construction Revenue

1,000,000
15,000,000

15,000,000
15,000,000
12,000,000

16,000,000

2007
2008
P55,000,000 P55,000,000
35,000,000
15,000,000
50,000,000
5,000,000
_____70%
3,500,000
_2,500,000
P 1,000,000

50,000,000
________
50,000,000
5,120,000
____100%
5,000,000
_3,500,000
P 1,500,000

2008
15,000,000
15,000,000
20,000,000
25,000,000
1,500,000
15,000,000

20,000,000
25,000,000

16,500,000

Problem 10 4
(a)
Cost incurred to date
Divide by total estimated cost
Percentage of Completion
(b)

Contract Price
Less: Total Estimated Cost
Cost incurred to date
Estimated costs to complete
Total
Estimated gross profit
Percentage of completion
Gross profit earned to date
Less: Gross profit earned in prior yrs.
Gross profit earned this year

2006
P 1,000,000
P 9,000,000
11.11%

2007
2008
P 5,500,000 P10,000,000
P11,000,000 _12,000,000
50%
83.33%

2006
P15,000,000

2007
2008
P15,000,000 P15,000,000

1,000,000
__8,000,000
__9,000,000
6,000,000
___11.11%
666,600
________
P 666,600

5,500,000 10,000,000
__5,500,000 __2,000,000
_11,000,000 _12,000,000
4,000,000
3,000,000
______50% ___83.33%
2000,000
9,500,000
___666,600 _2,000,000
P 1,333,400 P 500,000

174

(c)

Chapter 10

(1) Construction in progress (cost incurred)


Cash

1,000,000

(2) Accounts Receivable


Contract Billings

1,325,000

(3) Cash
Accounts Receivable

1,200,000

(4) Construction in progress (gross profit)


Cost of construction
Construction Revenue

666,600
1,000,000

1,000,000
1,325,000
1,200,000

1,666,600

Problem 10 5
2005
P14,000,000

(1)

Contract Price
Less: Total Estimated Cost
Cost incurred to date
6,500,000
Estimated cost to complete
__6,800,000
Total
_13,300,000
Estimated gross profit
700,000
Percentage of completion
___48.87%
Gross profit (loss) to date
342,090
Less: Gross profit (loss) in prior yrs. ________
Gross profit (loss) this year
P 342,090
(2)

2005

2006
P14,000,000

2007
2008
P14,000,000 P14,000,000

9,800,000
_3,900,000
13,700,000
300,000
___71.53%
214,590
___342,090
P( 127,500)

12,200,000
_1,900,000
14,100,000
( 100,000)
_____100%
( 100,000)
___214,590
P( 314,590)

2006

2007

13,900,000
________
13,900,000
100,000
____100%
100,000
( 100,000)
P 200,000
2008

Cost of construction
6,500,000
3,300,000
2,400,000
1,700,000
Construction in progress 342,090
127,500
314,590
200,000
Construction Revenue
6,842,090
3,172,500
2,085,410
1,900,000

Problem 10 6
(1)
Contract Price
Less: Total estimated costs
Cost incurred to date
Estimated costs to complete
Total
Estimated gross profit
Percentage of completion
Gross profit (loss) to date
Gross profit (loss) in prior yrs.
Gross profit (loss) this year

2005
P 6,000,000

2006
P 6,000,000

2007
P 6,000,000

3,400,000
_2,100,000
_5,500,000
500,000
___61.82%
309,100
________
P 309,100

5,950,000
___150,000
_6,100,000
( 100,000)
_______
( 100,000)
__309,100
P 409,100

6,150,000
________
_6,150,000
( 150,000)
________
( 150,000)
( 100,000)
P 50,000

Long-Term Construction Contracts

(2)
Cost of construction
Construction in progress
Construction Revenue
(3)

175

2005
2006
3,400,000
2,550,000
309,100
409,100
3,709,100
2,140,900

Cash
Accounts Receivable
Contract Billings
Construction in progress

400,000
6,000,000

2007
200,000

50,000
150,000

400,000
6,000,000

Problem 10 7
2006
P16,000,000

(1)

Contract Price
Less:Total Estimated Cost
Cost incurred to date
4,600,000
Estimated costs to complete
__9,640,000
Total
_14,240,000
Estimated gross profit
1,760,000
Engineer's estimate of comp.
______31%
Gross profit to date
545,600
Less: Gross profit earned in prior yrs. ________
Gross profit earned this yr.
P 545,600
(2)

(a) Construction on progress


Cash

2006
4,600,000

(b) Accounts receivable


Contract billings

5,000,000

(c) Cash
Accounts receivable

4,500,000

2007
P16,000,000

2008
P16,000,000

9,100,000
__5,100,000
_14,200,000
1,800,000
______58%
1,044,000
__545,600
P 498,410

14,350,000
_________
_14,350,000
1,650,000
_____100%
1,650,000
_1,044,000
P 606,000

4,600,000
5,000,000
4,500,000

2007
4,500,000
6,000,000
5,400,000

4,500,000
6,000,000
5,400,000

2008
5,250,000
5,000,000
6,100,000

5,250,000
5,000,000
6,100,000

(d) Cost of constructions


4,600,000
4,500,000
5,250,000
Construction in progress
545,600
498,400
606,000
Construction revenue
5,145,600
4,998,400
5,856,000
(e) Contract billings
16,000,000
Construction on progress
16,000,000
(3)

Zero Profit Method: 2008 Entres


(a) Construction in progress
Cash / accounts payable
(b) Accounts receivable
Contract billings

5,250,000
5,000,000

5,250,000
5,000,000

176

Chapter 10

(c) Cash
Accounts receivable

6,100,000

(d) Cost of construction


Construction in progress
Construction revenue

5,250,000
1,650,000

(e) Contract billings


Construction in progress
(4)

16,000,000

6,100,000

6,900,000
16,000,000

The following entry would be the only one different from (2).
2006
2007
2008
4,414,400
3,821,600
6,114,000
545,600
498,400
606,000
4,960,000
4,320,000
6,720,000

Cost of construction
Construction in progress
Construction revenue

Total estimated costs x estimated percentage of completion.


Problem 10 8

(1)
Contract Price
Less:Total Estimated Costs
Cost incurred to date
Estimated costs to complete
Total
Estimated gross profit (loss)
Less: Gross profit (loss) in prior yrs.
Gross profit (loss) this years
(2)

In 2008 when the project is completed.

2006
P6,500,000

2007
P6,500,000

2008
P6,500,000

2,150,000
_3,850,000
_6,000,000
500,000
________
P 520,000

5,250,000
_1,500,000
_6,750,000
(250,000)
___520,000
P( 250,000)

6,850,000
________
_6,850,000
(350,000)
_(250,000)
P( 600,000)

Franchise Accounting

177

CHAPTER 11
MULTIPLE CHOICE ANSWERS AND SOLUTIONS
11-1: b

11-2: c

11-3: a

11-4: b

11-5: a

11-6: b

11-7: d

11-8: d

No revenue is to be reported. Because the franchisor fails to render substantial


services to the franchisee as of December 31, 2008.
Initial franchise fee
Less: Cost of franchise
Net income

P5,000,000
____50,000
P4,950,000

The total initial franchise fee of P500,000 is to be recognized as earned because the
collectibility of the note for the balance is reasonably assured.
Cash downpayment
Collection of note applying to principal
Revenue from initial franchise fee

P 100,000
__200,000
P 300,000

Cash downpayment, January 2, 2008


Collection applying to principal, December 31, 2008
Total Collection
Gross profit rate [(5,000,000-500,000) 5,000,000]
Realized gross profit, December 31, 2008

P2,000,000
_1,000,000
3,000,000
_____90%
P2,700,000

Face value of the note (P1,200,000 - P400,000)


Present value of the note (P200,000 X 2.91)
Unearned interest income, July 1, 2008

P 800,000
__582,000
P 218,000

Initial franchise fee


Less: unearned interest income
Deferred revenue from franchise fee

P1,200,000
__218,000
P 982,000

Initial franchise fee


Continuing franchise fee (P400,000 X .05)
Total revenue
Cost
Net income

P 500,000
___20,000
520,000
___10,000
P 510,000

178

11-9: b

Chapter 11

Deferred Revenue from franchise fee:


Downpayment
Present value of the note (P1,000,000 X 2.91)
Less: Cost of franchise fee

P6,000,000
2,910,000

Deferred gross profit


Gross profit rate (6,910,000 8,910,000)

11-10: b

11-11: a

11-12: d

P8,910,000
_2,000,000
P6,910,000
77.55%

Downpayment (collection during 2008)


Gross profit rate

P6,000,000
___77.55%

Realized gross profit from initial franchise fee


Add: Continuing franchise fee (5,000,000 X .05)

P4,653,000
__250,000

Total
Less: Franchise expense
Operating income
Interest income, 12/31/05 (P2,910,000 X 14%) X 6/12
Net income

P4,903,000
___50,000
P4,853,000
__203,700
P5,056,700

Face value of the note receivable


Present value of the note receivable

P1,800,000
1,263,900

Unearned interest income

P 536,100

Initial franchise fee


Less: Unearned interest income

P3,000,000
__ 536,100

Deferred revenue from franchise fee

P2,463,900

Revenues from:
Initial franchise fee
Continuing franchise fee (P2,000,000 X .05)
Total revenue from franchise fees

P1,000,000
100,000
P1,100,000

Realized gross profit from initial franchise fee [(350,000 + 90,000) x 37%]
Continuing franchise fee (P121,000 + P147,500) x 5%

P 162,800
___13,425

Total revenue
Expenses

176,225
___42,900

Net operating profit


Interest income (P900,000 x 15%) x 6/12

133,325
___67,500

Net income

P 200,825

Franchise Accounting

11-13: c

11-14: a

11-15: c

11-16: a

179

Cash down-payment
Present of the note (P40,000 x 3.0374)

P 95,000
__121,496

Total

P 216,496

Initial franchise fee


Continuing franchise fee (P400,000 x 5%)

P 50,000
__20,000

Total revenue

P 70,000

Should be P80,000
Initial franchise fee down-payment (P100,000 / 5)
Continuing franchise fee (P500,000 x 12%)

P 20,000
__60,000

Total earned franchise fee

P 80,000

The unearned interest credited is the difference between the face value and the
present value of the notes receivable (900,000 720000).
The down payment of P600,000 is recognized as revenue since it is a fair
measure of the services already performed by the franchisor.

11-17: b

11-18:

Cora (P100,000 + P500,000)


Dora (P100,000 + P500,000)
Total

P 600,000
600,000
P1,200,000

Down payment (3,125,000 x 40%)


Present value of notes receivable ( 1,875,000/4) 468,750 x 3.04
Adjusted sales value of initial franchise fee
Direct cost of services
Gross profit

P1,250,000
1,425,000
2,675,000
802,500
1,872,500

Gross profit rate (1,872,500 2,675,000)

70%

180

Chapter 11

Date
Collection
Interest
1/1
6/30
468,750
171,000
12/30
468,750
135,270
Total collection applying to principal
Down payment
Total collection
Gross profit rate
Realized gross profit on
initial franchise fee
11-19: c

Principal
297,750
333,480
631,230
1,250,000
1,881,230
70%
1,316,861

Balance of PV of NR
P1,425,000
1,127,250
793,770

Franchise Accounting

181

SOLUTIONS TO PROBLEMS
Problem 11 1
a.

The collectibility of the note is reasonably assured.


Jan. 2:

July 31:

Cash ..... ..............................................................................12,000,000


Notes receivable................................................................. 8,000,000
Deferred Revenue from IFF. ........................................

20,000,000

Deferred cost of Franchises................................................ 2,000,000


Cash ..............................................................................

2,000,000

Nov. 30: Cash/AR ............................................................................


Revenue from continuing franchise fee (CFF)..............

29,000

Dec. 31: Cash / AR ..........................................................................


Revenue from CFF ........................................................

36,000

36,000

Cash .... .............................................................................. 2,800,000


Notes receivable ............................................................
Interest income (P8,000,000 x 10%) .............................

2,000,000
800,000

Adjusting Entries:
(1)
Cost of franchise revenue ........................................... 2,000,000
Deferred cost of franchises...................................

2,000,000

(2)

Deferred revenue from IFF .........................................20,000,000


Revenue from IFF...................................................
To recognize revenue from the initial franchise fee.

b.

29,000

20,000,000

The collectibility of the note is not reasonably assured.


Jan. 2 to Dec. 31 = Refer to assumption a.
Adjusting entry: to recognized revenue from the initial franchise fee (installment method)
(1)

(2)

To defer gross profit:


Deferred Revenue from IFF........................................20,000,000
Cost of Franchise Revenue...................................
Deferred gross profit Franchises .......................
GPR = P18,000 P20,000,000 = 90%
To recognize gross profit:
Deferred gross profit Franchises..............................12,600,000
Realized gross profit.............................................
(P14,000,000 X 90%)

2,000,000
18,000,000

12,600,000

182

a.

Chapter 11

Problem 11 2
Collection of the note is reasonably assured.
Jan. 5: Cash .. ..... .............................................................................. 600,000
Notes Receivable ................................................................... 1,000,000
Unearned interest income..................................................
Deferred revenue from F.F................................................
Face value of NR ............................................................................
Present value (P200,000 x P2,9906)...............................................
Unearned interest ............................................................................

179,718

Dec. 31: Cash / AR ..........................................................................


Revenue from CFF ........................................................
(P80,000 X 5%)

4,000

Cash .... ..............................................................................


Notes Receivable...........................................................

200,000

Adjusting Entries:
1) Unearned interest income .................................................. 119,624
Interest income............................................................
P598,120 x 20%

b.

179,718

3) Deferred revenue from FF................................................. 1,198,120


Revenue from FF ........................................................
Collection of the note is not reasonably assured.
Jan. 5 to Dec. 31 before adjusting entries Refer to Assumption a.
Dec. 31: Adjusting Entries:
1) Unearned interest income ................................................. 119,624
Interest income ...........................................................
2) Cost of franchise................................................................
Deferred cost of franchise...........................................

1,000,000
__598,120
401,880

Nov. 25: Deferred cost of Franchise ................................................


Cash ..............................................................................

2) Cost of Franchise...............................................................
Deferred cost of Franchise..........................................

401,880
1,198,120

179,718

179,718
4,000

200,000

119,624

179,718
1,198,120

119,624
179,718

3) Deferred revenue from FF................................................. 1,198,120


Cost of Franchise ........................................................
Deferred gross profit Franchise ...............................
GPR = 1,018,402 1,198,120 = 85%)

179,718
1,018,402

4) Deferred gross profit Franchise......................................578,319.60


Realized gross profit Franchise................................
(P600,000 + P200,000- P119,624) x 85%

578,319.60

Franchise Accounting

183

Problem 11 3
2007
July 1:

Cash.. ...... ..... .............................................................................. 120,000


Notes Receivable.......................................................................... 320,000
Unearned interest income ......................................................
Deferred revenue from FF .....................................................
Face value of NR.......................................................................... P320,000
Present value (P80,000 x 3.1699)................................................. _253,592
Unearned interest income............................................................. P 66,408

Sept. 1 to
Nov. 15: Deferred cost of franchise ............................................................
Cash .. ..... ..............................................................................
(P50,000 + P30,000)
Dec. 31: Adjusting Entry:
Unearned interest income.............................................................
Interest income ......................................................................
(P253,592 x 10% x 1/2)

80,000

12,680

2008
Jan. 10: Deferred cost of franchise ............................................................
Cash .. ..... ..............................................................................

50,000

July 1:

80,000

Cash.. ...... ..... ..............................................................................


Note receivable ......................................................................

Dec. 31: Adjusting Entries:


(1) Cost of franchise ....................................................................
Deferred cost of franchise .................................................

130,000

(2) Deferred revenue from FF .....................................................


Revenue from FF...............................................................

373,592

(3) Unearned interest income ......................................................


Interest income ..................................................................

25,360

66,408
373,592

80,000

12,680

50,000
80,000

130,000
373,592
25,360

184

Chapter 11

Problem 11 4
2008
Jan. 10: Cash.. ...... ..... .............................................................................. 6,000,000
Deferred revenue from FF. ....................................................

6,000,000

Jan. 10 to
July 15: Franchise expense ........................................................................ 2,250,000
Cash .. ..... ..............................................................................

2,250,000

Deferred revenue from FF............................................................ 4,000,000


Revenue from FF ...................................................................
Initial Franchise fee .....................................................................P6,000,000
Deficiency
Market value of costs (P180,000 90%) x 10 yrs.................( 2,000,000)
Adjusted initial fee (revenue) .......................................................P4,000,000
July 15: (a) Continuing expenses..............................................................
Cash / Accounts payable ...................................................

a)

b)

180,000

(b) Deferred revenue from FF ..................................................... 200,000


Revenue from CFF ............................................................
(P180,000 90%)
Problem 11 5
Adjusted initial franchise fee:
Total initial F.F.............................................................................
Less: Face Market value of kitchen equipment............................
Adjusted initial FF........................................................................
Revenues:
Initial FF.. ..... ..............................................................................
Sale of kitchen equipment ............................................................
Continuing F.F. (P2,000,000 x 2%) .............................................
Total . ...... ..... ..............................................................................
Expenses:
Initial expenses.............................................................................P 500,000
Cost of kitchen equipment............................................................ 1,500,000
Net income..... ..... ..............................................................................

4,000,000

180,000
200,000

P4,500,000
_1,800,000
P2,700,000
P2,700,000
1,800,000
___40,000
4,540,000
_2,000,000
P2,540,000

Journal Entries:
Jan. 2: Cash .. ..... .............................................................................. 1,500,000
Notes receivable..................................................................... 3,000,000
Deferred revenue from FF (adjusted SV)..........................
Revenue from FF (Market value of equipment)................

2,700,000
1,800,000

Cost of kitchen equipment ..................................................... 1,500,000


Kitchen equipment ............................................................

1,500,000

Franchise Accounting

185

Jan. 18: Franchise expense ........................................................................


Cash.... ..............................................................................

500,000

500,000

April 1: Cash ...... ..... ..............................................................................2,000,000


Notes receivable ................................................................

2,000,000

Dec. 31: Cash ...... ..... ..............................................................................1,000,000


Notes receivable ................................................................

1,000,000

Cash / Account receivable............................................................


Revenue from continuing FF.............................................

40,000

Deferred revenue from FF............................................................ 2,700,000


Revenue from FF...............................................................

40,000
2,700,000

Problem 11 6
Recognition of initial franchise fee (IFF) (6 mos. after opening)
Revenue from initial FF:
Total initial FF ..... ..............................................................................P2,500,000
Less: Deficiency in continuing FF (Sch. 1) ........................................ 160,000
Expense (costs of initial services) ...............................................................
Net income .. ... ...... ..... ..............................................................................
Schedule 1 Estimated deficiency in CFF
(1)
Yr. of
Estimated
Contract
Continuing FF
1
P220,000
2
220,000
3
220,000
4
220,000
5
220,000
6
150,000
7
150,000
8
150,000
9
90,000
10
90,000

(2)
Market Value
of Continuing Services
P250,000
250,000
250,000
125,000
125,000
125,000
125,000
125,000
125,000
125,000

2,340,000
__700,000
P1,640,000

(Excess of 2 over 1)
Deficiency
P 30,000
30,000
30,000

35,000
__35,000
P160,000

Recognition of revenue from CFF and costs:


Years 1-3
Revenue from CFF ........................ P250,000
Expenses . ...... ..... ......................... _200,000
Net income..... ..... ......................... P 50,000

Years 4-5
P220,000
_100,000
P120,000

Years 6-8
P150,000
_100,000
P 50,000

Years 9-10
P125,000
_100,000
P 25,000

186

Chapter 11

Problem 11 7

Revenues:
Initial FF (Sch. 1)
Interest income
Continuing FF
Others
Expenses:
Initial expenses
Continuing expense
Others
Net Income

1/12/2008

6/1/2008

7/1/2008

6/30/2009

62,500

80,000

287,200

45,490*
48,000

( 50,000)
P 12,500

( 68,000)
P 12,000

( 70,000)

P217,200

( 36,000)

P 57,490

* P454,900 x 10% = P45,490


Schedule 1: Computation of initial FF to the recognized:
Total initial fee ...... ...................................................................................................
Less:
Interest unearned on the note ........................................................................
Market value of inventory ............................................................................
Market value of equipment ...........................................................................
Deficiency in continuing costs......................................................................
Adjusted initial FF .. ...................................................................................................
A.

B.

P750,000
( 145,100)
( 80,000)
( 62,500
( 175,200)
P287,200

A
B
B
C

Unearned Interest:
Face value of the note ..........................................................................................
Present value (120,000 x 3.7908) ........................................................................
Unearned interest .................................................................................................

P600,000
454,900
P145,100

rounded

Market value of equipment and inventory:


Equipment (P50,000 80%)................................................................................
Inventory... ...... ...................................................................................................

P 62,500
80,000

Income from Sales:


Sales Price. ...... ..........................................
Cost.... ...... ...... ..........................................
Net income ...... ..........................................
C.

Equipment
P62,500
50,000
P12,500

Analysis of Continuing costs:


Market value of costs is P4,000/Mo. or P48,000 / yr.
Continuing Fees:
Years 1-4
Gross revenues ..........................................
P330,000/mo.
Gross fees per month ..................................
P 2,475/mo.
Gross fees per year......................................
Market value of continuing costs ................
Deficiency per year .....................................
Number of years .........................................
Deficiency
..........................................
Total deficiency for 20 years is P175,200

P 29,700
( 48,000)
( 18,300)
x4
P( 73,200)

Inventory
P80,000
68,000
P12,000

Total
P142,500
118,000
P 24,500

Years 5-16
P450,000/mo.
P 3,375/mo.

Years 17-20
P500,000/mo.
P 3,750/mo.

P 40,500
( 48,000)
(
7,500)
x 12
P( 90,000)

P 45,000
( 48,000)
(
3,000)
x4
P( 12,000)

Franchise Accounting

Dates of Revenue Recognition: .....................................................


January 12, 2008 ............................................................
June 1, 2008 ...................................................................
July 1, 2008 ....................................................................
June 30, 2009 .................................................................

187

Types of Revenue
Sale of equipment
Sale of inventory
Initial FF (as adjusted0
Interest income and
continuing revenue.

CHAPTER 12
MULTIPLE CHOICE
12-1:

d. This is recorded when the working fund is replenished.

12-2:

c.
Sales
Cost of goods sold:
Purchases
Merchandise inventory, end
Gross profit
Expenses
Net income (loss)

12-3:

P800,000
180,000

620,000
P 80,000
198,000
P (118,000)

b
Sales
Cost of goods sold (P70,000 / 140%)
Gross profit
Less: Samples (P8,000 P6,000)
Expenses
Net income

12-4:

P 700,000

P 2,000
2,800

P 70,000
50,000
P 20,000
4,800
P 15,200

a
Sales
Cost of goods sold
Gross profit
Expenses (P9,000 + P4,500)
Net income

12-5:

12-6:

12-7:

12-8

a
Shipment of merchandise to home office
Equipment sent to home office
Expenses assigned to branch by the home office
Cash remittance to home office
Home office account balance

P 100,000
72,000
P 28,000
13,500
P 14,500

P 80,000
50,000
8,000
(40,000)
P 98,000

12-9: d
12-10: a
Home Office account balance before closing, Dec. 32, 1008
P 35,000
Net income (loss)
Sales
P147,000
Cost of cost goods sold
Shipment to branch
P135,000
Inventory, 12/31
18,500 116,500
Gross profit
P 30,500
Expenses
13,500
17,000
Home Office account balance (Investment in Branch account balance) P 52,000
Shipment to Branch account has no beginning balance, because this was closed at the end
of 2008.
12-11: b

Jan. 1, 2008
P 6,000
86,000
74,000
P166,000

Petty cash fund


Accounts receivable
Inventory
Home Office account balance

Jan. 1, 2009
P 6,000
98,000
82,000
P186,000

12-12: d

Unadjusted balances, Dec. 31


Remittance in transit
Shipment in transit
Cash collections of home office
Adjusted balances, Dec. 31

(Branch Books)
Home Office
P 21,320
7,280
400)
P 28,200
(

(Home Office Books)


Investment in Branch
P 38,600
(10,400)
P 28,200

12-13: a
Unadjusted balance Investment in Branch account, 12/31
Charge for advances by president
Erroneous entry for merchandise allowance
Share in advertising expense
Unadjusted balance Home Office account, 12/31

P430,000
(5,500)
( 600)
(9,000)
P414,900

12-14: a
Unadjusted balances, 12/31
Shipment in transit
Collection of HO A/R by branch
Error in recording of branch profit
Returns of merchandise in transit
Adjusted balances, 12/31

(Branch Books)
Home Office
P 97,350
6,150

P103,500

12-15: a

(Branch Books)
Home Office
Unadjusted balances
P25,550
Error in recording shipment to Cavity branch
(12,000)
Error in recording shipment to Tagaytay branch 15,000
Branch AR collected by home office
Merchandise returns in transit
( 1,200)
Error in recording branch profit
( 3,600)
Adjusted balances
P23,750

12-16: c
Unadjusted balance- Investment in Branch account
Remittance in transit
Shipment in transit
Expenses allocated
Error in recording remittance
Error in recording shipments
Unadjusted balance Home Office account

Unadjusted balances,
Remittance in transit
Shipment in transit
Expenses allocated
Unrecorded HO collection of AR
Error in recording shipments
Adjusted balances
12-17 a
Unadjusted balances
Branch AR collected by Home Office
Shipments in transit
Acquisition of furniture
Merchandise returns
Cash remittance in transit
Adjusted balances

( Branch Books)
Home Office
P 44,000
20,000
5,000
(3,000)
9,000
P 75,000
(Branch Books)
Home Office
P 440,000
( 8,000)
32,000

P 464,000

(Home Office Books)


Investment in Branch
P 84,000
25,000
900
( 6,400)
P103,500
(Home Office Books)
Investment in Branch
P27,350
(3,000)
P23,750
P 85,000
(10,000)
(20,000)
( 5,000)
3,000
( 9,000)
P 44,000
(HomeOffice Books)
Investment in Branch
P 85,000
(10,000)

P 75,000
(Home Office Books)
Investment in Branch
P 496,000
(12,000)
(15,000)
( 5,000)
P 464,000

PROBLEMS
Problem 12-1
Home Office Books
1. Investment in branch
Cash

30,000

2. Investment in branch
Shipment to branch

75,000

Branch Books
30,000

Cash

Home office

30,000

Shipment from home office


Home office

75,000

3. No entry

Purchases
Accounts payable

10,000

4. No entry

Accounts receivable
Sales

5. Shipment to branch
Investment in branch

2,000

75,000

2,000

Home office
2,000
Shipment from home office

6. No entry

Cash

7. No entry

Accounts payable
Cash

8. No entry

Salaries
Rent
Utilities
Other operating expenses
Cash

9. Investment in branch
Accumulated depn

10. Cash
Investment in branch
11. Cash

Investment in branch

12. Investment in branch


Branch income

7,500

65,000
3,000
10,000

7,500

65,000
3,000
10,000

125,000

Accounts receivable

Depreciation
Rent
Insurance
Home office
Home office
Cash
Home office
Accounts receivable

105,000
7,000
10,000
5,000
2,000
12,000
1,500
5,000
1,000
65,000
3,000

Sales
125,000
Inventory, end
5,000
Shipment from HO
Purchases
Salaries
Rent
Utilities
Other operating expenses
Home office

30,000
75,000
10,000
125,000
2,000
105,000
7,000

29,500

7,500
65,000
3,000

73,000
10,000
10,000
10,000
2,000
12,500
10,000

Problem 12-2
a.

Books of the Branch


1.

Cash
Merchandise inventory
Home office

200,000
350,000

2.

Merchandise inventory
Accounts payable

400,000

3.

Accounts receivable
Sales

650,000

Cost of goods sold


Merchandise inventory

425,000

Cash

600,000

4.

5.

b.

Accounts receivable

Advertising expense
Sales commission
Other expense
Cash

400,000
650,000
425,000
600,000

40,000
65,000
45,000

Accounts payable
Home office
Cash

150,000

370,000
120,000

490,000

Manila Sales Naga Branch


Income Statement
Year Ended December 31, 2008
Sales
Cost of goods sold
Gross profit
Expenses:
Advertising expense
Sales commissions
Other expenses
Net income

c.

550,000

P650,000
425,000
225,000
P40,000
65,000
45,000

150,000
P 75,000

Manila Sales Naga Branch


Balance Sheet
December 31, 2008
Cash
Accounts receivable
Merchandise inventory
Total assets

P160,000
50,000
325,000
P535,000

Accounts payable
Home office

Total liabilities and capital

30,000
505,000
P535,000

Problem 12-3
Home Office Books

(1) Adjusting Entries

a. Investment in branch
Cash

63,750

b. Investment in branch
Shipment to branch

75,300

c. Accounts receivable
Sales

157,500

d. Purchases
Accounts payable

183,750

e. Cash

170,400

Accounts receivable

f. Accounts payable
Cash
g. Expenses
Cash
Furniture & fixtures branch
Investment in branch
h. Cash

Investment in branch

63,750

12,000
80,100

15,000

63,750

75,300

157,500

Accounts receivable
Sales

99,000

183,750

Purchases
33,750
Accounts payable
33,750

39,900
12,000
80,100

Cash

80,100
Accounts receivable

Home office
Cash

80,100

Accounts payable
Cash

18,375

75,300
99,000

80,100
80,100
18,375

Home office
Cash

12,000

12,000

Expenses
Cash

i. Retained earnings
Cash

63,750

Shipment from HO
Home office

186,000

39,900

Home office

73,300

170,400

186,000

Cash

Branch Books

27,000

27,000

15.000

(2) Adjusting Entries


j. Expenses
Acc. Depreciation

1,750

k. Investment in branch
Acc. Depn Br. F & F

975

l. Prepaid expenses
Expenses

375

m. Expenses
Accrued expenses

150

1,750
975

Expenses
Home office

375

Prepaid expenses
Expenses

150

Expenses
Accrued expenses

975
1,125
450

975
1,125
450

Home Office Books


n. Sales
Shipments to branch
Merchandise inv., 12/31
Merchandise inv. 1/1
Purchases
Expenses
Income summary

157,500
75,300
72,750

o. Branch loss
Investment in branch

2,100

p. Income summary
Branch loss

2,100

q. Income summary
Retained earnings

3.

Closing Entries

60,180
183,750
41,445
20,175

18,075

2,100

Branch Books
Sales
99,000
Merchandise inv., 12/31 35,250
Income summary
2,100
Purchases
33,750
Shipment from HO
75,300
Expenses
27,300
Home office
2,100
Income summary

2,100

2,100
18,075

Individual Financial Statements


Cebu Company Home Office
Income Statement
Year Ended December 31, 2008
Sales
Cost of sales
Merchandise inventory, 1/1
Purchases
Goods available for sale
Shipment to branch
Goods available for own sale
Merchandise inventory, 12/31
Gross profit
Expenses
Net operating income
Branch income (loss)
Net income

P157,500
P 60,180
183,750
P243,930
( 75,300)
P168,630
( 72,750)

95,880
P 61,620
41,445
P 20,175
( 2,100)
P 18,075

Cebu Company Branch


Income Statement
Year Ended December 31, 2008
Sales
Cost of sales
Purchases
Shipments from home office
Goods available for sale
Merchandise inventory, 12/31
Gross profit
Expenses
Net income (loss)

P 99,000
P 33,750
75,300
P109,050
35,250

73,800
P 25,200
27,300
P( 2,100)

Cebu Company Home Office


Balance Sheet
December 31, 2008
Assets
Cash
Accounts receivable
Merchandise inventory, 12/31
Prepaid expenses
Furniture and fixtures
Less: Accumulated depreciation
Branch furniture and fixtures
Less: Accumulated depreciation
Investment in branch
Total assets
Liabilities and Stockholders Equity
Liabilities
Accrued expenses
Accounts payable
Total liabilities
Stockholders Equity
Capital stock
Retained earnings
Total liabilities and stockholders equity

P30,000
8,370
P12,000
975

P 34,800
28,575
72,750
3,075
21,630
11,025
45,825
P217,680

P 2,025
31,950
P 33,975
P 75,000
108,705

183,705
P217,680

Cebu Company Branch


Balance Sheet
December 31, 2008
Assets
Cash
Accounts receivable
Merchandise inventory, 12/31
Prepaid expenses
Total assets

P 6,375
18,000
35,250
1,125
P61,650

Liabilities and Capital


Accounts payable
Home office
Total liabilities and capital

P 450
15,375
P61,650

4.

Combined Financial Statements


Cebu Company
Combined Income Statement
Year Ended December 31, 2008
Sales
Cost of sales
Merchandise inventory, 1/1
Purchases
Goods available for sale
Merchandise inventory, 12/31
Gross profit
Expenses
Combined net income

P256,500
P 60,180
217,500
P277,680
108,000

169,680
P 86,820
68,745
P 18,075

Cebu Company
Balance Sheet
December 31, 2008
Assets
Cash
Accounts receivable
Merchandise inventory
Prepaid expenses
Furniture and fixtures
Less: accumulated depreciation
Total assets

P 41,175
47,475
108,000
4,200

P42,000
9,345

32,655
P233,505

Liabilities and Stockholders Equity


Accrued expenses
Accounts payable
Capital stock
Retained earnings
Total liabilities and stockholders equity

2,475
47,325
75,000
108,705
P233,505

Problem 12-4
Branch Books
Sales
145,000
Inventory, 12/31
60,000
Inventory, 1/1
Shipments from HO
Expenses
Income summary
Income summary
Home office

22,000

(a) and (b) Closing Entries

18,000
145,000
20,000
23,000

22,000

Home Office Books

Sales
560,000
Inventory, 12/31
90,000
Shipments to branch
145,000
Inventory, 1/1
Purchases
Expenses
Income summary
Investment in branch
Branch income

22,000

45,000
540,000
90,000
120,000
22,000

Branch income
22,000
Income summary
22,000
Income summary
142,000
Retained earnings
142,000

CG Corporation
Combined Statement Working Paper
Year Ended December 31, 2008
Eliminations

Debits
Cash
Accounts receivable
Inventory, 1/1
Investment in branch
Equipment (net)
Purchases
Shipments from HO
Expenses
Total debits

Home
Office
36,000
54,000
45,000
70,000
95,000
540,000
90,000
930,000

Branch

Debit

7,000
29,000
18,000

Credit

(2) 70,000

145,000
20,000
219,000

(1)145,000

Income
Statement
Dr (Cr)

63,000
540,000

95,000

150,000
371,000

Credits
Accounts payable
Home Office
Capital stock
Retained earnings, 1/1
Sales
Shipments to branch
Total credits

54,000
144,000
560,000
145,000
930,000

145,000

Inventory, 12/31 (IS)

90,000

60,000

27,000

4,000
70,000

219,000

31,000

(2) 70,000
(705,000)

(1)145,000

215,000

Total liabilities & equity


1.
2.

43,000
83,000

110,000

Inventory 12/31 (BS)


Total assets

Net income

Balance
Sheet

215,000

54,000
144,000

(150,000)
142,000

142,000
371,000

To eliminate shipments to branch and shipments from HO


To eliminate reciprocal accounts.

10

Problem 12-5
(1)

Oro Company
Working Paper for Combined Statements
Year Ended December 31, 2008

Debits
Cash
Notes receivable
Accounts receivable (net)
Inventories
Furniture & fixtures (net)
Investment in Branch
Cost of goods sold
Operating expenses
Totals
Credits
Accounts payable
Common stock
Retained earnings
Home Office
Sales
Totals

Home
Office

Branch

63,000
10,500
120,600
143,700
72,150
124,050
300,750
104,250

128,700
32,850

939,000

275,700

Income
Statements
Dr (CR)

21,900
55,950
36,300

61,500
300,000
37,500

124,050
151,650

540,000
939,000

Eliminations
Debit
Credit

275,700

84,900
10,500
176,550
45,000
72,150

(2)135,000
(2)135,000

(1)124,050

Balance
Sheet

564,050
137,100
389,100
61,500
300,000
37,500

(1)124,050
289,050

(691,650)
289,050

Net Income

9,900

(1) To eliminate shipments


(2) To eliminate reciprocal accounts.

(9,900)
389,100

Closing Entries
2.

Branch Books

Sales
Income Summary
Cost of goods sold
Operating expenses
Home Office
Income summary

3. Home Office Books


151,650
9,900

9,900

128,700
32,850
9,900

Branch loss
9,900
Investment in Branch

9,900

Income summary
Branch loss

9,900

9,900

11

Problem 12-6
a.

b.

Investment in Branch account (Home Office Books)


Unadjusted balance
Error in recording cash transfer, April 8
Cash transfer recorded in subsequent year, Dec. 31
Error in recording allocated depreciation, Dec. 31
Adjusted balance

P138,200
( 45,000)
( 15,000)
6,000
P 84,200

Home Office account (Branch Books)


Unadjusted balance
Error in recording salary allocation, April 5
Error in recording inventory transfer, July 6
Unrecorded allocated depreciation, Dec. 31
Adjusted balance

P(93,000)
( 200)
12,000
( 3,000)
P(84,200)

Adjusting Entries
Home Office Books

Other income
Investment in branch
Rizal

45,000

Cash

15,000

Investment in branchRizal

Investment in branch
Accumulated depn

Branch Books

45,000

15,000
6,000

6,000

Salary expense
Home office

200

200

Home office
12,000
Shipments from HO

12,000

Depreciation expense
Home office

3,000

3,000

Problem 12-7
a.

b.

Investment in Branch account (Home Office Books)


Unadjusted balance, Dec. 31
Cash remittance in transit
Merchandise returns in transit
Adjusted balance, Dec. 31

P166,400
(30,000)
(12,000)
P124,400

Home Office account (Branch Books)


Unadjusted balance, Dec. 31
Error in recording expense
Shipment in transit
Supplies charged to branch
Collection of branch receivable
Adjusted balance, Dec. 31

P103,200
7,200
24,000
8,000
( 18,000)
P124,400

Adjusting Entries

Home Office Books


Cash
30,000
Shipment to branch
12,000
Investment in branch
42,000

Branch Books
Shipment from HO
24,000
Supplies
8,000
Expenses
7,200
Accounts receivable
18,000
Home office
21,200

12

Problem 12-8
(1)

Reconciliation Statement

(Home Office Books) (Branch Books)


Investment in Branch
Home Office

Unadjusted balances, 1/31


Advertising charged to branch
Home office AR collected by branch
Shipment in transit
Error in recording receipt of merchandise
Understatement of depreciation
Remittance in transit, 1/31
(2)

P59,720
600
( 432)
(12,800)
P47,088

P 43,268
480
(

180)

P 47,088

Adjusting Entries
Home Office Books

Retained earnings
Cash
Accounts receivable
Investment in branch

432
12,800

Branch Books

Advertising
480
Shipments from HO
3,520
600
Shipment from HO
12,632
Home office

180
3,820

Problem 12-9
(1)

Branch Books
Adjusting Entries
Shipment from home office
Operating expenses (P4,200 + P3,900)
Home office

57,600
8,100

65,700

Closing Entries
Sales
Inventory, 12/31 (P64,580 + P57,600)
Inventory, 1/1
Shipment from HO (P623,200 + P57,600)
Operating expenses
Income summary

778,200
122,180

Income summary
Home office

116,990

47,800
680,800
54,790
116,990
116,900

13

(2)

Home Office Books


Accounts receivable
Investment in branch
Cash (P20,000 + P19,200)

470
330

Investment in branch
Branch income
(3)

116,990

800
116,900

Reconciliation Statement

Unadjusted balances, 12/31


Error in recording remittance to branch
Shipment in transit
Expenses charged to branch
Branch net income
Freight erroneously charged to branch
Cash remittance in transit to HO
Adjusted balances, 12/31

Home Office Books


Branch Books
(Investment in Branch) (Home Office)
P 206,344
P 140,974
20,000
57,600
8,100
116,990
116,990
(
470)
( 19,200)
P 323,664
P 323,664

Problem 12-1111
a.

P 2,000
Sales (P 27,000 + P 33,000 + P 26,000) .
Cost of Goods Sold (P 36,000 + P 18,000) .
Gross Profit
Rent Expense ..
Property Tax Expense
Depreciation Expense
Miscellaneous Expense .
General Corporate Expense
Net Income

b.

P 86,000
(54,000)
P 32,000

P 4,000
5,000
4,000
11,000
6,000 (30,000)
P 2,000

P 180,000
Initial Transfers .
June Inventory Shipment ..
Property Tax Payment ..
September Inventory Shipment
Expense Allocation ..
Cash Transfer ...
Balance in Home Office/Branch Accounts (correct) ..

P 188,000
18,000
5,000
26,000
6,000
(63,000)
P 180,000

14

c.

Journal Entries Tarlac Branch


1/10/08

1/20/08
2/1/08

4/1/08

5/1/08
6/5/08
7/6/08
9/9/08

10/1/08

Cash .
Inventory .
Equipment
Home Office
Rent Expense
Cash .
Cash ..
Sales
Cost of Goods Sold ..
Inventory .
Cash .
Sales ...
Cost of Goods Sold .
Inventory
Miscellaneous Expenses .
Cash ...
Inventory .
Home office ...
Property Tax Expense .
Home Office ..
Inventory
Home Office .

Cash
Sales .
Cost of Goods Sold ..
Inventory ..
11/1/08
Miscellaneous Expenses ...
Cash .
12/22/08 Home Office
Cash .
12/31/08 Depreciation Expense .
Accumulated depreciation ..
12/31/08 General Corporate Expenses
Home Office ..

30,000
36,000
122,000
4,000
27,000
18,000
33,000
18,000
7,000
18,000
5,000
26,000

26,000
18,000
4,000
63,000
4,000
6,000

188,000
4,000
27,000
18,000
33,000
18,000
7,000
18,000
5,000
26,000

26,000
18,000
4,000
63,000
4,000
6,000

15

d.

TARLAC BRANCH
Balance Sheet
December 31, 2008
Assets
Cash .
Inventory .
Equipment ...
Accumulated Depreciation .
Total Assets
Equity
Home Office* ..

P 122,000
(4,000)

P 38,000
26,000
118,000
P 182,000
P 182,000

*Home office balance is P 180,000 as computed in Part b plus the P 2,000 net
income for the period.

16

CHAPTER 13
MULTIPLE CHOICE
13-1:

13-2:

a
Goods available for sale:
At billed price (P30,000 + P180,000)
At cost (P210,000 / 120%)
Balance of Allowance for Overvaluation account before adjustment

13-3:

c
Inter-company inventory profit (IIP) before closing
Less: IIP from shipment from home office
Billed price
Cost (P300,000 / 120%)
IIP from beginning inventory at billed price
Divided by
Cost of branchs beginning inventory

13-4:

P300,000
250,000

Billed Price
P15,000
110,000

%
150%
150%

Cost
P10,000
73,333

5,000

150%

3,333

50,000
P 16,000
20%
P 80,000

Overvaluation
P 5,000
36,667
P41.667
1,667
P40,000

b
Shipment to branch, at billed price
Shipping cost
Total cost
Sold (50%)
Inventory

13-6:

P 66,000

a
Beginning inventory from HO
Shipments
Balance before adjustment
Ending inventory from HO
Required adjustments

13-5:

P210,000
175,000
P 35,000

P375,000
2,000
P377,000
188,500
P188,500

a
Shipment to branch, at cost
Shipping cost
Billed price
Sold (50%)
Inventory, at billed price

P312,500
2,000
P314,500
157,250
P157,250

17

13-7:

c
Home office account balance after closing branch profit
Less: branch profit
Investment in branch account balance before closing branch profit

13-8:

d
Branch ending inventory, at billed price
Acquired from home office, at billed price:
Cost (P6,000 / 20%)
Mark-up
Purchased from outsiders

13-9:

P765,000
130,000
P635,000

P 50,000
P30,000
6,000

36,000
P 14,000

b
Cost of goods sold Home office
Cost of goods sold Branch:
Billed price
Less: overvaluation (P110,000 P90,000)
Combined cost of goods sold

P590,000
P300,000
20,000

280,000
P870,000

13-10: c
13-11: d
Overvaluation of branch ending inventory acquired from HO:
Billed price
Cost (P28,600 / 130%)
Adjusted balance of allowance for overvaluation account

P 28,600
22,000
P 6,600

13-12: b
Shipment from home office
Expenses
Cash remittance to home office
Home Office account balance before closing

P 90,000
17,000
(70,000)
P 37.000

13-13: b
Shipment to branch, at cost
Ending inventory, at cost (P70,000 / 30%)
Cost of goods sold
Freight (P6,000 x P50,400/P72,000)
Total

P 72,000
( 21,600)
P 50,400
4,200
P 54,600

13-14: b (20% of P30,000)


13-15: b (P151,200 / 140%)

18

13-16: c
Sales
Cost of goods sold
Shipments from home office (P151,200/140%) P108,000
Inventory, 1/1 (P28,350 / 140%)
20,250
Inventory, 12/31 (P25,200 / 140%)
( 18,000)
Gross profit
Expenses
Branch profit as far as the home office is concerned

P270,000

110,250
P159,750
90,000
P 69,750

13-17: c
Unsold merchandise
Less: Merchandise acquired from home office, at billed price
Merchandise acquired from outsiders
Merchandise acquired from home, at cost (P7,500 / 20%)
Branch inventory at cost, 12/31

P 60,000
45,000
P 15,000
37,500
P 52,500

13-18: a
Branch inventory, 1/1
Acquired from home office at billed price:
Overvaluation [P99,900 (P390,000 P300,000)]
Cost (P9,900 / 30%)
Purchases from outsiders

P 54,600
P 9,900
33,000

42,900
P 11,700

13-19: c
Acquired from home office [(P60,000 x 80%) 120%]
Acquired from outsiders (P60,000 x 20%)
Branch inventory, 12/31 at cost

P 40,000
12,000
P 52,000

13-20: b
Sales (P148,000 + P144,000)
Cost of sales at cost to home office:
Shipment from home office (P108,000 / 120%) P90,000
Purchases
52,000
Inventory, 12/31 (no. 19 above)
(52,000)
Gross profit
Expenses (P76,000 + P24,000)
Branch net income (actual)

P192,000

90,000
P102,000
100,000
P 2,000

13-21: b
Allowance for overvaluation account balance
Overvaluation on the shipment (P200,000 x 25%)
Overvaluation on the branch beginning inventory
Cost of branch beginning inventory (P7,500 / 25%)
Branch beginning inventory at billed price

P 57,500
50,000
P 7,500
30,000
P 37,500
19

13-22: b
Sales
Cost of goods sold cost to home office
Beginning inventory
P 30,000
Shipment from home office
200,000
Ending inventory (P40,000 / 125%)
( 32,000)
Gross profit
Expenses
Branch net income as far as the home office is concerned

P400,000

198,000
P202,000
100,000
P102,000

13-23: b
Branch inventory, 1/1
Acquired from home- at billed price
Overvaluation [P24,000 (P80,000 P60,000)] P 4,000
At cost [(P4,000 (P20,000 / P60,000)]
12,000
Acquired from outsiders

P 20,000
16,000
P 4,000

13-24: a
Sales
Cost of sales (at cost to home office)
Inventory, 1/1 (P12,000 + P4,000)
P16,000
Shipments from home office
60,000
Purchases
30,000
Inventory, 12/31 [(P20,000133 1/3%) +P6,000] (21,000)
Gross profit
Expenses
Branch net income (actual)

P200,000

85,000
P115,000
60,000
P 55,000

13-25: a
Inventory, 1/1
Shipments from home office
Overvaluation
Cost of goods available for sale
Percentage of mark-up (P72,500 / P362,500)

P 75,000
360,000
( 72,500)
P362,500
20%

13-26: b

20

13-27: a
Billing percentage above cost (P20,000 / P80,000)
Branch inventory, 6/1 at cost (P12,000 / 125%)
Home office inventory, 6/1
Purchases
Goods available for sale
Inventory, 6/30 at cost:
Branch (P10,000 / 125%)
Home office
Combined cost of goods sold

25%
P 9,600
40,000
160,000
P209,600
P 8,000
60,000

68,000
P141.600

13-28: d
Sales
Cost of goods sold
Gross profit
Expenses
Combined net income

P450,000
141,600
P308,400
150,000
P158,400

13-29: d
Sales
Cost of goods sold:
Inventory, 1/1: Home office
P57,500
Branch (P22,250 / 125%) 17,800 P 75,300
Purchases
410,000
Goods available for sale
P 485,300
Inventory, 12/31: Home office
P71,250
Branch (P29,250/120%) 24,375
95,625
Gross profit
Expenses
Combined net income

P687,500

389,675
P297,825
241,750
P 56,075

13-30: a
Sales
Cost of goods sold:
Inventory, 1/1:
Home office
Branch [P15,000 + (P49,000 / 122.5%)]
Purchases
Goods available for sale
Inventory, 12/31:
Home office
Branch [P11,000 + (P52,000 / 133 1/3%)]
Gross profit
Expenses
Combined net income

P669,000
P160,000
55,000

P110,000
50,000

P215,000
460,000
P675,000
160,000

515,000
P154,000
145,000
P 9,000

21

13-31: a
The entries made by the branch to record the interbranch transfer of merchandise are:
Books of Branch 1:
Home office
19,500
Freight in
3,500
Shipment from home office
16,000
Books of Branch 3:
Shipment from home office
16,000
Freight in
4,000
Cash
2,500
Home office
17,500
Therefore the home office would make the following entry:
Investment in Branch 3
17,500
Excess freight
2,000
Investment in Branch 1
13-32: a
Unadjusted balances
Error in recording shipment
Error in recording expense
Unrecorded cash remittance
Adjusted balances

(Home office books)


Investment in branch
77,000
(31,000)
46,000

19,500
(Branch books)
Home office
61,000
(10,000)
5,000
46,000

13-33: c
13-34: a
Home office books
Inv in Bacolod 25,000
Inv in Cebu
25,000

Cebu branch books


Home office 25,000
Cash
25,000

Inv in Bacolod 34,300


Inv in Cebu
34,300

Home office 34,300


SD
700
AR
35,000

Inv in Bacolod 62,500


Home office 212,500
Expenses
150,000
Expenses
37,500
Inv in Cebu
212,500
Cash
250,000

Bacolod branch books


Cash
25,000
Home office 25,000
Cash

34,300
Home office 34,300

Expenses
62,500
Home office
62,500

Inv in Cebu 253,000


Freight in
3,000
S to branch
200,000 S from HO 250,000
Allowance
50,000
Home office
253,000
Cash
3,000
Inv in Bacolod 252,700
Home office 253,000
Excess freight
300
S from H
253,000
Inv in Cebu
253,000

22

(Home office books)


Investment in Cebu Branch
25,000
34,300
212,500
253,000
253,000 524,800
271,800

(Bacolod branch books)


Home Office
25,000
34,300
62,500
252,700
374,500

23

PROBLEMS
Problem 13-1
(a)

Journal Entries
Home Office Books

Branch Books

(1) Investment in branch


Cash

18,000

(2) Investment in branch


Cash

3,000

(3) Investment in branch


Shipment to branch
Allowance for overValuation
(4)

100,000

18,000

Equipment
Home office

18,000

3,000

Rent expense
Home office

3,000

80,000

Shipment from HO
Home office

100,000

No entry

Operating expenses
Cash

(b)

Investment in branch

3,000
100,000

20,000

Cash
(5) Cash

18,000

60,000

60,000

Sales

Home office
Cash

11,000
105,000
60,000

11,000
105,000
60,000

Working Paper Elimination Entries


(1)

Home office
61,000
Investment in branch
To eliminate reciprocal accounts computed
as follows:
Equipment purchased
P 18,000
Rent paid
3,000
Inventory shipped
100,000
Cash transfer
( 60,000)
Balance
P 61,000

(2)

Shipment to branch
Allowance for overvaluation of branch inventory
Shipment from home office
To eliminate inter-company shipments

(3)

Inventory, 12/31 (Income statement)


Inventory, 12/31 (Balance Sheet)
To reduce inventory, 12/31 to cost.

80,000
20,000

5,000

61,000

100,000

5,000

24

(c)

Closing Entries Branch Books


Sales
Inventory, 12/31
Rent expense
Shipment from home office
Operating expenses
Income summary
Income summary
Home office

105,000
25,000

16,000

3,000
100,000
11,000
16,000
16,000

Problem 13-2
a.

Branch Books

Equipment
Shipment from home office
Cash
Home office

50,000
60,000
10,000

Purchases
Cash or accounts payable

30,000

Prepaid rent
Home office

10,000

Cash
Accounts receivable
Sales

40,000
50,000

Advertising expense
Salary expense
Cash

8,000
5,000

Home office
Cash

10,000

Home office
Accounts receivable

3,000

Rent expense
Prepaid rent

5,000

120,000
30,000
10,000

90,000

13,000
10,000
3,000
5,000

25

Home Office Books


-

Investment in branch
120,000
Equipment
Shipment to branch
Allowance for overvaluation of branch inventory
Cash
To record assets sent to branch

50,000
40,000
20,000
10,000

Investment in branch
Cash
To record rent expense of the branch

10,000

10,000

Cash

10,000

Cash

3,000

b.

Income Statement

Investment in branch
To record cash remittance from branch
Investment in branch
To record collection of branch receivable.

Sales
Cost of goods sold
Shipment from home office at cost
Purchases
Goods available for sale
Ending inventory:
From home office (1/3)
From outsiders (1/4)
Gross profit
Expenses:
Advertising expense
Salary expense
Rent expense
Net income

10,000

3,000

P90,000
P40,000
30,000
70,000
P13,333
7,500

(20,833)
P 8,000
5,000
5,000

49,167
P40,833

18,000
P22,833

Problem 13-3
a.

Investment in Branch account beginning balance


Cash transfer
Inventory transfer
Rent allocated
Expenses allocated
Inventory transfer
Transportation allocated
Unadjusted balance Investment in Branch account

P 86,000
( 32,000)
34,500
1,000
3,000
46,000
3,000
P141,500

26

b.

Home Office account beginning balance


Inventory transfer
Rent allocated
Expenses allocated
Inventory transfer (error made)
Cash transfer
Home Office account unadjusted balance

c.

Reconciliation Statement
Unadjusted balances, 1/31
Unrecorded cash transfer
Error in recording transfer (overstated)
Expense allocation not recorded
Adjusted balances, 1/31

P 54,000
34,500
1,000
3,000
64,000
( 74,000)
P 82,500
Investment in Branch
P141,500
( 74,000)
P 67,500

Home Office
P 82,500
18,000
( 3,000)
P 67,500

Problem 13-4
a.

b.

Books of Branch X
Shipment from home office
Freight-in
Home office

5,000
300

Home office
Shipment from office

5,800

5,800

Books of Branch Y
Shipment from home office
Freight-in
Home office

c.

5,300

5,000
600

5,600

Books of the Home Office


Investment in branch X
Shipment to branch X
Cash

5,300

Investment in branch Y
Inter-branch freight expense
Investment in branch X

5,000
600

Shipment to branch X
Shipment to branch Y

5,000

5,000
300

5,600
5,000

27

Malakas Company
Combination Worksheet
Year Ended December 31, 2008

Debits
Cash
Accounts receivable
Inventory, 12/31
Investment in branch
Land, bldg, and equipment
Shipment from office
Purchases
Depreciation expense
Advertising expense
Rent expense
Miscellaneous expense
Inventory, 1/1
Total debits

Malakas

Davao

25,000
108,000
209,000
207,000
340,000
348,000
25,000
36,000
12,000
40,000
175,000
1,525,000

18,000
25,000
42,000
112,000
96,000
8,000
15,000
5,000
20,000
35,000
376,000

Credits
Accumulated depreciation
Accounts payable
Notes payable
Home office

80,000
37,000
220,000
-

16,000
15,000
176,000

Common stock
Retained earnings, 1/1
Sales
Shipment to branch
Inventory, 12/31

100,000
240,000
529,000
110,000
209,000

127,000
42,000

Adjustments and
Eliminations
Debit
Credit

(4) 14,000

(5) 16,000
(7)207,000

(3) 14,000

(6)110,000

(1) 9,000
(1) 6,000
(1) 2,000

(7)207,000

(2) 10,000

452,000
348,000
33,000
60,000
23,000
62,000
200,000

877,000
96,000
52,000
220,000
-

(655,000)
(4) 14,000

Combined net income

(230,000)

(179,000)
(409,000)

1,525,000

376,000

388,000

100,000

(249,000)
(179,000)

Combined retained earnings


Totals

43,000
133,000
249,000

(1) 17,000
(3) 14,000

(2) 10,000
(6)110,000
(5) 16,000

Income
Retained
Statement Earnings Balance
Dr (Cr)
Dr (Cr)
Sheet

388,000

(409,000)
877,000

Adjustments and Elimination Entries


(1)

Advertising expense
Rent expense
Miscellaneous expenses
Home office
Unrecorded expenses allocated to the branch

9,000
6,000
2,000

17,000

(2)

Retained earnings, 1/1


10,000
Inventory, 1-1
To eliminate unrealized inventory profit of preceding year

10,000

(3)

Shipment from home office


Home office
Unrecorded shipments

14,000

14,000

28

(4)

Inventory, 12/31 (debits)


Inventory (credits)
Shipment not yet received by the branch

14,000

(5)

Inventory, 12/31 (debits)


Inventory (credits)
To reduce ending inventory to cost

16,000

(6)

Shipment to branch
Shipment from home office
To eliminate inter-company shipments

110,000

(7)

Home office
Investment in branch
To eliminate reciprocal accounts

207,000

14,000

16,000

110,000

207,000

Problem 13-6
a.

Eliminating Entries
(1)

Home office
Investment in branch Silver

395,000

(2)

Home office
Investment in branch Opal

260,000

(3)

Unrealized intra-company profit Silver


Unrealized intra-company profit Opal
Inventory from home office

20,000
16,000

(4)

Inventory
Inventory from home office

90,000

(5)

Unrealized intra-company profit Silver


Equipment

40,000

395,000
260,000

36,000
90,000
40,000

29

Ginto Company
Balance Sheet Working Paper
December 31, 2008

Cash
Accounts receivable
Inventory
Inventory from home office
Land
Buildings and equipment
Investment in branch Silver
Investment in branch Opal
Total debits
Accumulated depreciation
Accounts payable
Bonds payable
Common stock
Retained earnings
Home office
Unrealized intra-company profit
Silver
Opal
Total credits

b.

Home
Office
81,000
100,000
260,000

Silver
Branch
20,000
40,000
50,000
70,000

Opal
Branch
15,000
25,000
44,000
56,000

70,000
700,000
395,000
260,000
1,866,000

30,000
350,000

20,000
200,000

560,000

360,000

280,000
110,000
400,000
300,000
700,000
-

120,000
45,000

80,000
20,000

395,000

260,000

60,000
16,000
1,866,000

560,000

360,000

Eliminations
Debit
Credit
(4) 90,000

( 3) 36,000
(4) 90,000
(5) 40,000
(1)395,000
(2)260,000

120,000
1,210,000
2,055,000
480,000
175,000
400,000
300,000
700,000

(1)395,000
(2)260,000
(3) 20,000
(5) 40,000
(3) 16,000
821,000

Combined
116,000
165,000
444,000

821,000

2,055,000

Ginto Company
Combined Balance Sheet
December 31, 2008
Assets
Cash
Accounts receivable
Inventory
Land
Buildings and equipment
Less: Accumulated depreciation
Total assets
Liabilities and Stockholders Equity
Liabilities
Accounts payable
Bonds payable
Total liabilities
Stockholders Equity
Common stock
Retained earnings
Total liabilities and stockholders equity

P1,210,000
480,000

P 116,000
165,000
444,000
120,000
730,000
P1,575,000

P 175,000
400,000
P 575,000
P 300,000
700,000

1,000,000
P1,575,000

30

Problem 13-7
a.

b.

Books of Branch P
Shipment from home office
Freight-in
Home office

8,000
50

Home office
Shipment from home office
Freight-in
Cash

8,120

8,000
50
70

Books of Branch Q
Shipment from home office
Freight-in
Home office

c.

8,050

8,000
80

8,080

Books of Home Office


Investment in branch P
Shipment to branch P
Cash

8,050

Investment in branch Q
Inter-branch freight expense
Investment in branch P

8,080
40

Shipment to branch - P
Shipment to branch Q

8,000

8,000
50

8,120
8,000

Problem 13-8
Debits:
Cash = P36,000 (add the book values and include the P9,000 transfer in transit)
Accounts receivable = P118,000
Inventory, 12/31 = P151,000 (branch balance would be P81,000 when the shipment in transit is
included. This balance must be adjusted to cost of P54,000
(P81,000 150%) and then add to home office balance of P97,000.
Investment in branch = 0 (eliminated)
Land, buildings and equipment = P460,000
Shipment from home office = 0 (eliminated)
Purchases = P429,000
Depreciation expense = P28,000 (add the two book values and the year-end allocation)
Advertising expense = P58,000 (add the two book values and the year-end allocation)
Rent expense = P30,000 (add the two book values and the year-end allocation)
Miscellaneous expense = P100,000 (add the two book values and the year-end allocation)
Inventory, 1/1 = P145,000 (branch balance is adjusted to cost of P24,000 (P36,000 / 150%),
and then added to home office balance.
Total debits = P1,555,000 (add the above totals)
31

Credits
Accumulated depreciation = P108,000
Accounts payable = P104,000
Notes payable = P180,000
Home office = 0 (eliminated)
Common stock = P60,000 (home office balance)
Retained earnings, 1/1 = P248,000 (home office balance after reduction of P12,000 unrealized
profit in beginning inventory of branch. Cost is P24,000
(P36,000 / 150%) which indicates the P12,000 unrealized.
Sales = P704,000
Shipment to branch = 0 (eliminated)
Inventory, 12/31 = P151,000
Total credits = P1,555,000 (add the above totals)
Reconciliation Statement
Investment in Branch account balance (Home office books)
Unrecorded cash transfer
Adjusted balance

P177,000
( 9,000)
P168,000

Home Office account balance (Branch books)


Inventory transfer in transit
Expense allocated not yet recorded
Adjusted balance

P123,000
21,000
24,000
P168,000

Problem 13-9
Home Office Books
(1) Investment in branch
Shipment to branch
Unrealized inventory profit
(2) Cash
Investment in branch
Closing entries:
(3) Sales
Inventory, 12/31
Shipment to branch
Purchases
Expenses
Income summary
(4) Investment in branch
Branch income summary
Branch income summary
Investment in branch

Case A
60,000

61,200
130,000
8,000
60,000

13,000

60,000
61,200

150,000
17,200
30,800
13,000

Unrealized inventory profit


Branch income summary
Income summary
Income summary
Retained earnings

Case B
75,000

61,200
130,000
8,000
60,000

500
13,500

43,800

43,800

43,800

60,000
15,000
61,200

150,000
17,200
30,800

500
500
13,000
43,800

Case C
90,000

61,200
130,000
8,000
60,000

14,000
27,000

43,800

60,000
30,000
61,200

150,000
17,200
30,800

14,000
14,000
13,000
43,800

32

Ilocos Branch Books


Case A
(1) Shipment from home office
Home office

60,000

(2) Accounts receivable


Sales

81,000

(3) Cash

64,000

Accounts receivable

(4) Expenses
Cash

14,000

(5) Home office


Cash

61,200

60,000
81,000
64,000
14,000
61,200

Case B
75,000
81,000
64,000
14,000
61,200

Case C

75,000
81,000
64,000
14,000
61,200

90,000
81,000
64,000
14,000
61,200

90,000
81,000
64,000
14,000
61,200

Closing entries
(6) Sales
Inventory 12/31
Shipment from HO
Expenses
Income summary

81,000
6,000

(7) Income summary


Home office

13,000

Home office
Income summary

60,000
14,000
13,000

81,000
7,500
500

75,000
14,000

81,000
9,000
14,000

90,000
14,000

13,000
500

500

14,000

14,000

33

Working Paper for Combined Financial Statements


December 31, 2008

Income Statement
Sales
Merchandise inventory, 12/31
Shipment to branch
Total credits
Shipment from home office
Purchases
Expenses
Total debits
Net income(loss) carried forward
Retained Earnings Statement
Net income (loss) from above
Retained earnings, 12/31 Carried forward
Balance Sheet
Cash (overdraft)
Accounts receivable
Merchandise inventory, 12/31
Investment in branch
Total debits
Accounts payable
Unrealized inventory profit
Capital stock
Retained earnings, from above
Home office
Total credits

Home Office
130,000
8,000
60,000
198,000

Branch
81,000
9,000
90,000

Eliminations
Debit
Credit

211,000
14,000
225,000

(3) 3,000
(2) 60,000

150,000
17,200
167,200
30,800

14,000
104,000
(14,000)

150,000
31,200
181,200
43,800

30,800

(14,000)

43,800

30,800

(14,000)

43,800

39,000
45,000
8,000
28,800
120,800

(11,200)
17,000
9,000

27,800
62,000
14,000
103,800

20,000
30,000
40,000
30,800
120,800

90,000

Combined

(2) 90,000

(3) 3,000
(1) 28,800

14,800
(2) 30,000
(14,000)
28,800
14,800

(1) 28,800
121,800

121,800

20,000
40,000
43,800
103,800

34

Problem 13-10
(1)

Consolidated Working Paper

Debits
Cash
Inventories

Home
Office

Branch A

Branch B

33,000
70,000

22,000
21,000

13,000
15,000

Other current assets


Investment in Branch A
Investment in Branch B
Cost of sales *

50,000
45,000
42,000
80,000

25,000

23,000

57,000

45,000

Expenses

90,000
410,000

25,000
150,000

20,000
116,000

40,000
100,000
50,000

15,000

11,000

45,000

30,000

Credits
Current liabilities
Capital stock
Retained earnings, Jan. 1
Home Office
Allow. for overvaluation of
Branch inv. Branch A
Allow. for overvaluation of
Branch inv. Branch B
Sales
Net income

13,000
12,000
195,000
410,000

Adj. & Elim.


(dr) Cr

Income
Statement

68,000

A (12,000)
B 8,000
D 45,000
D 42,000
B (8,000)
C 25,000

Balance
Sheet

110,000
98,000
(165,000)
(135,000)

276,000
66,000
100,000
50,000

A 12,000
D (87,000)
C (13,000)

90,000
150,000

75,000
116,000

C (12,000)

360,000
60,000

60,000
276,000

Book value of cost of sales from home office and branches

Home Office
Inventory, January 1,
Purchases
Shipment to branch
Shipment from home office
Goods available for sale
Inventory, Dec. 31
Cost of sales

P 80,000
160,000
( 90,000)
P150,000
( 70,000)
P 80,000

Investment in
Branch A

Investment in
Branch B

P 18,000

P24,000

60,000
P 78,000
( 21,000)
P 57,000

36,000
P 60,000
(15,000)
P 45,000

35

(2) Reconciliation of Home Office and Investment in Branch accounts.

Unadjusted balances, Dec.31

Books of Home Office


Investment
Investment
In Branch A In Branch B
P 45,000
P 42,000

Books of
Books of
Branch A
Branch B
Home Office Home Office
P 45,000
P 30,000

Shipments in transit to Branch B


Branch Profit (Schedule 1)
Adjusted balances, December 31

12,000
8,000
P 53,000

10,000
P 52,000

8,000

10,000

P 53,000

P 52,000

Schedule 1:
Sales
Cost of sales:
Beginning inventory
Shipment from home office
Goods available for sale
Ending inventory
Cost of sales
Gross profit
Expenses
Net profit

Branch A
P90,000

Branch B
P75,000

P18,000
60,000
78,000
21,000
57,000
33,000
25,000
P 8,000

P24,000
48,000
72,000
27,000
45,000
30,000
20,000
P10,000

36

CHAPTER 14
MULTIPLE CHOICE
14-1:

a
Purchase price (8,000 shares x P30)
Direct acquisition cost
Contingent consideration
Acquisition cost

14-2:

P240,000
4,000
5,000
P249,000

a
Purchase price
Direct acquisition cost
Acquisition cost
Less: Fair value of net assets acquired
Goodwill

14-3:

P250,000
50,000
P300,000
180,000
P120,000

c
Purchase price (100,000 shares x P36)
Direct acquisition cost
Contingent consideration
Acquisition cost

14-4:

14-5:

P3,600,000
100,000
20,000
P3,720,000

b
Purchase price (600,000 shares x P50)
Direct acquisition cost
Acquisition cost
Less: goodwill recorded
Fair value of net assets acquired

P30,000,000
300,000
P30,300,000
6,120,000
P24,180,000

Capital stock issued (at par)

P30,000,000

c
Purchase price
Legal fees
Acquisition cost
Less: Fair value of net assets acquired
Current assets
Plant assets
Liabilities
Income from acquisition

P2,550,000
25,000
P2,575,000
P1,100,000
2,200,000
( 300,000)

3,000,000
P( 425,000)

37

14-6:

a (at fair value at date of acquisition)

14-7:

d
Abel net income, January to December (P80,000 + P1,320,000) P1,400,000
Cain net income, April to December
400,000
Total net income
P1,800,000

14-8:

a
Acquisition cost
Less: Fair value of net assets acquired
Cash
Inventory
Property, plant and equipment
Liabilities
Income from acquisition

14-9

P 800,000
P 160,000
380,000
1,120,000
( 360,000)

1,300,000
P (500,000)

a
Acquisition cost
Less: Fair value of net assets acquired (P600,000 P188,000)
Goodwill
Avons assets
Bells assets at fair value
Total assets

P 700,000
412,000
P 288,000
2,000,000
600,000
P2,888,000

14-10: b
Debit to Investment in Stock
Brokers fee
Pre-acquisition audit fee
Legal fees for the combination
Total

P 50,000
40,000
32,000
P 122,000

Debit to expenses:
General administrative costs
Other indirect costs
Total

P 15,000
6,000
P 21,000

Debit to APIC
Audit fee for SEC registration of stock issue
SEC registration fee for stock issue
Total

P 46,000
5,000
P 51,000

38

14-11: d
Acquisition costs:
Cash
Stocks issued at fair value
Contingent liabilities
Total
Less: fair value of net assets acquired:
Cash
Inventories
Other current assets
Plant assets (net)
Current liabilities
Other liabilities
Goodwill

P200,000
330,000
70,000
P600,000
P40,000
100,000
20,000
180,000
(30,000)
(40,000)

Total assets after combination:


Total assets before combination
Cash paid
Registration and issuance costs of shares issued
Polos assets after combination
Assets acquired at fair values
Goodwill
Total assets after combination

270,000
P330,000
P 760,000
(200,000)
( 30,000)
P 530,000
340,000
330,000
P1,200,000

14-12: d
Acquisition cost
Less: Fair value net assets acquired
Goodwill

P1,400,000
1,350,000
P 50,000

14-13: a
Acquisition cost
Less: Fair value of net identifiable assets acquired:
Current assets
P 80,000
Non-current assets
120,000
Liabilities
( 20,000)
Income from acquisition

P160,000

Non- current assets

P120,000

180,000
P(20,000)

14-14: c
Acquisition cost
Less: Fair value of identifiable assets acquired:
Cash
P 60,000
Merchandise inventory
142,500
Plant assets (net)
420,000
Liabilities
(135,000)
Goodwill

P600,000

487,500
P112,500

39

14-15: b
Acquisition cost
Less: Fair value of identifiable assets acquired
Goodwill
MMs net assets at book value
PPs net assets at fair value
Total assets after combination

P1,000,000
800,000
P 200,000
1,200,000
800,000
P2,200,000

14-16: c, Under the purchase method assets are recorded at their fair values (P225.000)
14-17: d
Capital stock issued at par (10,000 shares x P10)
APIC (10,000 shares x P40)
Total

P100,000
400,000
P500,000

14-18: d, net assets are recorded at their fair values.


14-19: a
Income from acquisition
Fair value of net assets acquired P2,000,000 P400,000)
Acquisition cost

P 100,000
1,600,000
1,500,000

Shares to be issued (P1,500,000 P40)

37,500 shares

14-20: d

14-21:

Goodwill
Fair value of net assets acquired
Acquisition cost

P 200,000
1,600,000
P1,800,000

Shares to be issued (P1,800,000 P40)

45,000 shares

Total assets of Pablo before acquisition at book value


Total assets acquired from Siso at fair value (100,000 +440,000)
Total assets
Less: cash paid (15,000 + 25,000)
Total assets after cash payment
Goodwill to be recognized (Sched 1)
Total assets after combination

P 700,000
540,000
1,240,000
40,000
1,200,000
195,000
1,395,000

Sched 1: Acquisition cost:


Purchase price (30,000 shares x P20) 600,000
Direct cost
25,000
Contingent consideration
50,000
Fair value of net assets acquired (540,000 60,000)
Goodwill

675,000
480,000
195,000

40

14-22:

Stockholders equity before acquisition


Capital stock issued at par (30,000 shares x P10)
APIC (50,000 +300,000) 15,000
Stockholders equity after acquisition

14-23: a
Acquisition cost
Less: fair value of net assets acquired
Goodwill

650,000
300,000
335,000
1,285,000
B Company
P4,400,000
4,150,000
P 250,000

Total goodwill recorded (250,000 + 268,000)


14-24: a
A Company
B Company
C Company
Cash paid for combination expenses
Goodwill (see 14-23)
Total assets after combination
14-25: a
Stockholders equity before acquisition
Capital stock issued at par (229,000 shares x P10)
Additional paid-in-capital [(229,000 x 12) 10,000]
Indirect cost (reduction from retained earnings)
Stockholders equity after acquisition

C Company
P638,000
370,000
P268,000
518,000
5,250,000
6,800,000
900,000
(30,000)
518,000
13,438,000
P1,300,000
2,290,000
2,738,000
(20,000)
6,308,000

41

PROBLEMS
Problem 14-1
1.

Books of Big Corporation


Accounts receivable
Inventories
Property, plant and equipment
Current liabilities
Income from acquisition
Cash
To record acquisition of net assets of Small.

120,000
140,000
300,000

Computation of Income from Acquisition:


Acquisition cost (P500,000 + P5,000)
Less: Fair value of net identifiable assets acquired:
Accounts receivable
P120,000
Inventories
140,000
Property, plant and equipment
300,000
Current liabilities
( 50,000)
Income from acquisition
2.

50,000
5,000
505,000

P505,000

510,000
P( 5,000)

Books of Small Corporation


Cash
Current liabilities
Accounts receivable
Inventories
Property, plant and equipment
Retained earnings
To record sale of net assets to Big.

500,000
50,000

Common stock
Retained earnings
Cash
To record liquidation of the corporation.

200,000
300,000

120,000
100,000
280,000
50,000

500,000

42

Problem 14-2
Cash
Inventory
Building and equipment net
Patent
Accounts payable
Cash
Income from acquisition
To record acquisition of the net assets at fair values.

50,000
150,000
300,000
200,000

Computation of Income from Acquisition


Acquisition cost (P565,000 + P5,000)
Less: Fair value of net identifiable assets acquired
Total assets
P700,000
Accounts payable
( 30,000)
Income from acquisition

30,000
570,000
100,000

P570,000
670,000
P(100,000)

Problem 14-3
Cash and receivables
Inventory
Building and equipment
Goodwill
Accounts payable
Common stock, P10 par value
Additional paid-in capital
Cash
To record acquisition of net assets acquired.

50,000
200,000
300,000
65,000

Computation of Goodwill
Purchase price (6,000 shares x P90)
Direct acquisition cost
Acquisition cost
Less: fair value of net identifiable assets acquired
Total assets
P550,000
Accounts payable
( 50,000)
Goodwill

50,000
60,000
480,000
25,000

P540,000
25,000
P565,000
500,000
P 65,000

43

Problem 14-4
(1)

Cash
Accounts receivable
Inventory
Land
Building and equipment
Bond discount
Goodwill
Accounts payable
Bonds payable
Common stock, P10 par value
Additional paid-in capital
Cash (P10,000 + P3,000)
To record purchase of net assets of Tan.

60,000
100,000
115,000
70,000
350,000
20,000
108,000

Computation of Goodwill
Purchase price (12,000 shares x P50)
Professional fees (P10,000 + P3,000)
Acquisition cost
Less: Fair value of net identifiable assets acquired
Total assets
P695,000
Total liabilities
( 190,000)
Goodwill

10,000
200,000
120,000
480,000
13,000

P600,000
13,000
P613,000
505,000
P108,000

(2)

Additional paid-in capital


6,000
Cash
To record costs of issuing and registering of shares issued
(P5,000 + P1,000)

(3)

Expenses
Cash
To record indirect acquisition costs.

6,000

9,000

9,000

Problem 14-5
1.
2.
3.
4.
5.
6.
7.

Common stock:: P200,000 + (8,000 shares x P10)


Cash and receivables: P150,000 + P40,000
Land: P100,000 + P85,000
Building and equipment net: P300,000 + P230,000
Goodwill: (8,000 shares x P50) - P355,000
APIC: P20,000 + (8,000 shares x P40)
Retained earnings

P280,000
190,000
185,000
530,000
45,000
340,000
330,000

44

Problem 14-6
Combined Balance Sheet
After acquisition
Cash and receivables
Inventory
Building and equipment
Accumulated depreciation
Goodwill
Total assets
Accounts payable
Bonds payable
Common stock P10 Par value
Additional paid-in capital
Retained earnings(including income from acquisition)
Total liabilities and stockholders equity

Based on P40/share Based on P20/share


P 350,000
P 350,000
645,000
645,000
1,050,000
1,050,000
(200,000)
(200,000)
180,000
P2,025,000
P1,845,000
P 140,000
485,000
450,000
550,000
400,000
P2,025,000

Computation of Goodwill Based on P40 per share:


Acquisition cost (15,000 shares x P40)
Less: Fair value of net identifiable assets (P545,000 P125,000)
Goodwill
Computation of Income from Acquisition Based on P20 per share:
Acquisition cost (15,000 shares x P20)
Less: Fair value of net identifiable assets
Income from acquisition (added to retained earnings of Red)

P 140,000
485,000
450,000
250,000
520,000
P1,845,000

P600,000
420,000
P180,000
P300,000
420,000
P(120,000)

Problem 14-7
(a)

Combined Balance Sheet


January 1, 2008

ASSETS
Cash and receivables
Inventory
Land
Plant and equipment
Less: Accumulated depreciation
Goodwill
Total assets
LIABILITIES AND STOCKHOLDERS EQUITY
Current liabilities
Capital stock, P20 par value
Capital in excess of par
Retained earnings
Total liabilities and stockholders equity

P540,000
150,000

P 110,000
142,000
115,000
390,000
13,000
P 770,000
P 100,000
214,000
216,000
240,000
P 770,000

45

Computation of Goodwill
Acquisition cost
Less: Fair value of net identifiable assets acquired
(P217,000 P20,000)
Goodwill
(b)

P210,000
197,000
P 13,000

Stockholders Equity section


(1) With 1,100 shares issued
Capital stock: P200,000 + (1,100 shares x P20)
Capital in excess of par: P20,000 + (1,100 x P280)
Retained earnings
Total

P222,000
328,000
240,000
P790,000

(2) With 1,800 shares issued


Capital stock: P200,000 + (1,800 shares x P20)
Capital in excess of par: P20,000 + (1,800 x P280)
Retained earnings
Total

P 236,000
524,000
240,000
P1,000,000

(3) With 3,000 shares issued


Capital stock: P200,000 + (3,000 shares x P20)
Capital in excess of par: P20,000 + (3,000 x P280)
Retained earnings
Total

P260,000
860,000
240,000
P1,360,000

Problem 14-8
Revenue
Net income
Earnings per share
(a)
(b)
(c)
(d)
(e)

2007 (a)
P1,400,000
500,000
P 5.00

2008
P1,800,000 (b)
545,000
P 4.84 (d)

2009
P2,100,000
700,000
P 5.60 (e)

Separate figures for Dollar Transport only.


P2,000,000 P200,000
P620,000 - P55,000
P545,000 / 112,000 shares (100,000 + 125,000) 2
P700,000 / 125 shares

46

Problem 14-9
a.

Books of Peter Industries


Cash
28,000
Accounts receivable
258,000
Inventory
395,000
Long-term investments
175,000
Land
100,000
Rolling stock
63,000
Plant and equipment
2,500,000
Patents
500,000
Special licenses
100,000
Discount on equipment trust notes
5,000
Discount on debentures
50,000
Goodwill
244,700
Allowance for bad debts
Current payables
Mortgage payables
Premium on mortgage payable
Equipment trust notes
Debenture payable
Common stock
APIC common
Cash (direct acquisition cost)
To record acquisition of assets and liabilities at fair values.
Computation of Goodwill
Purchase price (180,000 shares x P14)
Direct acquisition cost
Acquisition cost
Less: fair value of net identifiable assets acquired
Total assets
P4,112,500
Total liabilities
(1,702,200)
Goodwill
Expenses
Cash
To record indirect cost.

42,000

6,500
137,200
500,000
20,000
100,000
1,000,000
180,000
2,298,000
135,000

P2,520,000
135,000
P2,655,000
2,410,300
P 244,700
42,000

47

b.

Books of HCC:
Common stock
APIC Common
Treasury stock
To record retirement of treasury stock.
P7,500 = P5 x 1,500 shares
P4,500 = P12,000 P7,500
Investment in stock - Peter
Allowance for bad debts
Accumulated depreciation
Current payable
Mortgage payable
Equipment trust notes
Debentures payable
Discount on bonds payable
Cash
Accounts receivable
Inventory
Long-term investments
Land
Rolling stock
Plant and equipment
Patents
Special licenses
Gain on sale of assets and liabilities
To record sale of assets and liabilities to Peter.

7,500
4,500

2,520,000
6,500
614,000
137,200
500,000
100,000
1,000,000

Common stock
592,500
APIC Common
495,500
APIC Retirement of preferred
22,000
Retained earnings
1,410,000
Investment in stock Peter
To record retirement of HCC stock and distribution of
Peter Industries stock:
P592,500 = P600,000 - P7,500
P495,500 = P500,000 P4,500
P1,410,000 = P220,000 + P1,189,900

12,000

40,000
28,000
258,000
381,000
150,000
55,000
130,000
2,425,000
125,000
95,800
1,189,900

2,520,000

48

Problem 14-10
a.

Increase in capital stock (P240,00 P200,000)


Increase in APIC (P420,000 P60,000)
Value of shares issued

P 40,000
360,000
P 400,000

b.

Total assets after combination


Total assets of Subic before combination
Total fair value of assets of Clark before combination

P1,130,000
650,000
P 480,000

Total liabilities after combination


P220,000
Total liabilities of Subic before combination
(140,000)
Fair value of Clarks net assets (including goodwill)
Less: Goodwill
Fair value of Clarks net assets before combination

( 80,000)
P 400,000
55,000
P 345,000

c.

Par value of common stock after combination


Par value of common stock before combination
Increase in par value
Divided by par value per share
Number of shares issued

P 240,000
200,000
P 40,000
P5
8,000 shares

d.

Value of shares computed in (a)


Number of shares issued computed in
Market price per share

P 400,000
8,000
P
50

Problem 14-11
a.

Inventory reported by Son at date of combination was P70,000


(325,000 P20,000 P55,000 P140,000 P40,000)

b.

Fair value of total assets reported by Son:


Fair value of cash
Fair value of accounts receivable
Fair value of inventory
Buildings and equipment reported following purchase
Buildings and equipment reported by Papa
Fair value of Sons total assets

c.

P570,000
(350,000)

P 20,000
55,000
110,000
220,000
P405,000

Market value of Sons bond:


Book value reported by Son
Bond premium reported following purchase
Market value of bond

P100,000
5,000
P105,000

49

d.

Shares issued by Papa Corporation:


Par value of stock following acquisition
Par value of stock before acquisition
Increase in par value of shares outstanding
Divide by par value per share
Number of shares issued

e.

f.

P190,000
(120,000)
P 70,000

P5
14,000

Market price per share of stock issued by Papa Corporation


Par value of stock following acquisition
Additional paid-in capital following acquisition

P190,000
262,000

Par value of stock before acquisition


Additional paid-in capital before acquisition
Market value of shares issued in acquisition
Divide by number of shares issued
Market price per share

P120,000
10,000

P452,000
(130,000)
P322,000
14,000
P 23.00

Goodwill reported following the business combination:


Market value of shares issued by Papa
Fair value of Sons assets
Fair value of Sons liabilities:
Accounts payable
P 30,000
Bond payable
105,000
Fair value of liabilities
Fair value of Sons net assets
Goodwill recorded in business combination
Goodwill previously on the books of Papa
Goodwill reported

P405,000

(135,000)

g.

Retained earnings reported by Son at date of combination was P90,000


(P325,000 P30,000 P100,000 P50,000 P55,000)

h.

Papas retained earnings of P120,000 will be reported.

i.

1.

Investment account
Additional paid-in capital
Cash

17,000
9,800

P322,000

(270,000)
P 52,000
30,000
P 82,000

26,800

2.

Goodwill previously computed


Merger costs added to investment account
Total goodwill reported

P82,000
17,000
P99,000

3.

Additional paid-in capital reported following combination


Stock issue costs
Total additional paid-in capital reported

P262,000
(9,800)
P252,200

50

CHAPTER 15
MULTIPLE CHOICE
15-1:

a
Acquisition cost
Less: Book value of interest acquired (100%)
Difference
Allocation:
Property and equipment
Other assets
Long-term debt
Goodwill

15-2:

P4,000,000
3,200,000
800,000
P(750,000)
150,000
(200,000)

c
Acquisition cost
Less: Book value of interest acquired (P280,000 x 90%)
Difference
Allocation to plant assets (P40,000 x 90%)
Goodwill

15-3:

P 350,000
252,000
98,000
(36,000)
P 62,000

c
Plant assets Pall Company
Plant assets Mall Company
Consolidated

15-4:

( 800,000)
P -0-

P 220,000
180,000
P 400,000

a
Acquisition cost
Less: Book value of interest acquired (P560,000 P70,000)
Difference
Allocation:
Inventory
P 25,000
Property and equipment
( 35,000)
Income from acquisition

P495,000
490,000
5,000
(10,000)
P( 5,000)

51

15-5:

b
Acquisition cost
Less: Book value of interest acquired (P320,000 x 80%)
Difference
Allocation:
Inventory (P20,000 x 80%)
P(16,000)
Land (P10,000 x 80%)
8,000
Mortgage payable (P5,000 x 80%)
( 4,000)
Goodwill

15-6:

15-7:

15-8:

( 12,000)
P 67,000

a
Inventory (P360,000 + P130,000)

P490,000

Plant and equipment (P500,000 + P420,000)

P920,000

a
Building

P180,000

Land

P 90,000

d
Sons stockholders equity
Minority interest proportionate share
Minority interest in net assets of subsidiary

15-9:

P355,000
256,000
P 79,000

P400,000
20%
P 80,000

d
Acquisition cost
Less: Book value of interest acquired (P145,000 x 75%)
Difference
Allocation to accounts payable (P5,000 x 75%)
Goodwill

P160,000
108,750
51,250
3,750
P 55,000

Therefore:
Total assets (P800,000 + P300,000 + P55,000)
Total liabilities (P250,000 + P15,000 + P160,000 + P5,000)

P1,155,000
570,000

15-10: b (P900,000 x 1%)

52

15-11: a
Controlling (Parent) interest:
Shares acquired (P120,000 / P120)
Divided shares outstanding (P125,000 /P100)
Parents interest

1,000 shares
1,250
80%

Minority interest in net assets of subsidiary (P200,000 x 20%) P40,000


15-12: a
Goodwill
Book value of interest acquired (P100,000 / 20%) x 80%
Investment cost

P250,000
400,000
P650,000

15-13: b
Net assets on the date of acquisition (P247,095 + P43,605)
Adjustments of assets excluding goodwill:
Inventories
P6,630
Plant and equipment
48,450
Patent
7,650
Net assets at fair value

P290,700

62,730
P353,430

15-14: d (P500,000 + P300,000)


15-15: b
Acquisition cost
Less: Book value of interest acquired (P250,000 x 80%)
Difference
Allocated to plant and equipment (P50,000 x 80%)
Goodwill

P260,000
200,000
60,000
(40,000)
P 20,000

15-16: a (The retained earnings of the parent only).


15-17: a (The stockholders equity of the parent only).
15-18: b (P50,000 + P10,000)
15-19: d (P380,000 + P150,000)

53

15-20: d
Cash and cash equivalent (P70,000 + P90,000)
Inventory (P100,000 + P60,000)
Property and equipment (P500,000 + P300,000)
Goodwill
Total assets

P 160,000
160,000
800,000
20,000
P1,140,000

15-21: d
Fair value of the reporting unit
Fair value of net assets (excluding goodwill)
Implied goodwill
Carrying value of goodwill (P450,000 P390,000)
Impairment loss

P 485,000
440,000
45,000
60,000
P 15,000

15-22: b
Fair value of the reporting unit
Fair value of the net assets (P590,000 P100,000)
Implied goodwill to be recorded
Carrying value of goodwill
Impairment loss

P 540,000
490,000
50,000
150,000
P 100,000

15-23: a

The amount reported is equal to Primos retained earnings of P567,000

15-24: a

100% [P138, 000 (P320, 000 P140, 000)]

15-25: a

(340,000- 200,000)

15-26: b
Cash
Accounts receivable
Inventories (see 15-25)
Equipment (800,000 - 500,000)
Accounts payable
Fair value of net assets

40,000
20,000
140,000
300,000
(40,000)
460,000

15-27: a
Net asset acquired (320,000 x 70%)
Differential allocated to inventory
Differential allocated to equipment
Differential allocation to goodwill
Minority interest (140,000 x30%)
Amount paid by Parent

224,000
40,000
100,000
10,000
(42,000)
332,000

54

PROBLEMS
Problem 15-1
a.

Investment in Solo Company stock


Cash
To record acquisition of 90% (90,000 / 100,000)
of the outstanding shares of Solo.

b.

Working paper elimination entries:


(1)

(2)

1,080,000

Common stock Solo


400,000
Retained earnings Solo
500,000
Investment in Solo company stock
Minority interest in net assets of subsidiary
To eliminate Solos equity accounts at date of acquisition.
Inventories
30,000
Plant assets
60,000
Goodwill
189,000
Investment in Solo company stock
Minority interest in net assets of subsidiary
To allocate difference
Computation and allocation of difference:
Acquisition cost
Less: Book value of interest acquired
Common stock (P400,000 x90%)
Retained earnings (P500,000 x 90%)
Difference
Allocation:
Inventories
Plant assets
Total
Minority interest (P90,000 x10%)
Goodwill

1,080,000

810,000
90,000

270,000
9,000

P1,080,000
P360,000
450,000
(30,000)
(60,000)
(90,000)
9,000

810,000
P 270,000

( 81,000)
P 189,000

55

Problem 15-2
a.

Investment in Straw stock


Cash
To record acquisition of 100% of Straw stock.

b.

Acquisition cost
Less: Book value of interest acquired (100%)
Difference
Allocation (100%:
Inventories
Land
Building
Equipment
Patents
Goodwill

c.

600,000

600,000
P600,000
420,000
180,000

P( 40,000)
( 80,000)
150,000
( 20,000)
( 20,000)

( 10,000)
P170,000

Working paper elimination entries:


(1)

(2)

Common stock Straw


Retained earnings Straw
Investment in Straw stock
To eliminate equity accounts of Straw at
date of acquisition.

100,000
320,000

Inventories
Land
Equipment
Patents
Goodwill
Buildings
Investment in Straw stock
To allocate difference.

40,000
80,000
20,000
20,000
170,000

420,000

150,000
180,000

56

Problem 15-3
a.

Investment in Soto stock


Cash
To record acquisition of 90% stock of Sotto.

b.

Acquisition cost
Less: Book value of interest acquired (P900,000 x 90%)
Difference
Allocation:
Current assets
P 50,000
Property and equipment
(100,000)
Long-term debt
( 40,000)
Total
P( 90,000)
Minority interest (10% thereof)
9,000
Goodwill

c.

950,000

950,000
P950,000
810,000
140,000

(81,000)
P 59,000

Working paper elimination entries:


(1)

(2)

Common stock Sotto


100,000
APIC Sotto
200,000
Retained earnings Sotto
600,000
Investment in Sotto stock
Minority interest in net assets of subsidiary
To eliminate equity accounts of Sotto at date of
acquisition.
Property, plant and equipment
100,000
Goodwill
59,000
Long-term debt
40,000
Current assets
Investment in Sotto stock
Minority interest in net assets of subsidiary
To allocate difference.

810,000
90,000

45,000
140,000
14,000

57

Problem 15-4
Paco Company and Subsidiary
Consolidated Balance Sheet
January 2, 2008
Current assets
Property, plant and equipment
Other assets
Total assets

P475,000
285,000
70,000
P830,000

Current liabilities
Mortgage payable
Common stock
Additional paid-in capital
Retained earnings (including income from subsidiary of P20,000)
Total liabilities and stockholders equity

P280,000
85,000
200,000
65,000
200,000
P830,000

Computation of income from acquisition:


Investment cost (20,000 shares x P6)
Less: Book value of interest acquired
Common stock
Retained earnings
Difference
Allocated to property and equipment
Income from acquisition

P120,000
P35,000
80,000

115,000
P 5,000
(25,000)
P(20,000)

Problem 15-5
Under the purchase method, the investment cost is equal to the fair value of stock issued by Palo
(P250,000) plus direct acquisition cost (P10,000) or a total of P260,000. The P20,000 stock issue
cost is treated as a reduction from the additional paid-in capital. The entry to record the
acquisition of stock is as follows:
Investment in Solo stock
Common stock, at par
Additional paid-in capital
Cash (direct acquisition cost)
Additional paid-in capital
Cash

260,000

20,000

100,000
150,000
10,000
20,000

58

Palo Company and Subsidiary


Consolidated Balance Sheet
December 31, 2008
Cash
Receivables
Inventory
Property and equipment net
Goodwill
Total assets

P 70,000
120,000
170,000
340,000
30,000
P730,000

Current liabilities
Long-term liabilities
Common stock
Additional paid-in capital
Retained earnings, 12/31
Total liabilities and stockholders equity

P 30,000
120,000
210,000
150,000
220,000
P730,000

Computation of goodwill:
Acquisition cost
Less: Book value of interest acquired (P90,000 + P100,000)
Difference
Allocated to equipment
Goodwill

P260,000
190,000
70,000
(40,000)
P 30,000

Problem 15-6
a.
Investment in Seed Company stock
Cash
To record acquisition of 100% of Seed company stock.
Allocation schedule:
Acquisition cost
Less: Book value of interest acquired
Difference
Allocation:
Inventory
Plant assets
Long-term liabilities
Income from acquisition

b.

Working paper elimination entries


(1)
Common stock Seed
Additional paid-in capital Seed
Retained earnings Seed
Investment in Seed stock
To eliminate equity accounts of Seed at
date of acquisition.
(2)

350,000

350,000
P350,000
320,000
30,000

P(20,000)
(80,000)
40,000

(60,000)
P930,000)

100,000
40,000
180,000

Inventory
20,000
Plant assets
80,000
Long-term debt
Investment in Seed stock
Retained earnings Pill (income from acquisition)
To allocate difference.

320,000

40,000
30,000
30,000

59

Pill Corporation and Subsidiary


Consolidated Working Paper
May 31, 2008 Date of Acquisition

Assets
Cash
Accounts receivable
Inventories
Investment in Seed company
Plant assets
Total
Liabilities & Stockholders
Equity
Current liabilities
Long-term debt
Common stock:
Pill
Seed
Additional paid-in capital
Pill
Seed
Retained earnings
Pill
Seed
Total

Pill
Corporation

Seed
Company

200,000
700,000
1,400,000
350,000

10,000
60,000
120,000

2,850,000
5,500,000

610,000
800,000

500,000
1,000,000

80,000
400,000

1,500,000
1,200,000
1,300,000
5,500,000

Eliminations

Debit

(2) 20,000
(2) 80,000

& adjustment

Credit

(1)320,000
(2) 30,000

(2) 40,000

100,000

(1)100,000

40,000

(1) 40,000

180,000
800,000

(1)180,000
420,000

Consolidated
210,000
760,000
1,540,000
3,540,000
6,050,000

580,000
1,440,000
1,500,000
1,200,000

(2) 30,000

1,330,000

420,000

6,050,000

Problem 15-7
a.

Accounts Receivable
Cash

b.

Investment in Sea Company stock


Common stock ((30,000 shares x P20)
Investment in Sea Company stock
Common stock
Current liabilities

70,000
600,000
40,000
30,000

70,000
600,000

70,000

60

Pop Corporation and Subsidiary


Working Paper for Consolidated Balance Sheet
April 30, 2008 Date of acquisition

Assets
Cash
Accounts receivable net
Inventories
Investment in Sea Company
Plant assets
Goodwill
Total
Liabilities & Stockholders
Equity
Current liabilities
Long-term debt
Common stock
Pop
Sea
Additional paid-in capital
Retained earnings
Pop
Sea
Minority interest in net assets
Of subsidiary
Total

Pop
Corporation

Sea
Company

50,000
230,000
400,000
640,000

80,000
270,000
350,000

1,300,000

560,000

2,620,000

1,260,000

380,000
800,000

250,000
600,000

(3) 70,000

100,000
360,000

(1)100,000
(1)360,000

1,070,000

370,000

2,620,000

Adjustments

Debit

(2) 90,000
(2)220,000
(2) 80,000

& Eliminatio

Credit

(3) 70,000
(1)328,000
(2)312,000

(2) 20,000

130,000
430,000
840,000
2,080,000
80,000
3,560,000

560,000
1,420,000
1,070,000

(50,000)

(1) 50,000

1,260,000

(1) 82,000
(2) 58,000
920,000

920,000

Consolidated

370,000
140,000
3,560,000

(1) To eliminate equity accounts of Sea Company on the date of acquisition.


(2) To allocate difference, computed as follows:
Acquisition cost
Less: Book value of interest acquired (P410,000 x 80%)
Difference
Allocation:
Inventories
P( 90,000)
Plant assets
(220,000)
Long-term debt
20,000
Total
P(290,000)
Minority interest (20%
58,000
Goodwill
(3) To eliminate intercompany receivables and payables.

P640,000
328,000
312,000

232,000
P 80,000

61

Problem 15-8
1. Acquisition cost
Less: Book value of interest acquired
Common stock
APIC
Retained earnings
Difference
Allocation:
Inventory
Land
Building
Equipment
Bonds payable

P500,000
P100,000
200,000
230,000
P( 20,000)
( 10,000)
50,000
60,000
( 50,000)

530,000
( 30,000)

30,000

2. P Company and Subsidiary

Consolidated Working Paper


January 1, 2008 Date of acquisition

Debits
Cash
Accounts receivable
Inventory
Land
Building
Equipment
Investment in S Company
Total
Credits
Accounts payable
Bonds payable
Common stock P Company
Common stock S Company
APIC S Company
Retained earnings P Co.
Retained earnings S Co.
Total

P
Company

S
Company

Adjustments

& Eliminations

Debit

Credit

300,000
200,000
200,000
100,000
600,000
800,000
500,000
2,700,000

50,000
100,000
80,000
50,000
400,000
200,000

150,000

60,000
290,000

(2) 50,000

100,000
200,000

(1)100,000
(1)200,000

230,000
880,000

(1)230,000
640,000

1,500,000
1,050,000
2,700,000

880,000

(2) 20,000
(2) 10,000
(2) 30,000

(2) 50,000
(2) 60,000
(1)530,000

Consolidated
350,000
300,000
300,000
160,000
950,000
940,000
3,000,000
210,000
240,000
1,500,000

640,000

1,050,000
3,000,000

(1) To eliminate equity accounts of S Company.


(2) To allocate difference.

62

Problem 15-9
1.

2.

Acquisition cost
Less: Book value of interest acquired
Common stock (P100,000 x 80%)
APIC (P200,000 x 80%)
Retained earnings (P230,000 x 80%)
Difference
Allocation
Inventory
Land
Building
Equipment
Bonds payable
Total
Minority interest (20%)
Goodwill

P500,000
P 80,000
160,000
184,000
P (20,000)
(10,000)
50,000
60,000
(50,000)
P 30,000
( 6,000)

424,000
P 76,000

24,000
P100,000

P Company and Subsidiary


Consolidated Working Paper
January 2, 2008 Date of acquisition

Debits
Cash
Accounts receivable
Inventory
Land
Building
Equipment
Investment in S Company
Goodwill
Total
Credits
Accounts payable
Bonds payable
Common stock P Co.
Common stock S Co.
APIC S Co.
Retained earnings P Co.
Retained earnings S Co.
Minority interest in net
Assets of subsidiary
Total

P
Company

S
Company

300,000
200,000
200,000
100,000
600,000
800,000
500,000

50,000
100,000
80,000
50,000
400,000
200,000

2,700,000

880,000

150,000

60,000
290,000

(2) 50,000

100,000
200,000

(1)100,000
(1)200,000

230,000

(1)230,000

880,000

(2) 6,000
716,000

1,500,000
1,050,000

2,700,000

(1) To eliminate equity accounts of S Company


(2) To allocate difference

Adjustments

& Eliminations

Debit

Credit

(2) 20,000
(2) 10,000

(2)100,000

(2) 50,000
(2) 60,000
(1)424,000
(2) 76,000

Consolidated
350,000
300,000
300,000
160,000
950,000
940,000
100,000
3,100,000
210,000
240,000
1,500,000
1,050,000

(1)106,000
716,000

100,000
3,100,000

63

Problem 15-10
1.

2.

Acquisition cost
Less: Book value of interest acquired (100%)
Difference
Allocation
Inventory
Land
Equipment
Long-term investment in MS
Income from acquisition

P542,000
670,000
(128,000)
P (10,000)
(40,000)
20,000
(15,000)

( 45,000)
P(173,000)

P Company and Subsidiary


Consolidated Working Paper
January 2, 2008 Date of acquisition

Assets
Cash
Accounts receivable
Inventory
Land
Equipment
Investment in S Company
Long-term investment in MS
Total
Liabilities & Stockholders
Equity
Accounts payable
Common Stock P Co.
Common Stock S Co.
APIC P Co.
Retained earnings P Co.
Retained earnings S Co.
Total

P
Company

S
Company

100,000
200,000
150,000
50,000
300,000
542,000
100,000
1,442,000

100,000
150,000
130,000
80,000
200,000

175,000
400,000

115,000

200,000
667,000
1,442,000

125,000
785,000

200,000
470,000
785,000

Adjustments

& Eliminations

Debit

Credit

(2) 10,000
(2) 40,000
(2)128,000
(2) 15,000

(2) 20,000
(1)670,000

200,000
350,000
290,000
170,000
480,000
240,000
1,730,000

290,000
400,000

(1)200,000
(1)470,000
863,000

Consolidated

(2)173,000

200,000
840,000

863,000

1,730,000

(1) To eliminate equity accounts of S Company.


(2) To allocate difference

64

CHAPTER 16
MULTIPLE CHOICE
16-1:

d, because no impairment of goodwill is recognized.

16-2:

d, consolidated net income will decrease due to amortization of the allocated difference
which is not the goodwill (P60,000 / 10 years).

16-3:

d, computed as follows:
Subsidiarys net income
Amortization of the allocated difference
Minority interest in net income of subsidiary

16-4:

16-5:

P150,000
( 20,000)
P130,000

c
Acquisition cost (P500,000 + P40,000)
Less: Book value of interest acquired
Difference

P540,000
480,000
P 60,000

Cost Method
Acquisition cost
P540,000
Parents share of subsidiarys net income
Dividends received from subsidiary
Amortization of allocated difference (P60,000/20) Investment account balance, Dec. 31, 2008
P540,000

Equity Method
P540,000
120,000
( 48,000)
( 3,000)
P609,000

a
Net assets of Sol, January 2, 2008
Increase in earnings:
Net income
Dividends paid (P60,000 / 75%)
Net assets of Sol, Dec. 31, 2008

P300,000
P160,000
80,000

80,000
P380,000

Minority interest in net assets of subsidiary (P380,000 x 25%) P 95,000


16-6:

a
Punos net income
Dividend income (P40,000 x 90%)
Salas net income
Consolidated net income

P145,000
(36,000)
120,000
P229,000

65

16-7:

d
Peters net income from own operation
Peters share of Sellers net income
MINIS (P200,000 x 25%)
Consolidated net income attributable to parent

16-8:

a
Investment in Son, Jan. 1
Pops share of Sons net income (100%)
Dividends received (100%)
Amortization of allocated difference to
Equipment (P38,000 / 10)
Investment in Son, Dec. 31

16-9:

P1,000,000
200,000
( 50,000)
P1,150,000

2006
P310,000
150,000
( 60,000)

2007
P396,200
180,000
(60,000)

( 3,800)
P396,200

( 3,800)
P512,400

2008
P512,400
200,000
( 60,000)
( 3,800)
P648,600

a
Sys net income
Amortization of allocated difference
Adjusted net income of Sy

P300,000
( 60,000)
P240,000

Minority interest in net income of subsidiary (P240,000 x 10%) P 24,000


16-10: a. Under the equity method consolidated retained earnings is equal to the retained
earnings of the parent company.
16-11: c
Retained earnings, Jan. 2, 2008 Puzon
Consolidated net income attributable to parent:
Net income Puzon
P200,000
Net income Suarez
40,000
Dividend income (P20,000 x 80%)
(16,000)
MINIS (P40,000 x 20%)
( 8,000)
Dividends paid Puzon
Consolidated retained earnings, Dec. 31, 2008

P500,000

216,000
( 50,000)
P666,000

16-12: c
Acquisition cost
Less: Book value of interest acquired
Difference
Allocation due to undervaluation of net assets
Goodwill ( not impaired)

P1,700,000
1,260,000
P 440,000
( 40,000)
P 400,000

66

16-13: d
Net assets of Suazon, Jan. 2, 2008
Increase in earnings (P190,000 P125,000)
Net assets of Suazon, Dec. 31, 2008
Unamortized difference to plant assets (P100,000 P10,000)
Adjusted net assets of Suazon, Dec. 31, 2008

P1,000,000
65,000
P1,065,000
90,000
P1,175,000

Minority interest in net assets of subsidiary (1,175,000 x 20%) P 231,000


16-14: b
Prestos net income from own operations
Prestos share of Storks net income (P80,000 P23,000)
MINIS (P57,000 x 10%)
Consolidated net income attributable to parent

P140,000
57,000
( 5,700)
P191,300

16-15: b
Investment in Siso stock (at acquisition cost)

P600,000

Dividend income (P30,000 x 5%)

P 1,500

16-16 d
Consolidated net income:
Pepes net income from own operations
Sisons adjusted net income:
Net income -2008
Amortization of allocated difference
to equipment (P20,000 / 5)
Consolidated net income

P210,000
P67,000
4,000

Consolidated retained earnings:


Pepes retained earnings, Jan.2, 2007
Consolidated net income attributable to parent 2007
Pepes NI from own operations
P185,000
Sisons adjusted NI;
Net income 2007
P40,000
Amortization -2007
4,000 36,000
MINIS (P36,000 x 30%)
(10,800)
Dividends paid ,2007 - Pepe
Pepes retained earnings, Jan. 2, 2008
Consolidated net income attributable to parent 2008:
Consolidated net income (see above)
P273,000
MINIS (P63,000 x 30%)
( 18,900)
Dividends paid, 2008 Pepe
Consolidated retained earnings, Dec. 31, 2008

63,000
P273,000
P701,000

210,200
( 50,000)
P861,200
254,100
( 60,000)
P1,055,300

67

16-17: b
Acquisition cost
Less: Book value of interest acquired
Allocated to building
Consolidated retained earnings
Retained earnings, Jan. 1, 2008 Pepe
Consolidated net income attributable to parent:
Net income Precy
Adjusted net income of Susy:
Net income of Susy
P100,000
Amortization (P70,000 / 10) 2
( 3,500)
MINIS (P96,500 x 30%)
Dividends paid Precy
Consolidated retained earnings, Dec. 31, 2008

P700,000
630,000
P 70,000
P550,000
P275,000
96,500
(28,950)

342,550
( 70,000)
P822,550

Minority interest in net assets of subsidiary


Stockholders equity of Susy, June 30, 2008
Increase in earnings- net income (7/1 to 12/31)
Stockholders equity, Dec. 31, 2008
Unamortized difference (P70,000 P3,500)
Adjusted net assets of Susy, Dec. 31, 2008

P 900,000
100,000
P1,000,000
66,500
P1,066,500

Minority interest in net assets of subsidiary (P1,066,500 x 30%)

P 319,950

16-18: a
Goodwill
Acquisition cost
Less: Book value of interest acquired (P1,320,000 P320,000)
Goodwill (not impaired)

P1,200,000
1,000,000
P 200,000

Consolidated retained earnings under the equity method is equal to the retained
earnings of the parent company, P1,240,000.
16-19: b
Net income Pablo
Dividend income (P40,000 x 70%)
Sitos net income
MINIS (P70,000 x 30%)
Consolidated net income attributable to parent

P130,000
(28,000)
70,000
(21,000)
P151,000

68

16-20: c
Consolidated net income 2008
Net income Ponce
Dividend income (P15,000 x 60%)
Solis net income
MINIS (P40,000 x 40%)
Consolidated net income attributable to parent 2008
Consolidated retained earnings 2008
Retained earnings, Jan. 2, 2007- Ponce
Consolidated net income attributable to parent 2007:
Net income Ponce
Dividend income (P30,000 x 60%)
Solis net income
MINIS (P35,000 x 40%)
Dividends paid, 2007 Ponce
Consolidated retained earnings, Dec. 31, 2007
Consolidated net income attributable to parent 2008
Dividends paid. 2008 Ponce
Consolidated retained earnings, Dec. 31, 2008
16-21 a

P 90,000
(9,000)
40,000
(16,000)
P105,000
P 400,000
P70,000
(18,000)
35,000
( 14,000)

Acquisition cost
Less: Book value of interest acquired (220,000 x 80%)
Difference
Allocated to:
Depreciable assets (30,000 80%)
(37,500)
Minority interest ( 37,500 x 20%)
7,500
Goodwill

P216,000
176,000
40,000

Polo net income from own corporation


Seed net income from own operation:
Net income
Amortization (37,500 10%)
Total
Goodwill impairment lost
Consolidated net income

P 95,000

16-22: a
Retained earnings 1/1/08 Polo
Consolidated net income attributed to parent:
Consolidated net income
MINI (35,000 3,750) x 20%
Total
Dividends paid- Polo
Consolidated retained earnings 12/31/08
16-23: a

35,000
(3,750)

75,000
(25,000)
P450,000
105,000
(30,000)
P525,000

(30,000) = 80%
10,000

31,250
126,250
(8,000)
118,250
P520,000

118,250
6,250

112,000
632,000
(46,000)
586,000

(35,000 3750) x 20%

69

16-24: a
Seed stockholders equity, January 2, 2008 (80,000 + 140,000)
Undistributed earnings 2008 (35,000 15,000)
Unamortized difference (37,500 - 3750)
Seed stockholders equity (net asset), December 31, 2008
MINAS (273,750 20%)
16-25: a

220,000
20,000
33,750
273,750
54,750

(see no. 16-22)

16-26: a
Acquisition cost
Less: Book value of interest acquired (280,000 x 70%)
Difference
Allocation:
to depreciable assets
(50,000)
MINAS (30%)
15,000
Retained earnings, 1/1/08-Sisa company
Retained earnings, 1/1/07-Sisa company (squeeze)
Increase
Amortization- prior years (50,000 10 years)
Adjusted increase in earnings of Sisa (21,000/30% )
16-27: a
Retained earnings 1/1/08- Pepe
Retained earnings 1/1/08- Sisa
230,000
Adjustment and elimination:
Date of acquisition
(155,000)
Undistributed earnings to MINAS
(21,000)
Amortization- prior year
(5,000)
Consolidated retained earnings 1/1/08
16-28: a
Pepe company net income
Sisa company net income
Dividend income (10,000 x 70%)
Amortization- 2008
Consolidated net income
16-29: a
Consolidated retained earnings 1/1/08(see 16 27)
Consolidated net income attributable to parent:
Consolidated net income (see 16-28)
133,000
MINIS (25,000 5,000) 30%
(6,000)
Dividend paid- Pepe company
Consolidated retained earnings 12/31/08

231,000
196,000
35,000
35,000
230,000
155,000
75,000
(5,000)
70,000
520,000

49,000
569,000
120,000
25,000
(7,000)
(5,000)
133,000
569,000
127,000
(50,000)
646,000

70

PROBLEMS
Problem 16-1
a.

Since Pasig paid more than the P240,000 fair value of Sibols net assets, all allocations
are based on fair value with the excess of P10,000 assigned to goodwill. The
amortizations of the allocated difference are as follows:
Annual
Allocated to
Allocation
Life
Amortization
Building
Equipment

P 50,000
(20,000)

10 years
5 years

P 5,000
(4,000)

Building:
Allocation, Jan. 1, 2004
Amortization during past years -2004 to 2005 (P5,000 x 2)
Amortization for the current year 2006
Allocation, Dec. 31, 2006

P 50,000
(10,000)
( 5,000)
P 35,000

Equipment
Allocation, Jan. 1, 2004
Amortization during past years 2004 to 2005 (P4,000 x 2)
Amortization for the current year 2006
Allocation, Dec. 31, 2006

P(20,000)
8,000
4,000
P( 8,000)

b.

Since Pasig paid P20,000 less than the P240,000 fair value of Sibols net assets, a
negative difference arises. Under PFRS 3 (Business combination), the allocation of the
negative difference to the non-current assets, excluding long-term investments in
marketable securities is no longer permitted. The negative difference is immediately
amortized in profit or loss (income from acquisition). Therefore, the allocation assigned
to building and equipment is the same as in (a) above.

c.

Same as in (a) above. Except that the negative goodwill amortized to income is P60,000.

d.

Neither allocations nor amortization are found in a pooling of interests.

Problem 16-2
a.

No entry is to be recorded by Holly during 2005 under the cost method.


Allocation schedule Date of acquisition
Difference
Allocation:
Inventory
Land
Equipment
Discount on notes payable
Total
Minority interest (10%)
Goodwill (not impaired)

P240,000
P ( 5,000)
(75,000)
(60,000)
(50,000)
P(190,000)
19,000

171,000
P 69,000

71

Amortization of differential:
Inventory sold
Land sold
Equipment (P60,000/15 years)
Discount on notes payable
Total
b.

P 5,000
75,000
4,000
7,500
P91,500

Working paper elimination entries


(1)

(2)

(3)

(4)

Common stock State


500,000
Premium on common stock State
100,000
Retained earnings State
120,000
Investment in State stock
Minority interest in net assets of subsidiary
To eliminate equity accounts of State on the date
of acquisition.
Inventory
5,000
Land
75,000
Equipment
60,000
Discount on notes payable
50,000
Goodwill
69,000
Investment in State stock
Minority interest in net assets of subsidiary
To allocate difference.

648,000
72,000

240,000
19,000

Cost of goods sold


5,000
Gain on sale of land
75,000
Operating expenses (depreciation)
4,000
Interest expense
7,500
Inventory
5,000
Land
75,000
Equipment
4,000
Discount on notes payable
7,500
To amortize allocated difference.
Minority interest in net asset of subsidiary
2,350
Minority interest in net income of subsidiary
2,350
To recognize minority share in the net income (loss)
of State.
Computed as follows:
Net income
P 68,000
Adjustments for total amortization
91,500
Adjusted net income (loss)
P(23,500)
Minority interest share (P23,500 x 10%)

P 2,350

72

Problem 16-3
a.

Consolidated Buildings
Profit Company (at book value)
Simon Corporation (at fair value)
Amortization of differential (P120,000 / 6 years)
Total

P 900,000
560,000
( 20,000)
P1,440,000

b.

Consolidated Retained Earnings, Dec. 31, 2008


Retained earnings, Jan. 1 Profit Company
Consolidated net income (per c below)
Dividends paid Profit Company
Total

P 600,000
380,000
(80,000)
P 900,000

c.

Consolidated net income, Dec. 31, 2008


Total revenues (P700,000 + P400,000)
Total expenses (P400,000 + P300,000)
Amortization
Total

P1,100,000
(700,000)
( 20,000)
P 380,000

d.

Consolidated Goodwill [(P680,000 P480,000)- P120,000]

80,000

Problem 16-4
Allocation Schedule
Acquisition cost
Less: Book value of interest acquired
Difference
Allocation:
Equipment
Buildings
Goodwill (not impaired)

P206,000
140,000
P 66,000
P(40,000)
10,000

(30,000)
P 36,000

a.

Investment in Stag Company 12/31/06 (at acquisition cost)

P 206,000

b.

Minority Interest in Net Assets of Subsidiary (MINAS)

P -0-

c.

Consolidated Net Income


Net income from own operations Pony (P310,000 P198,000) P 112,000
Net income from own operations Stag (P104,000 P74,000)
30,000
Amortization
( 4,500)
Total
P 137,500

d.

Consolidated Equipment
Total book value (P320,000 + P50,000)
Allocation
Amortization (P5,000 x 3 years
Total

P 370,000
40,000
(15,000)
P 395,000

73

e.

Consolidated Buildings
Total book value
Allocation
Amortization (P500 x 3 years)
Total

P 288,000
( 10,000)
1,500
P 279,500

f.

Consolidated Goodwill (not impaired)

g.

Consolidated Common Stock (Pony)

P 290,000

h.

Consolidated Retained Earnings


Retained earning, Dec. 31, 2008 Pony
P 410,000
Add: Ponys share of Stags adjusted increase in earnings
Net earnings 2008 (P30,000 P20,000)
P10,000
Amortization
( 4,500)
5,500
Total
P 415,500

36,000

Problem 16-5
a.

Retained Earnings, Dec. 31, 2008 Sison


Stockholders equity, Dec. 31, 2008 Sison (P232,000/40%)
Stockholders equity, Jan. 1, 2005 Sison
Increase in earnings
Retained earnings, Jan. 1, 2005 Sison
Retained earnings, Dec. 31, 2008 Sison

b.

Consolidated Retained Earnings Dec. 31, 2008


Retained earnings, Jan. 1, 2005 - Perez
Net income 2005 to 2008
Dividends paid 2005 to 2008
Retained earnings, Dec. 31, 2008
Add: Perez share of adjusted net increase in Sisons
Retained earnings
P80,000
Amortization (P8,333 x 4)
(33,332)
Adjusted
P46,668
Perez interest
60%
Total
Allocation Schedule
Acquisition cost
Less: Book value of interest acquired (P500,000 x 60%)
Difference
Allocation:
Depreciable assets (P50,000 / 60%)
P(83,333)
Minority interest (40%)
33,333
Amortization per year (P83,333/10 years)

P 580,000
(500,000)
P 80,000
200,000
P 280,000
P 600,000
100,000
( 45,000)
P 655,000

28,000
P 683,000
P350,000
300,000
P 50,000
(50,000
P 8,333

74

Problem 16-6
a.

Working Paper Elimination Entries, Dec. 31, 2008


(1)

Dividend income
Dividends declared Short
To eliminate intercompany dividends.

10,000

(2)

Common stock Short


Retained earnings Short
Investment in Short Company
To eliminate equity accounts of Short at
date of acquisition

100,000
50,000

(3)

Depreciable asset
Investment in Short Company
To allocate difference.

(4)

Depreciation expense
Depreciable asset
To amortize allocated difference

30,000

5,000

10,000

150,000

30,000

5,000

75

b.

Pony Corporation and Subsidiary


Consolidation Working Paper
December 31, 2008
Pony
Corporation

Short
Company

200,000
10,000
210,000
25,000
105,000
130,000
80,000

120,000
120,000
15,000
75,000
90,000
30,000

230,000
80,000
310,000
40,000

50,000
30,000
80,000
10,000

270,000

70,000

285,000

Balance Sheet
Cash
Accounts receivable
Inventory
Depreciable asset (net)
Investment in Short stock

15,000
30,000
70,000
325,000
180,000

5,000
40,000
60,000
225,000

20,000
70,000
130,000
575,000
-

Total

620,000

330,000

Accounts payable
Notes payable
Common stock
Pony
Short
Retained earnings, Dec. 31
From above
Total

50,000
100,000

40,000
120,000

Income Statement
Sales
Dividend income
Total
Depreciation
Other expenses
Total
Net income carried forward
Retained Earnings
Retained earnings, Jan. 1
Net income from above
Total
Dividends declared
Retained earnings, Dec. 31
Carried forward

200,000
270,000
620,000

Adjustments

& Eliminations

Debit

Credit

320,000
320,000
45,000
180,000
225,000
95,000

(1) 10,000
(3)

5,000

(2) 50,000
(1) 10,000

(3) 30,000

Consolidated

(4) 5,000
(2)150,000
(3) 30,000

230,000
95,000
325,000
40,000

795,000
90,000
220,000

100,000

(2)100,000

70,000
330,000

195,000

200,000

195,000

285,000
795,000

Problem 16-7
a.

Working Paper Elimination Entries


(1)

(2)

(3)

Dividend income
Minority interest in net assets of subsidiary
Dividends declared Sisa

8,000
2,000

Common stock Sisa


100,000
Retained earnings Sisa
50,000
Investment in Sisa stock
Minority interest in net assets of subsidiary
Minority interest in net income of subsidiary
Minority interest in net assets of subsidiary

6,000

10,000

120,000
30,000
6,000

76

b.

Popo Corporation and Subsidiary


Consolidated Working Paper
December 31, 2008
Popo
Corporation
Income Statement
Sales
200,000
Dividend income
8,000
Total revenue
208,000
Depreciation expense
25,000
Other expenses
105,000
Total expenses
130,000
Net income
78,000
MI in net income of Sub.
Net income carried forward
78,000
Retained Earnings
Retained earnings, 1/1
Net income from above
Total
Dividends declared
Retained earnings, 12/31
Carried forward

Sisa
Company
120,000
120,000
15,000
75,000
90,000
30,000
30,000

Adjustments

& Eliminations

Debit

Credit

(1)

8,000

(3)

6,000

Consolidated
320,000
320,000
40,000
180,000
220,000
100,000
( 6,000)
94,000

230,000
78,000
308,000
40,000

50,000
30,000
80,000
10,000

268,000

70,000

284,000

173,000
500,000
120,000
793,000

105,000
300,000

278,000
800,000
1,078,000

Accumulated depreciation
Current liabilities
Long-term debt
Common stock
Retained earnings , 12/31
From above
MI in net assets of Subsidiary

175,000
50,000
100,000
200,000

75,000
40,000
120,000
100,000

268,000

70,000

Total

793,000

405,000

Balance Sheet
Current assets
Depreciable assets
Investment in Sisa stock
Total

(2) 50,000
(1) 10,000

(2)120,000

405,000

250,000
90,000
220,000
200,000

(2)100,000
(1)

2,000
166,000

230,000
94,000
324,000
40,000

(2) 30,000
(3) 6,000
166,000

284,000
34,000
1,078,000

77

c.

Consolidated Financial Statements


Popo Corporation and Subsidiary
Consolidated Balance Sheet
December 31, 2008
Assets
Current assets
Depreciable assets
Less: Accumulated depreciation
Total assets
Liabilities and Stockholders Equity
Current liabilities
Long-term debt
Total liabilities
Stockholders Equity
Common stock
Retained earnings, 12/31
Minority interest in net assets of subsidiary
Total liabilities and stockholders equity

P800,000
250,000

P278,000
550,000
P828,000
P 90,000
220,000
P310,000

P200,000
284,000
34,000

518,000
P828,000

Popo Corporation and Subsidiary


Consolidated Income Statement
Year Ended December 31, 2008
Sales
Expenses:
Depreciation expense
Other expenses
Consolidated net income
Minority interest in net income of subsidiary
Consolidated net income attributable to parent

P320,000
P 40,000
180,000

220,000
P100,000
6,000
P 94,000

Popo Corporation and Subsidiary


Consolidated Retained Earnings
Year Ended December 31, 2008
Retained earnings, Jan. 1 Popo
Consolidated net income attributable to parent
Total
Dividends paid Popo
Consolidated retained earnings, Dec. 31

P230,000
94,000
P324,000
40,000
P284,000

78

Problem 16-8
a.

Palo Corporation and Subsidiary


Consolidation Working Paper
December 31, 2008
Palo
Corporation

Sebo
Company

300,000
19,000
319,000
210,000
25,000
23,000
258,000
61,000

150,000

230,000
61,000
291,000
20,000

50,000
20,000
70,000
10,000

271,000

60,000

272,000

Balance Sheet
Cash
Accounts receivable
Inventory
Buildings and equipment
Investment in Sebo stock

37,000
50,000
70,000
300,000
229,000

20,000
30,000
60,000
240,000

57,000
80,000
130,000
540,000
-

Goodwill
Total

686,000

350,000

105,000
40,000
70,000
200,000

65,000
20,000
55,000
150,000

(2)150,000

271,000
686,000

60,000
350,000

239,000

Income Statement
Sales
Investment Income
Total revenues
Cost of goods sold
Depreciation expense
Other expenses
Total cost and expenses
Net income carried forward
Retained Earnings
Retained earnings, Jan. 1
Net income from above
Total
Dividends declared
Retained earnings, Dec. 31
carried forward

Accumulated depreciation
Accounts payable
Taxes payable
Common stock
Retained earnings, Dec. 31
from above
Total

150,000
85,000
20,000
25,000
130,000
20,000

Adjustments

& Eliminations

Debit

Credit

450,000
450,000
295,000
45,000
48,000
388,000
62,000

(1) 19,000

(2) 50,000
(1) 10,000

(3) 20,000

Consolidated

(1) 9,000
(2)200,000
(3) 20,000

230,000
62,000
292,000
20,000

20,000
827,000
170,000
60,000
125,000
200,000

239,000

272,000
827,000

79

b.

Consolidated Financial Statements


Palo Corporation and Subsidiary
Consolidated Income Statement
Year Ended December 31, 2008
Sales
Cost of goods sold
Gross profit
Expenses:
Depreciation expenses
Other expenses
Consolidated net income

P450,000
295,000
155,000
P45,000
48,000

93,000
P 62,000

Palo Corporation and Subsidiary


Consolidated Retained Earnings
Year Ended December 31, 2008
Retained earnings, January 1 Palo
Consolidated net income
Total
Dividends paid Palo
Retained earnings, December 31

P230,000
62,000
292,000
20,000
P272,000

Palo Corporation and Subsidiary


Consolidated Balance Sheet
December 31, 2008
Assets
Cash
Accounts receivable
Inventory
Buildings and equipment
Less: Accumulated depreciation
Goodwill
Total
Liabilities and Stockholders Equity
Accounts payable
Taxes payable
Common stock
Retained earnings, Dec. 31
Total

P540,000
170,000

P 57,000
80,000
130,000
370,000
20,000
P657,000
P 60,000
125,000
200,000
272,000
P657,000

80

Problem 16-9
1.

Acquisition cost
Less: Book value of interest acquired (80%)
Common stock (P300,000 x 80%)
Retained earnings (P400,000 x 80%)
Difference
Allocation:
Inventories
Land
Building
Equipment
Patents
Total
Minority interest (20%)
Goodwill (not impaired)

P756,000
P240,000
320,000
P( 30,000)
( 50,000)
(100,000)
75,000
( 40,000)
P(145,000)
29,000

560,000
P196,000

(116,000)
P 80,000

Working Paper Elimination Entries - December 31, 2006(not required)


(1)

(2)

(3)

(4)

Investment income
Minority interest in net assets of subsidiary
Dividends declared S
Investment in S Company

50,000
54,800

Common stock S
300,000
Retained earnings, Jan. 1 S
400,000
Investment in S Co.
Minority interest in net assets of subsidiary

560,000
140,000

Inventories
30,000
Land
50,000
Building
100,000
Patents
40,000
Goodwill
80,000
Equipment
Investment in S Company
Minority interest in net assets of subsidiary

75,000
196,000
29,000

Cost of goods sold


Inventory
Equipment (P75,000 / 10)
Expenses (amortization)
Buildings (P100,000 / 20)
Patents (P40,000 / 10)

(5)

94,800
10,000

30,000
7,500
1,500

Minority interest in net income of subsidiary


23,700
Minority interest in net assets of subsidiary
To established minority share in subsidiary net income.
Computed as follows:
Net income S Co.
P150,000
Amortization
31,500
Adjusted net income
P118,500

30,000

5,000
4,000
23,700

81

MINIS (P118,500 x 20%)


2.

P Company and Subsidiary


Consolidated Working Paper
Year Ended December 31, 2008

Income Statement
Sales
Cost of sales
Gross profit
Expenses
Operating income
Investment income
Net /consolidated income
MI interest in net income of
Subsidiary
Net income carried forward
Retained earnings
Retained earnings, 1/1
Net income from above
Total
Dividends declared
Retained earnings, 12/31
Carried forward
Balance Sheet
Cash
Accounts receivable
Inventories
Land
Buildings (net)
Equipment (net)
Patent
Investment in S Co. stock
Goodwill
Total
Accounts payable
Common stock
Additional paid-in capital
Retained earnings, 12/31
from above
MI in net assets of subsidiary
Total

P 23,700

P
Company

S
Company

1,000,000
400,000
600,000
360,000
240,000
94,800
334,800

500,000
150,000
350,000
200,000
150,000
150,000

334,800

150,000

600,000
334,800
934,800
100,000

400,000
150,000
550,000
50,000

834,800

500,000

834,800

200,000
150,000
100,000

100,000
50,000
40,000
150,000
200,000
450,000
-

300,000
200,000
140,000
200,000
295,000
680,500
36,000
-

298,000
810,800

1,558,800

1,090,000

124,000
200,000
400,000

190,000
300,000
-

834,800

500,000

1,558,800

1,090,000

Adjustments

& Eliminations

Debit

Credit

1,500,000
580,000
920,000
561,500
358,500
358,500

(4) 30,000
(4)

Consolidated

1,500

(1) 94,800
(5) 23,700

(23,700)
334,800

(2)400,000

600,000
334,800
934,800
100,000

(1) 50,000

(3) 30,000
(3) 50,000
(3)100,000
(4) 7,500
(3) 40,000

(3) 80,000

(4) 30,000
(4) 5,000
(3) 75,000
(4) 4,000
(1) 54,800
(2)560,000
(3)196,000

314,000
200,000
400,000

(2)300,000

(1) 10,000
466,200

80,000
1,931,500

(2)140,000
(3) 29,000
(5) 23,700
466,200

834,800
182,700
1,931,500

82

Problem 16-10
a.

Investment in Sally Products Co.


Cash
To record acquisition of 80% stock of Sally.
Cash

160,000

Dividend income
To record dividends received from Sally (P10,000 x 80%)
b.

8,000

160,000

8,000

Working Paper Eliminating Entries Dec. 31, 2008


Allocation schedule:
Acquisition cost
Less: Book value of interest acquired (P150,000 x 80%)
Difference
Allocated to building and equipment
P (50,000)
Minority interest (20%)
10,000
(1)

(2)

Dividend income
Minority interest in net assets of subsidiary
Dividends declared Sally

8,000
2,000

P160,000
120,000
40,000
(40,000)

10,000

Common stock Sally


100,000
Retained earnings, 1/1 Sally
50,000
Investment in Sally Products
Minority interest in net assets of subsidiary

120,000
30,000

(3)

Building and equipment


50,000
Investment in Sally Products
Minority interest in net assets of subsidiary

40,000
10,000

(4)

Depreciation expense
Accumulated depreciation Bldg

(5)

Accounts payables
Cash and receivables

(6)

Minority interest in net income of subsidiary


5,000
Minority interest in net assets of subsidiary
Computed as follows:
Net income Sally
P30,000
Amortization
(5,000)
Adjusted net income
P25,000
MINIS (P25,000 x 20%)
P 5,000

5,000
10,000

5,000
10,000
5,000

83

c.

Pilar Corporation and Subsidiary


Consolidation Working Paper
December 31, 2008

Income Statement
Sales
Dividend income
Total revenue
Cost of goods sold
Depreciation expense
Inventory losses
Total cost and expenses
Net /consolidated income

Pilar
Corporation

Sally Wood
Products

200,000
8,000
208,000

100,000

120,000
25,000
15,000
160,000
48,000

50,000
15,000
5,000
70,000
30,000

100,000

MI interest in net income of


subsidiary (MINIS
Net income carried forward
Retained earnings statement

Retained earnings, 1/1


Net income from above
Total
Dividends declared
Retained earnings, 12/31
carried forward

Balance Sheet
Cash and receivables
Inventory
Land
Buildings and equipment
Investment in Sally
Total
Accumulated depreciation
Accounts payable
Notes payable
Common stock
Retained earnings from above
MI in net assets if subsidiary

Total

48,000

30,000

298,000
48,000
346,000
30,000

90,000
30,000
120,000
10,000

316,000

110,000

81,000
260,000
80,000
500,000
160,000

65,000
90,000
80,000
150,000

1,081,000

385,000

205,000
60,000
200,000
300,000
316,000

105,000
20,000
50,000
100,000
110,000

1,081,000

385,000

Adjustments

& Eliminations

Debit

Credit

(1)

8,000

(4)

5,000

(6)

5,000

Consolidated
300,000
300,000
170,000
45,000
20,000
235,000
65,000
(5,000)
60,000

(2) 50,000
(1) 10,000

338,000
60,000
398,000
30,000
368,000

(5) 10,000
(3) 50,000

(5) 10,000

(2)120,000
(3) 40,000
(4)

5,000

(2)100,000
(1)

2,000

230,000

(2) 30,000
(3) 10,000
(6) 5,000
230,000

136,000
350,000
160,000
700,000
1,346,000
315,000
70,000
250,000
300,000
368,000
43,000

1,346,000

84

Problem 16-11
a.

Eliminating entries:
E(1)

Dividend Income
Dividends Declared
Eliminate dividend income from subsidiary.

20,000

E(2)

Common Stock Star Company


Retained Earnings, January 1
Differential
Investment in Star Company Stock
Eliminate investment balance at date
of acquisition.

150,000
50,000
20,000

E(3)

Goodwill
Retained Earnings, January 1
Differential
Assign differential at beginning of year

20,000

220,000

8,000
12,000

20,000

Porno Corporation and Star Company


Consolidated Workingpaper
December 31, 2008
_____Item_____
Income Statement
Sales
Dividend income
Credits
Cost of goods sold
Depreciation expense
Other expenses
Debits
Net income, carry forward
Retained Earnings Statement
Retained earnings, Jan. 1
Net income, from above
Dividends declared
Retained earnings, Dec. 31,
carry forward

Light
Corporation

Star
Company
200,000

Eliminations
Debit
Credit

350,000
20,000
370,000
270,000
25,000
21,000
(316,000)
54,000

200,000
135,000
20,000
10,000
(165,000)
35,000

262,000

60,000

54,000
316,000
(20,000)

35,000
95,000
(20,000)

296,000

75,000

(1) 20,000

__

20,000

____

Consolidated
550,000
_______
550,000
405,000
45,000
31,000
(481,000)
69,000

(2) 50,000
(3) 12,000
20,000
___

(1) 20,000

82,000

20,000

260,000
69,000
329,000
(20,000)
309,000

85

Balance Sheet
Cash
Accounts receivable
Inventory
Buildings and equipment
Investment in Star Company
stock
Differential
Goodwill
Debits
Accumulated depreciation
Accounts payable`
Taxes payable
Common stock
Light Corporation
Star Company
Retained earnings, from above
Credits

46,000
55,000
75,000
300,000

30,000
40,000
65,000
240,000

76,000
95,000
140,000
540,000

220,000
696,000
130,000
20,000
50,000
200,000
296,000
696,000

375,000

(2)220,000
(2) 20,000 (3) 20,000
(3) 8,000

85,000
30,000
35,000
150,000
75,000
375,000

8,000
859,000
215,000
50,000
85,000

(2)150,000
82,000
260,000

200,000
20,000
260,000

309,000
859,000

86

87

CHAPTER 17
17-1:

MULTIPLE CHOICE

Consolidated sales
Sales Papa
Sales San
Elimination of inter-company sales
Consolidated sales
Consolidated cost of goods sold
Cost of goods sold Papa
Cost of goods sold San
Eliminations:
Realized profit in beginning inventory
Unrealized profit in ending inventory
Intercompany purchases
Consolidated cost of goods sold
17-2:

4,000)
10,000
( 50,000)
P 636,000

60,000
( 10,000)
P
50,000
20%
P 10,000

d
Net income from own operation Pat
Pats share of adjusted net income of Susan:
Net income Susan
P200,000
Realized profit in beginning inventory
(P112,000 x 50%/150%)
37,500
Unrealized profit in ending inventory
(P33,000 x 50%/150%)
(11,000)
Consolidated net income
Attributable to minority interest (P226,500 x 30%)
Attributable to parent

17-4:

P 490,000
190,000

c
Net income Sisa
Unrealized profit in ending inventory upstream
Adjusted net income Sisa
Minority interest proportionate share
Minority interest in net income of subsidiary

17-3:

P 900,000
500,000
( 50,000)
P 1,350,000

P 200,000

226,500
P 426,500
67,950
P 358,550

b
Net income from own operations- Patton
Unrealized profit in ending inventory DS (P200,000 x .25)
Realized income
Solis net loss
Consolidated net income

P 300,000
(50,000)
250,000
(150,000)
P 100,000

87

17-5:

d
Pardos share of Santos net income (P300,000 x 75%)
Unrealized profit in ending inventory Upstream
(P200,000 x 25%/125%) x 75%
Realized profit in beginning inventory Upstream
(P150,000 x 25%/125%) x 75%
Investment income account balance, Dec. 31, 2008

17-6:

17-7:

d
Net income from own operation Puzon
Suazons adjusted net income:
Net income
Unrealized profit in ending inventoryUpstream (P25,000 x 40%)
Consolidated net income
MINIS (P100,000 x 25%)
Attributable to parent
b
Net income from own operation Pat
Unrealized profit in ending inventory:
2008 (P20,000 x .40)
2009 (P30,000 x .50)
Realized profit in beginning inventory
Realized income
Sun net income
Consolidated net income

17-8:

( 30,000)
22,500
P 217,500
P 200,000

P110,000
( 10,000)

100,000
P 300,000
(25.000)
P 275,000

2008
P 500,000

2009
P 550,000

(8,000)
492,000
200,000
P 692,000

(15,000)
8,000
543,000
225,000
P 768,000

a
Net income from own operation Pip
Adjusted net income of Sol:
Net income
P 250,000
Realized profit in beginning inventoryUpstream (P40,000 x 40%)
16,000
Unrealized profit in ending inventoryUpstream (P70,000 x 30%)
( 21,000)
Consolidated net income - 2008

17-9:

P 225,000

a
Net income from own operations Popo
Unrealized profit in ending inventory Downstream
Realized separate net income Popo
Popos share of Sottos adjusted net income:
Net income
P 360,000
Realized profit in beginning inventoryUpstream
10,000
MINIS (P370,000 x 5%)
Attributable to parent

P 400,000

245,000
P 645,000
P 500,000
( 15,000)
P 485,000

370,000
( 18,500)
P 836,500

88

17-10: a
Stockholders equity Sands, Dec. 31, 2008
P5,500,000
Unamortized difference (P1,000,000 P200,000)
800,000
Adjusted stockholders equity (net assets) Sands
P6,300,000
Minority interest in net assets of subsidiary (P6,300,000 x 40%)P2,520,000
17-11: d
Gross profit rate Short (P110,000 / P200,000)

55%

Inventories
Inventory from outsiders Power
Inventory from outsiders Short
Powers inventory acquired from Short at cost:
[P5,000 (P5,000 x 55%)}
Consolidated ending inventories
Investment income
Powers share of Shorts net income (P50,000 x 75%)
Unrealized profit in ending inventory upstream
(P5,000 x 55%) x 75%
Realized profit in beginning inventory upstream
(P10,000 x 55%) x 75%
Investment income, Dec. 31, 2008

P 5,000
25,000
2,250
P 32,250
P 37,500
( 2,063)
4,125
P 39,562

Investment in Short Company


Acquisition cost (P80,000 x 80%)
Unrealized profit in ending inventory
Realized profit in beginning inventory
Investment in Short Company, Dec. 31, 2008

P 60,000
( 2,063)
4,125
P 62,062

Minority interest in net assets of subsidiary


Stockholders equity, Dec. 31, 2006 Short
Realized profit in beginning inventory (P10,000 x 55%)
Unrealized profit in ending inventory (P5,000 x 55%)
Adjusted net assets of Short, Dec. 31, 2006
Minority interest
MINAS

P 80,000
5,500
( 2,750)
P 82,750
25%
P 20,687.50

17-12: b
Gross profit rate of Sit (P200,000 / P500,000)
Net income from own operations Pit
Adjusted net income of Sit:
Net income
P 75,000
Realized profit in beginning inventoryUpstream (P40,000 x 40%)
16,000
Unrealized profit in ending inventoryUpstream (P25,000 x 40%)
( 10,000)
Consolidated net income
MINIS (P281,000 x 10%)
Attributable to parent

40%
P 200,000

81,000
P 281,000
( 8,100)
P 272,900

89

17-13: b
Gross profit of Sir (P120,000 / P400,000)

30%

Consolidated cost of sales


Cost of sales Pig
Cost of sales Sir
Eliminations:
Realized profit in beginning inventory (P70,000 x 30%)
Unrealized profit in ending inventory (P60,000 x 30%)
Intercompany purchases
Consolidated cost of sales
Consolidated net income
Net income from own operations Pig
Pigs share of Sirs adjusted net income:
Net income
Realized profit in beginning inventory
Unrealized profit in ending inventory
Consolidated net income
MINIS (P83,000 x 10%)
Attributable to parent

( 21,000)
18,000
(200,000)
P 677,000
P 200,000

P 80,000
21,000
(18,000)

17-14: a

2006
Pal Corp net income
150,000
Intercompany profit in ending inventory:
2006
(14,000)
2007
2008
Pal net income from own operation
136,000
Solo net income from own operation
100,000
Consolidated net income
236,000
MINIS:
2006(100,000 14,000) x 40%
34,400
2007(90,000 +14,000 21,000) 40%
2008(160,000 + 21,000 24,000) 40%
Consolidated NI attributable to Parent 201,600

17-15: a
Acquisition cost
Less: book value of interest acquired (400,000 x 60%)
Difference
Allocated to Equipment
( 20,000)
MINAS (40%)
8,000
Total sales
Intercompany sales (30,000 + 80,000)
Consolidated sales

P 600,000
280,000

83,000
283,000
(8,300)
P 274,700

2007
240,000
14,000
(21,000)
233,000
90,000
323,000
33,200
289,800

2008
300,000
21,000
( 24,000)
297,000
160,000
427,000

62,800
394,200
252,000
240,000
12,000
(12,000)
600,000
(110,000)
490,000

90

17-16: c
Total cost of goods sold (250,000 +120,000)
370,000
Adjustments due to intercompany sale:
COGS charged for intercompany sale (20,000 + 50,000) 70,000
COGS charged by: Star (30,000 6,000)
24,000
Polo (80,000 20,000)
60,000
Total
154,000
Cost of goods sold for consolidated entity:
20,000 x (24,000/30,000)
(16,000)
50,000 x (60,000/80,000)
(37,500) (100,500)
Consolidated cost of goods sold
269,500
17-17: c
Polo Corp. net income from own operation (105,000 25,000)
Unrealized profit in ending inventory-DS (6,000 x 10/30)
Adjusted Polo Corp. net income from own operation
Star Corp. net income from own operation:
Net income
45,000
Unrealized profit in EI-US (20,000 x 30/80)
(7,500)
Amortization (20,000/10 years)
(2,000)
Consolidated net income
MINIS (35,500 x 40%)
Attributable to Parent
17-18: a
Pepsi net income from own operation
Sarsi net income
90,000
Unrealized profit in EI (45,000 x 60/180)
(15,000)
Consolidated net income
MINIS (75,000 x 30%)
Consolidated net income attributable to Parent-2007
17-19: a
Inventory-Pepsi
P 30,000
Less: unrealized profit in books of Sarsi:
(135,000 90,000) x (30,000/135,000)
(10,000)
Inventory-Sarsi
P110,000
Less: unrealized profit in books of Pepsi:
(280,000 140,000) x (110,000/280,000)
(55,000)
Consolidated inventory 12/31/08
17-20: a
Cost of goods sold on sale of inventory on hand-1/1/08:
[45,000 x (120,000/180,000)]
Cost of goods sold on purchases from Sarsi- 2008
[(135,000 30,000) x (90,000/135,000)]
Cost of goods sold on purchases from Pepsi- 2008
[(280,000 110,000) x (140,000/280,000)]
Consolidated cost of goods sold-2008

80,000
(2,000)
78,000

35,500
113,500
(14,200)
99,300
160,000
75,000
235,000
(22,500)
212,500

20,000
55,000
75,000

30,000
70,000
85,000
185,000

91

17-21: b
Pepsi net income
Sarsi net income
Realized profit in beginning inventory - 2008
Unrealized profit in ending inventory- Sarsi
Unrealized profit in ending inventory- Pepsi
Consolidated net income

220,000
85,000
15,000
(10,000)
(55,000)
255,000

92

PROBLEMS
Problem 17-1
a.

b.

Consolidated Net Income


Net income from own operations P Company
S Co. adjusted net income:
Net income S
Unrealized profit in ending inventory
Upstream (P9,000 x 50/150)
Realized profit in beginning inventoryUpstream (P6,000 x 50/150)
Consolidated net income

P200,000
P30,000
(3,000)
2,000

Minority Interest in Net Income of Subsidiary


Adjusted net income - S Co.
Minority interest
Minority interest in net income of subsidiary

29,000
P229,000
P 29,000
x 30%
P 8,700

Problem 17-2
a.

Consolidated Net Income


Net income from own operations P Co.
Realized profit in beginning inventory Downstream
(P10,500 x 40/140)
Adjusted net income
S Company adjusted net income:
Net income S
Unrealized profit in ending inventoryUpstream (P8,000 x 25%)
Consolidated net income

P100,000
3,000
P103,000
P90,000
(2,000)

88,000
P191,000

b.

Minority Interest in Net Income of Subsidiary


Adjusted net income S Co.
Minority interest
MINIS

P88,000
x 20%
P17,600

c.

Minority Interest in Net Assets of Subsidiary


Stockholders equity , Jan. 1, 2008 S Company
Increase in earnings 2008 (P90,000 P35,000)
Unrealized profit in ending inventory Upstream
Stockholders equity, Dec. 31, 2008 S Company
Minority interest
MINAS

P350,000
55,000
(2,000)
P403,000
x 20%
P 80,600

93

Problem 17-3
a.

b.

Net Assets, Dec. 31, 2008 S Co.


Minority interest per consolidated balance sheet, 12/31
Unrealized profit in ending inventory Upstream
(P36,000 x 25/125) x 20%
Minority interest per books S Co.
Divided by
Net assets- S Co.
Price Paid
Net assets S Co., Dec. 31, 2008
Net income S Co.
Net assets S Co., Jan. 1, 2008
Parents interest
Book value of interest acquired
Difference
Price paid

P158,560
1,440
P160,000
20%
P800,000
P800,000
(160,000)
P640,000
x 80%
P512,000
20,000
P532,000

Problem 17-4
The computation of the selected consolidation balances are affected by the inter-company profit
in downstream intercompany sales as computed below:
Unrealized profit in ending inventory, Dec. 31, 2007 Downstream
Intercompany profit (P120,000 P72,000)
Inventory left at year end
Unrealized profit, Dec. 31, 20057

P 48,000
x 30%
P 14,400

Unrealized profit in ending inventory, Dec. 31, 2008 Downstream


Intercompany profit (P250,000 P200,000)
P 50,000
Inventory left at year end
x 20%
Unrealized profit, Dec. 31, 2008
P 10,000
a.

b.

c.

Consolidated Sales
Apo
Bicol
Intercompany sales 2008
Total
Cost of goods sold
Apos book value
Bicols book value
Intercompany sales-2008
Realized profit in beginning inventory 2008
Unrealized profit in ending inventory 2008
Consolidated cost of goods sold
Operating expenses
Apo
Bicol
Total

P800,000
600,000
(250,000)
P1,150,000
P 535,000
400,000
(250,000)
( 14,400)
10,000
P 680,600
P 100,000
100,000
P 200,000

94

d.

Dividend Income 0 (eliminated)

e.

Minority Interest in Net Income of Subsidiary (P100,000 x 30%)

f.

Inventory
Apo
Bicol
Unrealized profit in ending inventory, Dec. 31, 2008
Consolidated inventory
Minority Interest in Net Assets of Subsidiary
Stockholders equity , Jan. 1, 2008 Bicol
Increase in earnings in 2008 (P100,000 P50,000)
Stockholders equity, Dec. 31, 2008 Bicol
Minority interest
MINAS

g.

P 30,000
P 298,000
700,000
(10,000)
P 988,000
P 950,000
50,000
P1,000,000
x 30%
P 300,000

Problem 17-5
P Company and Subsidiary
Consolidated Income Statement
Year Ended December 31, 2008
Sales (P2,000,000 + P1,000,000 P600,000)
Cost of goods sold (Schedule 1)
Gross profit
Expenses
Income before income tax
Provision for income tax
Consolidated net income after income tax
Attributable to minority interest (Schedule 2)
Attributable to parent

P2,400,000
704,000
1,696,000
600,000
1,096,000
440,000
656,000
44,000
P 612,000

Schedule 1:
Cost of sales P Company
Purchases from S Company
Intercompany profit in beginning inventory (P60,000 x 25%)
Intercompany profit in ending inventory (P76,000 x 25%)
Total
Cost of sales S Company
Consolidated cost of sales

P 800,000
(600,000)
( 15,000)
19,000
P 204,000
500,000
P 704,000

Schedule 2:
Net income S Company
Realized profit in beginning inventory Upstream
Unrealized profit in ending inventory Upstream
Adjusted net income
Minority interest
MINIS

P 180,000
15,000
(19,000)
P 176,000
x 25%
P 44,000

95

Problem 17-6
a.

Working Paper Eliminating Entries


(1)

(2)

(3)

Dividend income
Minority interest in net assets of subsidiary (20%)
Dividends declared- D (P32,000 / 80%)
To eliminate intercompany dividends.

32,000
8,000

Common stock S
90,000
Retained earnings S
220,000
Investment in S Co. stock
Minority interest in net assets of subsidiary
To eliminate equity accounts of S on the date of
acquisition.
Minority interest in net assets of subsidiary
Retained earnings, Jan. 1
Cost of goods sold
To eliminate realized profit in beginning inventory

4,000
16,000

(4)

Sales

150,000

(5)

Minority interest in net income of subsidiary


Minority interest in net assets of subsidiary
To establish minority interest in net income of S Co.
computed as follows:
Sales
Cost and expenses (P140,000 +P20,000)
Net income
Realized profit in beginning inventory Upstream
Unrealized profit in ending inventory Upstream
Adjusted net income
Minority interest
MINIS

Cost of goods sold


Inventory, Dec. 31 (P45,000 x 33.33%)
To eliminated intercompany sales and unrealized
profit in ending inventory.

40,000

248,000
62,000

20,000

135,000
15,000

8,000

8,000

P200,000
160,000
40,000
20,000
(15,000)
P 45,000
x 20%
P 9,000

b.

Consolidated Net Income


Net income from own operations (P250,000 P205,000)
S Company adjusted net income
Consolidated net income

P 45,000
45,000
P 90,000

c.

Minority Interest in Net Assets of Subsidiary (MINAS)


Stockholders equity, Dec. 31, 2008 S Company
Adjusted net income 2008
Adjusted net assets, Dec. 31, 2008 S Co.
Minority interest
MINAS

P 310,000
45,000
P 355,000
x 20%
P 71,000

96

Problem 17-7
a.

Consolidated Sales
Reported total sales (P600,000 + P510,000)
Intercompany sales (P140,000 + P240,000)
Consolidated sales

P1,170,000
(380,000)
P 790,000

b.

Consolidated Cost of Goods Sold


Cost of goods sold:
Pato (P660,000 / 140%)
Sales (P510,000 / 120%
Amount to be eliminated (P128,000 + P232,000) see entry below
Total

P 471,429
425,000
( 360,000)
P 536,429

Elimination of intercompany sales and intercompany profit in inventory:


Downstream Sales
Sales
Inventory (P42,000 x 40/140)
Cost of goods sold
Upstream Sales
Sales
Inventory (P48,000 x 20/120)
Cost of goods sold
c.

d.

140,000

240,000

Consolidated Net Income


Net income from own operations Pato
Unrealized profit in ending inventory Downstream
Adjusted net income Pato
Add: Adjusted net income of Sales Co.
Net income
P20,000
Unrealized profit in ending inventory Upstream (8,000)
Consolidated net income
Consolidated Inventory, Dec. 31, 2008
Inventory reported Pato
Inventory reported Sales
Unrealized profit in ending inventory (P8,000 + P12,000)
Consolidated inventory

12,000
128,000

8,000
232,000
P 70,000
(12,000)
P 58,000
12,000
P 70,000

P 48,000
42,000
(20,000)
P 70,000

97

Problem 17-8
a.

b.

c.

Unrealized Profit in Beginning Inventory


Beginning inventory - Downstream
Gross profit rate (P240,000/ P400,000)
Unrealized profit in beginning inventory

P 100,000
x 60%
P 60,000

Unrealized Profit in Ending Inventory


Ending inventory Downstream (P200,000 x 80%)
Gross profit rate
Unrealized profit in ending inventory

P 160,000
x 60%
P 96,000

Intercompany Sales
Sales P Company
Sales S Company
Intercompany sales 2008
Consolidated sales

P2,000,000
1,000,000
(400,000)
P2,600,000

Intercompany Cost of Sales


Cost of sales P Company
Cost of sales S Company
Intercompany purchases
Intercompany profit in beginning inventory
Intercompany profit in ending inventory
Consolidated cost of sales

P 800,000
600,000
(400,000)
( 60,000)
96,000
P1,036,000

Parents interest (40,000 shares / 50,000 shares)


P Company Entries 2008:
(1)
Investment in S Company stock
Income from subsidiary
To record Ps share of S Co. income
(P120,000 x 80%)

80%
96,000

(2)

Cash

(2)

Income from subsidiary


36,000
Investment in S Company stock
To adjust income from subsidiary for intercompany
profit in :
Ending inventory
(96,000)
Beginning inventory
60,000
Net adjustment
( 36,000)

Investment in S Company stock


To record dividends received from S
(P60,000 x 80%)

48,000

96,000

48,000

36,000

98

d.

Working Paper Eliminating Entries:

(1)

Income from subsidiary


Minority interest in net assets of subsidiary
(P60,000 x 20%)
Dividends declared S
Investment in S Company stock
To eliminate intercompany dividends.

(2)

e.

60,000
12,000

Common stock S Co.


500,000
Retained earnings S Co.
860,000
Investment in S Company stock
Minority interest in net assets of subsidiary
To eliminate equity accounts of S Company as of
beginning of year.

(3)

Goodwill
Investment in S Company stock
To allocate difference to goodwill.

(4)

Retained earnings Jan. 1


60,000
Cost of sales
To eliminate realized profit in beginning inventoryDownstream.

(5)

Cost of sales
96,000
Inventories
To eliminate unrealized profit in ending inventoryDownstream.

(6)

Sales

(7)

Accounts payable
50,000
Accounts receivable
To eliminate intercompany payables and receivables.

(8)

Minority interest in net income of subsidiary


24,000
Minority interest in net assets of subsidiary
To establish minority share of S net income
(P120,000 x 20%)

Cost of sales
To eliminate intercompany sales.

60,000

400,000

Consolidated Net Income


Net Income from own operations P Company (P480,000 P60,000)
Realized profit in beginning inventory
Unrealized profit in ending inventory
Adjusted net income P Compay
S Company net income
Consolidated net income

60,000
12,000

1,088,000
272,000

60,000

60,000

96,000

400,000

50,000

24,000

P420,000
60,000
( 96,000)
P384,000
120,000
P504,000

99

CHAPTER 18
MULTIPLE CHOICE
18-1:

a
Equipment at original cost
Accumulated depreciation:
Time of sale
Current depreciation based on
Original cost (P500,000/10 years

18-2:

P500,000
P250,000
50,000

b
Net income Sol
Unrealized gain on sale of computer, Dec. 31
Adjusted net income
Minority interest proportionate share
Minority interest in net income of subsidiary (MINIS)

18-3:

b
Net income from own operations Prime
Unrealized gain Downstream
Realized net income Prime
Second Company net income
Consolidated net income

18-4:

P300,000

P100,000
( 30,000)
P 70,000
30%
P 21,000
2005
P200,000
(30,000)
P170,000
100,000
P270,000

2006
P250,000
__P250,000
150,000
P400,000

c
Net income Saw
Unrealized loss-Upstream
Realized loss ((P12,000 / 5) x 6/12
Adjusted net income Saw

P100,000
12,000
( 1,200)
P110,800

Minority interest in net income of subsidiary (P110,800 x 25%) P 27,700


18-5:

c
Equipment at original cost
Accumulated depreciation:
Time of sale
Current depreciation (P900,000/10)

P1,000,000
P360,000
90,000

P 450,000

100

18-6:

a
Adjusted net income Susie (P12,000 / 40%)
Add back: Unrealized gain Upstream
Net income of Susie 2008

18-7:

a
Original cost
Amount debited to Truck account
Selling price of the truck Amount paid

18-8:

P100,000
(48,000)
P 52,000

c
Net income Po
Unrealized gain, Dec. 31 DS
Net income from own operation Po
Net income of So
Consolidated net income, Dec. 31, 2008
MINIS (P180,000 x 20%)
Attributable to parent

18-9:

P 30,000
90,000
P120,000

P200,000
(30,000)
270,000
180,000
P350,000
(36,000)
P314,000

b
Stockholders equity, Jan. 1, 2008 Sy
Increase in earnings 2008 (P65,000 P30,000)
Stockholders equity, Dec. 31, 2008 Sy

P1,000,000
35,000
P1,035,000

Minority interest in net assets of subsidiary (P1,035,000 x 20%)P 207,000


18-10: b
Consolidated net income attributable to parent:
Net income Pink
Unrealized gain, July 1- Downstream
Realized gain, Dec. 31 (P50,000 / 10) x 6/12
Realized net income Pink
Sodas adjusted net loss:
Net loss
P(40,000)
Unrealized loss, 1/1 Upstream
15,000
Realized loss, 12/31 (P15,000/5)
( 3,000)
Consolidated net income, Dec. 31, 2008
MI in net loss of subsidiary (P28,000 x 20%)
Attributable to parent

P300,000
( 50,000)
2,500
P252,500

(28,000)
P224,500
5,600
P230,100

101

Minority interest in net assets of subsidiary


Net assets, Jan. 1, 2008 (P1,240,000 / 80%)
Decrease in earnings:
Net loss
Dividends paid
Net assets, Dec. 31, 2008
Unrealized loss, Dec. 31 (Upstream)
Adjusted net assets, Dec. 31, 2008

P1,550,000
P40,000
30,000

( 70,000)
P1,480,000
( 12,000)
P1,468,000

Minority interest in net assets of subsidiary (P1,468,000 x 20%)P 293,600


18-11: a
Net assets, Dec. 31, 2008
Minority interest, Dec. 31, 2008
Add: MI share of unrealized profit in ending inventory -Upstream
(P36,000 x 20%) x 20%
MI share of unrealized gain on sale of equipment- Upstream
(P60,000 x 20%) (P12,000 / 5)
Minority interest before adjustment
Net assets Steve, Dec.31, 2008 (P200,000 / 20%)
Investment in Steve Company stock Equity method
Acquisition cost:
Net assets, Dec. 31, 2008
Less: net income steve
MINIS
P36,960
MI share of unrealized profit in ending
Inventory Upstream
1,440
MI share of unrealized gain on sale of
Equipment Upstream
9,600
MINIS per book
P48,000
Divided by
20%
Net assets, Jan. 1, 2008
Parents proportionate share
Book value of interest acquired
Add: difference
Purchase price (acquisition cost)
Add: Investment income
Peters share of Steve net income (P240,000 x 80%)
Unrealized profit in ending inventory Downstream
(P24,000 x 20%/120%) x 100%
Unrealized profit in beginning inventory Upstream
(P36,000 x 25/125%) x 80%
Unrealized gain on sale of equipment Upstream
(P48,000 9,600)
Investment in Steve Company, Dec. 31, 2008

P188,960
1,440
9,600
P200,000
P1,000,000

P1,000,000

240,000
P 760,000
x 80%
P 608,000
20,000
P 628,000
P 192,000
( 4,000)
( 5,760)
( 38,400)
P 771,840

102

18-12: a
Net income from own operations Pipe
Pipes share of Smokers adjusted net income:
Net income
Unrealized gain, July 1, 2008 Upstream
Realized gain, Dec. 31, 2008 (P50,000/5)x
Consolidated net income, Dec. 31, 2008
18-13: d
Net income from operations Parent
Parents share of adjusted net income of Sub:
Net income
Unrealized gain Upstream
Realized gain: 2007 (P9,000/3) x
2008 (P9,000/3)
Adjusted net income
Consolidated net income
MINIS
Attributable to parent

P400,000
P100,000
(50,000)
5,000

55,000
P455,000

2007
P100,000

2008
P120,000

P 60,000
( 9,000)
750
P 51,750
P151,750
(10,350)
P141,400

P 75,000
3,000
P 78,000
P198,000
(15,600)
P182,400

18-14: d
Investment in Sili Company stock Equity method
Acquisition cost
Investment income net of dividends 2005 to 2007:
Increase in earnings (P500,000 P200,000) x 75%
Investment income, Dec. 31, 2007:
Share of Silis net income (P60,000 x 75%)
45,000
Unrealized gain on sale of land Downstream
(15,000)
Unrealized loss on sale of building Downstream
10,000
Realized loss on sale of building (P10,000 / 5) x 75% ( 1,500)
Investment income, Dec. 31, 2008:
Share of Silis net income (P70,000 x 75%)
52,500
Realized loss (P10,000 / 5)
(2,000)
Dividends received:
2007: (P10,000 x 75%)
7,500
2008: (P20,000 x 75%)
15,000
Investment in Sili Company stock, Dec. 31, 2008

P500,000
225,000

38,500
50,500
(22,500)
P791,500

103

18-15: a
Investment in Saw Company stock, Dec. 31, 2008
Acquisition cost
Investment income 2002 to 2006:
Increase in earnings (P500,000 P300,000) x 90%
Investment income 2007 (see above)
Investment income 2008:
Powers share of Saws net income (P120,000 x 90%) P108,000
Realized loss on sale of warehouse (P20,000/2) x 90%
(9,000)
Dividends received:
2007: ( P20,000 x 90%)
P 18,000
2008: ( P30,000 x 90%)
27,000
Investment in Saw Company stock account balance 12/31/08

P550,000
180,000
101,250
99,000
(45,000)
P885,250

104

PROBLEMS

Problem 18-1
Computation of the missing amounts in the working paper eliminations for P Corporation and S
Company:
(1)
P640 (P3,200 x 20%)
(2)
P2,560 (P3,200 x 80%)
(3)
P1,600 (P800 x 2)
(4)
P320 (P1,600 x 20%)
(5)
P1,280 (P1,600 x 80%)
(6)
P3,200 (P800 x 4)
Problem 18-2
a.
Consolidated Net Income
Net income from own operations P Company
Unrealized gain on sale of equipment, Dec. 31 Downstream
Adjusted net income P Co,
S Company net income
Consolidated net income

P200,000
(30,000)
P170,000
180,000
P350,000

b.

Minority interest in net income of subsidiary (P180,000 x 20%)

P 36,000

c.

Minority Interest in Net Assets of Subsidiary:


Stockholders equity, Jan. 1, 2008 S Company
Increase in earnings 2008 (P180,000 P60,000)
Stockholders equity, Dec. 31, 2008 S Company
Minority interest
Minority interest in net assets of subsidiary

P 900,000
120,000
P1,020,000
x 20%
P 204,000

Problem 18-3
Pony Corporation and Subsidiary
Consolidated Income Statement
Year Ended December 31, 2008
Sales (P500,000 + P300,000)
Gain on sale of machinery (schedule 1)
Total revenue
Cost of sales P200,000 + P130,000)
Gross profit
Expenses:
Depreciation (P50,000 +P30,000 P5,000)
P 75,000
Other expenses (P80,000 + P140,000)
220,000
Consolidated net income
Attributable to minority interest (P190,000 + P5,000) +10,000) x 25%
Attributable to parent

P800,000
20,000
820,000
330,000
490,000
295,000
785,000
(28,750)
P266,250

105

Schedule 1:
Selling price Dec. 28, 2008
Book value (P65,000 5) x3
Gain on sale
Unrealized gain (P25,000 P15,000)
Total gain

P36,000
26,000
10,000
10,000
P20,000

Problem 18-4
a.

b.

Consolidated Net Income


Net income from own operations P Company
Adjusted net income of S Company:
Net income S
Unrealized gain, 4/1/08 - Upstream
Realized gain, 12/31/08 (P30,000/5) x 9/12
Consolidated net income
MINIS (P124,500 x 20%)
Attributable to parent
Minority Interest in Net Assets of Subsidiary
Stockholders equity , Jan. 1, 2008 S Company
Increase in adjusted earnings 2008:
Net earnings (P150,000 P50,000)
Unrealized gain 12/31 (P30,000 P4.500)
Stockholders equity, Dec. 31, 2008
Minority interest
MINAS

P300,000
P150,000
( 30,000)
4,500

124,500
424,500
(24,900)
P399,600

P800,000
P100,000
(25,500)

74,500
P874,500
x 20%
P114,900

Problem 18-5
a.

b.

Consolidated Net Income - 2008


Net income from own operations BJ
Gain on sale of machine, July 1 - Downstream
Realized gain, Dec. 31 (P50,000 / 10) x 6/12
Adjusted net income BJ
Net income (loss) of DK:
Net income (loss) DK
Loss on sale of truck , Jan. 1 - Upstream
Realized loss, Dec. 31 (P15,000 / 5)
Consolidated net income
Minority Interest in Net Income of Subsidiary
Net loss from own operations DK
Upstream loss on sale of truck
Realized loss on sale of truck
Adjusted net loss
Minority interest
MI in net loss of subsidiary

P300,000
(50,000)
2,500
P252,500
P(40,000)
15,000
( 3,000)

(28,000)
P224,500
P (40,000)
15,000
( 3,000)
P ( 28,000)
x 20%
P ( 5,600)

106

c.

Minority Interest in Net Assets of Subsidiary


Net assets, Jan. 1, 2006 (P1,240,000 / 80%)
Increase in earnings (loss) -2006 (P40,000 + P30,000)
Net assets, Dec. 31, 2006
Unrealized loss Upstream (P15,000 P3,000)
Adjusted net assets
Minority interest
MINAS

P1,550,000
(70,000)
P1,480,000
12,000
P1,492,000
x 20%
P 298,400

Problem 18-6
Texas Company and Subsidiary
Consolidated Income Statement
Year Ended December 31, 2008
Sales
Cost of goods sold
Gross profit
Expenses (P200,000 + P100,000 P8,000 )
Consolidated net income
Attributable to minority interest (P150,000 x 25%)
Attributable to parent

P1,500,000
650,000
850,000
292,000
P 558,000
37,500
P 520,500

Adjustment for expenses (depreciation) = P40,000 / 5 years.


Problem 18-7
a.

Leo Company and Subsidiary


Consolidated Balance Sheet Working Paper
December 31, 2007
Leo
Company
101,000
80,000
150,000
400,000
141,000

Taurus
Corporation
20,000
40,000
90,000
300,000

Total debits

872,000

450,000

Accumulated depreciation
Accounts payable
Notes payable
Common stock
Retained earnings

135,000
90,000
200,000
100,000
347,000

85,000
25,000
90,000
200,000
50,000

Cash and receivables


Inventory
Land
Building and equipment
Investment in stock Taurus

MI in net assets in Subsidiary


Total

872,000

450,000

Adjustments

& Eliminations

Debit

Credit

(2) 10,000
(3) 9,000
(3) 15,000

(1)150,000
(2) 6,000

Consolidated
121.000
120,000
250,000
709,000
1,200,000

(3) 24,000

244,000
115,000
290,000
100,000
347,000

(1)100,000
(2) 4,000

104,000

(1)200,000
(1) 50,000

1,200,000

107

(1) To eliminate equity accounts of subsidiary


(2) To intercompany gain on sale of land.
(3) To eliminate intercompany gain on sale of equipment debited to Investment account and restore equipment to
its original book value.

b.

Leo Company and Subsidiary


Consolidated Balance Sheet
December 31, 2008
Cash and receivables
Inventory
Land
Building and equipment
Less: Accumulated depreciation
Total assets

P121,000
120,000
250,000

P709,000
244,000

465,000
P956,000

Accounts payable
Notes payable
Common stock stock
Retained earnings
Minority interest in net assets of subsidiary
Total liabilities and equity

P115,000
290,000
100,000
347,000
104,000
P956,000

Problem 18-8
a.

Working Paper Elimination Entries Dec. 31, 2008


(1)

(2)

Dividend income
Minority interest in net assets of subsidiary
Dividends declared Jupiter
To eliminate intercompany dividends

4,000
1,000

Common stock Jupiter


100,000
Retained earnings Jupiter
50,000
Investment in Jupiter stock
Minority interest in net assets of subsidiary
To eliminate equity accounts of Jupiter as of the
date of acquisition

(3)

Goodwill
Investment in Jupiter Stock
To allocate difference to goodwill

40,000

(4)

Retained earnings Jan. 1


8,000
Minority interest in net assets of subsidiary
2,000
Land
To eliminate unrealized gain on sale of land Upstream.

5,000

120,000
30,000

40,000

10,000

108

(5)

Gain on sale of equipment


Building and equipment
Accumulated depreciation
To eliminate gain on sale of equipment

20,000
5,000

2,000

25,000

(6)

Accumulated depreciation
Depreciation
To adjust excess depreciation

(7)

Accounts payable
7,000
Accounts receivable
To eliminate intercompany payables and receivables.

7,000

(8)

Minority interest in net income of subsidiary


6,000
Minority interest in net assets of subsidiary
(P40,000 10,000) x 20%

6,000

2,000

109

b.

Vincent Company and Subsidiary


Consolidation Working Paper
December 31, 2008

Income Statement
Sales
Gain on sale of equipment
Dividend income
Total revenues
Cost of goods sold
Depreciation
Other expenses
Total cost and expenses
Net/consolidated income
MI in net income of subsidiary
Net income carried forward
Retained Earnings Statement
Retained earnings, Jan.1
Net income from above
Total
Dividends declared
Retained earnings, Dec. 31
Carried forward
Balance Sheet
Cash and receivables
Inventory
Land
Buildings and equipment
Investment in Jupiter stock
Goodwill
Total
Accumulated depreciation
Accounts payable
Bonds payable
Common stock
Retained earnings from above
MI in net assets of subsidiary
Total

Vincent
Company

Jupiter
Company

240,000
20,000
4,000
264,000
140,000
25,000
15,000
180,000
84,000

120,000

84,000

40,000

294,000

105,000

84,000
378,000
30,000

40,000
145,000
5,000

348,000

140,000

113,000
260,000
80,000
500,000
160,000

35,000
90,000
80,000
150,000

1,113,000

355,000

205,000
60,000
200,000
300,000
348,000

45,000
20,000
50,000
100,000
140,000

1,113,000

355,000

120,000
60,000
15,000
5,000
80,000
40,000

Adjustments

& Eliminations

Debit

Credit

(5) 20,000
(1) 4,000
(6)

(8)

2,000

6,000

(2) 50,000
(4) 8,000

Consolidated
360,000
360,000
200,000
38,000
20,000
258,000
102,000
(6,000)
96,000
341,000

(1)

5,000

96,000
437,000
30,000
407,000

(7)
(5)

5,000

(3) 40,000
(6)
(7)

2,000
7,000

7,000

(4) 10,000
(2)120,000
(3) 40,000

(5) 25,000

(2)100,000
(1)
(4)

1,000
2,000

(2) 30,000
(8) 6,000

245,000

245,000

141,000
350,000
150,000
655,000
40,000
1,336,000
273,000
73,000
250,000
300,000
407,000
33,000
1,336,000

110

c.

Consolidated Financial Statements


Vincent Company and Subsidiary
Consolidated Balance Sheet
December 31, 2008
Assets
Cash and receivables
Inventory
Land
Buildings and equipment
Less: Accumulated depreciation
Goodwill
Total assets
Liabilities and Stockholders equity
Liabilities
Accounts payable
Bonds payable
Total liabilities
Stockholders Equity
Common stock
Retained earnings
Minority interest in net assets of subsidiary
Total liabilities and stockholders equity

P655,000
273,000

P 141,000
350,000
150,000
382,000
40,000
P1,063,000

73,000
250,000
P 323,000
P300,000
407,000
33,000

740,000
P1,063,000

Vincent Company and Subsidiary


Consolidated Income Statement
Year Ended December 31, 2008
Sales
Cost of goods sold
Gross profit
Expenses: Depreciation
Other expenses
Consolidated net income
Attributable to minority interest
Attributable to parent

P 38,000
20,000

P 360,000
200,000
160,000
58,000
102,000
6,000
P 96,000

Vincent Company and Subsidiary


Consolidated Retained Earnings
Year Ended December 31, 2008
Retained earnings, Jan. 1 Vincent
Retained earnings, Jan. 1 Jupiter
Total
Consolidated net income attributable to parent
Dividends declared Vincent
Consolidated retained earnings

P 294,000
47,000
341,000
96,000
( 30,000)
P 407,000

111

Problem 18-9
(a)

P100,000 (the common stock of Phantom only)

(b)

P140,000

P250,000 (P593,000 P343,000)

(d)

P100,000 (P126,000 P35,000) + [(P25,000 + P85,000) - P101,000]

(e)

(f)

Purchase price, Jan. 1, 2008


Undistributed earnings from 1/1/05 to 1/1/08:
(P80,000 P30,000) x 60%
Undistributed income for 2008 (P30,000 P20,000) x 60%
Total
Adjustments:
Unrealized gain on sale of land Downstream (g)
Unrealized gain on sale of equipment Upstream
(P9,000 P3,000) x 60%
Adjusted Investment account balance, Dec. 31, 2008

(g)

P7,000 (P70,000 + P90,000) P153,000

(h)

(i)

P510,000 [P345,000 + P150,000 + (P60,000 P45,000)]

(j)

P278,000 = P180,000 + P80,000 + [(P60,000/5) x 4 ]


Less [(P45,000 / 3) x 2 years]

(k)

Retained earnings, Dec. 31, 2008


Less: Share of unrealized profit on sale of equipment:
Gain record [P45,000 (P60,000 x 3/5)]
Realized in 2008 (P9,000 / 3)
Unrealized
Phantoms interest
Consolidated retained earnings

(l)

P105,000
30,000
6,000
P141,000
(7,000)
(3,600)
P 70,400

P380,000
P9,000
3,000
P6,000
x 60%

Net income Shadow, 2008 (P250,000 P220,000)


Realized gain on sale of building c Dec. 31, 2006 Upstream
Adjusted net income
Minority interest
Minority interest in net income of subsidiary

3,600
P376,400
P 30,000
3,000
P 33,000
x 40%
P 13,200

112

Problem 18-10
Supporting computations
(1)

Allocation schedule (purchase price)


Less: Book value of interest acquired (P350,000 x 60%)
Difference
Allocated to patents (P120,000 x 60%)
Goodwill

P 372,000
210,000
P 162,000
( 72,000)
P 90,000

Amortization of patents (P120,000 / 12)

P 10,000

(2)

Unrealized gain on intercompany sale of building Upstream, Jan. 1, 2006:


Unrealized gain at date of sale (P80,000 P30,000)
P 50,000
Realized gain (P50,000 / 5) x 2 years
(20,000)
Unrealized gain as of Jan. 1, 2008
P 30,000

(3)

Realized profit from intercompany sale of inventory Downstream, 1/1/08:


Remaining inventory as of Dec. 31, 2007
P 50,000
Gross profit rate on sales 2007 (P30,000 / P150,000)
x 20%
Realized profit as of Jan. 1, 2008
P 10,000

(4)

Unrealized profit from intercompany sale of inventory Downstream, 12/31/08


Remaining inventory as of Dec. 31, 2008
P 40,000
Gross profit rate on sales 2008 (P48,000 / P160,000)
x 30%
Unrealized profit as of Dec. 31, 2008
P 12,000

Consolidated balances 2008


a.

Cost of goods Sold


Cost of goods sold Apex
Cost of goods sold Small
Intercompany sale of inventory 2008
Realized profit on beginning inventory
Unrealized profit on ending inventory
Consolidated

P 460,000
205,000
(160,000)
( 10,000)
12,000)
P 507,000

b.

Operating Expenses
Operating expenses Apex
Operating expenses Small
Amortization (No. 1 above)
Excess depreciation (P50,000 / 5 years)
Consolidated

P 170,000
70,000
10,000
(10,000)
P 240,000

113

c.

Consolidated Net Income


Sales (after elimination of intercompany sales)
Cost of goods sold (a)
Operating expenses (b)
Minority interest in net income of subsidiary:
Net income Small
Realized gain on sale of building Upstream
Adjusted net income
Minority interest
Attributable to parent

P 840,000
(507,000)
(240,000)
P25,000
10,000
P35,000
x 40%

( 14,000)
P 79,000

d.

Consolidated Retained Earnings, Jan. 1, 2008


Retained earnings, Jan. 1, 2008 Apes
P 690,000
Amortization of patents 2002 to 2007 (P10,000 x 6)
(60,000)
Unrealized profit on inventory, 2007 Downstream
(10,000)
Unrealized gain on sale of building, 1/1/08 - Upstream (P30,000 x 60%) (18,000)
Consolidated retained earnings, Jan. 1, 2008
P 602,000

e.

Consolidated Inventory
Inventory Apex
Inventory Small
Unrealized profit in inventory Dec. 31, 2008
Consolidated inventory

P 233,000
229,000
( 12,000)
P 450,000

f.

Consolidated Building
Buildings Apex
Buildings Small
Unrealized gain, Jan. 1, 2006
Realized gain, 2006 2008 (P10,000 x 3 )
Consolidated buildings

P 308,000
202,000
(50,000)
30,000
P 490,000

g.

Consolidated Patents
Patents Small
Allocation
Amortization, 2002 2008 (P10,000 x 7)
Consolidated patents (net)

P 20,000
120,000
( 70,000)
P 70,000

h.

Consolidated Common Stock = P300,000 (Apex common stock)

i.

Minority Interest in Net Assets of Subsidiary


Stockholders equity Small, Dec. 31, 2008 (P100,000 + P420,000)
Unrealized gain on sale of building, Dec. 31,2008 Upstream
Adjusted net assets, Dec. 31, 2008
Minority interest
Minority interest in net assets of subsidiary

P 520,000
(20,000)
P 500,000
x 40%
P 200,000

114

Problem 18-11
a.

Working Paper Elimination Entries


(1)

Retained earnings Jan. 1


Investment in Duke
To adjust Investment account for unrealized profit
in inventory on Dec. 31, 2005 (P10,000 x 60%)

6,000

(2)

Income from Duke Company


Minority interest in net assets of subsidiary
Dividends declared Duke
Investment in Duke
To eliminate intercompany dividends.

84,000
24,000

(3)

Common stock Duke


320,000
APIC Duke
90,000
Retained earnings, 1/1 Duke
620,000
Investment in Duke (60%)
Minority interest in net assets of subsidiary (40%)
To eliminate equity accounts of Duke as of beginning of year.

(4)

Goodwill
Investment in Duke
To allocate difference

(5)

Impairment loss
Goodwill
To reduce goodwill for impairment.

(6)

Sales

(7)

Investment in Duke
6,000
Minority interest in net assets of subsidiary
4,000
Cost of goods sold
To eliminate realized profit in beginning inventory Upstream

Cost of goods sold


To eliminated intercompany sales

100,000

5,000

200,000

6,000

60,000
48,000

618,000
412,000

100,000

5,000

200,000

10,000

(8)

Cost of goods sold


12,000
Inventory
To eliminate unrealized profit in ending inventory Upstream

12,000

(9)

Investment in Duke
Land
To eliminate gain on sale of land Downstream

40,000

40,000

(10)

Liabilities
Accounts receivable
To eliminate intercompany debt.

40,000

(11)

Minority interest in net income of subsidiary


Minority interest in net assets of subsidiary
(P140,000 + 10,000 P12,000 P5,000) x 40%

53,200

40,000

53,200

115

b.

Minority Interest in Net Income of Subsidiary


Net income Duke
Realized profit in beginning inventory Upstream
Unrealized profit in ending inventory Upstream
Impairment loss
Adjusted net income Duke
Minority interest
MINIS

c.

Minority Interest in Net Assets of Subsidiary


Stockholders equity, 1/1/08 Duke (P320,000 + P90,000 + 620,000)
Increase in earnings 2008 (P140,000 P60,000)
P80,000
Unrealized profit in ending inventory
(12,000)
Realized profit in beginning inventory
10,000
Goodwill impairment loss
( 5,000)
Adjusted net assets, 12/31/08
Minority interest
MINAS

d.

P140,000
10,000
( 12,000)
( 5,000)
P133,000
x 40%
P 53,200

Consolidated Net Income


Net income from own operations Baron (P284,000 P84,000)
Unrealized gain on sale of land
Adjusted net income- Baron
Adjusted net income of Duke (P133,000 x 60%)
Consolidated net income

P1,030,000

73,000
P1,103,000
x 40%
P 441,200
P 200,000
(10,000)
P 190,000
133,000
P 323,000

Problem 18 12
Pluto Corporation and Subsidiary Star Corporation
Comparative Consolidated Income Statement
Years Ended December 31, 2007 and 2008
.
.
Sales
Cost of goods sold
Gross profit
Operation expenses
Consolidated net income
Minority interest in net income of subsidiary
Attributable to equity holders of Pluto
Supporting computations:
.
.
Consolidated sales:
Combined sales
Less: intercompany sales
Consolidated sales

2008
P800,000
442,000
358,000
178,000
180,000
10,000
P170,000

December 31

2008
P850,000
(50,000)
P800,000

2007
P660,000
368,000
292,000
138,000
154,000
10,000
P144,000

2007
P700,000
(40,000)
P660,000

.
.
.
.
.
.
.
.
.
.

116

Consolidated cost of goods sold:


Combined costs of good sold
Intercompany sales
Unrealized profit in ending inventory
Unrealized profit in beginning inventory
Consolidated cost of goods sold

P490,000
(50,000)
10,000
(8,000)
P442,000

P400,000
(40,000)
8,000
P368,000

.
.

Consolidated operating expenses


Combined operating expenses
Realized gain on sale of equipment (P10,000/.2)
Consolidated operating expenses

P180,000
(2,000)
P178,000

P140,000
(2,000)
P138,000

.
.

Minority interest in net income of subsidiary


Star Companys reported net income
Gain on upstream sale of land
Unrealized gain in upstream, inventory sales
Realized net income
Minority interest
Minority interest in net income of subsidiary

P65,000
(5,000)
(10,000)
P50,000
20%
P10,000

P50,000
P50,000
20%
P10,000

.
.
.

117

CHAPTER 19
Multiple Choice
19-1:

d.
Direct exchange rate:
December 1
December 31
Decrease in forex rate

1 2.22 yen =
1 2.70 yen =

P
P

Forex gain (200,000 yen x P0.08)

19-2:

19-3:

16,000

c.
Forex rate, December 1
Forex rate, December 31
Increase in forex rate

Forex gain (1,500,000 yen x P0.02)

0.45
0.47
0.02
30,000

d.
September 30:
Forex rate, September 1
Forex rate, September 30
Decrease in forex rate

P
P

Forex gain (200,000 hkg.$ x P0.02)

December 31:
Forex rate, October 1
Forex rate, December 30
Increase in forex rate

Forex loss (200,000 hkg.$ x P0.03)

19-4:

0.45
0.37
0.08

5.61
5.59
0.02
4,000

5.59
5.62
0.03

(6,000)

c.
Forex loss on importation of merchandise:
Peso equivalent, January 10, 2004
Peso equivalent, April 20, 2004
Forex loss (increase)
Forex loss on notes payable:
Peso equivalent, September 1, 2004
Peso equivalent, December 31, 2004
Forex loss on principal
Add: Forex loss on interest
Based on P 3,2000,000
Based on P 300,000,000 (P3,000,000x10%x4/12)
Forex loss
Total forex loss (P 8,000 + P220,000)

P
P

600,000
608,000
(8,000)

P 3,000,000
3,200,000
P (200,000)
P 120,000
100,000

20,000
P (220,000)
P (228,000)

118

19-5:

a.
Direct forex rate Transaction date (P 1 $0.018)
Direct forex rate Balance sheet date (P 1 $0.017)
Direct forex rate Settlement date (P 1 $0.020)
Forex gain (loss), 2004
Transaction date ($10,000 x P55.5555)
Balance sheet ($10,000 x P 58.8235)
Forex loss (increase)
Forex gain (loss), 2005
Balance sheet date ($10,000 x P58.8235)
Settlement data ($10,000 x P 50.00)
Forex gain (decrease)

19-6:

55.5555
58.8235
50.0000

555,555
588,235
P ( 32,680)
P
P

588,235
500,000
88,235

b.
Adjusted value of accounts receivable, 6/30
Peso equivalent, 7/27
Forex loss

19-7:

P
P

315,000
300,000
(15,000)

a.
2004
Forex rate, 11/5/04
Forex rate, 12/31/04
Decrease in forex rate
Payable in foreign currency
Forex gain
2005
Forex rate, 12/31/04
Forex rate, 1/15/05
Decrease in forex rate
Payable in foreign currency
Forex loss

19-8:

a. (1000,000 FC x P 0.85)

19-9:

c. (50,000 FC x P 0.6498)

P
P
P
P
P
P

0.4295
0.4245
0.0050
50,000
250
0.4245
0.4345
0.0100
50,000
(500)

19-10: b
Forward rate, 3/31/04
Selling spot rate, 4/30/04
Decrease
Forward contract receivable
Forex loss

0.25
0.22
P
0.03
100,000 FC
P
3,000

119

19-11: d. forex gain (loss) on purchase commitments is based on the changes in the forward rates.
Forward rates December 31, 2004
90-day forward rate

.0055
.0055

On December 31, 2004, no changes in forward rates occurred, so no forex gains (losses) are to be
recognized on December 31, 2004 under both transactions.

19-12: b.
Forward contract receivable (P100,000 Baht x P1.650)
Spot rate (100,000 Baht x P1.600)
Forex loss

165,000
160,000
P
(5,000)

19-13: d.
Import transaction Based on spot rates:
12/31/04: Forex loss [1,000,000 Francs x (P6.01 P6.16)]
Forward Contract Based on forward rates:
12/31/04: Forex gain [1,000,000 Francs x (P6.06 P6.07)]

P (150,000)
P

Net forex loss

10,000

P (140,000)

19-14: b.
12/31/04: Forex gain [$5,000 x (P56.50 P56.60)]
3/31/04 : Forex loss:
Forward contract receivable ($5,000 x P56.60)
Settlement at spot rate ($5,000 x P56.32)

P
P 283,000
281,600

Net forex loss

500
(1,400)

(900)

7,800
8,200
(400)

19-15: a.
Increase in forward rates:
Forward contract receivable, 11/1/04 (10,000 fc x P.78)
Forward contract receivable, 12/31/04 (10,000 fc x P82)
Forex loss

19-16: b. Increase in forward rates [100,000 x (P.90 P.93)]


19-17: c
Gain from increase in intrinsic value of put option
Loss from decrease in fair value of available for sale securities
Loss from decrease in time value of the option
Net loss on hedging activity 12/31/07

100
(100)
(60)
(60)

19-18: a

120

19-19: a
12/01/08:
12/31/08:

A$ 70,000/P42,000= 1.667 A$ to P1.00


A$ 70,000/P41,700= 1.679 A$ to P1.00

19-20: a, A$70,000 x P.57 (December 31 forward rate)


19-21: a, The balance will not change, because it is denominated in Philippine peso.
19-22: a
P82,000/KRW 400,000 = P.205
The P82,000 is the amount of the peso payable to bank. This amount is computed
using the forward rate.

121

Problems
Problem 19-1

Accounts
Receivable
Case 1

NA

Case 2

Accounts
Payable
P 160,000 (a)

P 38,000

NA

Foreign
Currency
Transactions
Exchange Loss

Foreign
Currency
Transactions
Exchange Gain

NA

P 20,000 (b)

NA

P 2,000 (d)

Case 3

NA

P 13,500 (e)

P 1,500 (f)

NA

Case 4

P 6,250 (g)

NA

P 1,250 (h)

NA

(a)
(b)
(c)
(d)
(e)
(f)
(g)
(h)

$40,000 x P4.00
$40,000 x (P4.00 P4.50)
$20,000 x P1.90
$20,000 x (P1.90 P1.80)
$30,000 x P.45
$30,000 x (P.45 P.40)
$2,500,000 x P.0025
$2,500,000 x (P.0025 P.003)

Problem 19-2
a.

May 1

Inventory (or purchases)


Accounts payable
Foreign purchases denominated in
Philippine pesos.

800,000

June 20

Accounts payable
Cash
Settlement.

800,000

July 1

Accounts receivable
Sales
Foreign sales denominated in
Philippine pesos.

500,000

August 10

Cash
Accounts receivable
Collections.

500,000

800,000

800,000

500,000

500,000

122

b.

May 1

Inventory (or purchases)


800,000
Accounts payable
Foreign purchases denominated in yen:
P800,000 / P.40 = 2,000,000 yen

June 20

Foreign currency transaction loss


Accounts payable
P900,000 = 2,000,000 yen x P.45
800,000 = 2,000,000 yen x P.40
P100,000

100,000

Accounts payable
Cash or foreign currency
Settlement denominated in yen.

900,000

July 1

Accounts receivable
500,000
Sales
Foreign sale denominated in Hongkong $
P500,000 / P5.20 = 96,154 Hkg $

August 10

Accounts receivable
Foreign currency transaction gain
P501,924 = 96,154 Hkg. $ x P 5.22
500,000 = 96,154 Hkg. $ x P 5.20
P 1,924
Cash or foreign currency
Accounts receivable
Collections

1,924

501,924

800,000

100,000

900,000

500,000

1,924

501,924

Problem 19-3
a.

No net exposure between November 1 and March 1. Michael, Inc. has hedged its foreign currency
purchase commitment with a forward contract to receive an equal number of foreign currency
units.

b.

November 1:

Forward contract receivable


3,076,800
Forward contract payable
To record forward contract at forward rate:
240,000 Ringgit x P12.82

December 31:

Forex loss
4,800
Forward contract receivable
To record forex loss for the decrease in
forward rate, P240,000 x P.02

3,076,800

4,800

123

December 31:

Firm commitment for merchandise


4,800
Forex gain
To record increase in fair value of the
Purchase commitment, and resultant
gain or the decrease in the forward rate.

March 1:

Forward contract payable


3,076,800
Cash
To record settlement of forward contract.

3,076,800

Cash (240,000 x P12.86)


3,086,400
Forex loss (240,000 x P.02)
4,800
Forward contract receivable
To record receipt of 240,000 Ringgit when
the spot rate is P12.86.

3,091,200

Firm commitment for merchandise


Forex gain
To record change in value of the firm
commitment.

4,800

Purchases (240,000 x P12.82)


3,076,800
Firm commitment for merchandise
Cash
To record purchases of merchandise.

4,800

4,800

9,600
3,086,400

Problem 19-4
June 1:

Purchases
460,000
Accounts payable
To record purchases ( 1,000,000 x P.46).
Forward contract receivable (fc)
480,000
Forward contract payable
To record purchase of 1,000,000 for delivery
in 60 days at forward rate of P.48.

June 30:

Forex loss
20,000
Accounts payable
To record forex loss for the increase in spot
rate, 1,000,000 x (P.46 P.48)
Forward contract receivable
20,000
Forex gain
To record forex gain for the increase
in forward rate, 1,000,000 x (P.48 P.50).

460,000

480,000

20,000

20,000

124

August 1:

Accounts payable
Forex loss ( 1,000,000 x P.03)
Cash ( 1,000,000 x P.51)
To record settlement.

480,000
30,000

510,000

Cash ( 1,000,000 x P.51)


510,000
Forex gain
Forward contract receivable
To record receipt of 1,000,000 at spot rate

10,000
500,000

Forward contract payable


480,000
Cash
To record settlement of forward contract.

480,000

Accounts receivable
1,280,000
Sales
To record sale (100,000 Rial x P12.80).

1,280,000

Problem 19-5
December 1:

Forward contract receivable


1,240,000
Forward contract payable (fc)
To record forward contract to sell 100,000 Rial
at a 90-day forward rate of P12.40.
December 31:

Forex loss
10,000
Accounts receivable
To adjust receivable for the decrease in spot rate
and record forex loss, 100,000 Rial x (P12.80 P 12.70).
Forex loss
20,000
Forward contract payable (FC)
To record forex gain for the increase in forward rate,
100,000 Rial x (P12.40 P12.60).

March 1:

Cash
1,290,000
Forex gain(100,000 Rial x P.20)
Accounts receivable
To record collection of accounts receivable at spot rate.
Forward contract payable (FC)
Forex loss
Cash (100,000 Rial x P12.60)
To record delivery of 100,000 Rial.

1,260,000
30,000

Cash
1,240,000
Forward contract receivable
To record collection for forward contract.

1,240,000

10,000

20,000

20,000
1,270,000

1,290,000

1,240,000

125

Problem 19-6
October 1:

Forward contract receivable


Forward contract payable (fc)
(15,000 Baht x P1.16)

17,400

December 31:

Forex loss
Forward contract payable (fc)
15,000 Baht x (P1.16 P1.17).

150

Firm commitment for materials


Forex gain
To record increase in fair value of sales
commitment.

150

April 1:

Cash
17,400
Forward contract receivable
To record collection of forward contract.
Forward contract payable
17,550
Forex gain
Cash /fc (15,000 Baht x P1.16)
To record delivery of 15,000 Baht at forward rate
of P1.16.
Forex loss
Firm commitment for materials
Cash/fc (15,000 Baht x P1.18)
Sales
To record sales.

150
17,700

17,400

150

150

17,400

150
17,400

150
17,700

Problem 19-7
Contract 1:
October 1:

Forward contract receivable (fc)


160,000
Forward contract payable
To record forward contract to buy 400,000 at P40.

December 31:

Forward contract receivable (fc)


4,000
Forex gain
To record forex gain for the increase in forward
rate of P.01.

April 1:

Cash ( 400,000 x P.43)


172,000
Forward contract receivable (fc)
Forex gain
To record receipt of 400,000 at spot rate of
P.43.
Forward contract payable
160,000
Cash
To record payment of forward contract.

160,000

4,000

164,000
2,000

160,000

Contract 2:

126

December 1:

Forward contract receivable


9,200
Forward contract payable (fc)
To record forward contract to sell 2 million Rupiah
at P.0046.

December 31:

Forward contract payable


200
Forex gain
To record forex gain for the decrease in forward
Rate by P.0001.

March 1:

Cash
9,200
Forward contract receivable
To record settlement of forward contract.

9,200

Forward contract payable (fc)


9,000
Forex loss
800
Cash
To record payment of 2 million Rupiah at spot
rate of P.0049.

9,800

9,200

200

Problem 19-8
1.

Investment in Siam
Cash
To record purchase of 40% of Siam Company.

1,920,000

Cash

123,200
Investment in Siam
To record dividends from Siam for 20 x 1 (P308,000 x 40%)

1,920,000

123,200

Investment in Siam
243,200
Other comprehensive income-translation adjustment 128,800
Income from Siam
372,000
To record income from Siam for 20x1 computed as follows:
Share of reported income (P930,000 x 40%)
P 372,000
Share of equity adjustment (P322,000 x 40%)
128,800
2a.

Cash (fc)

b.

Loan payable (fc)


60,000
Other comprehensive income-translation adjustment
To adjust loan to current rate (P1.55 P1.50) x 1,200,000.

c.

Interest expense
91,500
Interest payable (fc)
Forex gain
To accrue interest expense (1,200,000 x 10% x year x P1.525)
And record interest payable (1,200,000 x 10% x year x P 1.50).

Loans payable (fc)


To record loan of 1,200,000 NT dollar at P1.55.

1,860,000

1,860,000

60,000

90,000
1,500

127

Problem 19-9
1.

Cash (fc)

168,000
Accounts receivable (fc)
Forex gain
To record collection of 100,000 Baht from Queens Company.

167,000
1,000

Forward contract payable (fc)


167,000
Forex loss
1,000
Cash (fc)
To record delivery of 100,000 Baht in settlement of the
forward contract denominated in Baht.

168,000

Cash

Forward contract receivable


To record receipt of Phil. Pesos in settlement of the
forward contract receivable.
2.

164,000

Forward contract payable


Cash
To record payment of forward contract payable.

76,000

Cash (fc)
Forex loss

75,000
500

Forward contract receivable (fc)


To record collection of forward contract receivable:
(10,000,000 Rupiah x P.00750)

Accounts payable (fc)


75,500
Cash (fc)
Forex gain
To record payment of accounts payable to Indon Co.
(1,000,000 Rupiah x P.00750)

164,000

76,000

75,500

75,000
500

Problem 19-10
1.

Schedule of forward contract items at December 31, 2004 balance sheet.


Current assets:
Forward contract receivable (Siam hedge: in Phil. pesos)
Forward contract receivable (Indon hedge: 10,000,000 x P.0077)
Forward contract receivable (Speculation in Yen: 200,000 x P.670)
Change in value of firm commitment

P 168,000
77,000
134,000
1,000

Current liabilities:
Accounts payable (Indon account: 10,000,000 x P.0077)
P 77,000
Forward contract payable (Siam hedge: 100,000 Baht x P1.690)
169,000
Forward contract payable (Speculation in Yen: payable in Phil. pesos)
130,000

128

2.

Forex gain or loss for 2004:


Indon:

P2,000 loss on account payable offset by P2,000 gain on


Forward contract receivable

Siam:

Forex loss is offset by the change in the value of firm


commitment

Speculation: The speculation is accounted for at the forward rate


throughout the life of the contract. Therefore, the forward
contract receivable is adjusted to P 134,000 (the rate for
60-day futures at December 31 and the P4,000 gain is
recognized).
Forex gain for 2004 in the income statement

4,000
P 4,000

Problem 19-11
a. Entry to record the purchase of the call options on November 30, 2007
November 30, 2007
Call Options
Cash
Purchase call options for 10,000 barrels
of oil at a premium of P2 per barrel for
March 1, 2008. The options are at the money
of P30 per barrel; therefore, the entire
P20,000 is time value

20,000

20,000

b. Adjusting entry on December 31, 2007:


December 31, 2007
Loss on hedge activity
Call options
Record the decrease in the time value
of the options to current earnings.
Call options
Other comprehensive income
Record the increase in the intrinsic value
of the options to other comprehensive income.

14,000

10,000

14,000

10,000

c. Entries to record March 1, 2008, expiration of options, the sales of option, and the purchase
of oil.
March 1, 2008
Loss on hedge activity
Call options
Record the decrease in the time value
of the options to current earnings.
The options have expired.

6,000

6,000

129

Call options
Other comprehensive income
Record the increase in the intrinsic value
of the options to other comprehensive income.

20,000

Cash
Call options
Record the sale of the call options.

30,000

Oil inventory
Cash
Record the purchase of 10,000 barrels
of oil at the spot price of P33 per barrel.

330,000

20,000

30,000

330,000

d. June 1, 2008, entries to record the sale of the oil and other entries:
June 1, 2008
Cash
Sales
Record the sale of 10,000 barrels
of oil at P34 per barrel

340,000

Cost of goods sold


Oil inventory
Recognize the cost of the oil sold.

330,000

Other comprehensive income- reclassification


Cost of goods sold
Reclassify into earnings the other
comprehensive income from the cash flow hedge.

30,000

340,000

330,000

30,000

130

CHAPTER 20
MULTIPLE CHOICE
20-1:

b
Bad debt expense (S$ 6,000 x P28.20)
P169,200
Amortization of patents (S$ 4,000 x P28.20)
112,800
Rent expense (S$ 10,000 x P28.20)
282,000
Total
P564,000
Average rate (P28.20) is used to translate all expenses since this is a reasonable
estimation.

20-2:

b
Machinery [(24,000 Ringgit 10) x P10.42]
Equipment [(12,000 Ringgit 10) x P10.42]
Total depreciation

20-3:

d
Accounts receivable
Prepaid expenses
Property and equipment (net)
Total

20-4:

P 25,008
12,504
P 37,512

P120,000
55,000
275,000
P450,000

a
Depreciation expense (H$ 12,000 x P5.80)
Bad debts (H$ 8,000 x P5.80)
Rent (H$ 20,000 x P5.80)
Total

P 69,600
46,400
116,000
P232,000

Average rate for the year is used in translating depreciation expense because this is more
reasonable estimation than the rate when the related asset was acquired (P4.80).
20-5:

d
[25,000 LCU x (1 2)]

20-6:

20-7:

d
Long-term receivable: [1,500,000 LCU x (1 1.5 LCU)]

P1,000,000

Long-term debt: [2,400,000 LCU x (1 1.5 LCU)]

P1,600,000

b (NT Dollar 10,000 x P1.70)

131

20-8:

20-9:

b
Beginning inventory
Purchases
Goods available for sale
Ending inventory
Cost of goods sold

40,000 Rupee
300,000
340,000
30,000
310,000 Rupee

Translated cost of goods sold (310,000 Rupee x P.5745)

P178,095

c
NZ Dollar
20,000

Net assets, 1/1/05


Increase in net assets:
Net income, 2005 (30,000 20,000) 10,000
Net assets 12/31/05
30,000
Net assets at current rate
30,000
Translation adjustment, 2005 (positive)

Rate
P15

Phil Peso
P300,000

P19

190,000
P490,000
630,000
P140,000

P21

20-10: b
Equipment [800,000 x (1 50)]

P16,000

Accumulated depreciation [560,000 x (1 50)]

P11,200

Depreciation [80,000 x (1 50)]

P 1,600

20-11: a (25,000 Rupee x P1.24)


20-12: d (5,000 Rupee x P1.30)
20-13: c
Investment cost, Jan. 1, 2005
Less: Book and fair value of net assets acquired
(300,000 Rp x P1.20)
Goodwill
Goodwill
Impairment
Balance

Pesos
P42,000
4,340 (3,500 Rp x P1.24)
P37,660

Translated balance (31,500 Rp x P1.32)


Less: umimpaired goodwill
Translation adjustment

P402,000
360,000
P 42,000
Rupee
35.000 (P42,000 / P1.20)
3,500
31,500
P41,580
37,660
P 3,920

132

20-14: b
Translation adjustment from translating the trial balance
Translation adjustment from translating goodwill (per 20-13)
Total translation adjustment

P12,000 Cr
3,920 Cr
P15,920

20-15: b
Investment in Subsidiary account, Jan. 1, 2005
Share in subsidiary net income [(800,000 yen x 70%) x P.57]
Translation adjustment (P25,000 x 70%)
Share of subsidiary dividends [(50,000 yen x 70%) x P.59]
Investment in Subsidiary account, December 31, 2005

P1,600,000
319,200
17,500
(
20,650)
P1,916,050

20-16: d
20-17: a
Initial inventory transfer date:
Selling price
Cost
Profit
Balance sheet date (75,000 x 1.70)
20-18: a

Phil Peso

Thailand Baht

P120,0001.60
(80,000)
40,000

75,000 B

127,500

(P127,500 40,000)

20-19: a
Net asset beginning
Net income
Net asset translated at rate:
During the year
At end of year

Yen
200,000
200,000
400,000
400,000

Exchange Rate
.44
.46
.48

Translation adjustment (credit)


20-20: a

75,000 B

Phil Peso
88,000
92,000
180,000
192,000
(12,000)

(70,000 rupee x P1.50)

20-21: c
Investment cost
Book value of interest acquired (1,100,000 x 1.10) x .80
Goodwill

P1,210,000
968,000
242,000

133

PROBLEMS
Problem 20-1
a.
Pilipino Company
Translation Working Paper
December 31, 2005
Cash
Accounts receivable
Inventory
Plant and equipment
Cost of sales
Operating expenses
Depreciation expenses
Total
Accumulated Other Comprehensive Income Translation Adjustment
Total debits
Accumulated depreciation
Accounts payable
Common stock
Retained earnings, Jan. 1
Sales
Total credits

Yen
40,000
120,000
100,000
700,000
360,000
140,000
60,000
1,520,000

Exchange Rate
.40 CR
.40 CR
.40 CR
.40 CR
.425 AR
.425 AR
.425 AR

Phil. Pesos
16,000
48,000
40,000
280,000
153,000
59,500
25,500
622,000
25,000
647,000

240,000
80,000
200,000
400,000
600,000
1,5200,000

.40
.40
.44
.44
.425

CR
CR
HR
HR
AR

P96,000
32,000
88,000
176,000
255,000
647,000

CR Current Rate
AR Average Rate
HR Historical Rate

b. Proof of Translation Adjustment


Net assets at beginning of year
Adjustment for changes in net assets
Position during year
Net income for the year
Net assets translated at rates in effect
For those items
Net assets at end of year
Change in translation adjustment during year
(to OCI) net decrease (debit)

Yen
600,000

Translation Rate
.44

Phil. Pesos
264,000

40,000

.425

17,000

640,000

.40

281,000
256,000
25,000

Accumulated OCI translation adjustment,1/1

-0-

Accumulated OCI translation adjustment,


Dec. 31 (debit)

25,000

134

Problem 20-2
(1)

Trial Balance Translation

Thailand
Baht
7,000
20,000
5,000
25,000
100,000
70,000
10,000
30,000
15,000
282,000

Cash
Accounts receivable (net)
Receivable from Davao
Inventory
Plant and equipment
Cost of goods sold
Depreciation expense
Operating expenses
Dividends paid
Total debits
Accumulated depreciation
Accounts payable
Bonds payable
Common stock
Sales
Total
Accumulated other comprehensive
Income Translation adj. (credit)
Total credits

Translation
Rate
1.60 CR
1.60 CR
1.60 CR
1.60 CR
1.60 CR
1.50 AR
1.50 AR
1.50 AR
1.54 HR

10,000
12,000
50,000
60,000
150,000
282,.000

1.60
1.60
1.80
1.46
1.50

Philippine
Pesos
11,200
32,000
8,000
40,000
160,000
105,000
15,000
45,000
23,000
439,300

CR
CR
CR
HR
AR

16,000
19,200
80,000
87,600
225,000
427,800
11,500
439,300

CR Current Rate
AR Average Rate
HR Historical Rate

(2)

Proof of Translation Adjustment

Net assets at beginning of year


Adjustments for changes in net
asset position during year:
Net income for year (sch. 1)
Dividends paid
Net assets translated at:
Rates during year
Rates at end of year
Change in OCI translation adj.
during year Net increase
Accumulated OCI translation
adjustment Jan. 1
Change in OCI translation
adjustment, Dec. 31 (credit)

Thailand
Baht
60,000

Translation
Rate
1.46

Philippine
Pesos
87,600

40,000
(15,000)

1.50
1.54

60,000
(23,100)

85,000

1.60

124,500
136,000
11,500
-011,500

135

Schedule 1:
Sales
Cost of goods sold
Depreciation expense
Operating expenses
Net income

150,000 Thailand Baht


( 70,000)
( 10,000)
( 30,000)
40,000 Thailand Baht

(b)
The change in the translation adjustment of P11,500 is included as a credit in the other
comprehensive income on the Statement of Comprehensive Income. The other comprehensive
income is then accumulated and reported in the stockholders equity section of the consolidated
balance sheet as presented below:
Net assets

P136,000

Common stock
Retained earnings, Dec. 31
Accumulated Other Comprehensive Income
Total

P 87,600
36,900
11,500
P136,000

Problem 20-3
a.

Translation Work paper

Cash
Accounts receivable
Inventory
Plant and equipment
Cost of sales
Operating expenses
Depreciation expense
Dividends
Total debits
Accumulated depreciation
Accounts payable
Common stock
Retained earnings, Jan. 1
Sales
Total
Accumulated OCI Translation
Adjustment
Total credits

Brunei $
1.600
2,500
4,000
35,000
17,000
7,000
3,000
1,500
71,600
9,000
2,600
20,000
10,000
30,000
71,600

Exchange
Rate
33 CR
33 CR
33 CR
33 CR
31 AR
31 AR
31 AR
32 HR

Philippine
Pesos
52,800
82,500
132,500
1,155,000
527,000
217,000
93,000
48,000
2,307,300

AR
CR
HR
HR
AR

297,000
85,800
600,000
300,000
930,000
2,212,800

31
33
30
30
31

94,500
2,307,300

136

Proof of Translation Adjustment (not required)

Net assets at beginning of year


Adjustment for net assets position
during the year:
Net income
Dividends paid
Net assets translated at rates
in effect for those items
Net assets at end of year
Change in translation adjustment during
Year to OCI net increase (credit)
Accumulated OCI translation adj. 1/1
Accumulated OCI translation
Adjustment 12/31 (credit)

b.

Brunei $
30,000

Translation
rate Philippine Pesos
30
900,000

3,000
(1,500)

31
32

93,000
(48,000)

31,500

33

945,000
1,039,500
94,500
-094,500

Parent Company entries affecting Investment in Moslem Co. (equity method)

Jan. 2:

Investment in Moslem Co.


Cash
To record investment cost.

Oct. 15:

Cash

48,000

Dec. 31:

Investment in Moslem Co.


Investment income
To record equity in income of Moslem

93,000

Investment in Moslem Co.


Other Comprehensive Income Translation
adjustment
To record parents share of change in translation
Adjustment

94,500

Investment in Moslem Co.


To record dividends received

900,000

900,000

48,000

93,000

94,500

137

Problem 20-4
UK Company
Translation Working Paper
Year Ended December 31, 2005

Income Statement
Sales
Cost of sales
Depreciation expense
Other expenses
Net income carried forward
Retained Earnings Statement
Balance, 1/1
Net income from above
Balance, 12/31
Balance Sheet
Cash
Accounts receivable
Inventories, at cost
Prepaid expenses
Property, plant and equipment (net)
Total assets
Accounts payable
Current portion of long-term debt
Long-term debt
Capital stock
Retained earnings from above
Total
Cumulative translation adjustment:
Balance, 1/1
Current translation adjustment
Balance, 12/31
Total liabilities and stockholders equity

In Pounds
90,000
(80,000)
(1,500)
(5,750)
2,750

Exchange
Rate
P67.50
67.50
67.50
67.50

2,500
2,750
5,250

In
Phil. Pesos

(A)
(A)
(A)
(A)

6,075,000
(5,400,000)
(101,250)
(388,125)
185,625

B
F

119,500
185,625
305,125

2,500
4,000
5,500
750
9,000
21,750

67.60
67.60
67.60
67.60
67.60

(C)
(C)
(C)
(C)
(C)

169,000
270,400
371,800
50,700
608,400
1,470,300

3,500
500
7,500
5,000
5,250

67.60
67.60
67.60
67.20

(C)
(C)
(C)
(H)

236,600
33,800
507,000
336,000
1,418,525

G
21,750

50,000
1,775
51,775
1,470,300

Translation Code:
C = Current rate
H = Historical rate
A = Average rate
B = Balance in Philippine pesos at the beginning of the year.
F = Per Income Statement

138

Problem 20-5
Goodluck Corporation
Foreign Exchange Translation Worksheet
Year Ended December 31, 2005

Cash
Marketable securities
Accounts receivable
Inventories
Property, plant and equip-net
Cost of goods sold
Depreciation expense
Other expenses
Totals

Trial
Balance
(In Pounds)
15,000
25,000
60,000
80,000
420,000
150,000
40,000
10,000
800,000

Accounts payable
Current portion of LT debt
Long-term debt
Sales
Other revenues
Capital stock
Retained earnings, 1/1
FC translation adjustment
Balance, 1/1
Current year
Net income

50,000
40,000
120,000
200,000
50,000
250,000
90,000

Totals

800,000

Exchange
Rate
0.95 C
0.95 C
0.95 C
0.95 C
0.95 C
0.90 A
0.90 A
0.90 A
0.95
0.95
0.95
0.90
0.90
0,87
G

C
C
C
A
A
H

G
B
B

Trial
Balance
(In Pesos)
14,250
23,750
57,000
76,000
399,000
135,000
36,000
9,000
750,000

Income
Statement
(In Pesos)

47,500
38,000
114,000
180,000
45,000
217,500
70,000
1,500
36,500
750,000

135,000
36,000
9,000
180,000

180,000
45,000

Balance
Sheet
(In Pesos)
14,250
23,750
57,000
76,000
399,000

570,000
47,500
38,000
114,000
217,500
70,000

(45,000)

1,500
36,500
45,000

180,000

570,000

Translation Code:
A = Average rate
B = Current rate
H = Historical rate
G = Given
B = Balancing amount

Problem 20-6
a. Direct and indirect exchange rates
January 1, 2007
December 31, 2007
December 31, 2008

Direct A$
P.03333=1
P.02857=1
P .025=1

Indirect
A$30=P1
A$35=P1
A$40=P1

The peso strengthened during 2007 because the number of A$ one Phil. Peso could acquire
at the end of the year (35) is greater than the number of A$ that could be acquired at the

139

beginning of the year (30); therefore, the value of the peso has increased relative to the A$
during 2007. The peso continued to strengthen during 2008.
b. Translated December 31, 2007, balance sheet:
Subsidiarys
Trial Balance
_ (in A$)__
A$ 100,000
400,000
680,000
1,000,000
R 2,230,000

Cash
Receivables
Inventory
Fixed assets
Total
Accumulated other
comprehensive income
translated adjustment (debit)
Total debits

Current payables
Long-term debt
Common stock
Retained earnings
Total credits

A$ 260,000
1,250,000
500,000
220,000
A$2,230,000

Direct
Exchange
Rate
P.02857
P.02857
P.02857
P.02857

Translated
Trial Balance
( in $)___
P 2,857
12,857
19,428
28,570
P 63,712

2,903
P 66,615
P.02857
P.02857
P.03333
P.03333

P 7,428
35,713
16,665
6,809
P 66,615

P.03333= average of beginning and ending exchange rates, rounded to 4 decimal points:
P.030945= [(P.03333 + P.02856) /2]
(Not required: Proof of translation adjustment (debit) of P 2,903)
___A$___
A$ 500,000

Net assets, 1/1/07


Adjustment for changes in
net assets during year:
Net income
220,000
Net assets translated at:
Rates during year
Rates at end of year
A$ 720,000
Change in translation
Adjustment during year (debit)

Translation
_ Rate_
P.03333
P.03095
P.02857

_Dollars_
P 16,665
6,809
P 23,474
(20,570)
P 2,904*

*Difference of P1 (P 2,904 P 2,903) due to rounding of exchange rates.

140

c. Translated December 31, 2008, balance sheet:

Cash
Receivables
Inventory
Fixed assets
Accumulated other
comprehensive incometranslation adjustment (debit)
Total debits

Subsidiarys
Trial Balance
(in A$)
A$
80,000
550,000
720,000
900,000
A$ 2,250,000

Direct
Exchange
__Rate
P.025
P.025
P.025
P.025

Translated
Trial Balance
(in P)__
P 2,000
13,750
18,000
22,500__
P56,250
5,635___
P61,885

(a)The retained earnings in pesos would begin with the December 31, 2007, peso
balance (P6,809) that would be carried forward. To this would be added 2008s net income
of A$90,000, which is the change in retained earnings in A$ multiplied by the 2008
exchange rate of P.02679 [(P.02857 + P.025/2)] which equals P2, 411. Therefore, translated
retained earnings on December 31, 2008, is P9, 220 (P9, 220= P6, 809 + P2, 411)
(Not required: Proof of translation adjustment (debit) of P5, 635)
Australian
Dollar
A$ 720,000

Net assets, 1/1/08


Adjustment for changes in
net assets during year:
Net income
90,000
Net assets translated at:
rates during year
Other comprehensive incomerate at end of year
A$ 810,000
Change in other comprehensive
income- translation
adjustment during year (debit)
Accumulated other comprehensive
income- translation adjustment, 1/1/08
Accumulated other comprehensive
income- translation adjustment, 12/31/08 (debit)
d.

Translation
_ Rate
P.02857
P.02679

Pesos___
P20, 570
2,411___
P22, 981

P.025

(20,250)__
P2, 731
2,904___
P5, 635

The P2, 731 change in the accumulated other comprehensive income- translation
adjustment during 2008 would be reported as a component of other comprehensive
income on 2008 statement of other comprehensive income.

141

CHAPTER 21
MULTIPLE CHOICE
21-1

21-2

21-3

21-4

21-5

21-6

21-7

21-8

21-9

21-10 c
21-11 d
21-12 b
21-13 b
21-14 a
21-15 a
Excess of income over expenses
Depreciation
Increase in due from national government agencies
Increase in prepaid rent
Increase in accounts payable
Cash provided by operating activities

P 200
70
( 10)
( 15)
30
P 275

142

Problem 21-1

PROBLEMS

1.

Memo entry in the RAOPS, RAOMO, RAOCO and RAOFE.

2.

Cash National Treasury MDS


Subsidy Income from National Government

3.

Memo entry in the RAOPS, RAOMO, RAOCO and RAOFE.

4.

Office equipment
Accounts payable

50,000

5.

Cash Disbursing Officer


Cash National Treasury MDS

40,000

6.

Salaries and wages Regular


Personal economic relief allowance (PERA)
Additional compensation
Due to BIR
Due to GSIS
Due to Pag-ibig
Due to Philhealth
Cash Disbursing Officer

44,000
3,000
3,000

Due to GSIS
Due to Pag-ibig
Due to Philhealth
Cash National Treasury MDS

5,500
400
600

Life and retirement contribution


Pag-ibig contribution
Philhealth contribution
Cash National Treasury MDS

5,500
400
600

Electricity
Telephone expense Landline
Accounts payable
Due to BIR
Cash National Treasury MDS

5,000
4,000
50,000

10.

Due to BIR
Subsidy income from national government

4,500

11.

Cash Collecting Officer


Sales revenue
Permit fees
Miscellaneous income

90,000

12.

Cash in Bank Local currency Current account


Cash Collecting Officer

90,000

7.

8.

9.

2,000,000

2,000,000

50,000
40,000

3,500
5,500
400
600
40,000

6,500

6,500

5,000
54,000
4,500
40,000
30,000
20,000
90,000

143

Problem 21-2
Building
1.

Memo entry in RAOCO.

2.

Advances to contractor
Cash National Treasury MDS

240

3.

Construction in progress Other Public Infrastructure


Advances to contractor
Accounts payable

400

4.

Accounts payable
Due to BIR
Cash National Treasury MDS

160

5.

Construction in progress Other Public Infrastructure


Accounts payable

400

6.

Accounts payable
Due to BIR
Cash National Treasury

400

7.

Due to BIR
Subsidy income from national government

8.

Office Building
Construction in progress OPI

80
800

240
240
160
40
120
400
40
360
80
800

Repairs of Building
1.

Memo entry in RAOCO.

2.

Construction materials inventory


Accounts payable

70

3.

Accounts payable
Due to BIR
Cash National Treasury MDS

70

4.

Construction in progress Other Public Infrastructure


Construction materials inventory

60

5.

Memo entry in the RAOCO

70
7
63
60

144

6.

Cash Disbursing Officer


Cash National Treasury MDS

36

7.

Construction in progress Other Public Infrastructure


Due to BIR
Cash Disbursing Officer

40

8.

Due to BIR
Cash National Treasury MDS

47

9.

Office building
Construction in progress OPI

100

36
4
36
47
100

Land:
1.

Memo entry in the RAOCO, P100.

2.

Land

3.

Accounts payable
Due to BIR
Cash National Treasury MDS

Accounts payable

100
100

100
10
90

145

Problem 21-3
(a)

Journal Entries:

1.

Memo entry in the Registry of Obligations and Allotments.

2.

Cash National Treasury MDS


Subsidy income from national government

3.

Memo entry in Registry of Obligations and Allotments.

4.

Office equipment
Accounts payable

120

5.

IT equipment and software


Accounts payable

30

6.

Prepaid rent
Cash National Treasury MDS

60

7.

Electricity expense
Cash National Treasury MDS

50

8.

Telephone expense Landline


Cash National Treasury MDS

40

9.

Petty cash fund


Cash National Treasury MDS

45

10.

Accounts payable
Due to BIR
Cash National Treasury MDS

120

11.

Accounts payable
Cash National Treasury MDS

30

12.

Due to BIR
Subsidy income from national government

12

13.

Cash Collecting Officer


Other service income
Sales revenue

50

14.

Cash in bank LCCA


Cash Collecting Officer

50

2,500

2,500

120
30
60
50
40
45
12
108
30
12
10
40
50

146

(b)

Pre-closing Trial Balance

Petty cash fund


Cash National Treasury MDS
Cash in Bank Local Currency Current Account
Prepaid rent
Office equipment
IT equipment and software
Other service income
Sales revenue
Subsidy income from national government
Electricity expense
Telephone expense landline
Total

45
2,167
50
60
120
30

50
40
2,562

10
40
2,512
2,562

Adjusting Entries
(1)

(2)

Depreciation Office equipment & software


Depreciation IT equipment
Accumulated depreciation Office equipment
Accumulated depreciation IT equip & software

20
5

Rent expense
Prepaid rent

30

20
5
30

Closing Entries:
(1)

Unused National Clearing Account (NCA)


Subsidy income from national government
Cash National Treasury MDS
NCA received during the year
Less: MDS check issued
Unused NCA

(2)

(3)

Income accounts:
Other service income
Sales revenue
Subsidy income from national government
Income and expense summary
Expense accounts:
Income and expense summary
Electricity expense
Telephone expense landline

2,167
2,500
333
2,167
10
40
345

90

(4)

Income and expense summary


Retained operating surplus

305

(5)

Retained operating surplus

305

Government equity

2,167

395

50
40
305

305

147

Problem 21-4
Agency VV
Statement of Income and Expenses
Year Ended December 31, 2008
Income:

Subsidy income from national government


Less: Reversion of unused NCA

P1,700
800

Less: Expenses
Salaries and wages Regular
Personnel Economic Relief Allowance
Additional compensation
Life and retirement insurance contribution
Pag-ibig contribution
Philhealth contgribution
Traveling expense Local
Office supplies expense
Electricity expense
Telephone expense landline
Janitorial services
Security services
Repairs and maintenance Office building
Depreciation Office building
Depreciation office equipment
Depreciation furniture and fixtures
Depreciation IT equipment and software
Net income over expenses

P 320
40
40
60
10
10
35
60
75
45
30
35
65
15
10
5
5

P900

860
P 40

148

Agency VV
Balance Sheet
As of December 31, 2008
ASSETS
Current Assets
Cash:
Cash in vault
Cash collecting officer
Cash disbursing officer
Petty cash fund
Cash in bank LCCA
Receivables:
Accounts receivable
Less: Allowance for doubtful accounts
Inventories:
Office supplies inventory
Other current assets
Long-term investment:
Investment in stock
Property, Plant and Equipment:
Land
Office building
Less: accumulated depreciation
Office equipment
Less: accumulated depreciation
Furniture and fixtures
Less: accumulated depreciation
IT equipment and software
Less: accumulated depreciation
Total assets
LIABILITIES AND EQUITY
Liabilities
Current liabilities
Accounts payable
Due to BIR
Due to GSIS
Due to Pag-ibig
Due to Philhealth
Other payables
Equity:
Government equity
Total liabilities and equity

P 200
500
1,000
150
350

P2,200

P 120
20

100
30
15
400

650
50
250
20
110
10
190
25

600
600
230
100
165

185
50
30
25
25
15

1,695
4,440

330
4,110
4,440

149

CHAPTER 22
Multiple Choice
22-1:

b.

22-2:

d.

(P500,000 P300,000)

The total tuition fees for educational and general purposes.


22-3:

d. (P1,240,000 P160,000)

22-4:

a.
Unrestricted cash contribution received from donors are to be reported as increase in net cash
provided by operation.

22-5:

d.
The remaining contribution of P5,000 on December 31, 2004.

22-6:

b.
Unregistered pledges from donors are treated as revenues at the time of the pledge.

22-7:

d.
Patient revenues
Nursing services
Professional services
Total revenues
Less:
Staff discounts
Allowances
Third party payors
Net revenues

P50,000
230,000
800,000

P 5,000,000
1,000,000
500,000
P 6,500,000
1,080,000
P 5,420,000

Bad debts are treated in the usual manner as expense.


22-8:

a.
As of July 31, 2004, all of the funds are properly includible in the Plan Funds, for a total of
P900,000.

22-9:

c. (P800,000 P110,000)

22-10: d.
Patient revenues (net of charity care)
Less: contractual adjustments
Net patient service revenues

P 600,000
200,000
P 400,000

22-11: c.

150

22-12: c.
The contributed services are debited to Salary Expense account and credited to Contribution
Revenue account.
22-13: c.
The net effect on unrestricted net assets of spending P10,000 on research is zero.
22-14: b. (P5,000,000 + P50,000)
The P1,000,000 contribution from the donor, who stipulated that the contribution be invested
Indefinitely, should be reported as permanently restricted revenue.
22-15: c.
22-16: b.
Both are treated as a financing activity on the statement of cash flows.
22-17: a.
Cash flows from operating activities would include both the cash received from patient service
Revenue of P300,000 and the cash received from gift shop sales of P25,000.
22-18: b.
Cash received from patient revenue (collection of receivables) and from tuition revenue are both
included in the amount reported for cash flows from operating activities. The other cash receipts
would be reported as increases in cash flows provided by financing activities.
22-19: b.
Expirations of donor restrictions on temporarily restricted net assets should be reported on the
Statement of operations as net assets released from restrictions.
22-20: c.
Current funds revenues include (1) all unrestricted gifts and other unrestricted resources earned
during the reporting period, and (2) restricted current funds to the extent that such funds were
expended for current operating purpose. Therefore, the amount that should be included in current
funds revenue is:
Unrestricted gifts received:
Expended
Not expended
Restricted gifts received
Expended
Total

P600,000
75,000
100,000
P775,000

151

Problems
Problem 22-1
1.

Pledges receivable
Allowance for uncollectible pledges
Contribution revenue

300,000

2.

Cash

260,000

3.

Cash
Fund raising expense
Fund raising revenue

40,000
5,000

4.

Investment

35,000

5.

Cash

6.

Salaries
Employee fringe benefits
Payroll taxes
Supplies
Telephone
Utilities
Rent
Conference, conventions and meetings
Cost of sales to public
Miscellaneous
Cash

7.

8.

Utilities
Salaries

Pledges receivable

Cash
Sales public revenue

Accounts payable or accrued expense payable

Fund Balance - Unrestricted


Fund balance Restricted to purchases
of new equipment.

5,000
90,000
15,000
16,000
7,000
1,500
6,000
10,000
5,000
1,000
3,000
1,000
5,000

10,000
270,000
260,000

45,000
35,000
5,000

154,500

6,000

10,000
10,000

152

Problem 22-2
(1)

Accounts receivable
Patient service revenues
To record gross patient service revenue for the month
at full rates.

80,000

Accounts receivable
Patient service revenues
To record receivable from Social Medicare.

2,500

Contractual adjustments
Accounts receivable
To record contractual adjustments allowed.

6,000

Doubtful accounts
Allowance for doubtful accounts
To provide allowances for doubtful accounts.

8,000

(2)

Salaries expense
Contribution revenues
To record donated services (10,000 200).

9,800

(3)

Pledges receivable
Contribution revenues
To record pledges received from donors.

5,000

Cash

3,500

Pledges receivable
To record pledges collected.
Provisions for doubtful pledges
Allowance for doubtful pledges
To provide doubtful pledges.
(4)

800

Cash

3,000

Plant assets
Cash
To record acquisition of new surgical equipment.

3,000

Fund balance
To record receipt of cash from restricted fund.

80,000

2,500

6,000

8,000

9,800

5,000

3,500

800

3,000

3,000

153

Problem 22-3
Plant Fund Ledger Account:
(1)

Equivalent

(2)

Buildings

Fund Balance
To record acquisitions of computers from unrestricted fund.
Cash
Mortgage notes payable
To record construction of new building financed in
part by 5% mortgage note payable.

50,000

2,000,000

50,000

250,000
1,750,000

Quasi-Endowment Fund Ledger Account:


(3)

Cash

Investments
Payable to Unrestricted fund
To record sale of investments at a gain, the use of
which is unrestricted.

110,000

100,000
10,000

Unrestricted Fund Ledger Account:


(1)

Undesignated fund balance


Cash
To record acquisitions of computers to be carried in
Plant Fund.

(2)

Cash

(3)

Receivable from quasi-endowment fund


Investment income
To record investment gain receivable.

Contribution revenues
To record receipt of unrestricted gift.

50,000

2,000,000

10,000

50,000

2,000,000

10,000

154

Problem 22-4
Nonprofit Trade Association
Statement of Activities
Year Ended June 30, 2004
Revenues and Gains:
Membership dues
Conferences and meetings
Publications and advertising sales
Special assessments
Investment income, including net gains
Total
Expenses:
Member services
Conferences and meetings
Technical services
Communications
General administration
Membership development
Increase in unrestricted net assets
Net assets, beginning of year
Net assets, end of year

P184,000
321,000
143,000
50,000
11,000
P709,000
P 56,000
166,000
218,000
61,000
154,000
27,000

682,000
27,000
285,000
P312,000

Nonprofit Trade Association


Statement of Financial Position
June 30, 2004
ASSETS
Current assets
Cash
Short-term investments
Accounts receivable (net)
Publications inventory
Total current assets
Long-term investments
Plant assets (net)
Other assets
Total assets

P 7,000
217,000
25,000
61,000
310,000
120,000
33,000
28,000
P491,000

LIABILITIES AND NET ASSETS


Current liabilities
Accounts payable and accrued liabilities
Deferred membership dues
Total current liabilities
Net assets (unrestricted)
Total liabilities and net assets

P 48,000
131,000
179,000
312,000
P491,000

155

Problem 22-5
Children Association
Statement of Activities
Year Ended December 31, 2004
Changes in unrestricted net assets:
Revenues and gains:
Contributions
Membership dues
Program service fees
Investment income
Total unrestricted revenues and gains
Expenses:
Programs
Management and general expenses
Fund raising
Increase in unrestricted net assets
Changes in temporarily restricted net assets:
Contributions
Expenses:
Management and general expenses
Fund raising expenses
Increase in temporarily restricted net assets
Increase in net assets
Net assets, beginning of year (P12,000 + P26,000 + P3,000)
Net assets, end of year

P320,000
25,000
30,000
10,000
P385,000
P270,000
47,000
8,000

325,000
P 60,000
P 15,000

P 4,000
1,000

5,000
P 10,000
P 70,000
41,000
P111,000

Children Association
Statement of Financial Position
December 31, 2004
ASSETS
Cash (P40,000 + P9,000)
Bequest and interest receivable (P5,000 + P1,000)
Pledges receivable (net) (P12,000 P3,000)
Investments, at cost
Total assets
LIABILITIES AND NET ASSETS
Liabilities
Accounts payable and accrued liabilities (P50,000 + P1,000)
Deferred revenues
Total liabilities
Net assets:
Unrestricted (P38,000 + P60,000)
Temporarily restricted (P3,000 + P10,000)
Total net assets
Total liabilities and net assets

P 40,000
6,000
9,000
100,000
P164,000
P 51,000
2,000
P 53,000
P 98,000
13,000
P111,000
P164,000

156

Problem 22-6
San Pedro Hospital
Statement of Financial Position
June 30, 2004
ASSETS
Current assets
Cash
Accounts receivable (net of allowance of P5,000)
Inventories
Prepaid expenses
Total current assets
Investments
Property, plant and equipment (net of accumulated depreciation of P140,000)
Total assets
LIABILITIES AND NET ASSETS
LIABILITIES
Current liabilities:
Accounts payable
Accrued expenses
Deferred revenues
Current portion of long-term debt
Total current liabilities
Mortgage payable
Total liabilities
NET ASSETS
Unrestricted
Temporarily restricted
Permanently restricted
Total net assets
Total liabilities and net assets

P 222,000
20,000
50,000
10,000
P 302,000
660,000
160,000
P1,122,000

45,000
17,000
11,000
24,000
P 97,000
125,000
P 222,000
P 148,000
232,000
520,000
P 900,000
P1,122,000

157

Vous aimerez peut-être aussi